Sunteți pe pagina 1din 140

RMM - Cyclic Inequalities Marathon 1 - 100

ROM A N IA N MAT HEMAT IC AL MAG AZINE

Founding Editor
DANIEL SITARU
Available online ISSN-L 2501-0099
www.ssmrmh.ro
SOCIETATEA DE ȘTIINȚE MATEMATICE DIN ROMÂNIA
ROMANIAN MATHEMATICAL SOCIETY

Filiala Mehedinți - Mehedinți Branch


www.ssmrmh.ro

RMM
CYCLIC INEQUALITIES
MARATHON
1 – 100

1
SOCIETATEA DE ȘTIINȚE MATEMATICE DIN ROMÂNIA
ROMANIAN MATHEMATICAL SOCIETY

Filiala Mehedinți - Mehedinți Branch


www.ssmrmh.ro

Proposed by
Daniel Sitaru – Romania, George Apostolopoulos - Messolonghi - Greece
Nguyen Viet Hung – Hanoi – Vietnam , Babis Stergioiu – Greece
Erdene Natsagdorj – Ulanbaatar-Mongolia , Abhay Chandra-India
Dorin Marghidanu – Romania, Vaggelis Stamatiadis-Greece
Le Viet Hung –Hai Lang-Vietnam, Sladjan Stankovic-Macedonia
Richdad Phuc-Hanoi-Vietnam, Hoang Le Nhat Tung – Hanoi – Vietnam
Anish Ray-Santaragachi-India, Marin Chirciu – Romania
Nguyen Phuc Tang - Dong Thap – Vietnam , Mihalcea Andrei Ștefan – Romania
Adil Abdullayev – Baku – Azerbaidian

2
SOCIETATEA DE ȘTIINȚE MATEMATICE DIN ROMÂNIA
ROMANIAN MATHEMATICAL SOCIETY

Filiala Mehedinți - Mehedinți Branch


www.ssmrmh.ro

Solutions by
Daniel Sitaru – Romania, Henry Ricardo - New York – USA
Kunihiko Chikaya – Tokyo – Japan, Imad Zak – Saida – Lebanon
Ngoc Minh Ngoc Bao-Gia Lan Province-Vietnam , Fotini Kaldi – Greece
Redwane El Mellass-Casablanca-Morroco, Sk Rejuan-West Bengal-India
Myagmarsuren Yadamsuren – Mongolia , Safal Das Biswas – Chinsurah – India
Kevin Soto Palacios – Huarmey-Peru , Nguyen Viet Hung – Hanoi – Vietnam
Soumitra Mandal-Chandar Nagore-India, Abhay Chandra – India
Marian Dincă – Romania , Ravi Prakash - New Delhi – India
Leonard Giugiuc-Romania, Seyran Ibrahimov – Maasilli – Azerbaidian
Abdallah El Farisi-Bechar-Algerie, Anh Tai Tran – Hanoi – Vietnam
Le Viet Hung- Quang Tri – Vietnam , Nirapada Pal-India,
Soumava Chakraborty – Kolkata – India, Anas Adlany-El Jadida-Morroco,
Phan Loc So'n-Quy - Nhon City – Vietnam, Saptak Bhattacharya-Kolkata-India,
Nguyen Minh Triet-Quang Ngai-Vietnam
Mihalcea Andrei Stefan-Romania, Soumitra Mandal-Chandar Nagore-India
Hoang Le Nhat Tung – Hanoi – Vietnam,
Khung Long Xanh - Da Nang – Vietnam
Nguyen Phuc Tang - Dong Thap – Vietnam, Sladjan Stankovic – Macedonia
Aditya Narayan Sharma – Kanchrapara – India,
Richdad Phuc – Hanoi – Vietnam
Pham Quy –Quang Ngai- Vietnam, Hamza Mahmood-Lahore-Pakistan

3
SOCIETATEA DE ȘTIINȚE MATEMATICE DIN ROMÂNIA
ROMANIAN MATHEMATICAL SOCIETY

Filiala Mehedinți - Mehedinți Branch


www.ssmrmh.ro

1. Given 𝒂, 𝒃, 𝒄, 𝒅 be positive real number such that 𝒂 = 𝟒


𝟏 𝟏 𝟏 𝟏
Prove that: + + + ≥ 𝒂𝟐 + 𝒃𝟐 + 𝒄𝟐 + 𝒅𝟐 .
𝒂𝒃 𝒃𝒄 𝒄𝒅 𝒅𝒂

Proposed by Sladjan Stankovic-Macedonia

Solution by Ngoc Minh Ngoc Bao-Gia Lan Province-VietNam


Without loss of generality, assume that: 𝒂 ≥ 𝒃 ≥ 𝒄 ≥ 𝒅.
𝟏 𝟏 𝟏 𝟏
Considering function: 𝒇 𝒂, 𝒃, 𝒄, 𝒅 = + + + − 𝒂 𝟐 − 𝒃𝟐 − 𝒄𝟐 − 𝒅𝟐
𝒂𝒃 𝒃𝒄 𝒄𝒅 𝒅𝒂
𝒂.𝒄 𝒂.𝒄 𝟒 𝟒 𝒂.𝒄 𝟐
We have: 𝒇 , 𝒃, ,𝒅 = + – − 𝒃𝟐 − 𝒅𝟐
𝟐 𝟐 𝒂.𝒄 𝒃 𝒂.𝒄 𝒅 𝟐

𝒂+𝒄 𝒂+𝒄 𝟐
𝟏 𝟏 𝟏 𝟏
⇒ 𝒇 𝒂, 𝒃, 𝒄, 𝒅 − 𝒇 , 𝒃, ,𝒅 = 𝒂 − 𝒄 + −
𝟐 𝟐 𝒂𝒄 𝒂 + 𝒄 𝒃 𝒅 𝟐
𝟏 𝟏 𝟏 𝟏
We need to prove that: + − ≥ 𝟎
𝒂𝒄 𝒂.𝒄 𝒃 𝒅 𝟐
𝟒
Use AM-GM inequality: 𝟒 = 𝒂 + 𝒃 + 𝒄 + 𝒅 ≥ 𝟒 𝒂𝒃𝒄𝒅 ⇒ 𝒂𝒃𝒄𝒅 ≤ 𝟏 ⇒ 𝒂𝟐 𝒄𝟐 ≤
𝟏.
𝟏 𝟏 𝟏 𝟏 𝟏 𝟏 𝟏 𝟏 𝟏 𝟏 𝒂𝟐 𝒄𝟐 𝟏
And + ≥ + ⇒ + − ≥ − ≥ − > 𝟎
𝒃 𝒅 𝒂 𝒄 𝒂𝒄 𝒂.𝒄 𝒃 𝒅 𝟐 𝒂𝟐 𝒄 𝟐 𝟐 𝒂𝟐 𝒄𝟐 𝟐
𝟐 𝟐
We prove that: 𝒇 𝒕, 𝒕, 𝒕, 𝒅 = + − 𝟑𝒕𝟐 − 𝒅𝟐 ≥ 𝟎. With 𝟑𝒕 + 𝒅 = 𝟒.
𝒕𝟐 𝒕𝒅

𝟐 𝟐 𝒕7𝟏 𝟐 𝟑𝟔7𝟒𝟖𝒕𝟐 .𝟏𝟐𝒕.𝟖 𝒕7𝟏 𝟐


Indeed 𝟐 + − 𝟑𝒕𝟐 − 𝒅𝟐 = ≥ . 𝑴𝒊𝒏𝒈 𝒕 ≥ 𝟎
𝒕 𝒅𝒕 𝒕𝟐 𝟒7𝟑𝒕 𝒕𝟐 𝟒7𝟑𝒕

𝟒. 𝟕
(With 𝒈 𝒕 = 𝟑𝟔 − 𝟒𝟖𝒕𝟐 + 𝟏𝟐𝒕 + 𝟖, and 𝑴𝒊𝒏𝒈 𝒕 = 𝒈 > 𝟎)
𝟗

4
SOCIETATEA DE ȘTIINȚE MATEMATICE DIN ROMÂNIA
ROMANIAN MATHEMATICAL SOCIETY

Filiala Mehedinți - Mehedinți Branch


www.ssmrmh.ro

2. If 𝒙, 𝒚, 𝒛, 𝒕, 𝒂, 𝒃, 𝒄 ∈ 𝟎, ∞ , 𝒙𝒚𝒛𝒕 = 𝒂𝟒 then:
𝒙𝒃 + 𝒚𝒄
𝟑𝒂 ≥ 𝟒 𝒂𝒃 + 𝒂𝒄
𝒚+𝒛+𝒕
Proposed by Daniel Sitaru – Romania
Solution 1 by Daniel Sitaru-Romania
Solution 2 by Myagmarsuren Yadamsuren – Mongolia

Solution 1 by Daniel Sitaru-Romania
𝟒
𝒙 + 𝒚 + 𝒛 + 𝒕 ≥ 𝟒 𝟒 𝒙𝒚𝒛𝒕 = 𝟒 𝒂𝟒 = 𝟒𝒂 (AM-GM)
𝒙𝒃 𝒙 𝒃
𝒙 𝒃7𝟏
≥ 𝒃7𝟐 = ≥
𝒚+𝒛+𝒕 𝟒 ⋅𝟑 𝒙 𝟑 ⋅ 𝟒𝒃7𝟐
𝟒𝒂 𝒃F𝟏 𝟒𝒃F𝟏 ⋅𝒂𝒃F𝟏 𝟒⋅𝒂𝒃F𝟏
≥ = = (1)
𝟑⋅𝟒𝒃F𝟐 𝟑⋅𝟒𝒃F𝟐 𝟑

𝒚𝒄 𝒙 𝒄
𝒙 𝟒7𝟏
≥ 𝒄7𝟐 = ≥
𝒚+𝒛+𝒕 𝟒 ⋅𝟑 𝒙 𝟒𝒄7𝟐 ⋅ 𝟑
𝟒𝒂 𝒄F𝟏 𝟒𝒄F𝟏 ⋅𝒂𝒄F𝟏 𝟒⋅𝒂𝒄F𝟏
≥ = = (2)
𝟑⋅𝟒𝒄F𝟐 𝟑⋅𝟒𝒄F𝟐 𝟑

By adding (1); (2):


𝒙𝒂 + 𝒚𝒄 𝟒 𝒃7𝟏
≥ 𝒂 + 𝒂𝒄7𝟏
𝒚+𝒛+𝒕 𝟑

Solution 2 by Myagmarsuren Yadamsuren – Mongolia
𝒙, 𝒚, 𝒛, 𝒕, 𝒂, 𝒃 ∈ 𝟎, ∞ 𝒙𝒚𝒛𝒕 = 𝒂𝟒
𝒙𝒃 .𝒚𝒄
Prove that: 𝑳𝑯𝑺 = 𝟑𝒂 ⋅ ≥ 𝟒 ⋅ 𝒂𝒃 + 𝒂𝒄
𝒚.𝒛.𝒕

5
SOCIETATEA DE ȘTIINȚE MATEMATICE DIN ROMÂNIA
ROMANIAN MATHEMATICAL SOCIETY

Filiala Mehedinți - Mehedinți Branch


www.ssmrmh.ro

𝑪𝒉𝒆𝒃𝒚𝒔𝒉𝒆𝒗
𝒙𝒃 + 𝒚𝒄 𝟏 𝟏
≥ ⋅ 𝒙𝒃 + 𝒙𝒄 ⋅
𝒚+𝒛+𝒕 𝟒 𝒙+𝒚+𝒛
𝑪𝒂𝒖𝒄𝒉𝒚7𝑺𝒄𝒉𝒘𝒂𝒓𝒛
𝟏 𝟒𝟐
≥ ⋅ 𝒙𝒃 + 𝒙𝒄 ⋅ =
𝟒 𝟑⋅ 𝒙+𝒚+𝒛+𝒕
𝑪𝒉𝒆𝒃𝒚𝒔𝒉𝒆𝒗
𝟒 𝒙𝒃 + 𝒙𝒄
= ⋅ ≥
𝟑 𝒙+𝒚+𝒛+𝒕
𝟒 𝟏 𝒙+𝒚+𝒛+𝒕 ⋅ 𝒙𝒃7𝟏 + 𝒙𝒄7𝟏
≥ ⋅ ⋅ =
𝟑 𝟒 𝒙+𝒚+𝒛+𝒕
𝑪𝒂𝒖𝒄𝒉𝒚
𝟏 𝟏 𝟒
𝒃7𝟏 𝒄7𝟏
= ⋅ 𝒙𝒃7𝟏 + 𝒙𝒄7𝟏 ≥ ⋅𝟒 𝒙𝒚𝒛𝒕 + 𝟒
𝒙𝒚𝒛𝒕 =
𝟑 𝟑
𝟒
= ⋅ 𝒂𝒃7𝟏 + 𝒂𝒄7𝟏
𝟑
𝒙𝒃 + 𝒚𝒄 𝟒
𝑳𝑯𝑺 = 𝟑 ⋅ 𝒂 ⋅ ≥ 𝟑𝒂 ⋅ ⋅ 𝒂𝒃7𝟏 + 𝒂𝒄7𝟏 = 𝟒 ⋅ 𝒂𝒃 + 𝒂𝒄
𝒚+𝒛+𝒕 𝟑

3. Prove that for any real numbers 𝒂, 𝒃 and 𝒄 the following inequality holds:
𝟖 𝒂𝟐 − 𝒃𝒄 𝒃𝟐 − 𝒄𝒂 𝒄𝟐 − 𝒂𝒃 ≤ 𝒂𝟐 + 𝒃𝟐 + 𝒄𝟐 𝟑
Proposed by Richdad Phuc-Vietnam
Solution by Leonard Giugiuc-Romania
𝒂𝟐 − 𝒃𝒄 𝒃𝟐 − 𝒄𝒂 𝒄𝟐 − 𝒂𝒃 = 𝒑 𝒂𝟑 + 𝒃𝟑 + 𝒄𝟑 − 𝒂𝟑 𝒃𝟑 + 𝒃𝟑 𝒄𝟑 + 𝒂𝟑 𝒄𝟑 ,
where 𝒑 = 𝒂𝒃𝒄.
But 𝒂𝟑 + 𝒃𝟑 + 𝒄𝟑 = 𝒂 + 𝒃 + 𝒄 𝟑
− 𝟑 𝒂 + 𝒃 + 𝒄 𝒂𝒃 + 𝒂𝒄 + 𝒃𝒄 + 𝟑𝒑 and
𝒂𝟑 𝒃𝟑 + 𝒃𝟑 𝒄𝟑 + 𝒂𝟑 𝒄𝟑 = 𝒂𝒃 + 𝒃𝒄 + 𝒄𝒂 𝟑
− 𝟑𝒑 𝒂 + 𝒃 + 𝒄 𝒂𝒃 + 𝒂𝒄 + 𝒃𝒄 + 𝟑𝒑𝟐
Case 1: 𝒂 + 𝒃 + 𝒄 = 𝟎 ⇒ 𝒂𝟑 + 𝒃𝟑 + 𝒄𝟑 = 𝟑𝒑 and
𝒂𝟑 𝒃𝟑 + 𝒃𝟑 𝒄𝟑 + 𝒂𝟑 𝒄𝟑 = 𝒂𝒃 + 𝒃𝒄 + 𝒄𝒂 𝟑
+ 𝟑𝒑𝟐 ⇒

6
SOCIETATEA DE ȘTIINȚE MATEMATICE DIN ROMÂNIA
ROMANIAN MATHEMATICAL SOCIETY

Filiala Mehedinți - Mehedinți Branch


www.ssmrmh.ro

⇒ 𝟖 𝒂𝟐 − 𝒃𝒄 𝒃𝟐 − 𝒄𝒂 𝒄𝟐 − 𝒂𝒃 = −𝟖 𝒂𝒃 + 𝒃𝒄 + 𝒄𝒂 𝟑 . But
𝒂𝟐 + 𝒃𝟐 + 𝒄𝟐 = −𝟐 𝒂𝒃 + 𝒂𝒄 + 𝒃𝒄 ⇒ 𝒂𝟐 + 𝒃𝟐 + 𝒄𝟐 𝟑
= −𝟖𝟗 𝒂𝒃 + 𝒃𝒄 + 𝒄𝒂 𝟑
So actually we’ve got the identity
𝟖 𝒂𝟐 − 𝒃𝒄 𝒃𝟐 − 𝒄𝒂 𝒄𝟐 − 𝒂𝒃 = 𝒂𝟐 + 𝒃𝟐 + 𝒄𝟐 𝟑 , hence proved.
Case 2: 𝒂 + 𝒃 + 𝒄 ≠ 𝟎. Assume WLOG
𝒂 + 𝒃 + 𝒄 = 𝟑 ⇒ 𝒂𝒃 + 𝒃𝒄 + 𝒄𝒂 = 𝟑 𝟏 − 𝒕𝟐 , 𝒕 ≥ 𝟎. We get:
𝒂𝟐 + 𝒃𝟐 + 𝒄𝟐 = 𝟑 𝟏 + 𝟐𝒕𝟐 , 𝒂𝟑 + 𝒃𝟑 + 𝒄𝟑 = 𝟑𝒑 𝟗𝒕𝟐 + 𝒑 and
𝒂𝟑 𝒃𝟑 + 𝒃𝟑 𝒄𝟑 + 𝒂𝟑 𝒄𝟑 = 𝟐𝟕 𝟏 − 𝒕𝟐 − 𝟐𝟕𝒑 𝟏 − 𝒕𝟐 + 𝟑𝒑𝟐 . Hence we need to
prove
𝟖 𝒕𝟐 − 𝟏 𝟑
+ 𝟖𝒑 ≤ 𝟏 + 𝟐𝒕𝟐 𝟑 . It’s well known that 𝒑 ≤ 𝟐𝒕𝟑 − 𝟑𝒕𝟐 + 𝟏. It’s
enough to show
𝟖 𝒕𝟐 − 𝟏 𝟑
+ 𝟖 𝟐𝒕𝟑 − 𝟑𝒕𝟐 + 𝟏 ≤ 𝟏 + 𝟐𝒕𝟐 𝟑

𝟑𝟔𝒕𝟒 − 𝟏𝟔𝒕𝟑 + 𝟔𝒕𝟐 + 𝟏 ≥ 𝟎 ⇔ 𝟐𝒕𝟐 𝟏𝟖𝒕𝟐 − 𝟖𝒕 + 𝟑 + 𝟏 ≥ 𝟎, which is clearly
true since
𝟐𝒕𝟐 𝟏𝟖𝒕𝟐 − 𝟖𝒕 + 𝟑 ≥ 𝟎 and 𝟏 > 𝟎.
Remark: From above, if 𝒂 + 𝒃 + 𝒄 = 𝟎 we have equality and if 𝒂 + 𝒃 + 𝒄 ≠ 𝟎
inequality is strict.

4. If 𝒂, 𝒃, 𝒄 ∈ 𝟎, ∞ then:
𝒂𝟐 𝒃𝟐 𝒃𝟐 𝒄𝟐 𝒄𝟐 𝒂𝟐 𝟏 𝟏 𝟏
+ + ≤ + +
𝒂 𝟓 + 𝒃𝟓 𝒃𝟓 + 𝒄𝟓 𝒄𝟓 + 𝒂 𝟓 𝒂 + 𝒃 𝒃 + 𝒄 𝒄 + 𝒂
Proposed by Daniel Sitaru – Romania

7
SOCIETATEA DE ȘTIINȚE MATEMATICE DIN ROMÂNIA
ROMANIAN MATHEMATICAL SOCIETY

Filiala Mehedinți - Mehedinți Branch


www.ssmrmh.ro

Solution by Kevin Soto Palacios – Huarmey-Peru


Si: 𝒂, 𝒃, 𝒄 ∈ < 0, ∞ >. Probar que:
𝒂𝟐 𝒃𝟐 𝒃𝟐 𝒄𝟐 𝒄𝟐 𝒂𝟐 𝟏 𝟏 𝟏
+ + ≤ + +
𝒂 𝟓 + 𝒃𝟓 𝒃𝟓 + 𝒄𝟓 𝒄𝟓 + 𝒂 𝟓 𝒂 + 𝒃 𝒃 + 𝒄 𝒄 + 𝒂
La desigualdad es equivalente:
𝟏 𝒂𝟐 𝒃𝟐 𝟏 𝒃𝟐 𝒄𝟐 𝟏 𝒂 𝟐 𝒄𝟐
− + − + − ≥ 𝟎
𝒂 + 𝒃 𝒂 𝟓 + 𝒃𝟓 𝒃 + 𝒄 𝒃𝟓 + 𝒄𝟓 𝒂 + 𝒄 𝒂 𝟓 + 𝒄𝟓
Tener presente lo siguiente:
𝒂𝟓 + 𝒃𝟓 = 𝒂 + 𝒃 𝒂𝟒 − 𝒂𝟑 𝒃 + 𝒂𝟐 𝒃𝟐 − 𝒂𝒃𝟑 + 𝒃𝟒 ⇔ (Cocientes Notables)
𝟏 𝒂𝟐 𝒃𝟐 𝒂𝟓 .𝒃𝟓 7 𝒂.𝒃 𝒂𝟐 𝒃𝟐
Sea: 𝑨 = − =
𝒂.𝒃 𝒂𝟓 .𝒃𝟓 𝒂𝟓 .𝒃𝟓 𝒂.𝒃

𝒂+𝒃 𝒂𝟒 + 𝒃𝟒 − 𝒂𝒃 𝒂𝟐 + 𝒃𝟐 𝒂𝟒 + 𝒃𝟒 − 𝒂𝒃 𝒂𝟐 + 𝒃𝟐
𝑨= =
𝒂𝟓 + 𝒃𝟓 𝒂 + 𝒃 𝒂𝟓 + 𝒃𝟓
Desde que: 𝒂, 𝒃, 𝒄 ∈ < 0, ∞ >. Por: 𝑴𝑨 ≥ 𝑴𝑮
𝒂𝟒 + 𝒂𝟒 + 𝒂𝟒 + 𝒃𝟒 ≥ 𝟒𝒂𝟑 𝒃 ∧ 𝒃𝟒 + 𝒃𝟒 + 𝒃𝟒 + 𝒃𝟒 ≥ 𝟒𝒃𝟑 𝒂
Sumando: 𝒂𝟒 + 𝒃𝟒 ≥ 𝒂𝒃 𝒂𝟐 + 𝒃𝟐 → 𝒂𝟒 + 𝒃𝟒 − 𝒂𝒃 𝒂𝟐 + 𝒃𝟐 ≥ 𝟎
𝒂𝟒 .𝒃𝟒 7𝒂𝒃 𝒂𝟐 .𝒃𝟐
Por la tanto: 𝑨 = ≥ 𝟎
𝒂𝟓 .𝒃𝟓

𝟏 𝒂𝟐 𝒃𝟐
→ − ≥ 𝟎
𝒂+𝒃 𝒂𝟓 + 𝒃𝟓

5. If 𝒂, 𝒃, 𝒄 ∈ 𝟎, ∞ then:
𝟒
𝒂𝟖 𝒃𝟖 + 𝒃𝟖 𝒄𝟖 + 𝒄𝟖 𝒂𝟖 ≥ 𝒂𝟓 𝒃𝟓 𝒄𝟓 𝟐𝟕 𝒂𝟒 + 𝒃𝟒 + 𝒄𝟒
Proposed by Daniel Sitaru – Romania

8
SOCIETATEA DE ȘTIINȚE MATEMATICE DIN ROMÂNIA
ROMANIAN MATHEMATICAL SOCIETY

Filiala Mehedinți - Mehedinți Branch


www.ssmrmh.ro

Solution by Kevin Soto Palacios-Huarmey-Peru


Si: 𝒂, 𝒃, 𝒄 ∈ < 0, ∞ >. Probar que:
𝟒
𝒂𝟖 𝒃𝟖 + 𝒃𝟖 𝒄𝟖 + 𝒄𝟖 𝒂𝟖 ≥ 𝒂𝟓 𝒃𝟓 𝒄𝟓 𝟐𝟕 𝒂𝟒 + 𝒃𝟒 + 𝒄𝟒
Desde que: 𝒂, 𝒃, 𝒄 ∈< 0, ∞ >
⇒ 𝒂𝟖 𝒃 𝟖 + 𝒃𝟖 𝒄𝟖 + 𝒄𝟖 𝒂 𝟖 ≥ 𝒂𝟒 𝒃 𝟒 𝒄𝟒 𝒂 𝟒 + 𝒃𝟒 + 𝒄𝟒
Por lao cual nos falta probar que:
𝟒
𝒂𝟒 𝒃𝟒 𝒄𝟒 𝒂𝟒 + 𝒃𝟒 + 𝒄𝟒 ≥ 𝒂𝟓 𝒃𝟓 𝒄𝟓 𝟐𝟕 𝒂𝟒 + 𝒃𝟒 + 𝒄𝟒
𝟒
⇒ 𝒂𝟒 + 𝒃𝟒 + 𝒄𝟒 ≥ 𝒂𝒃𝒄 𝟐𝟕 𝒂𝟒 + 𝒃𝟒 + 𝒄𝟒
⇒ 𝒂 𝟒 + 𝒃𝟒 + 𝒄𝟒 𝟒
≥ 𝒂𝒃𝒄 𝟒
⋅ 𝟐𝟕 𝒂𝟒 + 𝒃𝟒 + 𝒄𝟒
⇒ 𝒂 𝟒 + 𝒃𝟒 + 𝒄𝟒 𝟑
≥ 𝟐𝟕 𝒂𝒃𝒄 𝟒
→ Válido por: (MA≥MG).
𝟒
Por la tanto: 𝒂𝟖 𝒃𝟖 + 𝒃𝟖 𝒄𝟖 + 𝒄𝟖 𝒂𝟖 ≥ 𝒂𝟓 𝒃𝟓 𝒄𝟓 𝟐𝟕 𝒂𝟒 + 𝒃𝟒 + 𝒄𝟒
La igualdad se alcanza cuando: 𝒂 = 𝒃 = 𝒄
6. Let 𝒂, 𝒃, 𝒄 be positive real numbers such that:
𝟏 𝟏 𝟏
+ + = 𝟏.
𝟏+𝒂 𝟏+𝒃 𝟏+𝒄
Prove that
𝒂 − 𝟏 𝒃 − 𝟏 𝒄 − 𝟏 ≤ 𝟏
Proposed by Nguyen Viet Hung – Hanoi – Vietnam
Solution by Kevin Soto Palacios – Huarmey – Peru
𝟏 𝟏 𝟏
Sean: 𝒂, 𝒃, 𝒄 números reales positivos tal que : + + = 𝟏 (A)
𝟏.𝒂 𝟏.𝒃 𝟏.𝒄

Probar que:
𝒂 − 𝟏 𝒃 − 𝟏 𝒄 − 𝟏 ≤ 𝒂𝒃𝒄 (B)
𝒙.𝒚 𝒛.𝒙 𝒚.𝒛
Sea: 𝒂 = > 0, 𝑏 = > 0, 𝑐 = > 0
𝒛 𝒚 𝒙

9
SOCIETATEA DE ȘTIINȚE MATEMATICE DIN ROMÂNIA
ROMANIAN MATHEMATICAL SOCIETY

Filiala Mehedinți - Mehedinți Branch


www.ssmrmh.ro

Reemplazando en (A)
𝟏 𝟏 𝟏 𝒛 𝒚 𝒙
𝒙+𝒚 + 𝒛+𝒙 + 𝒚 + 𝒛 𝒙 + 𝒚 + 𝒛 𝒙 + 𝒚 + 𝒛 𝒙 + 𝒚 + 𝒛 = 𝟏
= + +
𝟏+ 𝟏+ 𝟏+
𝒛 𝒚 𝒙
Por la tanto (B) es equivalente:
𝒙+𝒚 𝒛+𝒙 𝒚+𝒛
𝒂−𝟏 𝒃−𝟏 𝒄−𝟏 = −𝟏 −𝟏 − 𝟏 ≤ 𝟏
𝒛 𝒚 𝒙
⇒ 𝒙 + 𝒚 − 𝒛 𝒛 + 𝒙 − 𝒚 𝒚 + 𝒛 − 𝒙 ≤ 𝒙𝒚𝒛
→ 𝒙 + 𝒚 − 𝒛 𝒛 + 𝒙 − 𝒚 𝒚 + 𝒛 − 𝒙 =
= 𝒙𝟐 − 𝒚 − 𝒛 𝟐
𝒚 + 𝒛 − 𝒙 =
= 𝒙𝟐 − 𝒚𝟐 − 𝒛𝟐 + 𝟐𝒚𝒛 𝒚 + 𝒛 − 𝒙
→ 𝒙𝟐 − 𝒚𝟐 − 𝒛𝟐 + 𝟐𝒚𝒛 𝒚 + 𝒛 − 𝒙 = 𝒙𝟐 𝒚 + 𝒙𝟐 𝒛 − 𝒙𝟑 − 𝒚𝟑 − 𝒚𝟐 𝒛 + 𝒚𝟐 𝒙 −
−𝒛𝟐 𝒚 − 𝒛𝟑 + 𝒛𝟐 𝒙 + 𝟐𝒚𝟐 𝒛 + 𝟐𝒚𝒛𝟐 − 𝟐𝒙𝒚𝒛
⇒ 𝒙 + 𝒚 − 𝒛 𝒛 + 𝒙 − 𝒚 𝒚 + 𝒛 − 𝒙 = −𝒙𝟑 − 𝒚𝟑 − 𝒛𝟑 +
+𝒙𝒚 𝒙 + 𝒚 + 𝒚𝒛 𝒚 + 𝒛 + 𝒛𝒙 𝒛 + 𝒙 − 𝟐𝒙𝒚𝒛 ≤ 𝒙𝒚𝒛
⇒ 𝒙𝟑 + 𝒚𝟑 + 𝒛𝟑 + 𝟑𝒙𝒚𝒛 ≥ 𝒙𝒚 𝒙 + 𝒚 + 𝒚𝒛 𝒚 + 𝒛 + 𝒛𝒙 𝒛 + 𝒙 →
→(Válido por desigualdad de Schur)
7. If 𝒂, 𝒃, 𝒄 ≥ 𝟏, then prove that:
𝟏 𝟏 𝟏 𝟏 𝟏 𝟏
+ + ≥ + +
𝟏 + 𝒂 𝟏 + 𝒃 𝟏 + 𝒄 𝟏 + 𝟒 𝒂𝒃𝟑 𝟏 + 𝟒 𝒃𝒄𝟑 𝟏 + 𝟒 𝒄𝒂𝟑
Proposed by Erdene Natsagdorj - Ulanbaatar Mongolia
Solution by Hung Nguyen Viet – Hanoi – Vietnam
We have known that if 𝒙 ≥ 𝟎, 𝒚 ≥ 𝟎 and 𝒙𝒚 ≥ 𝟏 then
𝟏 𝟏 𝟐
+ ≥
𝟏 + 𝒙 𝟏 + 𝒚 𝟏 + 𝒙𝒚

10
SOCIETATEA DE ȘTIINȚE MATEMATICE DIN ROMÂNIA
ROMANIAN MATHEMATICAL SOCIETY

Filiala Mehedinți - Mehedinți Branch


www.ssmrmh.ro

Applying this result step by step we obtain


𝟏 𝟑 𝟏 𝟏 𝟐 𝟐 𝟐
+ = + + ≥ +
𝟏 + 𝒂 𝟏 + 𝒃 𝟏 + 𝒂 𝟏 + 𝒃 𝟏 + 𝒃 𝟏 + 𝒂𝒃 𝟏 + 𝒃
𝟒 𝟒
≥ = 𝟒
𝟏+ 𝒂𝒃 ⋅ 𝒃 𝟏 + 𝒂𝒃𝟑
Similarly
𝟏 𝟑 𝟒
+ ≥ ,
𝟏 + 𝒃 𝟏 + 𝒄 𝟏 + 𝟒 𝒃𝒄𝟑
𝟏 𝟑 𝟒
+ ≥
𝟏 + 𝒄 𝟏 + 𝒂 𝟏 + 𝟒 𝒄𝒂𝟑
Adding up these three relations we get the desired inequality.
8. Prove the inequality holds for all positive real numbers 𝒂, 𝒃, 𝒄

𝒃𝟒 .𝒄𝟒 𝒄𝟒 .𝒂𝟒 𝒂𝟒 .𝒃𝟒


𝒃 + 𝒄 𝒂𝟐 + 𝒄 + 𝒂 𝒃𝟐 + 𝒂 + 𝒃 𝒄𝟐 ≥ 𝟑𝒂𝒃𝒄 + 𝒂 +𝒃 +𝒄
𝟐 𝟐 𝟐

Proposed by Nguyen Viet Hung – Hanoi – Vietnam


Solution by Kevin Soto Palacios – Huarmey – Peru
Siendo: 𝒂, 𝒃, 𝒄 números 𝑹. . Probar la siguiente desigualdad:

𝒃𝟒 .𝒄𝟒 𝒄𝟒 .𝒂𝟒 𝒂𝟒 .𝒃𝟒


𝒂𝟐 𝒃 + 𝒄 + 𝒃𝟐 𝒄 + 𝒂 + 𝒄𝟐 𝒂 + 𝒃 ≥ 𝟑𝒂𝒃𝒄 + 𝒂 +𝒃 +𝒄
𝟐 𝟐 𝟐

Partimos de la siguiente desigualdad:


𝟒
𝒂−𝒃 ≥ 𝟎 → 𝒂𝟒 + 𝒃𝟒 + 𝟔𝒂𝟐 𝒃𝟐 − 𝟒𝒃𝟑 𝒂 − 𝟒𝒂𝟑 𝒃 ≥ 𝟎
⇒ 𝟐 𝒂𝟒 + 𝒃𝟒 + 𝒂𝟐 𝒃𝟐 − 𝟐𝒂𝟑 𝒃 − 𝟐𝒃𝟑 𝒂 + 𝟐𝒂𝟐 𝒃𝟐 ≥ 𝒂𝟒 + 𝒃𝟒
𝒂𝟒 .𝒃𝟒
⇒ 𝟐 𝒂𝟐 − 𝒂𝒃 + 𝒃𝟐 𝟐
≥ 𝒂𝟒 + 𝒃𝟒 → 𝒂𝟐 − 𝒂𝒃 + 𝒃𝟐 ≥ … (A)
𝟐

Análogamente para los siguientes términos:

11
SOCIETATEA DE ȘTIINȚE MATEMATICE DIN ROMÂNIA
ROMANIAN MATHEMATICAL SOCIETY

Filiala Mehedinți - Mehedinți Branch


www.ssmrmh.ro

𝒃𝟒 .𝒄𝟒 𝒄𝟒 .𝒂𝟒
𝒃𝟐 − 𝒃𝒄 + 𝒄𝟐 ≥ … (B) ∧ 𝒄𝟐 − 𝒄𝒂 + 𝒂𝟐 ≥ … (C)
𝟐 𝟐

En consecuencia, la desigualdad inicial es equivalente:

𝒃 𝟒 + 𝒄𝟒 𝒄𝟒 + 𝒂𝟒 𝒂𝟒 + 𝒃𝟒
𝟑𝒂𝒃𝒄 + 𝒂 +𝒃 +𝒄 ≤
𝟐 𝟐 𝟐

≤ 𝟑𝒂𝒃𝒄 + 𝒂 𝒃𝟐 − 𝒃𝒄 + 𝒄𝟐 + 𝒃 𝒄𝟐 − 𝒄𝒂 + 𝒂𝟐 + 𝒄 𝒂𝟐 − 𝒂𝒃 + 𝒃𝟐

𝒃 𝟒 + 𝒄𝟒 𝒄𝟒 + 𝒂𝟒 𝒂𝟒 + 𝒃𝟒
⇒ 𝟑𝒂𝒃𝒄 + 𝒂 +𝒃 +𝒄 ≤
𝟐 𝟐 𝟐

≤ 𝒂𝟐 𝒃 + 𝒄 + 𝒃𝟐 𝒄 + 𝒂 + 𝒄𝟐 𝒂 + 𝒃 … (LQQD)

9. If 𝒂, 𝒃, 𝒄, 𝒙, 𝒚, 𝒛 ≥ 𝟎, 𝒂 + 𝒃 + 𝒄 = 𝟏 then:
𝟏 𝟏 𝟏 𝟗
+ + ≥
𝟏 + 𝒙𝒂 𝒚 𝒃 𝒛 𝒄 𝟏 + 𝒙 𝒃 𝒚 𝒄 𝒛 𝒂 𝟏 + 𝒙 𝒄 𝒚 𝒂 𝒛 𝒃 𝟑 + 𝒙 + 𝒚 + 𝒛
Proposed by Daniel Sitaru – Romania
Solution by Soumitra Mandal-Chandar Nagore-India
𝟏 𝟏+𝟏+𝟏 𝟐
≥ ≥
𝟏 + 𝒙𝒂 𝒚𝒃 𝒛𝒄 𝟑 + 𝒄𝒚𝒄 𝒙𝒂 𝒚𝒃 𝒛𝒄
𝒄𝒚𝒄

(Bergstrom’s inequality)
𝟐
𝟏+𝟏+𝟏
≥ 𝒂.𝒃.𝒄
=
𝒂𝒙 + 𝒃𝒚 + 𝒄𝒛
𝟑+ 𝒄𝒚𝒄 𝒂+𝒃+𝒄
𝟗 𝟗
= = =
𝟑+ 𝒄𝒚𝒄 𝒂𝒙 + 𝒃𝒚 + 𝒄𝒛 𝟑+ 𝒙+𝒚+𝒛 𝒂+𝒃+𝒄

12
SOCIETATEA DE ȘTIINȚE MATEMATICE DIN ROMÂNIA
ROMANIAN MATHEMATICAL SOCIETY

Filiala Mehedinți - Mehedinți Branch


www.ssmrmh.ro

𝟗
=
𝟑+𝒙+𝒚+𝒛
(Proved)
10. Let 𝒂, 𝒃, 𝒄 be positive real numbers such that 𝒂𝒃 + 𝒃𝒄 + 𝒄𝒂 = 𝟑. Prove that
𝒂𝟑 𝒃𝟑 𝒄𝟑
+ + ≥ 𝟑
𝒃𝟐 − 𝒃𝒄 + 𝒄𝟐 𝒄𝟐 − 𝒄𝒂 + 𝒂𝟐 𝒂𝟐 − 𝒂𝒃 + 𝒃𝟐
Proposed by Nguyen Viet Hung – Hanoi – Vietnam

Solution by Kevin Soto Palacios – Huarmey – Peru
Sean 𝒂, 𝒃, 𝒄 números ℝ. tal que: 𝒂𝒃 + 𝒃𝒄 + 𝒂𝒄 = 𝟑. Probar que:
𝒂𝟑 𝒃𝟑 𝒄𝟑
+ + ≥ 𝟑
𝒃𝟐 − 𝒃𝒄 + 𝒄𝟐 𝒄𝟐 − 𝒂𝒄 + 𝒂𝟐 𝒂𝟐 − 𝒃𝒂 + 𝒃𝟐
La desigualdad es equivalente:
𝒂𝟑 𝒃 + 𝒄 𝒃𝟑 𝒂 + 𝒄 𝒄𝟑 𝒂 + 𝒃
+ 𝟑 + 𝟑 ≥ 𝟑
𝒃 𝟑 + 𝒄𝟑 𝒂 + 𝒄𝟑 𝒂 + 𝒃𝟑
Para todos los ℝ. : “𝒂, 𝒃, 𝒄, 𝒙, 𝒚, 𝒛”, se cumple la siguiente desigualdad:

𝒃+𝒄 𝒙+ 𝒄+𝒂 𝒚+ 𝒂+𝒃 𝒛≥𝟐 𝒂𝒃 + 𝒃𝒄 + 𝒂𝒄 𝒙𝒚 + 𝒚𝒛 + 𝒛𝒙


(Demostrado anteriormente)
𝒂𝟑 𝒃𝟑 𝒄𝟑
Sean: 𝒙 = ,𝒚 = ,𝒛 =
𝒃𝟑 .𝒄 𝟑 𝒂𝟑 .𝒄 𝟑 𝒂𝟑 .𝒃𝟑

Asimismo:
𝒂𝟑 𝒃𝟑 𝒃𝟑 𝒄𝟑 𝒄𝟑 𝒂𝟑
𝒙𝒚 + 𝒚𝒛 + 𝒛𝒙 = 𝟑 ⋅ + ⋅ + ⋅
𝒃 + 𝒄𝟑 𝒂 𝟑 + 𝒄𝟑 𝒂 𝟑 + 𝒄𝟑 𝒂 𝟑 + 𝒃𝟑 𝒂 𝟑 + 𝒃𝟑 𝒃 𝟑 + 𝒄𝟑
Sean: 𝒎 = 𝒂𝟑 , 𝒏 = 𝒃𝟑 , 𝒑 = 𝒄𝟑
𝒎𝒏 𝒏𝒑 𝒑𝒎 𝟑
→ 𝒙𝒚 + 𝒚𝒛 + 𝒛𝒙 = + + ≥
𝒑+𝒏 𝒑+𝒎 𝒎+𝒑 𝒎+𝒏 𝒏+𝒎 𝒏+𝒑 𝟒

13
SOCIETATEA DE ȘTIINȚE MATEMATICE DIN ROMÂNIA
ROMANIAN MATHEMATICAL SOCIETY

Filiala Mehedinți - Mehedinți Branch


www.ssmrmh.ro

(Demostrado anteriormente)
Por la tanto:
𝒂𝟑 𝒃𝟑 𝒄𝟑
+ + ≥𝟐 𝟑 𝒙𝒚 + 𝒚𝒛 + 𝒛𝒙 ≥
𝒃𝟐 − 𝒃𝒄 + 𝒄𝟐 𝒄𝟐 − 𝒂𝒄 + 𝒂𝟐 𝒂𝟐 − 𝒃𝒂 + 𝒃𝟐

𝟗
≥𝟐 = 𝟑
𝟒


11. Let 𝒂, 𝒃, 𝒄 be positive real numbers such that 𝒂 + 𝒃 + 𝒄 = 𝟏. Prove that:
𝟏 𝟏 𝟏 𝟏 𝟏 𝟏
𝟏 + 𝒂𝒃 + 𝒃𝒄 + 𝒄𝒂 + + ≤ + +
𝒂 + 𝒃𝒄 𝒃 + 𝒄𝒂 𝒄 + 𝒂𝒃 𝒂 𝒃 𝒄
Proposed by Nguyen Viet Hung – Hanoi - Vietnam
Solution by Kevin Soto Palacios – Huarmey – Peru
Siendo 𝒂, 𝒃, 𝒄 números ℝ. , de tal manera que: 𝒂 + 𝒃 + 𝒄 = 𝟏. Probar que:
𝟏 𝟏 𝟏 𝟏 𝟏 𝟏
𝟏 + 𝒂𝒃 + 𝒃𝒄 + 𝒂𝒄 + + ≤ + +
𝒂 + 𝒃𝒄 𝒃 + 𝒄𝒂 𝒄 + 𝒂𝒃 𝒂 𝒃 𝒄
𝒂 + 𝒃𝒄 = 𝟏 − 𝒃 − 𝒄 + 𝒃𝒄 = 𝟏 − 𝒃 𝟏 − 𝒄 = 𝒂 + 𝒄 𝒂 + 𝒃
𝒃 + 𝒂𝒄 = 𝟏 − 𝒂 − 𝒄 + 𝒂𝒄 = 𝟏 − 𝒄 𝟏 − 𝒂 = 𝒂 + 𝒃 𝒄 + 𝒃
𝒄 + 𝒂𝒃 = 𝟏 − 𝒂 − 𝒃 + 𝒂𝒃 = 𝟏 − 𝒂 𝟏 − 𝒃 = 𝒃 + 𝒄 𝒂 + 𝒄
La desigualdad se puede expresar de la siguiente manera:
𝟏 𝟏 𝟏 𝟏 𝟏 𝟏
𝒂 + 𝒃𝒄 + 𝒃 + 𝒄𝒂 + 𝒄 + 𝒂𝒃 + + ≥ + +
𝒂 + 𝒃𝒄 𝒃 + 𝒄𝒂 𝒄 + 𝒂𝒃 𝒂 𝒃 𝒄
𝒂.𝒃𝒄 𝒂.𝒃𝒄 𝒃.𝒄𝒂 𝒃.𝒄𝒂 𝒄.𝒂𝒃 𝒄.𝒂𝒃
⇒ 𝟏+ + + +𝟏+ + + + 𝟏 ≥
𝒃.𝒄𝒂 𝒄.𝒂𝒃 𝒂.𝒃𝒄 𝒄.𝒂𝒃 𝒂.𝒃𝒄 𝒃.𝒄𝒂

𝒃+𝒄 𝒂+𝒄 𝒂+𝒃


≥ 𝟏+ + 𝟏+ + 𝟏+
𝒂 𝒃 𝒄

14
SOCIETATEA DE ȘTIINȚE MATEMATICE DIN ROMÂNIA
ROMANIAN MATHEMATICAL SOCIETY

Filiala Mehedinți - Mehedinți Branch


www.ssmrmh.ro

𝒂+𝒄 𝒂+𝒃 𝒃+𝒄 𝒂+𝒃 𝒃+𝒄 𝒂+𝒄


⇒ + + + + + ≤
𝒄+𝒃 𝒄+𝒃 𝒂+𝒄 𝒂+𝒄 𝒂+𝒃 𝒂+𝒃
𝒃+𝒄 𝒂+𝒄 𝒂+𝒃
≤ + +
𝒂 𝒃 𝒄
𝟏 𝟏 𝟏 𝟏 𝟏 𝟏
⇒ 𝒃+𝒄 − − 𝒂+𝒃 + 𝒂+𝒄 − − 𝒃+𝒄 +
𝒂 𝒂.𝒄 𝒂.𝒄 𝒃 𝒂.𝒃 𝒂.𝒃

𝟏 𝟏 𝟏
+ 𝒂+𝒃
− − 𝒂+𝒄 ≥ 𝟎
𝒄 𝒄+𝒃 𝒄+𝒃
𝒃+𝒄 𝒄 𝟏 𝒂+𝒄 𝒂 𝟏
⇒ − 𝒂+𝒃 + − 𝒃+𝒄 +
𝒂 𝒂+𝒄 𝒂+𝒄 𝒃 𝒂+𝒃 𝒂+𝒃
𝒂+𝒃 𝒃 𝟏
+ − 𝒂+𝒄 ≥ 𝟎
𝒄 𝒄+𝒃 𝒄+𝒃
𝟏 𝒃+𝒄 𝒄 𝟏 𝒂+𝒄 𝒂
⇒ − 𝒂+𝒃 + − 𝒃+𝒄 +
𝒂+𝒄 𝒂 𝒂+𝒃 𝒃

𝟏 𝒂+𝒃 𝒃
+ − 𝒂+𝒄 ≥ 𝟎
𝒃+𝒄 𝒄

𝟏 𝒃𝒄 + 𝒄𝟐 − 𝒂𝟐 − 𝒂𝒃 𝟏 𝒂𝒄 + 𝒂𝟐 − 𝒃𝟐 − 𝒃𝒄
⇒ + +
𝒂+𝒄 𝒂 𝒂+𝒃 𝒃
𝟏 𝒂𝒃 + 𝒃𝟐 − 𝒄𝟐 − 𝒂𝒄
+ ≥ 𝟎
𝒃+𝒄 𝒄
𝒄+𝒂 𝒄−𝒂 +𝒃 𝒄−𝒂 𝒂+𝒃 𝒂−𝒃 +𝒄 𝒂−𝒃
⇒ + +
𝒂 𝒂+𝒄 𝒃 𝒂+𝒃
𝒃+𝒄 𝒃−𝒄 +𝒂 𝒃−𝒄
+ ≥ 𝟎
𝒄 𝒃+𝒄
𝒄−𝒂 𝒂+𝒃+𝒄 𝒂−𝒃 𝒂+𝒃+𝒄 𝒃−𝒄 𝒂+𝒃+𝒄
⇒ + + ≥ 𝟎
𝒂 𝒂+𝒄 𝒃 𝒂+𝒃 𝒄 𝒃+𝒄
𝒄−𝒂 𝟏 𝒂−𝒃 𝟏 𝒃−𝒄 𝟏
⇒ + + ≥ 𝟎
𝒂+𝒄 𝒂 𝒂+𝒃 𝒃 𝒃+𝒄 𝒄

15
SOCIETATEA DE ȘTIINȚE MATEMATICE DIN ROMÂNIA
ROMANIAN MATHEMATICAL SOCIETY

Filiala Mehedinți - Mehedinți Branch


www.ssmrmh.ro

𝟐𝒂 𝟏 𝟐𝒃 𝟏 𝟐𝒄 𝟏
⇒ 𝟏− + 𝟏− + 𝟏− ≥ 𝟎
𝒂+𝒄 𝒂 𝒂+𝒃 𝒃 𝒃+𝒄 𝒄
𝟏 𝟏 𝟏 𝟐 𝟐 𝟐
⇒ + + ≥ + + (Lo cual probaremos)
𝒂 𝒃 𝒄 𝒂.𝒄 𝒂.𝒃 𝒃.𝒄

Desde que 𝒂, 𝒃, 𝒄 > 0: Por: 𝑴𝑨 ≥ 𝑴𝑮


𝟏 𝟏 𝟒
+ ≥ (A)
𝒂 𝒃 𝒂.𝒃
𝟏 𝟏 𝟒
+ ≥ (B)
𝒃 𝒄 𝒃.𝒄
𝟏 𝟏 𝟒
+ ≥ (C)
𝒂 𝒄 𝒂.𝒄
𝟏 𝟏 𝟏 𝟐 𝟐 𝟐
Sumando: (A) + (B) + (C) → + + ≥ + + (LQQD)
𝒂 𝒃 𝒄 𝒂.𝒄 𝒂.𝒃 𝒃.𝒄

12. Let 𝒂, 𝒃, 𝒄 be non-negative real numbers such that 𝒂 + 𝒃 + 𝒄 = 𝟑.


Prove that:

𝒂 𝟑 + 𝟏 + 𝒃𝟑 + 𝟏 + 𝒄𝟑 + 𝟏 ≤ 𝟔 𝒂 𝟐 + 𝒃𝟐 + 𝒄𝟐
Proposed by Nguyen Viet Hung – Hanoi – Vietnam
Solution by Kevin Soto Palacios – Huarmey – Peru
Siendo: 𝒂, 𝒃, 𝒄 números reales no negativos, de tal manera que: 𝒂 + 𝒃 + 𝒄 = 𝟑.
Probar que:

𝒂 𝟑 + 𝟏 + 𝒃𝟑 + 𝟏 + 𝒄𝟑 + 𝟏 ≤ 𝟔 𝒂 𝟐 + 𝒃𝟐 + 𝒄𝟐
La desigualdad se puede expresar como:
𝒂+𝟏 𝒂𝟐 − 𝒂 + 𝟏 + 𝒃 + 𝟏 𝒃𝟐 − 𝒃 + 𝟏 + 𝒄 + 𝟏 𝒄𝟐 − 𝒄 + 𝟏 ≤ 𝟔 𝒂𝟐 + 𝒃𝟐 + 𝒄 𝟐
Por la desigualdad de Cauchy Schwarz:
𝟐
𝒂+𝟏 𝒂 𝟐 − 𝒂 + 𝟏 + 𝒃 + 𝟏 𝒃𝟐 − 𝒃 + 𝟏 + 𝒄 + 𝟏 𝒄𝟐 − 𝒄 + 𝟏 ≤

≤ 𝒂 + 𝒃 + 𝒄 + 𝟑 𝒂 𝟐 + 𝒃𝟐 + 𝒄𝟐 − 𝒂 + 𝒃 + 𝒄 + 𝟑

16
SOCIETATEA DE ȘTIINȚE MATEMATICE DIN ROMÂNIA
ROMANIAN MATHEMATICAL SOCIETY

Filiala Mehedinți - Mehedinți Branch


www.ssmrmh.ro

𝟐
𝒂+𝟏 𝒂 𝟐 − 𝒂 + 𝟏 + 𝒃 + 𝟏 𝒃𝟐 − 𝒃 + 𝟏 + 𝒄 + 𝟏 𝒄𝟐 − 𝒄 + 𝟏 ≤

≤ 𝟔 𝒂 𝟐 + 𝒃𝟐 + 𝒄𝟐
⇒ 𝒂 𝟑 + 𝟏 + 𝒃𝟑 + 𝟏 + 𝒄𝟑 + 𝟏 ≤ 𝟔 𝒂𝟐 + 𝒃𝟐 + 𝒄𝟐 ... (LQQD)

13. Prove that for all positive real numbers 𝒂, 𝒃, 𝒄 :
𝒂+𝟏 𝟐 𝒃+𝟏 𝟐
𝒃+𝟏 𝟐 𝒄+𝟏 𝟐
𝒄+𝟏 𝟐 𝒂+𝟏 𝟐
+ + ≥𝟖 𝒂+𝒃+𝒄
𝒂𝒃 + 𝟏 𝒃𝒄 + 𝟏 𝒄𝒂 + 𝟏
Proposed by Nguyen Viet Hung – Hanoi – Vietnam
Solution by Kevin Soto Palacios – Huarmey – Peru
Probar para todos los números ℝ. 𝒂, 𝒃, 𝒄 la siguiente desigualdad:
𝟏+𝒂 𝟐 𝟏+𝒃 𝟐
𝟏+𝒃 𝟐 𝟏+𝒄 𝟐
𝟏 + 𝒄𝟐 𝟏 + 𝒂 𝟐
+ + ≥𝟖 𝒂+𝒃+𝒄
𝒂𝒃 + 𝟏 𝒃𝒄 + 𝟏 𝒄𝒂 + 𝟏
Realizamos los siguientes cambios de variables:
𝒑 = 𝒂 + 𝒃 + 𝒄 ∧ 𝒒 = 𝒂𝒃 + 𝒃𝒄 + 𝒂𝒄
Desde que: 𝒂, 𝒃, 𝒄 > 0
Por la desigualdad de Bergstrom’s:
𝟏+𝒂 𝟐 𝟏+𝒃 𝟐
𝟏+𝒃 𝟐 𝟏+𝒄 𝟐
𝟏+𝒄 𝟐 𝟏+𝒂 𝟐
+ + ≥
𝒂𝒃 + 𝟏 𝒃𝒄 + 𝟏 𝒄𝒂 + 𝟏
𝟐
𝟏+𝒂 𝟏+𝒃
≥ ≥ 𝟖𝒑
𝟑+𝒒
Por lo cual nos queda demostrar que:
𝟐
𝟐
⇒ 𝟏+𝒂 𝟏+𝒃 ≥ 𝟖𝒑 𝟑 + 𝒒 ⇔ 𝟑 + 𝟐𝒑 + 𝒒 ≥ 𝟐𝟒𝒑 + 𝟖𝒑𝒒

⇒ 𝟗 + 𝟒𝒑𝟐 + 𝒒𝟐 + 𝟏𝟐𝒑 + 𝟒𝒑𝒒 + 𝟔𝒒 ≥ 𝟐𝟒𝒑 + 𝟖𝒑𝒒 ⇔

17
SOCIETATEA DE ȘTIINȚE MATEMATICE DIN ROMÂNIA
ROMANIAN MATHEMATICAL SOCIETY

Filiala Mehedinți - Mehedinți Branch


www.ssmrmh.ro

⇔ 𝟒𝒑𝟐 + 𝒒𝟐 + 𝟗 − 𝟒𝒑𝒒 − 𝟏𝟐𝒑 + 𝟔𝒒 ≥ 𝟎


Pero ... 𝟒𝒑𝟐 + 𝒒𝟐 + 𝟗 − 𝟒𝒑𝒒 − 𝟏𝟐𝒑 + 𝟔𝒒 = 𝟐𝒑 − 𝒒 − 𝟑 𝟐
≥ 𝟎 ... (LQQD)
14. Prove that for all real numbers 𝒂, 𝒃, 𝒄:
𝒂+𝒃+𝒄 𝒂 𝒃 𝒄
≤ + +
𝟏+ 𝒂+𝒃+𝒄 𝟏+ 𝒃 + 𝒄 𝟏+ 𝒄 + 𝒂 𝟏+ 𝒂 + 𝒃
Proposed by Nguyen Viet Hung – Hanoi – Vietnam
Solution 1 by Abhay Chandra – India
Solution 2 by Marian Dincă – Romania

Solution 1 by Abhay Chandra – India
For real numbers we have 𝐚 + 𝐛 + 𝐜 ≥ 𝐚 + 𝐛 + 𝐜 which implies
𝐚 + 𝐛 + 𝐜 𝐚+𝐛+𝐜

𝐚 + 𝐛 + 𝐜 +𝟏 𝐚+𝐛+𝐜 +𝟏
Hence we are required to prove the following for all non-negative real numbers
𝐱 = 𝐚 ,𝐲 = 𝐛 ,𝐳 = 𝐜
𝐱 𝐱+𝐲+𝐳

𝟏+𝐲+𝐳 𝐱+𝐲+𝐳+𝟏
But from Cauchy – Schwarz on LHS we have
𝐱+𝐲+𝐳 𝟐
𝐋𝐇𝐒 ≥
𝐱 + 𝐲 + 𝐳 + 𝟐 𝐱𝐲 + 𝐲𝐳 + 𝐳𝐱
we are left to prove
𝟐
𝐱+𝐲+𝐳 ≥ 𝟐 𝐱𝐲 + 𝐲𝐳 + 𝐳𝐱 ⇒ 𝐱 𝟐 + 𝐲 𝟐 + 𝐳 𝟐 ≥ 𝟎
which is obviously true. Equality at 𝐱 = 𝐲 = 𝐳 = 𝟎 or 𝐚 = 𝐛 = 𝐜 = 𝟎.
Solution 2 by Marian Dincă – Romania

18
SOCIETATEA DE ȘTIINȚE MATEMATICE DIN ROMÂNIA
ROMANIAN MATHEMATICAL SOCIETY

Filiala Mehedinți - Mehedinți Branch


www.ssmrmh.ro

𝐚 𝐛 𝐜
+ + =
𝟏+ 𝐛 + 𝐜 𝟏+ 𝐜 + 𝐚 𝟏+ 𝐚 + 𝐛
𝐚 𝐛 𝐜
= +𝟏 + + + 𝟏 − 𝟑 =
𝟏+ 𝐛 + 𝐜 𝟏+ 𝐜 + 𝐚 𝟏+ 𝐚 + 𝐛
𝟏 𝟏 𝟏
= 𝟏+ 𝐚 + 𝐛 + 𝐜 + + − 𝟑 ≥
𝟏+ 𝐛 + 𝐜 𝟏+ 𝐜 + 𝐚 𝟏+ 𝐚 + 𝐛
𝟗
≥ 𝟏+ 𝐚 + 𝐛 + 𝐜 − 𝟑 use harmonic inequality
𝐜𝐢𝐜𝐥𝐢𝐜 𝟏. 𝐚.𝐛

𝟗.𝟗 𝐚 . 𝐛 . 𝐜 𝟑 𝐚.𝐛.𝐜 𝐚.𝐛.𝐜


= −𝟑= = 𝟐 ≥
𝟑.𝟐 𝐚 . 𝐛 . 𝐜 𝟑.𝟐 𝐚 . 𝐛 . 𝐜 𝟏. 𝐚.𝐛.𝐜
𝟑

𝐚 + 𝐛 + 𝐜 𝐚+𝐛+𝐜
≥ ≥
𝟏+ 𝐚 + 𝐛 + 𝐜 𝟏+ 𝐚+𝐛+𝐜
𝐱
because the function 𝐟 𝐱 = , is increasing for 𝐱 ≥ 𝟎 and
𝟏.𝐱

𝐚 + 𝐛 + 𝐜 ≥ 𝐚+𝐛+𝐜
done!
15. If 𝒂, 𝒃, 𝒄, 𝒅 ∈ 𝟎, 𝟏 , prove that:
𝒄.𝒅 𝒅.𝒂 𝒂.𝒃 𝒃.𝒄
𝒂+𝒃 𝟐 𝒃+𝒄 𝟐 𝒄+𝒅 𝟐 𝒅+𝒂 𝟐
+ + + > 2
𝟐 𝟐 𝟐 𝟐
Proposed by Dorin Marghidanu – Romania
Solution by Marian Dincă – Romania
𝐚, 𝐛, 𝐜, 𝐝 ∈ 𝟎, 𝟏
𝐜.𝐝
𝐚+𝐛 𝟐
> 𝟐
𝟐
𝐜𝐲𝐜

use Lema by Dorin Marghidanu


𝐱
𝐱𝐲 ≥ , for 𝐱, 𝐲 ∈ 𝟎, 𝟏
𝐱.𝐲

19
SOCIETATEA DE ȘTIINȚE MATEMATICE DIN ROMÂNIA
ROMANIAN MATHEMATICAL SOCIETY

Filiala Mehedinți - Mehedinți Branch


www.ssmrmh.ro

𝐜.𝐝 𝐚+𝐛
𝐚+𝐛 𝐜+𝐝 𝐚+𝐛 𝟐
𝟐 𝐚+𝐛
∈ 𝟎, 𝟏 , ∈ 𝟎, 𝟏 ⇒ ≥ =
𝟐 𝟐 𝟐 𝐚+𝐛 𝐜+𝐝 𝐚+𝐛+𝐜+𝐝
+
𝟐 𝟐
and similarly, we shall obtain:
𝐜.𝐝
𝐚+𝐛 𝟐 𝐚+𝐛
≥ = 𝟐
𝟐 𝐚+𝐛+𝐜+𝐝
𝐜𝐲𝐜 𝐜𝐲𝐜





16. If 𝒂, 𝒃, 𝒄 ∈ 𝟎, ∞ then:

𝟑 𝟐 𝟑 𝟐 𝟑
𝟏+ + 𝟐 ≥𝟏+𝟔 +𝟑
𝒂 𝒂 𝒂𝒃𝒄 𝒂𝒃𝒄 𝟐
Proposed by Daniel Sitaru – Romania
Solution by Soumitra Mandal - Chandar Nagore – India
Applying Holder’s Inequality
𝟑
𝟐 𝟑 𝟐 𝟑 𝟐 𝟑 𝟑 𝟐 𝟐 𝟐 𝟑 𝟑 𝟑 𝟑
𝟏+ + 𝟏+ + 𝟏+ + ≥ 𝟏+ ⋅ ⋅ + ⋅ ⋅
𝐚 𝐚𝟐 𝐛 𝐛𝟐 𝐜 𝐜𝟐 𝐚 𝐛 𝐜 𝐚𝟐 𝐛 𝟐 𝐜 𝟐

𝟑 𝟐 𝟑 𝟐 𝟑
⇔ 𝟏+ + 𝟐 ≥𝟏+𝟑 +𝟑
𝐚 𝐚 𝐚𝐛𝐜 𝐚𝐛𝐜 𝟐
𝐜𝐲𝐜

(proved)
𝟏 𝟏 𝟏
17. 𝐚, 𝐛, 𝐜 > 0 ∧ 𝑎𝑏 + 𝑏𝑐 + 𝑐𝑎 = 𝑎 + 𝑏 + 𝑐 ⇒ + + ≤ 𝟏
𝐚𝟐 .𝐛.𝟏 𝐛 𝟐 .𝐜.𝟏 𝐜 𝟐 .𝐚.𝟏

20
SOCIETATEA DE ȘTIINȚE MATEMATICE DIN ROMÂNIA
ROMANIAN MATHEMATICAL SOCIETY

Filiala Mehedinți - Mehedinți Branch


www.ssmrmh.ro

Proposed by Vaggelis Stamatiadis-Greece


Solution by Nguyen Viet Hung – Hanoi – Vietnam
By Cauchy – Schwarz inequality we have
𝟏 𝟏 + 𝐛 + 𝐜𝟐 𝟏 + 𝐛 + 𝐜𝟐
= = =
𝐚𝟐 + 𝐛 + 𝟏 𝐚𝟐 + 𝐛 + 𝟏 𝟏 + 𝐛 + 𝐜 𝟐 𝐚+𝐛+𝐜 𝟐
𝐜𝐲𝐜 𝐜𝐲𝐜 𝐜𝐲𝐜

𝟑 + 𝐚 + 𝐛 + 𝐜 + 𝐚𝟐 + 𝐛𝟐 + 𝐜 𝟐
=
𝐚+𝐛+𝐜 𝟐
It suffices to show that
𝟑 + 𝐚 + 𝐛 + 𝐜 + 𝐚𝟐 + 𝐛𝟐 + 𝐜 𝟐 ≤ 𝐚 + 𝐛 + 𝐜 𝟐
or
𝟑 ≤ 𝐚 + 𝐛 + 𝐜 (since 𝐚 + 𝐛 + 𝐜 = 𝐚𝐛 + 𝐛𝐜 + 𝐜𝐚)
But this is true by
𝟏
𝐚 + 𝐛 + 𝐜 = 𝐚𝐛 + 𝐛𝐜 + 𝐜𝐚 ≤ 𝐚 + 𝐛 + 𝐜 𝟐
𝟑
The proof is completed.

18. If 𝒂, 𝒃, 𝒄 ∈ 𝟎, ∞ , 𝒂 + 𝒃 + 𝒄 = 𝟏 then:
𝒂𝟑 𝒃𝟑 𝒄𝟑 𝟏
+ + ≥
𝟓𝒃 + 𝟕𝒄 𝟓𝒄 + 𝟕𝒂 𝟓𝒂 + 𝟕𝒃 𝟑𝟔
Proposed by Daniel Sitaru – Romania
Solution by Myagmarsuren Yadamsuren – Mongolia
𝒂𝟑 𝒃𝟑 𝒄𝟑 𝟏
𝑰= + + ≥
𝟓𝒃 + 𝟕𝒄 𝟓𝒄 + 𝟕𝒂 𝟓𝒂 + 𝟕𝒃 𝟑𝟔
𝑪𝒉𝒆𝒃𝒚𝒔𝒉𝒆𝒗 𝟑
𝟏 𝟑
𝟏 𝟏 𝟏
𝑰 ≥ ⋅ 𝒂+𝒃+𝒄 ⋅ + +
𝟐𝟕 𝟓𝒃 + 𝟕𝒄 𝟓𝒄 + 𝟕𝒂 𝟓𝒂 + 𝟕𝒃

21
SOCIETATEA DE ȘTIINȚE MATEMATICE DIN ROMÂNIA
ROMANIAN MATHEMATICAL SOCIETY

Filiala Mehedinți - Mehedinți Branch


www.ssmrmh.ro

𝑪𝒂𝒖𝒄𝒉𝒚7𝑺𝒄𝒉𝒘𝒂𝒓𝒛
𝟏 𝟏 𝟏 𝟏
= ⋅ + + ≥
𝟐𝟕 𝟓𝒂 + 𝟕𝒃 𝟓𝒃 + 𝟕𝒄 𝟓𝒄 + 𝟕𝒂
𝟏 𝟏+𝟏+𝟏 𝟐 𝟏
≥ ⋅ =
𝟐𝟕 𝟏𝟐 ⋅ 𝒂 + 𝒃 + 𝒄 𝟑𝟔
19. If 𝒂, 𝒃, 𝒄 ∈ 𝟎, ∞ then:

𝒂𝟐 𝒃𝟐 𝒂𝟐 𝒄𝟐 𝒃𝟐 𝒄𝟐
+ + + + + ≥𝟐 𝒂+ 𝒃+ 𝒄
𝒃 𝒂 𝒄 𝒂 𝒄 𝒃

Proposed by Daniel Sitaru - Romania


Solution 1 by Ravi Prakash - New Delhi – India
Solution 2 by Seyran Ibrahimov – Maasilli – Azerbaidian
Solution 1 by Ravi Prakash - New Delhi – India

𝐚𝟐 𝐛𝟐 𝐛𝟐 𝐜𝟐 𝐜𝟐 𝐚𝟐
+ + + + + ≥𝟐 𝐚+ 𝐛+ 𝐜
𝐛 𝐚 𝐜 𝐛 𝐚 𝐜

𝐚𝟐 𝐛𝟐 𝟏
+ − 𝐚+ 𝐛 = 𝐚 𝐚 + 𝐛 𝐛 − 𝐛 𝐚 − 𝐚 𝐛 =
𝐛 𝐚 𝐚𝐛
𝟏
= 𝐚−𝐛 𝐚− 𝐛 ≥ 𝟎
𝐚𝐛
𝐚𝟐 𝐛𝟐
Thus, + ≥ 𝐚 + 𝐛 (1)
𝐛 𝐚

Similarly

𝐛𝟐 𝐜𝟐
+ ≥ 𝐛 + 𝐜 (2)
𝐜 𝐛

and

22
SOCIETATEA DE ȘTIINȚE MATEMATICE DIN ROMÂNIA
ROMANIAN MATHEMATICAL SOCIETY

Filiala Mehedinți - Mehedinți Branch


www.ssmrmh.ro

𝐜𝟐 𝐚𝟐
+ ≥ 𝐚 + 𝐜 (3)
𝐚 𝐜

Adding (1), (2) and (3) we get the desired inequality.


Solution 2 by Seyran Ibrahimov – Maasilli – Azerbaidian
𝒂 𝒃 𝒂 𝒄 𝒃 𝒄 𝒃+𝒄 𝒂+𝒄 𝒂+𝒃
+ + + + + = + +
𝒃 𝒂 𝒄 𝒂 𝒄 𝒃 𝒂 𝒃 𝒄
𝒃 + 𝒄 ≥ 𝟐 𝒃𝒄
𝒂 + 𝒄 ≥ 𝟐 𝒂𝒄 𝑨𝑴 − 𝑮𝑴
𝒂 + 𝒃 ≥ 𝟐 𝒂𝒃

𝒃𝒄 𝒂𝒄
𝟐 +𝟐 ≥ 𝟒 𝒄
𝒂 𝒃

𝒂𝒄 𝒂𝒃
𝟐 +𝟐 ≥ 𝟒 𝒂
𝒃 𝒄

𝒃𝒄 𝒂𝒃
𝟐 +𝟐 ≥ 𝟒 𝒃
𝒂 𝒄

𝒃𝒄 𝒂𝒄 𝒂𝒃
𝑳𝑯𝑺 ≥ 𝟐 +𝟐 +𝟐 ≥𝟐 𝒂+ 𝒃+ 𝒄
𝒂 𝒃 𝒄

20. If 𝒂, 𝒃, 𝒄 ∈ 𝟎, ∞ and 𝒎, 𝒏 ∈ ℕ∗ , prove that:


𝒂𝒎.𝒏 𝒃𝒎.𝒏 𝒄𝒎.𝒏
+ + ≥ 𝒂 + 𝒃 + 𝒄
𝒃𝒎 𝒄𝒏7𝟏 𝒄𝒎 𝒂𝒏7𝟏 𝒂𝒎 𝒃𝒏7𝟏
Proposed by Dorin Mărghidanu – Romania
Solution by Soumitra Mandal - Chandar Nagore – India
Applying A.M. ≥ G.M.

23
SOCIETATEA DE ȘTIINȚE MATEMATICE DIN ROMÂNIA
ROMANIAN MATHEMATICAL SOCIETY

Filiala Mehedinți - Mehedinți Branch


www.ssmrmh.ro

𝒂𝒎u𝒏
.𝒎𝒃. 𝒏7𝟏 𝒄
𝒃𝒎 𝒄𝒏F𝟏
≥ 𝒂, similarly
𝒎.𝒏
𝒃𝒎u𝒏 𝒄𝒎u𝒏
.𝒎𝒄. 𝒏7𝟏 𝒂 .𝒎𝒂. 𝒏7𝟏 𝒃
𝒄 𝒂𝒏F𝟏
𝒎 𝒂 𝒃𝒏F𝟏
𝒎
≥ 𝒃 and ≥ 𝒄
𝒎.𝒏 𝒎.𝒏

now,

𝒂𝒎.𝒏
+𝒎 𝒂 + 𝒏−𝟏 𝒂 ≥ 𝒎+𝒏 𝒂
𝒃𝒎 𝒄𝒏7𝟏
𝒄𝒚𝒄𝒍 𝒄𝒚𝒄𝒍 𝒄𝒚𝒄𝒍 𝒄𝒚𝒄𝒍

so,
𝒂𝒎.𝒏
≥ 𝒂 + 𝒃 + 𝒄
𝒃𝒎 𝒄𝒏7𝟏
𝒄𝒚𝒄𝒍

(proved)
21. If 𝒂, 𝒃, 𝒄 ∈ −𝟏, 𝟏 then:
𝒂 + 𝒃 𝒃 + 𝒄 𝒄 + 𝒂 𝟐𝒂 𝟐𝒃 𝟐𝒄
+ + ≥ + +
𝟏 − 𝒄𝟐 𝟏 − 𝒂𝟐 𝟏 − 𝒃𝟐 𝟏 − 𝒃𝒄 𝟏 − 𝒄𝒂 𝟏 − 𝒂𝒃
Proposed by Daniel Sitaru – Romania
Solution 1 by Kevin Soto Palacios – Huarmey – Peru
Solution 2 by Ravi Prakash - New Delhi – India
Solution 1 by Kevin Soto Palacios – Huarmey – Peru
Si: 𝐚, 𝐛, 𝐜 ∈< −𝟏, 𝟏 >
𝐚 + 𝐛 𝐛 + 𝐜 𝐜 + 𝐚 𝟐𝐚 𝟐𝐛 𝟐𝐜
+ + ≥ + +
𝟏 − 𝐜𝟐 𝟏 − 𝐚𝟐 𝟏 − 𝐛𝟐 𝟏 − 𝐛𝐜 𝟏 − 𝐚𝐜 𝟏 − 𝐚𝐛
Por desigualdad de Cauchy:
𝐚 𝐚 𝟏 𝟏 𝟒
⇒ + = 𝐚 + ≥ 𝐚 ≥
𝟏 − 𝐜𝟐 𝟏 − 𝐛𝟐 𝟏 − 𝐜𝟐 𝟏 − 𝐛𝟐 (𝟐 − 𝐛 𝟐 − 𝐜 𝟐

24
SOCIETATEA DE ȘTIINȚE MATEMATICE DIN ROMÂNIA
ROMANIAN MATHEMATICAL SOCIETY

Filiala Mehedinți - Mehedinți Branch


www.ssmrmh.ro

𝟒 𝟐𝐚
≥ 𝐚 = ... (I)
𝟐7𝟐𝐛𝐜 𝟏7𝐛𝐜
𝐛 𝐛 𝟐𝐛 𝐜 𝐜 𝟐𝐜
⇒ + ≥ ... (II) ∧ + ≥ ... (III)
𝟏7𝐜 𝟐 𝟏7𝐚𝟐 𝟏7𝐚𝐜 𝟏7𝐚𝟐 𝟏7𝐛 𝟐 𝟏7𝐚𝐛

Sumando ... (I)+(II)+(III):


𝐚.𝐛 𝐛.𝐜 𝐜.𝐚 𝟐𝐚 𝟐𝐛 𝟐𝐜
+ + ≥ + + ... (LQQD)
𝟏7𝐜 𝟐 𝟏7𝐚𝟐 𝟏7𝐛 𝟐 𝟏7𝐛𝐜 𝟏7𝐚𝐜 𝟏7𝐚𝐛


Solution 2 by Ravi Prakash - New Delhi – India
Rewrite the inequality as:
𝟏 𝟏 𝟏 𝟏 𝟏 𝟏
𝐚 + + 𝐛 + + 𝐜 + ≥
𝟏 − 𝐛𝟐 𝟏 − 𝐜𝟐 𝟏 − 𝐜 𝟐 𝟏 − 𝐚𝟐 𝟏 − 𝐚𝟐 𝟏 − 𝐛 𝟐
𝟐𝐚 𝟐𝐛 𝟐𝐜
≥ + +
𝟏 − 𝐛𝐜 𝟏 − 𝐜𝐚 𝟏 − 𝐚𝐛
For −𝟏 < 𝐛, 𝐜 < 𝟏
𝟏 𝟏 𝟐 𝟏 𝟏 𝟏 𝟏
+ − = − + − =
𝟏 − 𝐛 𝟐 𝟏 − 𝐜 𝟐 𝟏 − 𝐛𝐜 𝟏 − 𝐛 𝟐 𝟏 − 𝐛𝐜 𝟏 − 𝐜 𝟐 𝟏 − 𝐛𝐜
𝐛 𝐛−𝐜 𝐜 𝟐 − 𝐛𝐜 𝐛 − 𝐜 𝐛 𝟏 − 𝐜 𝟐 − 𝐜 𝟏 − 𝐛𝟐
= + = =
𝟏 − 𝐛 𝟐 𝟏 − 𝐛𝐜 𝟏 − 𝐜 𝟐 𝟏 − 𝐛𝐜 𝟏 − 𝐛𝐜 𝟏 − 𝐛𝟐 𝟏 − 𝐜𝟐
𝐛−𝐜 𝟐
= ≥ 𝟎
𝟏 − 𝐛𝟐 𝟏 − 𝐜𝟐
Thus,
𝟏 𝟏 𝟐𝐚
𝐚 + ≥
𝟏 − 𝐛𝟐 𝟏 − 𝐜𝟐 𝟏 − 𝐛𝐜
Similarly for the second and third expressions on both sides.
22. If 𝒂, 𝒃, 𝒄 > 0 then:
𝒂 𝒃 𝒄
𝒂𝟒 𝒄𝟐 𝒆𝒃 + 𝒃𝟒 𝒂𝟐 𝒆 𝒄 + 𝒄𝟒 𝒃𝟐 𝒆𝒂 ≥ 𝟑𝒆𝒂𝟐 𝒃𝟐 𝒄𝟐

25
SOCIETATEA DE ȘTIINȚE MATEMATICE DIN ROMÂNIA
ROMANIAN MATHEMATICAL SOCIETY

Filiala Mehedinți - Mehedinți Branch


www.ssmrmh.ro

Proposed by Daniel Sitaru – Romania


Solution 1 Abdallah El Farisi-Bechar-Algerie
Solution 2 by Myagmarsuren Yadamsuren – Mongolia
Solution 3 by Safal Das Biswas – Chindahar – India
Solution 4 by Soumitra Mandal - Chandar Nagore – India
Solution 1 Abdallah El Farisi-Bechar-Algerie
The function 𝒙𝟐 𝒆𝒙 is convex for 𝒙 ≥ 𝟎 then we have for all 𝒂, 𝒃, 𝒄 > 0.
𝟐
𝟏 𝒂 𝟐 𝒂 𝟏 𝒃 𝒃 𝟏 𝒄 𝟐 𝒄
𝒆𝒃 + 𝒆𝒄 + 𝒆
𝒃
𝟑 𝒃 𝟑 𝒄 𝟑 𝒂
𝟏𝒂 𝟏𝒃 𝟏𝒄 𝟐 𝟏𝒂 𝟏𝒃 𝟏 𝒄
≥ + + 𝒆𝟑𝒃.𝟑 𝒄 .𝟑𝒂 ≥ 𝒆
𝟑𝒃 𝟑𝒄 𝟑𝒂

by MA-MG
Solution 2 by Myagmarsuren Yadamsuren – Mongolia
𝒂 𝒃 𝒄 𝑨𝑴7𝑮𝑴
𝟒 𝟐 𝒃 𝟒 𝟐 𝒄 𝟒 𝟐 𝒂
𝒂 𝒄 𝒆 + 𝒃 𝒄 𝒆 + 𝒄 𝒃 𝒄 ≥
𝟑 𝒂 𝒃 𝒄 𝟑 𝒂 𝒃 𝒄 𝑨𝑴7𝑮𝑴
≥𝟑⋅ 𝒂𝟔 ⋅ 𝒃𝟔 ⋅ 𝒄𝟔 ⋅ 𝒆𝒃 . 𝒄 .𝒂 = 𝟑𝒂 𝒃 𝒄 ⋅ 𝟐 𝟐 𝟐
𝒆𝒃 . 𝒄 .𝒂 ≥
𝟑 𝟑 𝒂𝒃𝒄
𝟐 𝟐 𝟐 𝟑⋅ ⋅ ⋅
≥ 𝟑𝒂 𝒃 𝒄 ⋅ 𝒆 𝒃𝒄𝒂 = 𝟑𝒆 ⋅ 𝒂𝟐 𝒃𝟐 𝒄𝟐
Solution 3 by Safal Das Biswas – Chindahar – India
𝒂 𝒃 𝒄
𝒂𝟒 𝒄𝟐 𝒆𝒃 + 𝒃𝟒 𝒂𝟐 𝒆 𝒄 + 𝒄𝟒 𝒃𝟐 𝒆𝒂 = 𝒌. Then by A.M ≥ G.M we have
𝟑 𝒂 𝒃 𝒄
𝒌≥𝟑 𝒂 𝒃 𝒄 𝒆𝒃 . 𝒄 .𝒂
𝟔 𝟔 𝟔

𝒂 𝒃 𝒄 𝒂 𝒃 𝒄 𝒂 𝒃 𝒄
u u u u
𝒃 𝒄 𝒂 . . 𝒃 𝒄 𝒂
𝟐 𝟐 𝟐
= 𝟑𝒂 𝒃 𝒄 𝒆 𝟑 again A.M ≥ G.M 𝒃 𝒄 𝒂
≥ 𝟏 then 𝒆 𝟑 ≥ 𝒆, thus
𝟑

𝒌 ≥ 𝟑𝒆𝒂𝟐 𝒃𝟐 𝒄𝟐

26
SOCIETATEA DE ȘTIINȚE MATEMATICE DIN ROMÂNIA
ROMANIAN MATHEMATICAL SOCIETY

Filiala Mehedinți - Mehedinți Branch


www.ssmrmh.ro

Solution 4 by Soumitra Moukherjee - Chandar Nagore – India


Applying A.M ≥ 𝐆. 𝐌,

𝒂
𝒂 𝟑 𝒂 𝒄𝒚𝒄𝒃
𝟒 𝟐 𝒃
𝒂 𝒄 𝒆 ≥𝟑 𝒂 𝟒 𝒄𝟐 𝒆 𝒄𝒚𝒄𝒃
= 𝟑𝒂𝟐 𝒃𝟐 𝒄𝟐 𝒆 𝟑 ≥ 𝟑𝒆𝒂𝟐 𝒃𝟐 𝒄𝟐
𝒄𝒚𝒄 𝒄𝒚𝒄

(proved)
23. Let 𝒂, 𝒃, 𝒄 be real numbers such that 𝐦𝐢𝐧 𝒂𝟐 + 𝒃𝟐 , 𝒃𝟐 + 𝒄𝟐 , 𝒄𝟐 + 𝒂𝟐 > 0.
Prove that
𝒂𝟐 − 𝒂𝒃 + 𝒃𝟐 𝒃𝟐 − 𝒃𝒄 + 𝒄𝟐 𝒄𝟐 − 𝒄𝒂 + 𝒂𝟐
+ + ≥ 𝟏 .
𝒃𝟐 + 𝒃𝒄 + 𝒄𝟐 𝒄𝟐 + 𝒄𝒂 + 𝒂𝟐 𝒂𝟐 + 𝒂𝒃 + 𝒃𝟐
Proposed by Nguyen Viet Hung – Hanoi – Vietnam
Solution by Leonard Giugiuc – Romania
𝟐
We have: 𝟐 𝐚 − 𝐛 ≥ 𝟎 ⇒ 𝟑 𝐚𝟐 − 𝐚𝐛 + 𝐛𝟐 ≥ 𝐚𝟐 + 𝐚𝐛 + 𝐛𝟐 > 𝟎.
Similarly,
𝟑 𝐚𝟐 − 𝐚𝐜 + 𝐜 𝟐 ≥ 𝐚𝟐 + 𝐚𝐜 + 𝐜 𝟐 > 𝟎 and 𝟑 𝐛𝟐 − 𝐛𝐜 + 𝐜 𝟐 ≥ 𝐛𝟐 + 𝐛𝐜 + 𝐜 𝟐 > 𝟎.
So that:

𝐚𝟐 − 𝐚𝐛 + 𝐛𝟐 𝐚𝟐 + 𝐚𝐛 + 𝐛𝟐 𝟑 𝐚𝟐 + 𝐚𝐛 + 𝐛 𝟐
𝟑 ≥ ≥𝟑 = 𝟑
𝐛 𝟐 + 𝐛𝐜 + 𝐜 𝟐 𝟐
𝐛 + 𝐛𝐜 + 𝐜 𝟐 𝐛 𝟐 + 𝐛𝐜 + 𝐜 𝟐
𝐜𝐲𝐜 𝐜𝐲𝐜 𝐜𝐲𝐜

24. If 𝒂, 𝒃, 𝒄 ≥ 𝟎 then:
𝒂 𝟐𝒃 + 𝟐7𝒄 + 𝒃 𝟐𝒄 + 𝟐7𝒂 + 𝒄 𝟐𝒂 + 𝟐7𝒃 ≥ 𝟐 𝒂 + 𝒃 + 𝒄
Proposed by Daniel Sitaru – Romania
Solution 1 by Abdallah El Farisi-Bechar – Algerie
Solution 2 by Ravi Prakash - New Dehi – India

27
SOCIETATEA DE ȘTIINȚE MATEMATICE DIN ROMÂNIA
ROMANIAN MATHEMATICAL SOCIETY

Filiala Mehedinți - Mehedinți Branch


www.ssmrmh.ro

Solution 3 by Soumitra Mandal - Chandar Nagore – India



Solution 1 by Abdallah El Farisi-Bechar – Algerie
The functions 𝟐𝒙 and 𝟐7𝒙 are convex for 𝒙 ≥ 𝟎 then for all 𝒂, 𝒃, 𝒄 > 0
we have:
𝒂 𝒃 𝒄
𝟐𝒃 + 𝟐𝒄 + 𝟐𝒂 +
𝒂+𝒃+𝒄 𝒂+𝒃+𝒄 𝒂+𝒃+𝒄
𝒂 𝒃 𝒄
+ 𝟐7𝒄 + 𝟐7𝒂 + 𝟐7𝒃 ≥
𝒂+𝒃+𝒄 𝒂+𝒃+𝒄 𝒂+𝒃+𝒄
𝒂𝒃 𝒃𝒄 𝒄𝒂 𝒂𝒃 𝒃𝒄 𝒄𝒂
. . 7 . .
≥𝟐 𝒂u𝒃u𝒄 𝒂u𝒃u𝒄 𝒂u𝒃u𝒄 +𝟐 𝒂u𝒃u𝒄 𝒂u𝒃u𝒄 𝒂u𝒃u𝒄 ≥ 𝟐 by MA-MG



Solution 2 by Ravi Prakash - New Dehi – India
𝒂𝒃u𝒃𝒄u𝒄𝒂
𝒂 𝟐𝒃 + 𝒃 𝟐𝒄 + 𝒄 𝟐𝒂 ≥ 𝒂 + 𝒃 + 𝒄 𝟐 𝒂u𝒃u𝒄 (1)
𝒂𝒄u𝒂𝒃u𝒃𝒄
7𝒄 7𝒂 7𝒃 7
𝒂 𝟐 +𝒃 𝟐 +𝒄 𝟐 ≥ 𝒂+𝒃+𝒄 𝟐 𝒂u𝒃u𝒄 (2)
Adding (1), (2) we get
𝒂 𝟐𝒃 + 𝟐7𝒄 + 𝒃 𝟐𝒄 + 𝟐7𝒂 + 𝒄 𝟐𝒂 + 𝟐7𝒃 ≥ 𝒂 + 𝒃 + 𝒄 𝟐𝜶 + 𝟐7𝜶 ≥
≥𝟐 𝒂+𝒃+𝒄
𝒂𝒃.𝒃𝒄.𝒄𝒂
where 𝜶 =
𝒂.𝒃.𝒄


Solution 3 by Soumitra Mandal - Chandar Nagore – India
We know, 𝒆𝒙 ≥ 𝒙 + 𝟏 and 𝒆7𝒙 ≥ 𝟏 − 𝒙 for all 𝒙 ≥ 𝟎

28
SOCIETATEA DE ȘTIINȚE MATEMATICE DIN ROMÂNIA
ROMANIAN MATHEMATICAL SOCIETY

Filiala Mehedinți - Mehedinți Branch


www.ssmrmh.ro

𝒂 𝟐𝒃 + 𝟐7𝒄 = 𝒂 𝒆𝒃 𝐥𝐨𝐠 𝟐 + 𝒆7𝒄 𝐥𝐨𝐠 𝟐 ≥ 𝒂 𝒃 𝐥𝐨𝐠 𝟐 + 𝟏 + 𝟏 − 𝒄 𝐥𝐨𝐠 𝟐 =


𝒄𝒚𝒄 𝒄𝒚𝒄 𝒄𝒚𝒄

= 𝟐𝒂 + 𝐥𝐨𝐠 𝟐 𝒂𝒃 − 𝒂𝒄 = 𝟐 𝒂 + 𝒃 + 𝒄 + 𝐥𝐨𝐠 𝟐 𝒂 𝒃 − 𝒄 + 𝒃 𝒄 − 𝒂 + 𝒄 𝒂 − 𝒃 =
𝒄𝒚𝒄

=𝟐 𝒂+𝒃+𝒄
(proved)
25. Prove that for all positive real numbers 𝒂, 𝒃, 𝒄

𝒂𝟐 + 𝟐 𝒃𝟐 + 𝟐 𝒄𝟐 + 𝟐
+ + ≥ 𝟑 .
𝒃+𝒄+𝟏 𝒄+𝒂+𝟏 𝒂+𝒃+𝟏

Proposed by Nguyen Viet Hung – Hanoi – Vietnam


Solution by Kevin Soto Palacios – Huarmey – Peru
Probar para todos los numerous 𝑹. 𝒂, 𝒃, 𝒄:

𝒂𝟐 + 𝟐 𝒃𝟐 + 𝟐 𝒄𝟐 + 𝟐
+ + ≥ 𝟑
𝒃+𝒄+𝟏 𝒄+𝒂+𝟏 𝒂+𝒃+𝟏

Por la desigualdad de Cauchy:


𝒂𝟐 + 𝟏 + 𝟏 𝟏 + 𝒃𝟐 + 𝟏 ≥ 𝒂 + 𝒃 + 𝟏 𝟐 … (A)
De forma análoga:
𝒃𝟐 + 𝟏 + 𝟏 𝟏 + 𝒄𝟐 + 𝟏 ≥ 𝒃 + 𝒄 + 𝟏 𝟐 … (B)
𝒄𝟐 + 𝟏 + 𝟏 𝟏 + 𝒂𝟐 + 𝟏 ≥ 𝒄 + 𝒂 + 𝟏 𝟐 … (C)
Multiplicando (A) (B) (C):
𝒂𝟐 + 𝟐 𝟐
𝒃𝟐 + 𝟐 𝟐
𝒄𝟐 + 𝟐 𝟐
≥ 𝒃+𝒄+𝟏 𝟐
𝒄+𝒂+𝟏 𝟐
𝒂 + 𝒃 + 𝟏 𝟐
⇒ 𝒂 𝟐 + 𝟐 𝒃𝟐 + 𝟐 𝒄𝟐 + 𝟐 ≥ 𝒃 + 𝒄 + 𝟏 𝒄 + 𝒂 + 𝟏 𝒂 + 𝒃 + 𝟏
De la desigualdad propuesta … Por: MA ≥ MG

29
SOCIETATEA DE ȘTIINȚE MATEMATICE DIN ROMÂNIA
ROMANIAN MATHEMATICAL SOCIETY

Filiala Mehedinți - Mehedinți Branch


www.ssmrmh.ro

𝟑
𝒂𝟐 .𝟐 𝒃𝟐 .𝟐 𝒄𝟐 .𝟐 𝒂𝟐 .𝟐 𝒃𝟐 .𝟐 𝒄𝟐 .𝟐
+ + ≥𝟑 ≥ 𝟑 … (LQQD)
𝒃.𝒄.𝟏 𝒄.𝒂.𝟏 𝒂.𝒃.𝟏 𝒃.𝒄.𝟏 𝒄.𝒂.𝟏 𝒂.𝒃.𝟏

26. If 𝒂, 𝒃 ∈ 𝟎, ∞ ; 𝒎, 𝒏 ∈ ℕ∗ then:
𝟏 𝟗

𝒎𝒂 + 𝒏𝒃 𝒏𝒂 + 𝒎𝒃 𝒎+𝒏 𝒂+𝒃+𝒄

Proposed by Daniel Sitaru – Romania


Solution 1 by Ravi Prakash - New Delhi – India
Solution 2 by Myagmarsuren Yadamsuren – Mongolia
Solution 1 by Ravi Prakash - New Delhi – India
𝒎𝒂 + 𝒏𝒃 + 𝒏𝒂 + 𝒎𝒃
≥ 𝒎𝒂 + 𝒏𝒃 𝒏𝒂 + 𝒎𝒃
𝟐
𝟏
⇒ 𝒎 + 𝒏 𝒂 + 𝒃 ≥ 𝒎𝒂 + 𝒏𝒃 𝒏𝒂 + 𝒎𝒃
𝟐
𝟏 𝟐
⇒ ≥
𝒎𝒂 + 𝒏𝒃 𝒏𝒂 + 𝒎𝒃 𝒎+𝒏 𝒂+𝒃
𝟏 𝟐 𝟏
≥ ≥
𝒎𝒂 + 𝒏𝒃 𝒏𝒂 + 𝒎𝒃 𝒎+𝒏 𝒂+𝒃
𝟐 𝒂 𝒂
≥ ⋅ =
𝒎+𝒏 𝒂+𝒃 𝒎+𝒏 𝒂+𝒃+𝒄
[AM ≥ HM]
Solution 2 by Myagmarsuren Yadamsuren – Mongolia
𝟏 𝑪𝒂𝒖𝒄𝒉𝒚

𝒎𝒂 + 𝒏𝒃 ⋅ 𝒏𝒂 + 𝒎𝒃
𝟐 𝟐 𝟏
≥ = ⋅ ≥
𝒎+𝒏 ⋅𝒂+ 𝒎+𝒏 ⋅𝒃 𝒎+𝒏 𝒂+𝒃

30
SOCIETATEA DE ȘTIINȚE MATEMATICE DIN ROMÂNIA
ROMANIAN MATHEMATICAL SOCIETY

Filiala Mehedinți - Mehedinți Branch


www.ssmrmh.ro

𝑪𝒂𝒖𝒄𝒉𝒚7𝑺𝒄𝒉𝒘𝒂𝒓𝒛 𝟐 𝟗 𝟗
≥ ⋅ =
𝒎+𝒏 𝟐⋅ 𝒂+𝒃+𝒄 𝒎+𝒏 ⋅ 𝒂+𝒃+𝒄
27. If 𝒂, 𝒃, 𝒄 ∈ 𝟎, ∞ then:
𝒂𝟑 𝒃𝟑 𝒄𝟑 𝟑
+ + ≥
𝒃𝟐 𝟓𝒂 + 𝟐𝒃 𝒄𝟐 𝟓𝒃 + 𝟐𝒄 𝒂𝟐 𝟓𝒄 + 𝟐𝒂 𝟕
Proposed by Daniel Sitaru – Romania
Solution 1 by Imad Zak – Saida – Lebanon
Solution 2 by Kevin Soto Palacios – Huarmey – Peru
Solution 3 by Soumitra Mandal – Chandar Nagore – India
Solution 1 by Imad Zak – Saida – Lebanon
𝟏 𝟗 𝟏
𝒇 𝒙 = + 𝐥𝐧 𝒙 −
𝒙 𝟓 + 𝟐𝒙 𝟒𝟗 𝟕


𝒙 − 𝟏 𝟑𝟔𝒙𝟐 + 𝟐𝟓𝟔𝒙 + 𝟐𝟒𝟓
𝒇 𝒙 =
𝟒𝟗𝒙𝟐 𝟐𝒙 + 𝟓 𝟐
𝒂 𝟎 𝟏 +∞

𝒇• 𝒙 − − − − − − − 𝟎 + + + + + + + + + + +
𝒇 𝒙 𝟎

⇒ 𝒇 𝒙 ≥ 𝟎 ∀ 𝒙 > 0
Now consider the inequality
𝒂𝟑 ?? 𝟑

𝒃𝟐 𝟓𝒂 + 𝟐𝒃 𝟕
𝒃 𝒄 𝒂
Let 𝒙 = 𝒚 = 𝒛 = ⇒ 𝒙𝒚𝒛 = 𝟏
𝒂 𝒃 𝒄
?? 𝟑
We get 𝒈 𝒙 ≥
𝟕

31
SOCIETATEA DE ȘTIINȚE MATEMATICE DIN ROMÂNIA
ROMANIAN MATHEMATICAL SOCIETY

Filiala Mehedinți - Mehedinți Branch


www.ssmrmh.ro

𝟗 𝟏 𝟗 𝟑
𝒈 𝒙 = 𝒇 𝒙 − 𝐥𝐧 𝒙 + = 𝒇 𝒙 − 𝐥𝐧 𝒙𝒚𝒛 +
𝟒𝟗 𝟕 𝟒𝟗 𝟕
𝟑 𝟑
= 𝒇 𝒙 −𝟎+ ≥𝟎−𝟎+
𝟕 𝟕
≪ = ≫ at 𝒙 = 𝒚 = 𝒛 = 𝟏 or 𝒂 = 𝒃 = 𝒄
Solution 2 by Kevin Soto Palacios – Huarmey – Peru
Siendo: 𝒂, 𝒃, 𝒄 ∈ < 0, ∞ >. Probar que:
𝒂𝟑 𝒃𝟑 𝒄𝟑 𝟑
+ + ≥
𝒃𝟐 𝟓𝒂 + 𝟐𝒃 𝒄𝟐 𝟓𝒃 + 𝟐𝒄 𝒂𝟐 𝟓𝒄 + 𝟐𝒂 𝟕
Por la desigualdad de Cauchy:
𝒂𝟑 𝒃𝟑 𝒄𝟑
+ + 𝒂𝒃𝟐 𝟓𝒂 + 𝟐𝒃 + 𝒃𝒄𝟐 𝟓𝒃 + 𝟐𝒄 + 𝒄𝒂𝟐 𝟓𝒄 + 𝟐𝒂 ≥
𝒃𝟐 𝟓𝒂.𝟐𝒃 𝒄𝟐 𝟓𝒃.𝟐𝒄 𝒂𝟐 𝟓𝒄.𝟐𝒂

≥ 𝒂 𝟐 + 𝒃𝟐 + 𝒄𝟐 𝟐
𝟐
𝒂𝟑 𝒃𝟑 𝒄𝟑 𝒂𝟐 .𝒃𝟐 .𝒄𝟐 𝟑
⇒ + + ≥ ≥
𝒃𝟐 𝟓𝒂.𝟐𝒃 𝒄𝟐 𝟓𝒃.𝟐𝒄 𝒂𝟐 𝟓𝒄.𝟐𝒂 𝟓 𝒂𝟐 𝒃𝟐 .𝟐𝒃𝟑 𝒂.𝟐𝒄𝟑 𝒃.𝟐𝒂𝟐 𝒄 𝟕

⇒ 𝟕 𝒂 𝟐 + 𝒃𝟐 + 𝒄𝟐 𝟐
≥ +𝟑 𝟓 𝒂𝟐 𝒃𝟐 + 𝟐𝒃𝟑 𝒂 + 𝟐𝒄𝟑 𝒃 + 𝟐𝒂𝟐 𝒄

⇒ 𝟔 𝒂𝟒 + 𝒃𝟒 + 𝒄𝟒 + 𝒂𝟒 + 𝒃𝟒 + 𝒄𝟒 + 𝟏𝟒 𝒂𝟐 𝒃𝟐 ≥ 𝟏𝟓 𝒂𝟐 𝒃𝟐 + 𝟔𝒃𝟑 𝒂 + 𝟔𝒄𝟑 𝒃 + 𝟔𝒂𝟑 𝒄

⇒ 𝒂𝟒 + 𝒃𝟒 + 𝒄𝟒 ≥ 𝒃𝟑 𝒂 + 𝒄𝟑 𝒃 + 𝒂𝟑 𝒄
Por: MA ≥ 𝑴𝑮: → 𝒂𝟒 + 𝒂𝟒 + 𝒂𝟒 + 𝒄𝟒 ≥ 𝟒𝒂𝟑 𝒄 … (A)
𝒃𝟒 + 𝒃𝟒 + 𝒃𝟒 + 𝒂𝟒 ≥ 𝟒𝒃𝟑 𝒄 … (B)
𝒄𝟒 + 𝒄𝟒 + 𝒄𝟒 + 𝒃𝟒 ≥ 𝟒𝒄𝟑 𝒃 … (C)
Sumando … (A) + (B) + (C):
⇒ 𝒂𝟒 + 𝒃𝟒 + 𝒄𝟒 ≥ 𝒃𝟑 𝒂 + 𝒄𝟑 𝒃 + 𝒂𝟑 𝒄 … (LQQD)
Solution 3 by Soumitra Mandal – Chandar Nagore – India

32
SOCIETATEA DE ȘTIINȚE MATEMATICE DIN ROMÂNIA
ROMANIAN MATHEMATICAL SOCIETY

Filiala Mehedinți - Mehedinți Branch


www.ssmrmh.ro

𝒂𝟑 𝐑𝐚𝐝𝐨𝐧‡ 𝐬 𝐈𝐧𝐞𝐪𝐮𝐚𝐥𝐢𝐭𝐲 𝒂+𝒃+𝒄 𝟑


≥ 𝟐

𝒃𝟐 𝟓𝒂 + 𝟐𝒃 𝒃 ⋅ 𝟓𝒂𝒃 + 𝟐𝒃𝟐
𝒄𝒚𝒄 𝒄𝒚𝒄
𝟑
𝒂+𝒃+𝒄
≥ 𝟐

𝒂+𝒃+𝒄 𝒄𝒚𝒄 𝟐𝒃𝟐 + 𝟓𝒂𝒃

[Applying Cauchy – Schwarz]


𝟑
𝒂+𝒃+𝒄
≥ 𝟐

𝒂+𝒃+𝒄 𝟐 𝒂+𝒃+𝒄 𝟐 + 𝒂𝒃 + 𝒃𝒄 + 𝒄𝒂
𝒂.𝒃.𝒄 𝟑 𝟑
≥ 𝟐 = (proved)
𝟏 𝟕
𝒂.𝒃.𝒄 𝟐 𝒂.𝒃.𝒄 𝟐 . 𝒂.𝒃.𝒄 𝟐
𝟑

28. Prove that for any positive real numbers 𝒂, 𝒃, 𝒄:


𝒂 𝒃 𝒄 𝒃𝒄 𝒄𝒂 𝒂𝒃
+ + ≥ 𝟐 + 𝟐 + 𝟐
𝒃 + 𝒄 𝒄 + 𝒂 𝒂 + 𝒃 𝒂 + 𝒃𝒄 𝒃 + 𝒄𝒂 𝒄 + 𝒂𝒃
Proposed by Nguyen Viet Hung – Hanoi – Vietnam
Solution by Kevin Soto Palacios – Huarmey – Peru
Probar para todos los numerous 𝑹. :
𝒂 𝒃 𝒄 𝒃𝒄 𝒄𝒂 𝒂𝒃
+ + ≥ 𝟐 + 𝟐 + 𝟐
𝒃 + 𝒄 𝒂 + 𝒄 𝒂 + 𝒃 𝒂 + 𝒃𝒄 𝒃 + 𝒄𝒂 𝒄 + 𝒂𝒃
Por la desigualdad de Nesbitt: 𝑹. :
𝒂 𝒃 𝒄 𝟏 𝒂 𝟏 𝒃 𝟏 𝒄 𝟑
+ + ≥ + + +
𝒃+𝒄 𝒂+𝒄 𝒂+𝒃 𝟐 𝒃+𝒄 𝟐 𝒂+𝒄 𝟐 𝒂+𝒃 𝟒
𝟏 𝒂 𝟏 𝒃 𝟏 𝒄 𝟑 𝒂 𝒃 𝒄 𝟑
⇒ + + ≥ → + + ≥
𝟐 𝒃+𝒄 𝟐 𝒂+𝒄 𝟐 𝒂+𝒃 𝟒 𝒃+𝒄 𝒂+𝒄 𝒂+𝒃 𝟐
Por transitividad:

33
SOCIETATEA DE ȘTIINȚE MATEMATICE DIN ROMÂNIA
ROMANIAN MATHEMATICAL SOCIETY

Filiala Mehedinți - Mehedinți Branch


www.ssmrmh.ro

𝒂 𝒃 𝒄 𝟏 𝒂 𝟏 𝒃 𝟏 𝒄 𝟑
+ + ≥ + + + ≥
𝒃+𝒄 𝒂+𝒄 𝒂+𝒃 𝟐 𝒃+𝒄 𝟐 𝒂+𝒄 𝟐 𝒂+𝒃 𝟒
𝒃𝒄 𝒄𝒂 𝒂𝒃
≥ 𝟐 + 𝟐 + 𝟐
𝒂 + 𝒃𝒄 𝒃 + 𝒄𝒂 𝒄 + 𝒂𝒃
Es suficiente probar:
𝟏 𝒂 𝟏 𝒃 𝟏 𝒄 𝟑 𝒃𝒄 𝒄𝒂 𝒂𝒃
+ + + ≥ 𝟐 + 𝟐 + 𝟐
𝟐 𝒃+𝒄 𝟐 𝒂+𝒄 𝟐 𝒂+𝒃 𝟒 𝒂 + 𝒃𝒄 𝒃 + 𝒄𝒂 𝒄 + 𝒂𝒃
𝒂 𝒃 𝒄 𝟑 𝟐𝒃𝒄 𝟐𝒄𝒂 𝟐𝒂𝒃
⇒ + + + ≥ 𝟐 + 𝟐 + 𝟐
𝒃 + 𝒄 𝒂 + 𝒄 𝒂 + 𝒃 𝟐 𝒂 + 𝒃𝒄 𝒃 + 𝒄𝒂 𝒄 + 𝒂𝒃
𝒂 𝒃 𝒄 𝟑 𝟐𝒂𝟐 𝟐𝒃𝟐 𝟐𝒄𝟐
⇒ + + + ≥ 𝟐− + 𝟐− + 𝟐−
𝒃.𝒄 𝒂.𝒄 𝒂.𝒃 𝟐 𝒂𝟐 .𝒃𝒄 𝒃𝟐 .𝒄𝒂 𝒄𝟐 .𝒂𝒃

𝒂𝟐 𝒃𝟐 𝒄𝟐 𝟐𝒂𝟐 𝟐𝒃𝟐 𝟐𝒄𝟐 𝟗


⇒ + + + 𝟐 + 𝟐 + 𝟐 ≥
𝒂𝒃 + 𝒂𝒄 𝒃𝒂 + 𝒃𝒄 𝒄𝒂 + 𝒄𝒃 𝒂 + 𝒃𝒄 𝒃 + 𝒄𝒂 𝒄 + 𝒂𝒃 𝟐
Por desigualdad de Cauchy:
𝒂𝟐 𝒂𝟐 𝒂𝟐 𝟗𝒂𝟐
⇒ + + ≥ … (A)
𝒂𝒃.𝒂𝒄 𝒂𝟐 .𝒃𝒄 𝒂𝟐 .𝒃𝒄 𝟐𝒃𝒄.𝟐𝒂𝟐 . 𝒂𝒄.𝒂𝒃

𝒃𝟐 𝒃𝟐 𝒃𝟐 𝟗𝒃𝟐
⇒ + + ≥ … (B)
𝒃𝒂.𝒃𝒄 𝒃𝟐 .𝒄𝒂 𝒃𝟐 .𝒄𝒂 𝟐𝒂𝒄.𝟐𝒃𝟐 . 𝒂𝒃.𝒃𝒄

𝒄𝟐 𝒄𝟐 𝒄𝟐 𝟗𝒄𝟐
⇒ + + ≥ … (C)
𝒄𝒂.𝒄𝒃 𝒄𝟐 .𝒂𝒃 𝒄𝟐 .𝒂𝒃 𝟐𝒂.𝟐𝒄𝟐 . 𝒃𝒄.𝒂𝒄

Sumando … (A) + (B) + (C):


𝒂𝟐 𝒃𝟐 𝒄𝟐 𝟐𝒂𝟐 𝟐𝒃𝟐 𝟐𝒄𝟐
⇒ + + + 𝟐 + + ≥
𝒂𝒃 + 𝒂𝒄 𝒃𝒂 + 𝒃𝒄 𝒄𝒂 + 𝒄𝒃 𝒂 + 𝒃𝒄 𝒃𝟐 + 𝒄𝒂 𝒄𝟐 + 𝒂𝒃
𝟗𝒂𝟐

𝟐𝒃𝒄 + 𝟐𝒂𝟐 + 𝒂𝒄 + 𝒂𝒃
𝟗𝒂𝟐 𝟗 𝒂 𝟐
𝟗
⇒ ≥ =
𝟐𝒃𝒄 + 𝒂𝟐 + 𝒂𝒄 + 𝒃𝒄 + 𝒂𝒃 𝟐 𝒃𝒄 + 𝒂𝟐 + 𝒂𝒃 + 𝒂𝒄 𝟐
(LQQD)

34
SOCIETATEA DE ȘTIINȚE MATEMATICE DIN ROMÂNIA
ROMANIAN MATHEMATICAL SOCIETY

Filiala Mehedinți - Mehedinți Branch


www.ssmrmh.ro

29. Let 𝒂, 𝒃, 𝒙, 𝒚, 𝒛, 𝒖, 𝒗, 𝒘 be positive real numbers such that 𝒙 + 𝒚 + 𝒛 = 𝟑.


Prove that:
𝒖𝟐 𝟏 𝒗𝟐 𝟏 𝒘𝟐 𝟏 𝟑
⋅ 𝒙
+ ⋅ 𝒚
+ ⋅ 𝒛

𝒖𝒘 𝒂𝒚 + 𝒃𝒛 𝒘𝒖 𝒂𝒛 + 𝒃𝒙 𝒖𝒗 𝒂𝒙 + 𝒃𝒚 𝒂+𝒃
Proposed by Nguyen Viet Hung – Hanoi – Vietnam
Solution by Soumitra Mandal – Chandar Nagore – India
𝒙 + 𝒚 + 𝒛 = 𝟑, 𝟑 ≥ 𝒙𝒚 + 𝒚𝒛 + 𝒛𝒙
𝒖𝟐 𝟏 𝟑 𝟗
⋅ 𝒙
≥ ≥
𝒖𝒘 𝒂𝒚 + 𝒃𝒛 𝟑
𝒙 𝒄𝒚𝒄 𝒙 𝒂𝒚 + 𝒃𝒛
𝒄𝒚𝒄 𝒄𝒚𝒄 𝒂𝒚 + 𝒃𝒛

Applying Weighted A.M ≥ G.M

𝒄𝒚𝒄 𝒙 𝒂𝒚 + 𝒃𝒛 𝒙u𝒚u𝒛
𝒙 𝟑
≥ 𝒂𝒚 + 𝒃𝒛 = 𝒂𝒚 + 𝒃𝒛 𝒙
𝒙+𝒚+𝒛
𝒄𝒚𝒄 𝒄𝒚𝒄

𝟏 𝟑
⇒ ≥
𝟑
𝒙 𝒄𝒚𝒄 𝒙 𝒂𝒚 + 𝒃𝒛
𝒄𝒚𝒄 𝒂𝒚 + 𝒃𝒛

𝒖𝟐 𝟏 𝟗 𝟗 𝟑
⋅ 𝒙
≥ = ≥
𝒖𝒘 𝒂𝒚 + 𝒃𝒛 𝒄𝒚𝒄 𝒙 𝒂𝒚 + 𝒃𝒛 𝒂 + 𝒃 𝒙𝒚 + 𝒚𝒛 + 𝒛𝒙 𝒂+𝒃
𝒄𝒚𝒄

(proved)
30. If 𝒂, 𝒃, 𝒄 > 0, 𝑎𝑏𝑐 = 1 then:
𝟏 𝟏 𝟏 𝟑
+ + ≥
𝒂𝟐 + 𝒂𝒃 + 𝒃𝟐 𝒃𝟐 + 𝒃𝒄 + 𝒄𝟐 𝒄𝟐 + 𝒄𝒂 + 𝒂𝟐 𝒂𝒃 𝟐 + 𝒃𝒄 𝟐 + 𝒄𝒂 𝟐

Proposed by Babis Stergioiu – Greece


Solution 1 by Imad Zak – Saida – Lebanon
Solution 2 by Soumitra Mandal - Chandar Nagore – India

35
SOCIETATEA DE ȘTIINȚE MATEMATICE DIN ROMÂNIA
ROMANIAN MATHEMATICAL SOCIETY

Filiala Mehedinți - Mehedinți Branch


www.ssmrmh.ro

Solution 3 by Le Viet Hung- Quang Tri – Vietnam


Solution 4 by Fotini Kaldi – Greece
Solution 1 by Imad Zak – Saida – Lebanon
𝒂, 𝒃, 𝒄 > 0, ∀ 𝒂𝒃𝒄 = 𝟏
prove that
𝟏 ?? 𝟑

𝒂𝟐 + 𝒂𝒃 + 𝒃𝟐 𝒂𝟐 𝒃𝟐
when 𝒂𝒃𝒄 = 𝟏 it is known that 𝒑 = 𝒂 ≥ 𝟑, 𝒒𝟐 ≥ 𝟑𝒑𝒓 = 𝟑𝒑𝒄
𝒄𝟐 𝒂𝟐 𝒃𝟐
𝑳𝑯𝑺 = + +
𝒂𝟐 𝒄𝟐 + 𝒂𝒃𝒄 𝒄 + 𝒃𝟐 𝒄𝟐 𝒂𝟐 𝒃𝟐 + 𝒂𝒃𝒄 𝒂 + 𝒂𝟐 𝒄𝟐 𝒃𝟐 𝒄𝟐 + 𝒂𝒃𝒄 𝒃 + 𝒂𝟐 𝒃𝟐
𝒄𝟐 𝒂𝟐 𝒃𝟐 𝑪7𝑩7𝑺 𝒂+𝒃+𝒄 𝟐
= 𝟐 𝟐 + + ≥
𝒂 𝒄 + 𝒄 + 𝒃𝟐 𝒄𝟐 𝒂𝟐 𝒃𝟐 + 𝒂 + 𝒂𝟐 𝒄𝟐 𝒃𝟐 𝒄𝟐 + 𝒃 + 𝒂𝟐 𝒃𝟐 𝟐 𝒂𝟐 𝒃𝟐 + 𝒂
𝒑𝟐 𝟗
= ≥
𝟐 𝒂𝟐 𝒃𝟐 + 𝒑 𝟐 𝒂𝟐 𝒃𝟐 + 𝒑
𝟗 ?? 𝟑 ??
We need to prove that ≥ or 𝟑 𝒂𝟐 𝒃𝟐 ≥ 𝟐 𝒂𝟐 𝒃𝟐 + 𝒑
𝟐 𝒂𝟐 𝒃𝟐 .𝒑 𝒂𝟐 𝒃 𝟐

?? ??
or 𝒂𝟐 𝒃𝟐 ≥ 𝒑 or 𝒂𝒃 𝟐
− 𝟐𝒑𝒓 ≥ 𝒑 ⇔
𝒒𝟐 ≥ 𝟑𝐩 true Q.E.D
≪=≫ at 𝒂 = 𝒃 = 𝒄 = 𝟏
Solution 2 by Soumitra Mandal - Chandar Nagore – India
Let 𝒙 = 𝒂𝒃, 𝒚 = 𝒃𝒄 and = 𝒄𝒂 ; 𝒙𝒚𝒛 = 𝟏
𝟏 𝟑

𝒂𝟐 + 𝒂𝒃 + 𝒃𝟐 𝒄𝒚𝒄 𝒂𝒃
𝟐
𝒄𝒚𝒄

𝟏 𝟑
⇔ 𝒛𝒙 𝒙𝒚 ≥ … (𝟏)
+ + 𝒙 𝒙𝟐 + 𝒚𝟐 + 𝒛𝟐
𝒄𝒚𝒄 𝒚 𝒛

36
SOCIETATEA DE ȘTIINȚE MATEMATICE DIN ROMÂNIA
ROMANIAN MATHEMATICAL SOCIETY

Filiala Mehedinți - Mehedinți Branch


www.ssmrmh.ro

Now,

𝟏 𝒚𝒛 𝟑 𝒙𝒚𝒛
𝒛𝒙 𝒙𝒚 = ≥ 𝟑
+ + 𝒙 𝒙 𝒚𝟐 + 𝒛𝟐 + 𝒚𝒛 𝒄𝒚𝒄 𝒙𝟐 + 𝒚𝟐 + 𝒙𝒚
𝒄𝒚𝒄 𝒚 𝒛 𝒄𝒚𝒄

𝟑 𝟗
= ≥ ≥
𝟑 𝟐 𝒙𝟐 + 𝒚𝟐 + 𝒛𝟐 + 𝒙𝒚 + 𝒚𝒛 + 𝒛𝒙
𝒄𝒚𝒄 𝒙𝟐 + 𝒚𝟐 + 𝒙𝒚

𝟗 𝟑
≥ ≥
𝟐 𝒙𝟐 + 𝒚𝟐 + 𝒛𝟐 + 𝒙𝟐 + 𝒚𝟐 + 𝒛𝟐 𝒙𝟐 + 𝒚𝟐 + 𝒛𝟐
Hence statement (1) is established
𝟏 𝟑

𝒂𝟐 + 𝒂𝒃 + 𝒃𝟐 𝒄𝒚𝒄 𝒂𝒃
𝟐
𝒄𝒚𝒄

(proved)
Solution 3 by Le Viet Hung- Quang Tri – Vietnam
First, we have:
𝟑 𝒂𝟐 − 𝒂𝒃 + 𝒃𝟐 ≥ 𝒂𝟐 + 𝒂𝒃 + 𝒃𝟐 ⇔ 𝟐 𝒂 − 𝒃 𝟐
≥ 𝟎
𝟏 𝟏 𝟏
⇒ ≥ ⋅
𝒂𝟐 + 𝒂𝒃 + 𝒃𝟐 𝟑 𝒂𝟐 − 𝒂𝒃 + 𝒃𝟐
𝟑 𝒃𝟐 − 𝒃𝒄 + 𝒄𝟐 ≥ 𝒃𝟐 + 𝒃𝒄 + 𝒄𝟐 ⇔ 𝟐 𝒃 − 𝒄 𝟐
≥ 𝟎
𝟏 𝟏 𝟏
⇒ ≥ ⋅
𝒃𝟐 + 𝒃𝒄 + 𝒄𝟐 𝟑 𝒃𝟐 − 𝒃𝒄 + 𝒄𝟐
𝟑 𝒄𝟐 − 𝒄𝒂 + 𝒂𝟐 ≥ 𝒄𝟐 + 𝒄𝒂 + 𝒂𝟐 ⇔ 𝟐 𝒄 − 𝒂 𝟐
≥ 𝟎
𝟏 𝟏 𝟏
⇒ ≥ ⋅
𝒄𝟐 + 𝒄𝒂 + 𝒂𝟐 𝟑 𝒄𝟐 − 𝒄𝒂 + 𝒂𝟐
𝟏 𝟏 𝟏 𝟏 𝟗
≥ ⋅ ≥ ⋅
𝒂𝟐 + 𝒂𝒃 + 𝒃𝟐 𝟑 𝒂𝟐 − 𝒂𝒃 + 𝒃𝟐 𝟑 𝒂𝟐 − 𝒂𝒃 + 𝒃𝟐

37
SOCIETATEA DE ȘTIINȚE MATEMATICE DIN ROMÂNIA
ROMANIAN MATHEMATICAL SOCIETY

Filiala Mehedinți - Mehedinți Branch


www.ssmrmh.ro

𝟑
= Cauchy – Schwarz
𝟐 𝒂𝟐 .𝒃𝟐 .𝒄𝟐 7 𝒂𝒃.𝒃𝒄.𝒄𝒂
𝟑
𝟑 𝒂𝒃.𝒃𝒄.𝒄𝒂 𝟑⋅𝟑 𝒂𝟐 𝒃𝟐 𝒄𝟐 𝟗
≥ ≥ 𝟐 = AM – GM
𝟐 𝒂𝟐 .𝒃𝟐 .𝒄𝟐 7 𝒂𝒃.𝒃𝒄.𝒄𝒂 𝒂𝒃.𝒃𝒄.𝒄𝒂 𝟐 𝒂𝟐 u𝒃𝟐 u𝒄𝟐 𝒂𝟐 .𝒃𝟐 .𝒄𝟐
𝟐

𝟗 𝟑
≥ =
𝟑 𝒂 𝟐 𝒃𝟐 + 𝒃𝟐 𝒄𝟐 + 𝒄𝟐 𝒂 𝟐 𝒂𝟐 𝒃𝟐 + 𝒃𝟐 𝒄𝟐 + 𝒄𝟐 𝒂 𝟐
Solution 4 by Fotini Kaldi – Greece
𝟑
𝑩 = 𝒂𝒃 + 𝒃𝒄 + 𝒄𝒂 ≥ 𝟑 𝒂𝒃 𝒃𝒄 𝒄𝒂 ⇒ 𝑩 ≥ 𝟑
𝟐 𝟐 𝟐 𝟑
𝑨 = 𝒂𝒃 + 𝒃𝒄 + 𝒄𝒂 ≥𝟑 𝒂𝒃 𝟐 𝒃𝒄 𝟐 𝒄𝒂 𝟐 ⇒ 𝑨 ≥ 𝟑
𝟑 𝒙𝟐 + 𝒚𝟐 + 𝒛𝟐 ≥ 𝒙 + 𝒚 + 𝒛 𝟐
≥ 𝟑 𝒙𝒚 + 𝒚𝒛 + 𝒛𝒙 ,
𝑨 ≥ 𝒂𝒃 𝒃𝒄 + 𝒂𝒃 𝒂𝒄 + 𝒂𝒄 𝒃𝒄 ⇒ 𝑨 ≥ 𝒂 + 𝒃 + 𝒄 ⇒
𝟏 𝟏
⇒ 𝟑𝑨 ≥ 𝟐𝑨 + 𝒂 + 𝒃 + 𝒄 ⇒ ≥
𝟐𝑨 + 𝒂 + 𝒃 + 𝒄 𝟑𝑨
𝒄𝟐 𝒂𝟐 𝒃𝟐
𝑳𝑯𝑺 = + + ≥
𝒂𝒄 𝟐 + 𝒄 + 𝒃𝒄 𝟐 𝒂𝒃 𝟐 + 𝒂 + 𝒂𝒄 𝟐 𝒃𝒄 𝟐 + 𝒃 + 𝒂𝒃 𝟐

𝒂+𝒃+𝒄 𝟐 𝟑 𝒂𝒃 + 𝒃𝒄 + 𝒄𝒂
≥ ≥
𝟐 𝒂𝒃 𝟐 + 𝒃𝒄 𝟐 + 𝒄𝒂 𝟐 + 𝒂 + 𝒃 + 𝒄 𝟐𝑨 + 𝒂 + 𝒃 + 𝒄
𝟗 𝟗 𝟑
≥ ≥ = ⇒
𝟐𝑨 + 𝒂 + 𝒃 + 𝒄 𝟑𝑨 𝑨
𝟏 𝟏 𝟏 𝟑
+ + ≥
𝒂𝟐 + 𝒂𝒃 + 𝒃𝟐 𝒃𝟐 + 𝒃𝒄 + 𝒄𝟐 𝒄𝟐 + 𝒄𝒂 + 𝒂𝟐 𝒂𝒃 𝟐 + 𝒃𝒄 𝟐 + 𝒄𝒂 𝟐


31. Let 𝒂, 𝒃, 𝒄 be positive real numbers such that 𝒂 ≤ 𝟐, 𝒂 + 𝒃 ≤ 𝟓 and
𝒂 + 𝒃 + 𝒄 ≤ 𝟏𝟏. Prove that
𝟏 𝟏 𝟏
+ + ≥ 𝟏
𝒂 𝒃 𝒄

38
SOCIETATEA DE ȘTIINȚE MATEMATICE DIN ROMÂNIA
ROMANIAN MATHEMATICAL SOCIETY

Filiala Mehedinți - Mehedinți Branch


www.ssmrmh.ro

Proposed by Nguyen Viet Hung – Hanoi – Vietnam


Solution by Kevin Soto Palacios – Huarmey – Peru
Siendo: 𝒂, 𝒃, 𝒄 números 𝑹. de tal manera que
𝒂 ≤ 𝟐, 𝒂 + 𝒃 ≤ 𝟓, 𝒂 + 𝒃 + 𝒄 ≤ 𝟏𝟏.
Probar que:
𝟏 𝟏 𝟏
+ + ≥ 𝟏
𝒂 𝒃 𝒄
Por la desigualdad de Cauchy:
𝟏 𝟏 𝟏 𝟏 𝟏 𝟏 𝟑𝟔 𝟑𝟔 𝟑𝟔
+ + + + + ≥ = ≥ = 𝟏 …
𝟑𝒂 𝟑𝒂 𝟑𝒂 𝟐𝒃 𝟐𝒃 𝒄 𝟗𝒂.𝟒𝒃.𝒄 𝒂.𝒃.𝒄 .𝟑 𝒂.𝒃 .𝟓𝒂 𝟏𝟏.𝟏𝟓.𝟏𝟎

(LQQD)
La igualdad se alcanza cuando 𝒂 = 𝟐, 𝒃 = 𝟑, 𝒄 = 𝟔.
32. Let 𝒙; 𝒚; 𝒛 > 0. Prove that:
𝒙 𝒚 𝒛 𝒙𝟐 𝒚𝟐 𝒛𝟐 𝒙𝟐 + 𝒙𝒚 + 𝒚𝟐
+ + + + ≥
𝒚 𝒛 𝒙 𝒚 𝒛 𝒙 𝒛
Proposed by Le Viet Hung –Hai Lang-Vietnam
Solution 1 by Ravi Prakash - New Delhi – India
Solution 2 by Soumava Chakraborty – Kolkata – India
Solution 3 by Imad Zak – Saida – Lebanon
Solution 4 by Daniel Sitaru – Romania
Solution 1 by Ravi Prakash - New Delhi – India
𝒙 𝒚 𝒛 𝒙𝟐 𝒚𝟐 𝒛𝟐 𝒙𝟐 + 𝒙𝒚 + 𝒚𝟐
+ + + + ≥
𝒚 𝒛 𝒙 𝒚 𝒛 𝒙 𝒛
𝒙 𝒚 𝒛 𝒙𝟐 𝒚𝟐 𝒛𝟐
𝑳𝑯𝑺 = + + + +
𝒚 𝒛 𝒙 𝒚 𝒛 𝒙

39
SOCIETATEA DE ȘTIINȚE MATEMATICE DIN ROMÂNIA
ROMANIAN MATHEMATICAL SOCIETY

Filiala Mehedinți - Mehedinți Branch


www.ssmrmh.ro

𝒙𝟐 + 𝒙𝒚 𝒙𝟑
= +
𝒛 𝒚𝟐
𝒙𝟐 .𝒙𝒚.𝒚𝟐 𝒙𝟑 𝒙𝟐
= + 𝟐
− (1)
𝒛 𝒚 𝒚

𝒙𝟐
Let 𝑬 = 𝒙−𝒚
𝒚𝟐

𝒙𝟐
= −𝟏 𝒙−𝒚 ∴ 𝒙−𝒚 =𝟎
𝒚𝟐
𝒙𝟐 − 𝒚𝟐 𝒙 − 𝒚
=
𝒚𝟐
𝒙+𝒚 𝒙−𝒚
= ≥ 𝟎
𝒚𝟐
Thus, form (1)
𝒙 𝒚 𝒛 𝒙𝟐 𝒚𝟐 𝒛𝟐 𝒙𝟐 + 𝒙𝒚 + 𝒚𝟐
+ + + + ≥
𝒚 𝒛 𝒙 𝒚 𝒛 𝒙 𝒛
Solution 2 by Soumava Chakraborty – Kolkata – India
𝒙𝟑 𝒙𝒚 𝒛𝟐 𝒙𝟐 𝒚𝟑 𝒚𝒛 𝒛𝒙 𝒚𝟐 𝒛𝟑
𝑳𝑯𝑺 = 𝟐 + + + + 𝟐+ + + +
𝒚 𝒛 𝒚 𝒛 𝒛 𝒙 𝒚 𝒙 𝒙𝟐
𝒙𝟐 𝒙𝒚 𝒚𝟐 𝒚𝟐 𝒚𝒛 𝒛𝟐 𝒛𝟐 𝒛𝒙 𝒙𝟐
𝑹𝑯𝑺 = + + + + + + + +
𝒛 𝒛 𝒛 𝒙 𝒙 𝒙 𝒚 𝒚 𝒚
𝒙𝟑 𝒚𝟑 𝒛𝟑 𝒚𝟐 𝒛𝟐 𝒙𝟐
𝑳𝑯𝑺 ≥ 𝑹𝑯𝑺 ⇔ + + ≥ + + (a)
𝒚𝟐 𝒛𝟐 𝒙𝟐 𝒛 𝒙 𝒚

𝒙𝟑 𝑨𝑴7𝑮𝑴 𝒙𝟐 𝒚𝟑 𝑨𝑴7𝑮𝑴 𝟐𝒚𝟐 𝒛𝟑 𝑨𝑴7𝑮𝑴 𝟐𝒛𝟐


Now, 𝟐
+𝒙 ≥ 𝟐 , 𝟐
+𝒚 ≥ , 𝟐
+𝒛 ≥
𝒚 𝒚 𝒛 𝒛 𝒙 𝒙

Adding the 3:
𝒙𝟑 𝒚𝟑 𝒛𝟑 𝒙𝟐 𝒚𝟐 𝒛𝟐
𝟐
+ 𝟐
+ 𝟐
+𝒙+𝒚+𝒛≥𝟐 + + (1)
𝒚 𝒛 𝒙 𝒚 𝒛 𝒙

40
SOCIETATEA DE ȘTIINȚE MATEMATICE DIN ROMÂNIA
ROMANIAN MATHEMATICAL SOCIETY

Filiala Mehedinți - Mehedinți Branch


www.ssmrmh.ro

𝒙𝟑 𝒚𝟑 𝒛𝟑 𝑹𝒂𝒅𝒐𝒏 𝒙.𝒚.𝒛 𝟑
Again 𝟐
+ 𝟐
+ ≥ = 𝒙 + 𝒚 + 𝒛
𝒚 𝒛 𝒙𝟐 𝒙.𝒚.𝒛 𝟐
(𝟐)

𝒙𝟑 𝒚𝟑 𝒛𝟑 𝒙𝟐 𝒚𝟐 𝒛𝟐
(1)+(2)⇒ 𝟐 𝟐
+ 𝟐
+ ≥𝟐 + +
𝒚 𝒛 𝒙𝟐 𝒚 𝒛 𝒙

𝒙𝟑 𝒚𝟑 𝒛𝟑 𝒙𝟐 𝒚𝟐 𝒛𝟐
⇒ 𝟐+ 𝟐+ 𝟐≥ + +
𝒚 𝒛 𝒙 𝒚 𝒛 𝒙
⇒ (a) is true (Proved)
Solution 3 by Imad Zak – Saida – Lebanon
𝟐
By Cauchy – Schwarz 𝑩 𝒙 + 𝒚 + 𝒛 ≥ 𝒙 + 𝒚 + 𝒛 ⇒ 𝑩 ≥ 𝒙 + 𝒚 + 𝒛 (1)
𝑩𝟐 𝟐
𝑨 𝒙+𝒚+𝒛 ≥𝑩 ⇒𝑨≥
𝒙
𝑩𝟐
So it is sufficient to prove ≥𝑩⇔𝑩≥ 𝒙 which is true by (1)
𝒙

≪=≫ holds for 𝒙 = 𝒚 = 𝒛


Solution 4 by Daniel Sitaru – Romania
𝒙𝟐 𝒛 𝒙𝟑 𝒛
𝑳𝑯𝑺 = ⋅ ≥ 𝑹𝑯𝑺
𝒙𝒚𝒛 𝒙𝒚𝒛
𝒙𝟐 𝒛 ⋅ 𝒙𝟑 𝒛 𝒙𝒚 𝒙𝟐 + 𝒙𝒚 + 𝒚𝟐

𝒙𝒚𝒛 𝟐 𝒙𝒚𝒛

𝒙𝟐 𝒛 ⋅ 𝒙𝟑 𝒛 ≥ 𝒙𝒚𝒛 ⋅ 𝒙𝒚 𝒙𝟐 + 𝒙𝒚 + 𝒚𝟐

𝒙𝟓 𝒛𝟐 ≥ 𝒙𝒚𝟐 𝒛𝟒

𝟓, 𝟎, 𝟐 ⋟ 𝟏, 𝟐, 𝟒 (𝑴𝒖𝒊𝒓𝒉𝒆𝒂𝒅)
33. If 𝒂, 𝒃, 𝒄 ∈ 𝟎, ∞ then:
𝟏𝟎𝒂𝟑 𝟏𝟎𝒃𝟑 𝟏𝟎𝒄𝟑
+ + ≥ 𝒂 + 𝒃 + 𝒄
𝟑𝒂𝟐 + 𝟕𝒃𝒄 𝟑𝒃𝟐 + 𝟕𝒄𝒂 𝟑𝒄𝟐 + 𝟕𝒂𝒃

41
SOCIETATEA DE ȘTIINȚE MATEMATICE DIN ROMÂNIA
ROMANIAN MATHEMATICAL SOCIETY

Filiala Mehedinți - Mehedinți Branch


www.ssmrmh.ro

Proposed by Daniel Sitaru – Romania


Solution 1 by Anas Adlany-El Jadida-Morroco
Solution 2 by Imad Zak – Saida – Lebanon
Solution 3 by Kevin Soto Palacios – Huarmey – Peru
Solution 4 by Soumitra Mandal –Chandar Nagore – India
Solution 1 by Anas Adlany-Morroco
WLOG, let 𝒂𝟐 = 𝟏.

𝟏𝟎𝒂𝟑 𝟏𝟎𝒂𝟑
≥ =
𝟑𝒂𝟐 + 𝟕𝒃𝒄 𝟐 𝒃 𝟐 + 𝒄𝟐
𝟑𝒂 + 𝟕
𝟐
𝒂𝟑 𝒂𝟑
= 𝟐𝟎 = 𝟐𝟎 ≥
𝟔𝒂𝟐 + 𝟕𝒃𝟐 + 𝟕𝒄𝟐 𝟕 − 𝒂𝟐
[Using Chebyshev’s inequality two times]
𝟐𝟎 𝟏 𝟐𝟎 𝟗
≥ 𝒂 𝒂𝟐 ≥ 𝒂 = 𝒂
𝟗 𝟕 − 𝒂𝟐 𝟗 𝟕 − 𝒂𝟐
as desired.
Solution 2 by Imad Zak – Saida – Lebanon
Homogeneous ⇒ let 𝒂𝒃𝒄 = 𝟏 the inequality is re-written as

𝒇 𝒂 ≥ 𝟎

𝟏𝟎𝒙𝟒 𝟕𝒙 𝒙𝟑 7𝟏
where 𝒇 𝒙 = −𝒙= which is convex
𝟑𝒙𝟑 .𝟕 𝟑𝒙𝟑 .𝟕
𝟐𝟏
⇒ 𝒇 𝒙 ≥ 𝒈 𝒙 = 𝒙 − 𝟏 the tangent at 𝒙 = 𝟏, we get
𝟏𝟎

𝟔𝟑 𝟔𝟑
𝒇 𝒂 ≥ 𝒈 𝒂 = − = 𝟎
𝟏𝟎 𝟏𝟎

42
SOCIETATEA DE ȘTIINȚE MATEMATICE DIN ROMÂNIA
ROMANIAN MATHEMATICAL SOCIETY

Filiala Mehedinți - Mehedinți Branch


www.ssmrmh.ro

q.e.d
equality at 𝒂 = 𝒃 = 𝒄
Solution 3 by Kevin Soto Palacios – Huarmey – Peru
By: Inequality Holder’s:
𝟏𝟎𝒂𝟑 𝟏𝟎𝒃𝟑 𝟏𝟎𝒄𝟑
+ + 𝟑𝒂𝟐 + 𝟕𝒃𝒄 + 𝟑𝒃𝟐 + 𝟕𝒂𝒄 + 𝟑𝒄𝟐 + 𝟖𝒂𝒃 𝟏 + 𝟏 + 𝟏 ≥
𝟑𝒂𝟐 + 𝟕𝒃𝒄 𝟑𝒃𝟐 + 𝟕𝒂𝒄 𝟑𝒄𝟐 + 𝟕𝒂𝒃

≥ 𝟏𝟎 𝒂 + 𝒃 + 𝒄 𝟑
It is enough prove that:
𝟐
𝟐 𝟐
𝟏𝟎 𝒂 + 𝒃 + 𝒄 𝟐
𝟑𝒂 + 𝟕𝒃𝒄 + 𝟑𝒃 + 𝟕𝒂𝒄 + 𝟑𝒄 + 𝟕𝒂𝒃 ≤ ⇒
𝟑
⇒ 𝒂𝟐 + 𝒃𝟐 + 𝒄𝟐 ≥ 𝒂𝒃 + 𝒃𝒄 + 𝒂𝒄
𝟏𝟎𝒂𝟑 𝟏𝟎𝒃𝟑 𝟏𝟎𝒄𝟑 𝟏𝟎 𝒂 + 𝒃 + 𝒄 𝟑
⇒ + + ≥ ≥
𝟑𝒂𝟐 + 𝟕𝒃𝒄 𝟑𝒃𝟐 + 𝟕𝒂𝒄 𝟑𝒄𝟐 + 𝟕𝒂𝒃 𝟑 𝟑𝒂𝟐 + 𝟕𝒃𝒄 + 𝟑𝒃𝟐 + 𝟕𝒂𝒄 + 𝟑𝒄𝟐 + 𝟕𝒂𝒃
𝟑
𝟏𝟎 𝒂 + 𝒃 + 𝒄
≥ 𝟐
= 𝒂 + 𝒃 + 𝒄
𝟏𝟎 𝒂 + 𝒃 + 𝒄

Solution 4 by Soumitra Mandal -Kolkata – India


𝟏𝟎𝒂𝟑 𝟏𝟎𝒂𝟑
⇒ ≥𝒂+𝒃+𝒄⇔ − 𝒂 ≥ 𝟎
𝟑𝒂𝟐 + 𝟕𝒃𝒄 𝟑𝒂𝟐 + 𝟕𝒃𝒄
𝒄𝒚𝒄 𝒄𝒚𝒄

𝟕 𝒂 𝒂+𝒃 𝒂−𝒄 +𝒂 𝒂−𝒃 𝒂+𝒄


⇔ ≥ 𝟎
𝟐 𝟑𝒂𝟐 + 𝟕𝒃𝒄
𝒄𝒚𝒄

𝟕 𝒂 𝒂−𝒃 𝒂+𝒄 𝒂 𝒂+𝒃 𝒂−𝒄


⇔ + ≥ 𝟎
𝟐 𝟑𝒂𝟐 + 𝟕𝒃𝒄 𝟑𝒂𝟐 + 𝟕𝒃𝒄
𝒄𝒚𝒄

𝟕 𝒂 𝒂−𝒃 𝒂+𝒄 𝒃 𝒃+𝒄 𝒃−𝒂


⇔ + ≥ 𝟎
𝟐 𝟑𝒂𝟐 + 𝟕𝒃𝒄 𝟑𝒃𝟐 + 𝟕𝒂𝒄
𝒄𝒚𝒄

𝟕 𝒂 𝒂+𝒄 𝒃 𝒃+𝒄
⇔ 𝒂−𝒃 − ≥ 𝟎
𝟐 𝟑𝒂𝟐 + 𝟕𝒃𝒄 𝟑𝒃𝟐 + 𝟕𝒂𝒄
𝒄𝒚𝒄

43
SOCIETATEA DE ȘTIINȚE MATEMATICE DIN ROMÂNIA
ROMANIAN MATHEMATICAL SOCIETY

Filiala Mehedinți - Mehedinți Branch


www.ssmrmh.ro

𝟕 𝟕𝒄 𝒂𝟑 − 𝒃𝟑 + 𝟑𝒂𝒃𝒄 𝒃 − 𝒂 + 𝟕𝒄𝟐 𝒂𝟐 − 𝒃𝟐
⇔ 𝒂−𝒃 ≥ 𝟎
𝟐 𝟑𝒂𝟐 + 𝟕𝒃𝒄 𝟑𝒃𝟐 + 𝟕𝒂𝒄
𝒄𝒚𝒄
𝟐 𝟐 𝟐
𝟕 𝟐 𝟕𝒄 𝒂 .𝒃 .𝟕𝒄 𝒂.𝒃 .𝟒𝒂𝒃𝒄
⇔ 𝒄𝒚𝒄 𝒂−𝒃 ≥ 𝟎, which is true
𝟐 𝟑𝒂𝟐 .𝟕𝒃𝒄 𝟑𝒃𝟐 .𝟕𝒂𝒄

Hence,
𝟏𝟎𝒂𝟑
≥ 𝒂 + 𝒃 + 𝒄
𝟑𝒂𝟐 + 𝟕𝒃𝒄
𝒄𝒚𝒄

(proved)
34. Let 𝒂, 𝒃, 𝒄 be positive real numbers such that 𝒂 + 𝒃 + 𝒄 = 𝟏. Prove that
𝒏
𝒂 𝒃 𝒄 𝟑
𝒏
+ 𝒏
+ 𝒏

𝒃+𝒄 𝒄+𝒂 𝒂+𝒃 𝟐
Proposed by Nguyen Viet Hung – Hanoi – Vietnam
Solution 1 by Kevin Soto Palacios – Huarmey – Peru
Solution 2 by Phan Loc So'n-Quy - Nhon City – Vietnam

Solution 1 by Kevin Soto Palacios – Huarmey – Peru
Siendo: 𝒂, 𝒃, 𝒄 números 𝑹. de tal manera que: 𝒂 + 𝒃 + 𝒄 = 𝟏. Probar que:
𝒏
𝒂 𝒃 𝒄 𝟑
𝒏
+ 𝒏
+ 𝒏

𝒃+𝒄 𝒄+𝒂 𝒂+𝒃 𝟐
La desigualdad puede ser equivalente:
𝒂𝒏u𝟏 𝒃𝒏u𝟏 𝒄𝒏u𝟏
⇒ 𝒏
+ 𝒏
+ ... (A)
𝒂 𝒃.𝒄 𝒃 𝒄.𝒂 𝒄 𝒂.𝒃 𝒏

De la desigualdad de Radon:
Si: 𝒙𝒌 , 𝒂𝒌 > 0, 𝑘 ∈ 𝟏, 𝟐, 𝟑, … , 𝒏 , 𝒏 ∈ ℕ ∧ 𝒎 > 0, se cumple lo siguiente:

44
SOCIETATEA DE ȘTIINȚE MATEMATICE DIN ROMÂNIA
ROMANIAN MATHEMATICAL SOCIETY

Filiala Mehedinți - Mehedinți Branch


www.ssmrmh.ro

𝒙𝒎.𝟏
𝟏 𝒙𝒎.𝟏
𝟐 𝒙𝒎.𝟏
𝒏 𝒙𝟏 + 𝒙𝟐 + ⋯ + 𝒙𝒏 𝒎.𝟏
⇒ 𝒎 + 𝒎 + ⋯+ 𝒎 ≥
𝒂𝟏 𝒂𝟐 𝒂𝒏 𝒂𝟏 + 𝒂𝟐 + ⋯ + 𝒂𝒏 𝒎
𝒙𝟏 𝒙𝟐 𝒙𝟑 𝒙𝒏
La igualdad se alcanza cuando: = = =⋯=
𝒂𝟏 𝒂𝟐 𝒂𝟑 𝒂𝒏

Por la desigualdad de Radon en (A) ... :


𝒂𝒏.𝟏 𝒃𝒏.𝟏 𝒄𝒏.𝟏 𝒂 + 𝒃 + 𝒄 𝒏.𝟏
𝒏
+ 𝒏
+ 𝒏
≥ 𝒏

𝒂 𝒃+𝒄 𝒃 𝒄+𝒂 𝒄 𝒂+𝒃 𝒂 𝒃+𝒄 +𝒃 𝒄+𝒂 +𝒄 𝒂+𝒃
𝟏 𝟑 𝒏
≥ 𝒏 =
𝟐 𝟐 𝟐
𝒂+𝒃+𝒄
𝟑
Solution 2 by Phan Loc So'n-Quy Nhon City – Vietnam
Asume: 𝒂 ≥ 𝒃 ≥ 𝒄.
𝒂 𝒃 𝒄 𝟏 𝟏 𝟏
≥ ≥ & ≥ ≥ .
𝒃+𝒄 𝒄+𝒂 𝒂+𝒃 𝒃+𝒄 𝒄+𝒂 𝒂+𝒃
By Cebyshev’s inequality:
𝒂 𝟏 𝒃 𝟏 𝒄 𝟏
⋅ 𝒏
+ ⋅ 𝒏
+ ⋅
𝒃+𝒄 𝒃+𝒄 𝒄+𝒂 𝒄+𝒂 𝒂+𝒃 𝒄+𝒂 𝒏
𝒏.𝟏
𝟏 𝒂 𝒃 𝒄 𝟏 𝟏 𝟏 𝟑
≥ + + 𝒏
+ 𝒏
+ 𝒏

𝟑 𝒃+𝒄 𝒄+𝒂 𝒂+𝒃 𝒂+𝒃 𝒃+𝒄 𝒄+𝒂 𝟐


35. If 𝒂, 𝒃, 𝒄 > 0 then:
𝟑
𝟐 𝒂+𝒃 +𝟓 𝒂𝟑 ≥ 𝟐𝟏 𝒂𝟐 𝒃 + 𝒃𝟐 𝒄 + 𝒄𝟐 𝒂

Proposed by Daniel Sitaru – Romania


Solution 1 by Kevin Soto Palacios – Huarmey – Peru
Solution 2 by Soumava Chakraborty – Kolkata – India
Solution 3 by Seyran Ibrahimov – Maasilli – Azerbaidjian

45
SOCIETATEA DE ȘTIINȚE MATEMATICE DIN ROMÂNIA
ROMANIAN MATHEMATICAL SOCIETY

Filiala Mehedinți - Mehedinți Branch


www.ssmrmh.ro


Solution 1 by Kevin Soto Palacios – Huarmey – Peru
Si: 𝒂, 𝒃, 𝒄 > 0. Probar que:
𝟑
𝟐 𝒂+𝒃 +𝟓 𝒂𝟑 ≥ 𝟐𝟏 𝒂𝟐 𝒃 + 𝒃𝟐 𝒄 + 𝒄𝟐 𝒂
𝟑 𝟑 𝟑
⇒𝟐 𝒂+𝒃 +𝟐 𝒃+𝒄 +𝟐 𝒄+𝒂 + 𝟓 𝒂𝟑 + 𝒃𝟑 + 𝒄𝟑 ≥ 𝟐𝟏 𝒂𝟐 𝒃 + 𝒃𝟐 𝒄 + 𝒄𝟐 𝒂
⇒ 𝟗 𝒂𝟑 + 𝒃𝟑 + 𝒄𝟑 + 𝟔𝒂𝒃 𝒂 + 𝒃 + 𝟔𝒃𝒄 𝒃 + 𝒄 + 𝟔𝒄𝒂 𝒄 + 𝒂 ≥
≥ 𝟐𝟏 𝒂𝟐 𝒃 + 𝒃𝟐 𝒄 + 𝒄𝟐 𝒂
⇒ 𝟗 𝒂𝟑 + 𝒃𝟑 + 𝒄𝟑 + 𝟔𝒃𝟐 𝒂 + 𝟔𝒄𝟐 𝒃 + 𝟔𝒂𝟐 𝒄 ≥ 𝟏𝟓 𝒂𝟐 𝒃 + 𝒃𝟐 𝒄 + 𝒄𝟐 𝒂
Desde que: 𝒂, 𝒃, 𝒄 > 0.
Por: 𝐌𝐀 ≥ 𝐌𝐆
⇒ 𝟔𝒂𝟑 + 𝟔𝒃𝟐 𝒂 ≥ 𝟏𝟐𝒂𝟐 𝒃 … (A)
𝟔𝒃𝟑 + 𝟔𝒄𝟐 𝒃 ≥ 𝟏𝟐𝒃𝟐 𝒄 … (B),
𝟔𝒄𝟑 + 𝟔𝒂𝟐 𝒄 ≥ 𝟏𝟐𝒄𝟐 𝒂 … (C)
Sumando: (A) + (B) + (C):
⇒ 𝟔 𝒂𝟑 + 𝒃𝟑 + 𝒄𝟑 + 𝟔𝒃𝟐 𝒂 + 𝟔𝒄𝟐 𝒃 + 𝟔𝒂𝟐 𝒄 ≥ 𝟏𝟐 𝒂𝟐 𝒃 + 𝒃𝟐 𝒄 + 𝒄𝟐 𝒂 … (D)
Por otro lado. Por: 𝐌𝐀 ≥ 𝐌𝐆
⇒ 𝒂𝟑 + 𝒂𝟑 + 𝒃𝟑 + 𝒃𝟑 + 𝒃𝟑 + 𝒄𝟑 + 𝒄𝟑 + 𝒄𝟑 + 𝒂𝟑 =
= 𝟑 𝒂𝟑 + 𝒃𝟑 + 𝒄𝟑 ≥ 𝟑𝒂𝟐 𝒃 + 𝟑𝒃𝟐 𝒄 + 𝟑𝒄𝟐 𝒂 … (E)
Finalmente sumando: (D) + (E) …
⇒ 𝟗 𝒂𝟑 + 𝒃𝟑 + 𝒄𝟑 + 𝟔𝒃𝟐 𝒂 + 𝟔𝒄𝟐 𝒃 + 𝟔𝒂𝟐 𝒄 ≥ 𝟏𝟓 𝒂𝟐 𝒃 + 𝒃𝟐 𝒄 + 𝒄𝟐 𝒂 … (LQQD)

Solution 2 by Soumava Chakraborty – Kolkata – India
Given inequality ⇔ 𝟐 𝟐 𝒂𝟑 + 𝟑 𝒂𝟐 𝒃 + 𝟑 𝒂𝒃𝟐 + 𝟓 𝒂𝟑 ≥ 𝟐𝟏 𝒂𝟐 𝒃
⇔𝟑 𝒂𝟑 + 𝟐 𝒂𝟐 𝒃 + 𝟐 𝒂𝒃𝟐 ≥ 𝟕 𝒂𝟐 𝒃 (A)

46
SOCIETATEA DE ȘTIINȚE MATEMATICE DIN ROMÂNIA
ROMANIAN MATHEMATICAL SOCIETY

Filiala Mehedinți - Mehedinți Branch


www.ssmrmh.ro

𝑨7𝑮
𝒂𝟑 + 𝒂𝟐 𝒃 + 𝒂𝒃𝟐 ≥ 𝟑𝒂𝟐 𝒃 𝒂𝟑 + 𝒂𝟐 𝒃 + 𝒂𝒃𝟐 ≥ 𝟑 𝒂𝟐 𝒃
𝑨7𝑮
𝒃𝟑 + 𝒃𝟐 𝒄 + 𝒃𝒄𝟐 ≥ 𝟑𝒃𝟐 𝒄 ⇒
𝑨7𝑮 𝟐 𝒂𝟑 + 𝟐 𝒂𝟐 𝒃 + 𝟐 𝒂𝒃𝟐 ≥ 𝟔 𝒂𝟐 𝒃 𝟏
𝒄𝟑 + 𝒄𝟐 𝒂 + 𝒄𝒂𝟐 ≥ 𝟑𝒄𝟐 𝒂
Again, (A-G) ⇒

𝒂𝟑 + 𝒂𝟑 + 𝒃𝟑 ≥ 𝟑𝒂𝟐 𝒃 𝟑 𝒂𝟑 ≥ 𝟑 𝒂𝟐 𝒃
𝒃𝟑 + 𝒃𝟑 + 𝒄𝟑 ≥ 𝟑𝒃𝟐 𝒄 ⇒
𝟑 𝟐
𝟑 𝟑 𝟑
𝒄 + 𝒄 + 𝒂 ≥ 𝟑𝒄 𝒂 𝟐 𝒂 ≥ 𝒂 𝒃 (𝟐)

(1) + (2) ⇒ 𝟑 𝒂𝟑 + 𝟐 𝒂𝟐 𝒃 + 𝟐 𝒂𝒃𝟐 ≥ 𝟕 𝒂𝟐 𝒃


⇒ (A) is true (Proved)

Solution 3 by Seyran Ibrahimov – Maasilli – Azerbaidjian

𝒂𝟑 + 𝒂𝟑 + 𝒂𝒃𝟐 + 𝒂𝒃𝟐 ≥ 𝟒𝒂𝟐 𝒃


𝒃𝟑 + 𝒃𝟑 + 𝒃𝒄𝟐 + 𝒃𝒄𝟐 ≥ 𝟒𝒃𝟐 𝒄 +
𝒄𝟑 + 𝒄𝟑 + 𝒄𝒂𝟐 + 𝒄𝒂𝟐 ≥ 𝟒𝒄𝟐 𝒂

− − − − − − − − − − − − − − −
𝒙 = 𝒂𝟑 + 𝒃𝟑 + 𝒄𝟑 + 𝒂𝒃𝟐 + 𝒃𝒄𝟐 + 𝒄𝒂𝟐 ≥ 𝟐𝒄𝒄𝟐 + 𝟐𝒃𝟐 𝒄 + 𝟐𝒄𝟐 𝒂
𝟑
𝟐 𝒂+𝒃 +𝟓 𝒂𝟑 ≥ 𝟐𝟏 𝒂𝟐 𝒃 + 𝒃𝟐 𝒄 + 𝒄𝟐 𝒂
𝒄𝒚𝒄𝒍 𝒄𝒚𝒄𝒍

𝟗𝒂𝟑 + 𝟗𝒃𝟑 + 𝟗𝒄𝟑 + 𝟔𝒂𝒃𝟐 + 𝟔𝒃𝒄𝟐 + 𝟔𝒄𝒂𝟐 ≥ 𝟏𝟓𝒂𝟐 𝒃 + 𝟏𝟓𝒃𝟐 𝒄 + 𝟏𝟓𝒄𝟐 𝒂


𝟑𝒂𝟑 + 𝟑𝒃𝟑 + 𝟑𝒄𝟑 + 𝟐𝒂𝒃𝟐 + 𝟐𝒃𝒄𝟐 + 𝟐𝒄𝒂𝟐 ≥ 𝟓𝒂𝟐 𝒃 + 𝟓𝒃𝟐 𝒄 + 𝟓𝒄𝟐 𝒂
𝒂𝟑 + 𝒃𝟑 + 𝒄𝟑 + 𝟐𝒙 ≥ 𝟓𝒂𝟐 𝒃 + 𝟓𝒃𝟐 𝒄 + 𝟓𝒄𝟐 𝒂
𝒂𝟑 + 𝒃𝟑 + 𝒄𝟑 ≥ 𝒂𝟐 𝒃 + 𝒃𝟐 𝒄 + 𝒄𝟐 𝒂
(proved)
36. If 𝒂, 𝒃, 𝒄 > 0 then:

47
SOCIETATEA DE ȘTIINȚE MATEMATICE DIN ROMÂNIA
ROMANIAN MATHEMATICAL SOCIETY

Filiala Mehedinți - Mehedinți Branch


www.ssmrmh.ro

𝟐
𝒂𝒄 + 𝒃𝒄 − 𝒄 𝒂𝒃 ≥ 𝒂𝟐 𝒃 𝟐 + 𝒃𝟐 𝒄𝟐 + 𝒄𝟐 𝒂 𝟐

Proposed by Daniel Sitaru – Romania


Solution by Ravi Prakash - New Delhi – India
𝟐
𝒂𝒄 + 𝒃𝒄 − 𝒄 𝒂𝒃 ≥ 𝒂𝟐 𝒃 𝟐 + 𝒃𝟐 𝒄𝟐 + 𝒄𝟐 𝒂 𝟐
𝟐
⇔ 𝟐𝒄𝟐 𝒂 + 𝒃 − 𝒂𝒃 − 𝒄𝟐 𝒂𝟐 − 𝒄𝟐 𝒃𝟐 ≥ 𝟎 (1)

Consider
𝟐
𝟐𝒄𝟐 𝒂 + 𝒃 − 𝒂𝒃 − 𝒄𝟐 𝒂 𝟐 − 𝒄𝟐 𝒃 𝟐

= 𝒄𝟐 𝟐 𝒂 + 𝒃 𝟐
+ 𝟐𝒂𝒃 − 𝟒 𝒂𝒃 𝒂 + 𝒃 − 𝒂𝟐 − 𝒃𝟐

= 𝒄𝟐 𝒂𝟐 + 𝒃𝟐 + 𝟔𝒂𝒃 − 𝟒 𝒂𝒃 𝒂 + 𝒃

= 𝒄𝟐 𝒂 + 𝒃 𝟐
− 𝟒 𝒂𝒃 𝒂 + 𝒃 + 𝟒𝒂𝒃
𝟐
= 𝒄𝟐 𝒂 + 𝒃 − 𝟐 𝒂𝒃
𝟒
= 𝒄𝟐 𝒂− 𝒃 ≥ 𝟎
Similarly for other two expressions.
Thus (1) is true.

37. If 𝒙, 𝒚, 𝒛 ∈ 𝟎, ∞ , 𝒙𝒚𝒛 = 𝟏 then:

𝟒 𝟑
𝒙𝟐 + 𝒚𝟐
𝒙 +𝒚 +𝒛 ≥ + 𝟑
𝒛
Proposed by Daniel Sitaru – Romania
Solution by Imad Zak – Saida – Lebanon
𝟏
Schur ⇒ 𝒙𝟒 + 𝒙𝒚𝒛 𝒙 + 𝒚 + 𝒛 ≥ 𝒙𝒚 𝒙𝟐 + 𝒚𝟐 but 𝒙𝒚𝒛 = 𝟏 ⇒ 𝒙𝒚 =
𝒛

48
SOCIETATEA DE ȘTIINȚE MATEMATICE DIN ROMÂNIA
ROMANIAN MATHEMATICAL SOCIETY

Filiala Mehedinți - Mehedinți Branch


www.ssmrmh.ro

𝒙𝟐 .𝒚𝟐
𝒙𝟒 + 𝒙≥ ... (1)
𝒛

𝒙𝟐 .𝒚𝟐
Our inequality: 𝒙𝟒 + 𝒙𝟑 + 𝒙≥ + 𝟑 !!
𝒛

𝒙𝟐 .𝒚𝟐
but (1) ⇒ ≤ 𝒙𝟒 + 𝒙
𝒛

so we need to prove that


?? ??
𝒙𝟒 + 𝒙𝟑 + 𝒙≥ 𝒙𝟒 + 𝒙+𝟑⇔ 𝒙𝟑 ≥ 𝟑 True
by AM-GM ≪=≫ 𝒙 = 𝒚 = 𝒛 = 𝟏
38. If 𝒂, 𝒃, 𝒄 > 0 then:

𝟐 𝒂+𝒃+𝒄 𝒂+𝒃 𝟐𝒂
≥ +
𝒂𝒃𝒄 𝟐𝒂𝒄 𝒄 𝒂+𝒃

Proposed by Daniel Sitaru – Romania


Solution 1 by Kevin Soto Palacios – Huarmey – Peru
Solution 2 by Soumava Chakraborty – Kolkata-India

Solution 1 by Kevin Soto Palacios – Huarmey – Peru
Si: 𝒂, 𝒃, 𝒄 > 0. Probar que:

𝟐 𝒂+𝒃+𝒄 𝒂+𝒃 𝟐𝒂
≥ +
𝒂𝒃𝒄 𝟐𝒂𝒄 𝒄 𝒂+𝒃

𝒂+𝒃 𝟐𝒂 𝒂+𝒃 𝒃 𝒂+𝒃 𝒂


+ ≤ + ≤
𝟐𝒂𝒄 𝒄 𝒂+𝒃 𝟐𝒂𝒃𝒄 𝟐𝒂𝒃𝒄
𝟐
𝒂+𝒃 𝒂+ 𝒃 𝟐 𝒂+𝒃
≤ ≤
𝟐𝒂𝒃𝒄 𝟐𝒂𝒃𝒄

49
SOCIETATEA DE ȘTIINȚE MATEMATICE DIN ROMÂNIA
ROMANIAN MATHEMATICAL SOCIETY

Filiala Mehedinți - Mehedinți Branch


www.ssmrmh.ro

𝟐
𝟐 𝒂+𝒃+𝒄 𝟐 𝒂+𝒃 𝟐 𝟐 𝒂+𝒃+𝒄 𝟐 𝒂+𝒃+𝒄
⇒ ≤ = =
𝒂𝒃𝒄 𝟐𝒂𝒃𝒄 𝟐𝒂𝒃𝒄 𝒂𝒃𝒄
LQQD
Solution 2 by Soumava Chakraborty – Kolkata-India

𝒂+𝒃 𝒂+𝒃 𝒃+𝒄 𝒄+𝒂


= + + =
𝟐𝒂𝒄 𝟐𝒂𝒄 𝟐𝒂𝒃 𝟐𝒃𝒄

𝒃 𝒂 + 𝒃 + 𝒄 𝒃 + 𝒄 + 𝒂 𝒄 + 𝒂 𝑪𝑩𝑺 𝒂 𝟐 𝒂 𝒂+𝒃+𝒄
= ≤ =
𝟐𝒂𝒃𝒄 𝟏 𝟐𝒂𝒃𝒄 𝒂+𝒃+𝒄

𝟐𝒂 𝟐𝒂 𝟐𝒃 𝟐𝒄
= + + ≤
𝒄 𝒂+𝒃 𝒄 𝒂+𝒃 𝒂 𝒃+𝒄 𝒃 𝒄+𝒂

𝑪𝑩𝑺 𝟏 𝟏 𝟏
≤ 𝟐 𝒂 + + ≤
𝒄 𝒂+𝒃 𝒂 𝒃+𝒄 𝒃 𝒄+𝒂

𝟏 𝟏 𝟏
≤ 𝟐 𝒂 + + =
𝒄 𝟐 𝒂𝒃 𝒂 𝟐 𝒃𝒄 𝒃(𝟐 𝒄𝒂)
𝑨𝑴7𝑮𝑴
∵𝒂+𝒃 ≥ 𝟐 𝒂𝒃

𝟏 𝟏
∴ ≤
𝒂 + 𝒃 𝟐 𝒂𝒃
etc

𝟏 𝟏 𝟏 𝟏
= 𝒂 + + =
𝒂𝒃𝒄 𝒄 𝒂 𝒃

50
SOCIETATEA DE ȘTIINȚE MATEMATICE DIN ROMÂNIA
ROMANIAN MATHEMATICAL SOCIETY

Filiala Mehedinți - Mehedinți Branch


www.ssmrmh.ro

𝟏 𝒂𝒃 + 𝒃𝒄 + 𝒄𝒂 𝒂
= 𝒂 = 𝒂𝒃 + 𝒃𝒄 + 𝒄𝒂 ≤
𝒂𝒃𝒄 𝒂𝒃𝒄 𝒂𝒃𝒄

𝑪7𝑩7𝑺 𝒂 𝒂 𝒂+𝒃+𝒄
≤ 𝒂 𝒂= =
(𝟐)
𝒂𝒃𝒄 𝒂𝒃𝒄 𝒂𝒃𝒄

𝒂.𝒃 𝟐𝒂 𝟐 𝒂.𝒃.𝒄
(1) + (2) ⇒ + ≤
𝟐𝒂𝒄 𝒄 𝒂.𝒃 𝒂𝒃𝒄

(Proved)

39. Let 𝒙, 𝒚, 𝒛 ∈ 𝟎, 𝟏 such that 𝒙𝒚 + 𝒚𝒛 + 𝒛𝒙 = 𝟏. Prove that:

𝒙 𝒚 𝒛 𝟗 𝟑
+ + ≥
𝟏 − 𝒚𝟐 𝟏 − 𝒛𝟐 𝟏 − 𝒛𝟐 𝟏 − 𝒙𝟐 𝟏 − 𝒙𝟐 𝟏 − 𝒚𝟐 𝟒
Proposed by Nguyen Viet Hung – Hanoi – Vietnam
Solution 1 by Kevin Soto Palacios – Huarmey – Peru
Solution 2 by Abhay Chandra – India

Solution 1 by Kevin Soto Palacios – Huarmey – Peru
Siendo: 𝒙, 𝒚, 𝒛 ∈< 0,1 > de tal manera que:
𝒙𝒚 + 𝒚𝒛 + 𝒛𝒙 = 𝟏. Probar que:

𝒙 𝒚 𝒛 𝟗 𝟑
+ + ≥
𝟏 − 𝒚𝟐 𝟏 − 𝒛𝟐 𝟏 − 𝒛𝟐 𝟏 − 𝒙𝟐 𝟏 − 𝒙𝟐 𝟏 − 𝒚𝟐 𝟒
Por la desigualdad de Cauchy:
𝒙𝟐 𝒚𝟐 𝒛𝟐 𝟗 𝟑
+ + ≥
𝒙 𝟏 − 𝒚𝟐 𝟏 − 𝒛𝟐 𝒚 𝟏 − 𝒛𝟐 𝟏 − 𝒙𝟐 𝒛 𝟏 − 𝒙𝟐 𝟏 − 𝒚𝟐 𝟒

51
SOCIETATEA DE ȘTIINȚE MATEMATICE DIN ROMÂNIA
ROMANIAN MATHEMATICAL SOCIETY

Filiala Mehedinți - Mehedinți Branch


www.ssmrmh.ro

Por la desigualdad de Cauchy:


𝒙𝟐 𝒙.𝒚.𝒛 𝟐
=
𝒙 𝟏7𝒚𝟐 𝟏7𝒛𝟐 𝒙.𝒚.𝒛 .𝒙𝒚𝒛 𝒙𝒚.𝒚𝒛.𝒛𝒙 7 𝒙.𝒚.𝒛 𝒙𝒚.𝒚𝒛.𝒛𝒙 .𝟑𝒙𝒚𝒛

𝒙𝟐 𝒚𝟐 𝒛𝟐
⇒ + + ≥
𝒙 𝟏 − 𝒚𝟐 𝟏 − 𝒛𝟐 𝒚 𝟏 − 𝒛𝟐 𝟏 − 𝒙𝟐 𝒛 𝟏 − 𝒙𝟐 𝟏 − 𝒚𝟐
𝟐
𝒙+𝒚+𝒛 𝟑 𝟗 𝟑
≥ ≥ =
𝟒𝒙𝒚𝒛 𝟏 𝟒
𝟑 ×
𝟑 𝟑
(LQQD)
Lo cual es cierto ya que, por MA ≥ MG:
𝟑
𝟐
𝟏
𝒙𝒚 + 𝒚𝒛 + 𝒛𝒙 ≥ 𝟑 𝒙𝒚𝒛 → ≥ 𝒙𝒚𝒛 ∧ 𝒙 + 𝒚 + 𝒛 ≥ 𝟑 𝒙𝒚 + 𝒚𝒛 + 𝒛𝒙 →
𝟑 𝟑
→ 𝒙 + 𝒚 + 𝒛 ≥ 𝟑
𝟏
La igualdad se alcanza cuando, 𝒙 = 𝒚 = 𝒛 =
𝟑

Solution 2 by Abhay Chandra – India


From AM ≥ GM, we have
𝒙 𝟐𝟕𝒙 𝟐𝟕𝒙 𝟐𝟕𝒙
+ 𝟏 − 𝒚𝟐 + 𝟏 − 𝒛𝟐 ≥
𝟏 − 𝒚𝟐 𝟏 − 𝒛𝟐 𝟖 𝟖 𝟒
which implies
𝒙 𝟐𝟕𝒙 𝟐
≥ 𝒚 + 𝒛𝟐
𝟏 − 𝒚𝟐 𝟏 − 𝒛𝟐 𝟖
Summing it up, we get
𝒙 𝟐𝟕𝒙 𝟐𝟕
≥ 𝒚𝟐 + 𝒛𝟐 = 𝒙 + 𝒚 + 𝒛 𝒙𝒚 + 𝒚𝒛 + 𝒛𝒙 − 𝟑𝒙𝒚𝒛
𝟏7𝒚𝟐 𝟏7𝒛𝟐 𝟖 𝟖

We are left to prove that

52
SOCIETATEA DE ȘTIINȚE MATEMATICE DIN ROMÂNIA
ROMANIAN MATHEMATICAL SOCIETY

Filiala Mehedinți - Mehedinți Branch


www.ssmrmh.ro

𝟐𝟕 𝟗 𝟑
𝒙 + 𝒚 + 𝒛 − 𝟑𝒙𝒚𝒛 ≥
𝟖 𝟒
𝟏
but 𝒙 + 𝒚 + 𝒛 ≥ 𝟑 and 𝒙𝒚𝒛 ≤ . Hence we are done. Equality at
𝟑 𝟑

𝟏
𝒙=𝒚=𝒛= .
𝟑
40. Let 𝒂, 𝒃, 𝒄 be positive real numbers such that:
𝒂𝟐 + 𝒃𝟐 + 𝒄𝟐 𝒂𝒃 + 𝒃𝒄 + 𝒄𝒂 = 𝒂𝒃𝒄 𝒂 + 𝒃 + 𝒄 + 𝟐 .
Prove that:

𝟗 𝒂 𝟗 + 𝒃𝟗 𝟗 𝒃𝟗 + 𝒄𝟗 𝟗 𝒄𝟗 + 𝒂 𝟗
+ + ≤ 𝟏 .
𝟐 𝟐 𝟐

Proposed by Nguyen Viet Hung – Hanoi – Vietnam


Solution by Kevin Soto Palacios – Huarmey – Peru
Siendo: 𝒂, 𝒃, 𝒄 números 𝑹. de tal manera que:
𝒂𝟐 + 𝒃𝟐 + 𝒄𝟐 𝒂𝒃 + 𝒃𝒄 + 𝒄𝒂 = 𝒂𝒃𝒄 𝒂 + 𝒃 + 𝒄 + 𝟐
Probar que:

𝟗 𝒂 𝟗 + 𝒃𝟗 𝟗 𝒃𝟗 + 𝒄𝟗 𝟗 𝒄𝟗 + 𝒂 𝟗
+ + ≤ 𝟏
𝟐 𝟐 𝟐

De la condición, se tiene lo siguiente:


⇒ 𝒂𝟐 𝒃𝒄 + 𝒃𝟐 𝒂𝒄 + 𝒄𝟐 𝒂𝒃 + 𝒂𝒃 𝒂𝟐 + 𝒃𝟐 + 𝒃𝒄 𝒃𝟐 + 𝒄𝟐 + 𝒄𝒂 𝒄𝟐 + 𝒂𝟐 =
= 𝒂𝟐 𝒃𝒄 + 𝒃𝟐 𝒂𝒄 + 𝒄𝟐 𝒂𝒃 + 𝟐𝒂𝒃𝒄
𝒂𝟐 + 𝒃𝟐 𝒃𝟐 + 𝒄𝟐 𝒄𝟐 + 𝒂 𝟐 𝒂 𝟑 + 𝒃𝟑 𝒃𝟑 + 𝒄𝟑 𝒄𝟑 + 𝒂 𝟑
⇒ + + =𝟐→ + + = 𝟐
𝒄 𝒂 𝒃 𝒂𝒃 𝒃𝒄 𝒄𝒂
Demostraremos lo siguiente:

53
SOCIETATEA DE ȘTIINȚE MATEMATICE DIN ROMÂNIA
ROMANIAN MATHEMATICAL SOCIETY

Filiala Mehedinți - Mehedinți Branch


www.ssmrmh.ro

𝟗 𝒂𝟗 + 𝒃𝟗 𝒂𝟑 + 𝒃𝟑
≤ → 𝒂𝟑 + 𝒃𝟑 𝟖
≥ 𝟐𝟖 𝒂𝟔 + 𝒃𝟔 − 𝒂𝟑 𝒃𝟑 𝒂𝟗 𝒃𝟗
𝟐 𝟐𝒂𝒃

⇒ 𝒂𝟐𝟒 + 𝟖𝒂𝟐𝟏 𝒃𝟑 + 𝟐𝟖𝒂𝟏𝟖 𝒃𝟔 + 𝟓𝟔𝒂𝟏𝟓 𝒃𝟗 + 𝟕𝟎𝒂𝟏𝟐 𝒃𝟏𝟐 + 𝟓𝟔𝒂𝟗 𝒃𝟏𝟓 +


+𝟐𝟖𝒂𝟔 𝒃𝟏𝟖 + 𝟖𝒂𝟑 𝒃𝟐𝟏 + 𝒃𝟐𝟒 ≥ 𝟐𝟓𝟔𝒂𝟏𝟓 𝒃𝟗 + 𝟐𝟓𝟔𝒃𝟏𝟓 𝒂𝟗 − 𝟐𝟓𝟔 𝒂𝒃 𝟏𝟐

⇒ 𝒂𝟐𝟒 + 𝟖𝒂𝟐𝟏 𝒃𝟑 + 𝟐𝟖𝒂𝟏𝟖 𝒃𝟔 − 𝟐𝟎𝟎𝒂𝟏𝟓 𝒃𝟗 + 𝟑𝟐𝟔𝒂𝟏𝟐 𝒃𝟏𝟐 − 𝟐𝟎𝟎𝒂𝟗 𝒃𝟏𝟓 +
+𝟐𝟖𝒂𝟔 𝒃𝟏𝟖 + 𝟖𝒂𝟑 𝒃𝟐𝟏 + 𝒃𝟐𝟒 ≥ 𝟎
Dividendo ÷ 𝒂𝟏𝟐 𝒃𝟏𝟐 , de tal manera a que el sentido no se altere, ya que: 𝒂, 𝒃 >
0:
𝒂𝟏𝟐 𝒂𝟏𝟐 𝒂𝟗 𝒃𝟗 𝒂𝟔 𝒃𝟔 𝒂𝟑 𝒃𝟑
⇒ 𝟏𝟐 + 𝟏𝟐 + 𝟖 𝟗 + 𝟗 + 𝟐𝟖 𝟔 + 𝟔 − 𝟐𝟎𝟎 𝟑 + 𝟑 + 𝟑𝟐𝟔 ≥ 𝟎
𝒃 𝒃 𝒃 𝒂 𝒃 𝒂 𝒃 𝒂
𝒂𝟑 𝒃𝟑
Sea: 𝟑
+ = 𝒙 ≥ 𝟐 (Válido por: MA ≥ MG). Por lo tanto:
𝒃 𝒂𝟑

𝒂 𝟔 𝒃𝟔
𝟔
+ 𝟔 = 𝒙𝟐 − 𝟐,
𝒃 𝒂
𝒂𝟗 𝒃𝟗
+ = 𝒙𝟑 − 𝟑𝒙,
𝒃𝟗 𝒂𝟗
𝒂𝟏𝟐 𝒂𝟏𝟐
𝟏𝟐
+ 𝟏𝟐 = 𝒙𝟐 − 𝟐 𝟐
− 𝟐
𝒃 𝒃
⇒ 𝒙𝟐 − 𝟐 𝟐
− 𝟐 + 𝟖 𝒙𝟑 − 𝟑𝒙 + 𝟐𝟖 𝒙𝟐 − 𝟐 − 𝟐𝟎𝟎𝒙 + 𝟑𝟐𝟔 ≥ 𝟎
⇒ 𝒙𝟒 + 𝟖𝒙𝟑 + 𝟐𝟒𝒙𝟐 − 𝟐𝟐𝟒𝒙 + 𝟐𝟕𝟐 ≥ 𝟎
Lo cual se puede expresar de la siguiente forma:
𝒙𝟒 − 𝟒𝒙𝟑 + 𝟒𝒙𝟐 + 𝟏𝟐𝒙𝟑 − 𝟒𝟖𝒙𝟐 + 𝟒𝟖𝒙 + 𝟔𝟖𝒙𝟐 − 𝟐𝟕𝟐𝒙 + 𝟐𝟕𝟐 ≥ 𝟎
⇒ 𝒙𝟐 𝒙 − 𝟐 𝟐
+ 𝟏𝟐𝒙 𝒙 − 𝟐 𝟐
+ 𝟔𝟖 𝒙 − 𝟐 𝟐
= 𝒙−𝟐 𝟐
𝒙𝟐 + 𝟏𝟐𝒙 + 𝟔𝟖 ≥ 𝟎
Luego:

54
SOCIETATEA DE ȘTIINȚE MATEMATICE DIN ROMÂNIA
ROMANIAN MATHEMATICAL SOCIETY

Filiala Mehedinți - Mehedinți Branch


www.ssmrmh.ro

𝟗 𝒂 𝟗 + 𝒃𝟗 𝟗 𝒃 𝟗 + 𝒄𝟗 𝟗 𝒄𝟗 + 𝒂 𝟗 𝒂𝟑 + 𝒃𝟑 𝒃𝟑 + 𝒄𝟑 𝒄𝟑 + 𝒂 𝟑
+ + ≤ + + = 𝟏
𝟐 𝟐 𝟐 𝟐𝒂𝒃 𝟐𝒃𝒄 𝟐𝒄𝒂

(LQQD)
41. Let 𝒂, 𝒃, 𝒄 be the side – lengths of a triangle. Prove that
𝒂𝒃𝒄 𝒃 + 𝒄 − 𝒂 𝒄 + 𝒂 − 𝒃 𝒂 + 𝒃 − 𝒄 ≥
≥ 𝒃𝟐 + 𝒄𝟐 − 𝒂𝟐 𝒄𝟐 + 𝒂𝟐 − 𝒃𝟐 𝒂𝟐 + 𝒃𝟐 − 𝒄𝟐 .
Proposed by Nguyen Viet Hung – Hanoi – Vietnam
Solution by Kevin Soto Palacios – Huarmey – Peru
Siendo 𝒂, 𝒃, 𝒄 los lados de un triángulo. Probar que:
𝒂𝒃𝒄 𝒃 + 𝒄 − 𝒂 𝒄 + 𝒂 − 𝒃 𝒂 + 𝒃 − 𝒄 ≥
≥ 𝒃𝟐 + 𝒄𝟐 − 𝒂𝟐 𝒄𝟐 + 𝒂𝟐 − 𝒃𝟐 𝒂𝟐 + 𝒃𝟐 − 𝒄𝟐 .
Recordar lo siguiente:
𝑨 𝑩 𝑪 𝒃+𝒄−𝒂 𝒄+𝒂−𝒃 𝒂+𝒃−𝒄
𝐬𝐢𝐧 𝐬𝐢𝐧 𝐬𝐢𝐧 =
𝟐 𝟐 𝟐 𝟖𝒂𝒃𝒄
Dividiendo ÷ 𝟖 𝒂𝒃𝒄 𝟐 a la desigualdad inicial se tiene:
𝒃+𝒄−𝒂 𝒄+𝒂−𝒃 𝒂+𝒃−𝒄

𝟖𝒂𝒃𝒄
𝒃 𝟐 + 𝒄𝟐 − 𝒂 𝟐 𝒄𝟐 + 𝒂𝟐 − 𝒃𝟐 𝒂 𝟐 + 𝒃𝟐 − 𝒄𝟐

𝟐𝒃𝒄 𝟐𝒄𝒂 𝟐𝒂𝒃
𝑨 𝑩 𝑪
⇒ 𝐬𝐢𝐧 𝐬𝐢𝐧 𝐬𝐢𝐧 ≥ 𝐜𝐨𝐬 𝑨 𝐜𝐨𝐬 𝑩 𝐜𝐨𝐬 𝑪
𝟐 𝟐 𝟐
1) Supongamos que sea un triángulo acutángulo:
𝐜𝐨𝐬 𝑨 + 𝑩 + 𝐜𝐨𝐬 𝑨 − 𝑩 𝟏 − 𝐜𝐨𝐬 𝑪 𝑪
𝐜𝐨𝐬 𝑨 𝐜𝐨𝐬 𝑩 = ≤ = 𝐬𝐢𝐧𝟐 ,
𝟐 𝟐 𝟐

55
SOCIETATEA DE ȘTIINȚE MATEMATICE DIN ROMÂNIA
ROMANIAN MATHEMATICAL SOCIETY

Filiala Mehedinți - Mehedinți Branch


www.ssmrmh.ro

𝑨 𝑩
𝐜𝐨𝐬 𝑩 𝐜𝐨𝐬 𝑪 ≤ 𝐬𝐢𝐧𝟐 ∧ 𝐜𝐨𝐬 𝑪 𝐜𝐨𝐬 𝑨 ≤ 𝐬𝐢𝐧𝟐
𝟐 𝟐
Luego, multiplicando:
𝑨 𝑩
𝟐
𝑪 𝟐
⇒ 𝐜𝐨𝐬 𝑨 𝐜𝐨𝐬 𝑩 𝐜𝐨𝐬 𝑪 ≤ 𝐬𝐢𝐧 𝐬𝐢𝐧 𝐬𝐢𝐧 ⇔
𝟐 𝟐 𝟐
𝑨 𝑩 𝑪
⇔ 𝐬𝐢𝐧 𝐬𝐢𝐧 𝐬𝐢𝐧 ≥ 𝐜𝐨𝐬 𝑨 𝐜𝐨𝐬 𝑩 𝐜𝐨𝐬 𝑪
𝟐 𝟐 𝟐
(LQQD)
2) Supongamos que sea un triángulo obtusángulo:
𝑪 ≥ 𝑩 ≥ 𝑨, 𝑪 ≥ 𝟗𝟎, 𝑨, 𝑩 ≤ 𝟗𝟎
𝑨 𝑩 𝑪
𝐜𝐨𝐬 𝑨 𝐜𝐨𝐬 𝑩 𝐜𝐨𝐬 𝑪 ≤ 𝟎 ∧ 𝐬𝐢𝐧 𝐬𝐢𝐧 𝐬𝐢𝐧 > 0 →
𝟐 𝟐 𝟐
𝑨 𝑩 𝑪
→ 𝐬𝐢𝐧 𝐬𝐢𝐧 𝐬𝐢𝐧 − 𝐜𝐨𝐬 𝑨 𝐜𝐨𝐬 𝑩 𝐜𝐨𝐬 𝑪 ≥ 𝟎
𝟐 𝟐 𝟐
(LQQD)
42. If 𝒂, 𝒃, 𝒄 > 0 , 𝒂𝟐 + 𝒃𝟐 + 𝒄𝟐 = 𝟑 then:

𝒂 𝒃𝟐 + 𝒄𝟐 ≤ 𝟔 𝒂𝒃 + 𝒃𝒄 + 𝒄𝒂

Proposed by Le Viet Hung - Quang Tri – Vietnam


Solution 1 by Hung Nguyen Viet – Hanoi – Vietnam
Solution 2 by Imad Zak – Saida – Lebanon

Solution 1 by Hung Nguyen Viet – Hanoi – Vietnam
Squaring both sides, the desired inequality becomes

𝒂 𝟐 𝒃 𝟐 + 𝒄𝟐 + 𝟐𝒂𝒃 𝒃𝟐 + 𝒄𝟐 𝒄𝟐 + 𝒂𝟐 ≤ 𝟔 𝒂𝒃 + 𝒃𝒄 + 𝒄𝒂
𝒄𝒚𝒄 𝒄𝒚𝒄

56
SOCIETATEA DE ȘTIINȚE MATEMATICE DIN ROMÂNIA
ROMANIAN MATHEMATICAL SOCIETY

Filiala Mehedinți - Mehedinți Branch


www.ssmrmh.ro

By AM-GM inequality we obtain

𝑳𝑯𝑺 ≤ 𝟐 𝒂𝟐 𝒃𝟐 + 𝒃𝟐 𝒄𝟐 + 𝒄𝟐 𝒂𝟐 + 𝒂𝒃 𝒂𝟐 + 𝒃𝟐 + 𝟐𝒄𝟐
𝒄𝒚𝒄

= 𝟐 𝒂 𝟐 𝒃 𝟐 + 𝒃𝟐 𝒄𝟐 + 𝒄𝟐 𝒂 𝟐 + 𝒂𝒃 𝟑 + 𝒄𝟐
𝒄𝒚𝒄

= 𝟐 𝒂𝟐 𝒃𝟐 + 𝒃𝟐 𝒄𝟐 + 𝒄𝟐 𝒂𝟐 + 𝟑 𝒂𝒃 + 𝒃𝒄 + 𝒄𝒂 + 𝒂𝒃𝒄 𝒂 + 𝒃 + 𝒄
It’s enough to show that
𝟐 𝒂𝟐 𝒃𝟐 + 𝒃𝟐 𝒄𝟐 + 𝒄𝟐 𝒂𝟐 + 𝒂𝒃𝒄 𝒂 + 𝒃 + 𝒄 ≤ 𝟑 𝒂𝒃 + 𝒃𝒄 + 𝒄𝒂 .
Indeed, we have
𝟑 𝒂𝒃 + 𝒃𝒄 + 𝒄𝒂 = 𝒂𝟐 + 𝒃𝟐 + 𝒄𝟐 𝒂𝒃 + 𝒃𝒄 + 𝒄𝒂
= 𝒂𝒃 𝒂𝟐 + 𝒃𝟐 + 𝒃𝒄 𝒃𝟐 + 𝒄𝟐 + 𝒄𝒂 𝒄𝟐 + 𝒂𝟐 + 𝒂𝒃𝒄 𝒂 + 𝒃 + 𝒄
≥ 𝟐 𝒂𝟐 𝒃𝟐 + 𝒃𝟐 𝒄𝟐 + 𝒄𝟐 𝒂𝟐 + 𝒂𝒃𝒄 𝒂 + 𝒃 + 𝒄
and we are done.
Solution 2 by Imad Zak – Saida – Lebanon
𝒂, 𝒃, 𝒄 > 0, 𝒂𝟐 = 𝟑 Prove that

𝒂 𝒂𝟐 + 𝒃𝟐 ≤ 𝟔 𝒂𝒃 ... (E)
First let’s prove
𝟐 𝒂𝟐 𝒂𝒃 ≥ 𝒂 𝒂𝒃 𝒂 + 𝒃 = 𝒂 𝒂 𝒂𝒃 − 𝟑𝒂𝒃𝒄 ... (F)
𝑳𝑯𝑺 − 𝑹𝑯𝑺 = 𝒂𝟑 𝒃 + 𝒂𝒃𝟑 − 𝟐𝒂𝟐 𝒃𝟐 + 𝒂𝟑 𝒄 + 𝒂𝒄𝟑 − 𝟐𝒂𝟐 𝒄𝟐 +
𝑨𝑴7𝑮𝑴
𝟑 𝟑 𝟐 𝟐
+ 𝒃 𝒄 + 𝒃𝒄 − 𝟐𝒃 𝒄 ≥ 𝟐𝒂𝟐 𝒃𝟐 − 𝟐𝒂𝟐 𝒃𝟐 = 𝟎

(F) is proved.
Now by weighted Jensen’s of weights 𝐚, 𝐛, 𝐜 on the concave function
𝒇 𝒙 = 𝒙, we have

57
SOCIETATEA DE ȘTIINȚE MATEMATICE DIN ROMÂNIA
ROMANIAN MATHEMATICAL SOCIETY

Filiala Mehedinți - Mehedinți Branch


www.ssmrmh.ro

𝒂 𝒄𝟐 + 𝒃𝟐 + 𝒃 𝒂 𝟐 + 𝒄𝟐 + 𝒄 𝒂 𝟐 + 𝒃𝟐
𝒂 𝒂𝟐 + 𝒃𝟐 ≤ 𝒂+𝒃+𝒄 𝒇 =
𝒂+𝒃+𝒄

= 𝒂 𝒂𝒃 𝒂 + 𝒃

according to (F).

𝒂 𝒂𝟐 + 𝒃𝟐 ≤ 𝟐 𝒂𝟐 𝒂𝒃 = 𝟔⋅ 𝒂𝒃

Q.E.D.
≪=≫ at 𝒂, 𝒃, 𝒄 = 𝟏, 𝟏, 𝟏

43. If 𝒂, 𝒃, 𝒄 ≥ 𝟏, 𝒂𝒃 + 𝒃𝒄 + 𝒄𝒂 = 𝟗 then:
𝒂𝟐 𝒃𝟐 𝒄𝟐
𝟐 𝟐 𝒂𝒃𝒄 𝟑
𝒂𝒂 ⋅ 𝒃 𝒃 ⋅ 𝒄 𝒄 𝟐 ≥
𝟑
Proposed by Daniel Sitaru – Romania
Solution 1 by Soumava Chakraborty – Kolkata – India
Solution 2 by Soumitra Mandal – Chandar Nagore – India
Solution 3 by Myagmarsuren Yadamsuren – Mongolia
Solution 1 by Soumava Chakraborty – Kolkata – India
Weighted GM-HM inequality ⇒
𝟏
𝒂𝟐 𝒃𝟐 𝒄𝟐 𝒂𝟐
𝒂𝟐 𝒂𝟐 𝒃𝟐 𝒄𝟐
𝒂 ⋅𝒃 ⋅𝒄 ≥ 𝟐 = + +
𝒂 𝒃𝟐 𝒄𝟐 𝒂 𝒂 ∑𝒂
+ +
𝒂 𝒃 𝒄
𝑩𝒆𝒓𝒈𝒔𝒕𝒓𝒐𝒎 𝒂 𝟐 𝒂 𝑨7𝑮 𝟑
≥ = ≥ 𝒂𝒃𝒄
𝟑 𝒂 𝟑
𝟑
𝒂𝟐 𝑨7𝑮 𝟑 𝒂𝟐 𝒃𝟐 𝒄𝟐
𝒂𝟐 𝒃𝟐 𝒄𝟐 𝟑 𝟑
∴𝒂 ⋅𝒃 ⋅𝒄 ≥ 𝒂𝒃𝒄 ≥ 𝒂𝒃𝒄

58
SOCIETATEA DE ȘTIINȚE MATEMATICE DIN ROMÂNIA
ROMANIAN MATHEMATICAL SOCIETY

Filiala Mehedinți - Mehedinți Branch


www.ssmrmh.ro

𝟑 𝟑
(∵ 𝒂𝒃𝒄 ≥ 𝟏, 𝒂𝟐 > 1, 3 𝒂𝟐 𝒃𝟐 𝒄𝟐 > 1, as 𝒂, 𝒃, 𝒄 ≥ 𝟏)
∴ it suffices to prove:
𝒂𝟐 𝒃𝟐 𝒄𝟐
𝟑
𝟑 𝟑 𝒂𝟐 𝒃𝟐 𝒄𝟐 𝒂𝒃𝒄 𝟑
𝒂𝒃𝒄 ≥
𝟑
𝟐 𝟑
𝒙𝟔 𝒙𝟑

𝟑𝒙𝟐
𝒙𝟑 𝟑 𝒙𝟐
𝟑 𝒙𝟑 𝟑
𝟑
⇔𝒙 ≥ ⇔ 𝒙 ≥ 𝒙 = 𝒂𝒃𝒄
𝟑 𝟑

𝟐
𝒙𝟑
𝟐
𝒙𝟐 𝒙𝟑 𝟑 𝟐 𝒙𝟑 𝒙𝟑
⇔𝒙 ≥ ⇔ 𝒙 𝐥𝐧 𝒙 ≥ 𝐥𝐧 (1)
𝟑 𝟑 𝟑

Case (1)
𝟏 𝒙𝟑 𝟑 𝒙𝟑 𝟏
≤ < 1 𝒙 = 𝒂𝒃𝒄 ≥ 𝟏 ⇒ ≥
𝟑 𝟑 𝟑 𝟑
∵ 𝒙 ≥ 𝟏, ∴ 𝐥𝐧 𝒙 ≥ 𝟎 ⇒ 𝑳𝑯𝑺 𝒐𝒇 𝟏 ≥ 𝟎
𝒙𝟑 𝒙𝟑 ⇒ 𝑳𝑯𝑺 > 𝑅𝐻𝑆 ⇒ (1) is true
∵ < 1, ∴ 𝐥𝐧 < 0 ⇒ 𝑹𝑯𝑺 𝒐𝒇 𝟏 < 0
𝟑 𝟑

Case 2
𝒙𝟑
≥ 𝟏
𝟑
𝑨7𝑮 𝟑
Now, 𝒂𝒃 + 𝒃𝒄 + 𝒄𝒂 ≥ 𝟑 𝒂𝟐 𝒃𝟐 𝒄𝟐
𝒙𝟐 𝒙𝟑
𝟐
⇒𝟑≥𝒙 ⇒𝟏≥ ⇒𝒙≥
𝟑 𝟑
𝟐 𝟐
𝟐 𝒙𝟑 𝒙𝟑 𝟐 𝒙𝟑 𝒙𝟑
∴𝒙 ≥ and also, 𝐥𝐧 𝒙 ≥ 𝐥𝐧 ⇒ 𝒙 𝐥𝐧 𝒙 ≥ 𝐥𝐧
𝟑 𝟑 𝟑 𝟑
𝟐
𝟐 𝒙𝟑 𝒙𝟑
(∵ 𝒙 , 𝐥𝐧 𝒙 , > 0 and 𝐥𝐧 ≥ 𝟎)⇒ (1) is true (Proved)
𝟑 𝟑

59
SOCIETATEA DE ȘTIINȚE MATEMATICE DIN ROMÂNIA
ROMANIAN MATHEMATICAL SOCIETY

Filiala Mehedinți - Mehedinți Branch


www.ssmrmh.ro

(Equality when 𝒂 = 𝒃 = 𝒄 = 𝟑, i.e., at 𝒙 = 𝟑)



Solution 2 by Soumitra Mandal – Kolkata – India
Let 𝑓 𝑥 = 𝑥 ¥ log 𝑥 for all 𝑥 ∈ 1, ∞
𝒇• 𝒙 = 𝟐𝒙 𝐥𝐨𝐠 𝒙 + 𝒙 > 0 for all 𝒙 ∈ 𝟏, ∞
𝒇•• 𝒙 = 𝟐 𝐥𝐨𝐠 𝒙 + 𝟑 > 0
Applying Jensen’s Inequality
𝟐 𝟐
𝒄𝒚𝒄 𝒂 𝐥𝐨𝐠 𝒂 𝒂+𝒃+𝒄 𝒂+𝒃+𝒄
≥ 𝐥𝐨𝐠
𝟑 𝟑 𝟑
now 𝒂 + 𝒃 + 𝒄 𝒂𝒃 + 𝒃𝒄 + 𝒄𝒂 ≥ 𝟗𝒂𝒃𝒄 ⇒ 𝒂 + 𝒃 + 𝒄 ≥ 𝒂𝒃𝒄
𝒂.𝒃.𝒄 𝟐 𝒂𝒃𝒄 𝟐
𝟐 𝟐 𝟐 𝒂𝒃𝒄 𝟑 𝟐 𝟐 𝟐 𝒂𝒃𝒄 𝟑
∴ 𝐥𝐨𝐠 𝒂𝒂 𝒃𝒃 𝒄𝒄 ≥ 𝐥𝐨𝐠 ⇒ 𝒂𝒂 ⋅ 𝒃𝒃 ⋅ 𝒄𝒄 ≥
𝟑 𝟑
(proved)
Solution 3 by Myagmarsuren Yadamsuren – Mongolia
𝒂𝟐 ⋅𝒃𝟐 ⋅𝒄𝟐 𝒂𝒃𝒄
1) 𝒂𝟐 𝐥𝐧 𝒂 + 𝒃𝟐 𝐥𝐧 𝒃 + 𝒄𝟐 𝐥𝐧 𝒄 ≥ 𝐥𝐧
𝟑 𝟑

𝑳𝑯𝑺 ⇒ 𝒇 𝒙 = 𝒙𝟐 ⋅ 𝐥𝐧 𝒙 ⇒ 𝒇•• 𝒙 ≥ 𝟎
𝒂𝟐 .𝒃𝟐 .𝒄𝟐 𝒂.𝒃.𝒄
JENSEN: 𝒂𝟐 𝐥𝐧 𝒂 + 𝒃𝟐 𝐥𝐧 𝒃 + 𝒄𝟐 𝐥𝐧 𝒄 ≥ 𝟑 ⋅ 𝐥𝐧
𝟑 𝟑

2) 𝑹𝑯𝑺: 𝒂 + 𝒃 + 𝒄 ⋅ 𝒂𝒃 + 𝒃𝒄 + 𝒄𝒂 ≥ 𝟗𝒂𝒃𝒄 (ASSURE)


𝑪𝒂𝒖𝒄𝒉𝒚 𝑪𝒂𝒖𝒄𝒉𝒚
𝟑
𝒂.𝒃.𝒄ª𝟑 𝒂𝒃𝒄
𝟑 ⇒ 𝒂.𝒃.𝒄 ⋅ 𝒂𝒃.𝒃𝒄.𝒄𝒂 ª𝟗𝒂𝒃𝒄
𝒂𝒃.𝒃𝒄.𝒄𝒂ª𝟑 𝒂𝒃𝒄 𝟐

𝒂 + 𝒃 + 𝒄 ⋅ 𝒂𝒃 + 𝒃𝒄 + 𝒄𝒂 ≥ 𝟗𝒂𝒃𝒄
𝟗 𝒂 + 𝒃 + 𝒄 ≥ 𝟗𝒂𝒃𝒄 ⇒ 𝒂 + 𝒃 + 𝒄 ≥ 𝒂𝒃𝒄 (*)
𝟐
(*) ⇒ 𝒂 + 𝒃 + 𝒄 ≥ 𝒂𝟐 𝒃𝟐 𝒄𝟐

60
SOCIETATEA DE ȘTIINȚE MATEMATICE DIN ROMÂNIA
ROMANIAN MATHEMATICAL SOCIETY

Filiala Mehedinți - Mehedinți Branch


www.ssmrmh.ro

𝒂𝟐 + 𝒃𝟐 + 𝒄𝟐 + 𝟐 𝒂𝒃 + 𝒃𝒄 + 𝒄𝒂 ≥ 𝒂𝟐 𝒃𝟐 𝒄𝟐

𝑪𝒂𝒖𝒄𝒉𝒚

𝒂𝟐 + 𝒃𝟐 + 𝒄𝟐 + 𝟐 𝒂𝒃 + 𝒃𝒄 + 𝒄𝒂 ≤ 𝟑 ⋅ 𝒂𝟐 + 𝒃𝟐 + 𝒄𝟐
𝟑 𝒂𝟐 + 𝒃𝟐 + 𝒄𝟐 ≥ 𝒂𝟐 𝒃𝟐 𝒄𝟐 (**)
∗ , ∗∗
𝒂𝟐 𝒃𝟐 𝒄𝟐 𝒂𝒃𝒄 𝟑⋅ 𝒂𝟐 .𝒃𝟐 .𝒄𝟐 𝒂.𝒃.𝒄
(*), (**) ⇒ 𝑹𝑯𝑺: ⋅ 𝐥𝐧 ≤ ⋅ 𝐥𝐧 = 𝑳𝑯𝑺
𝟑 𝟑 𝟑 𝟑


44. Prove the inequality holds for all positive real numbers 𝒂, 𝒃, 𝒄
𝟏 𝟏
+ +
𝒂𝟐 − 𝒂𝒃 + 𝒃𝟐 𝒃𝟐 − 𝒃𝒄 + 𝒄𝟐 𝒃𝟐 − 𝒃𝒄 + 𝒄𝟐 𝒄𝟐 − 𝒄𝒂 + 𝒂𝟐
𝟏 𝟏 𝟏 𝟏
+ ≤ + +
𝒄𝟐 − 𝒄𝒂 + 𝒂𝟐 𝒂𝟐 − 𝒂𝒃 + 𝒃𝟐 𝒂𝟒 𝒃𝟒 𝒄𝟒
Proposed by Nguyen Viet Hung – Hanoi – Vietnam
Solution by Kevin Soto Palacios – Huarmey – Peru
Probar para todos los números 𝑹. : 𝒂, 𝒃, 𝒄
𝟏 𝟏
+ +
𝒂𝟐 − 𝒂𝒃 + 𝒃𝟐 𝒃𝟐 − 𝒃𝒄 + 𝒄𝟐 𝒃𝟐 − 𝒃𝒄 + 𝒄𝟐 𝒄𝟐 − 𝒄𝒂 + 𝒂𝟐
𝟏 𝟏 𝟏 𝟏
+ ≤ + +
𝒄𝟐 − 𝒄𝒂 + 𝒂𝟐 𝒂𝟐 − 𝒂𝒃 + 𝒃𝟐 𝒂𝟒 𝒃𝟒 𝒄𝟒
Ahora bien ∀: 𝒙, 𝒚, 𝒛 ∈ ℝ, se cumple la siguiente desigualdad:
𝟏 𝟏 𝟏 𝟏 𝟏 𝟏
+ + ≤ + + → Siendo:
𝒙𝒚 𝒚𝒛 𝒛𝒙 𝒙𝟐 𝒚𝟐 𝒛𝟐

𝒙 = 𝒂𝟐 − 𝒂𝒃 + 𝒃𝟐 ,
𝒚 = 𝒃𝟐 − 𝒃𝒄 + 𝒄𝟐 ,
𝒛 = 𝒄𝟐 − 𝒄𝒂 + 𝒄𝟐
Por lo cual solo hace falta demostrar lo siguiente:

61
SOCIETATEA DE ȘTIINȚE MATEMATICE DIN ROMÂNIA
ROMANIAN MATHEMATICAL SOCIETY

Filiala Mehedinți - Mehedinți Branch


www.ssmrmh.ro

𝟏 𝟏 𝟏
+ + ≤
𝒂𝟐 − 𝒂𝒃 + 𝒃𝟐 𝟐 𝒃𝟐 − 𝒃𝒄 + 𝒄𝟐 𝟐 𝒄𝟐 − 𝒄𝒂 + 𝒂𝟐 𝟐

𝟐 𝟐 𝟐 𝟏 𝟏 𝟏
≤ + + ≤ + +
𝒂 𝟒 + 𝒃𝟒 𝒃𝟒 + 𝒄𝟒 𝒄𝟒 + 𝒂 𝟒 𝒂𝟒 𝒃𝟒 𝒄𝟒
Lo cual es cierto ya que:
𝟒
𝒂−𝒃 ≥ 𝟎 → 𝒂𝟒 + 𝒃𝟒 + 𝟔𝒂𝟐 𝒃𝟐 − 𝟒𝒂𝟑 𝒃 − 𝟒𝒃𝟑 𝒂 ≥ 𝟎
⇒ 𝟐 𝒂𝟒 + 𝒃𝟒 + 𝒂𝟐 𝒃𝟐 − 𝟐𝒂𝟑 𝒃 − 𝟐𝒃𝟑 𝒂 + 𝟐𝒂𝟐 𝒃𝟐 ≥ 𝒂𝟒 + 𝒃𝟒 →
→ 𝟐 𝒂𝟐 − 𝒂𝒃 + 𝒃𝟐 𝟐
≥ 𝒂𝟒 + 𝒃𝟒
Además por desigualdad de Cauchy: 𝒙, 𝒚, 𝒛 > 0, se cumple lo siguiente:
𝟏 𝟏 𝟏 𝟐 𝟐 𝟐
+ + ≥ + +
𝒙 𝒚 𝒛 𝒙+𝒚 𝒚+𝒛 𝒛+𝒙

45. Prove the inequality holds for all positive real numbers 𝒂, 𝒃, 𝒄
𝒂𝟐 𝒃𝟐 𝒃 𝟐 𝒄𝟐
+ 𝟐 +
𝒃𝟐 − 𝒃𝒄 + 𝒄𝟐 𝒄𝟐 − 𝒄𝒂 + 𝒂𝟐 𝒄 − 𝒄𝒂 + 𝒂𝟐 𝒂𝟐 − 𝒂𝒃 + 𝒃𝟐
𝒄𝟐 𝒂𝟐
+ 𝟐 ≤ 𝟑
𝒂 − 𝒂𝒃 + 𝒃𝟐 𝒃𝟐 − 𝒃𝒄 + 𝒄𝟐
Proposed by Hung Nguyen Viet-Hanoi-Vietnam
Solution by Le Viet Hung-Vietnam
Using AM-GM inequality:

𝟐 𝟐
𝟏 𝒂𝟐 − 𝒂𝒃 𝟐 + 𝒃𝟐 𝒂𝟐 + 𝒂𝒃 𝟐 + 𝒃𝟐
𝒂 − 𝒂𝒃 + 𝒃 = + ≥
𝟐 𝟐 𝟐−𝟏 𝟐+𝟏

𝟏 𝒂𝟒 + 𝒃𝟒
≥ 𝟐⋅ 𝒂𝟐 + 𝒃𝟐 − 𝟐𝒂𝟐 𝒃𝟐 =
𝟐 𝟐 𝟐

Similarly, we have:

62
SOCIETATEA DE ȘTIINȚE MATEMATICE DIN ROMÂNIA
ROMANIAN MATHEMATICAL SOCIETY

Filiala Mehedinți - Mehedinți Branch


www.ssmrmh.ro

𝒃𝟒 + 𝒄𝟒
𝒃𝟐 − 𝒃𝒄 + 𝒄𝟐 ≥
𝟐

𝟐 𝟐
𝒄𝟒 + 𝒂𝟒
𝒄 − 𝒄𝒂 + 𝒂 ≥
𝟐

𝒂𝟐 𝒃𝟐 𝟐𝒂𝟐 𝒃𝟐
⇒ ≤ =
𝒃𝟐 − 𝒃𝒄 + 𝒄𝟐 𝒄𝟐 − 𝒄𝒂 + 𝒂𝟐 𝒃 𝟒 + 𝒄𝟒 𝒄𝟒 + 𝒂 𝟒

𝒂𝟒 𝒃𝟒
= 𝟐 𝟒 ⋅
𝒄 + 𝒂 𝟒 𝒃𝟒 + 𝒄𝟒

and

𝒂𝟒 𝒃𝟒 𝒂𝟒 𝒃𝟒
𝟐 𝟒 ⋅ ≤ + = 𝟑
𝒄 + 𝒂 𝟒 𝒃𝟒 + 𝒄𝟒 𝒄𝟒 + 𝒂 𝟒 𝒃𝟒 + 𝒄𝟒

Inequality holds: 𝒂 = 𝒃 = 𝒄
46. If 𝒙, 𝒚, 𝒛 > 0 then:
𝟐
𝒙𝟐 𝒚𝟐 𝒛𝟐 𝒙 𝒚 𝒛 𝒙𝟑 𝒚𝟑 𝒛𝟑
𝟗 𝟐+ 𝟐+ 𝟐 >8 + + + + −𝟑
𝒚 𝒛 𝒙 𝒚 𝒛 𝒙 𝒚𝟑 𝒛𝟑 𝒙𝟑
Proposed by Daniel Sitaru – Romania
Solution 1 by Saptak Bhattacharya-Kolkata-India
Solution 2 by Nguyen Minh Triet-Quang Ngai-Vietnam

Solution 1 by Saptak Bhatacharya-Kolkata-India
𝒙 𝒚 𝒛
Put = 𝒂, = 𝒃, = 𝒄
𝒚 𝒛 𝒙

So, to show,

63
SOCIETATEA DE ȘTIINȚE MATEMATICE DIN ROMÂNIA
ROMANIAN MATHEMATICAL SOCIETY

Filiala Mehedinți - Mehedinți Branch


www.ssmrmh.ro

𝟗 𝒂 𝟐 + 𝒃𝟐 + 𝒄𝟐 𝟐
> 8 𝒂 + 𝒃 + 𝒄 𝒂𝟑 + 𝒃𝟑 + 𝒄𝟑 − 𝟑𝒂𝒃𝒄
Using 𝒂𝟑 + 𝒃𝟑 + 𝒄𝟑 − 𝟑𝒂𝒃𝒄 = 𝒂 + 𝒃 + 𝒄 𝒂𝟐 + 𝒃𝟐 + 𝒄𝟐 − 𝒂𝒃 − 𝒃𝒄 − 𝒄𝒂
And rearranging
𝒂 𝟐 + 𝒃𝟐 + 𝒄𝟐 𝟐
− 𝟖 𝒂𝒃 + 𝒃𝒄 + 𝒄𝒂 𝒂𝟐 + 𝒃𝟐 + 𝒄𝟐 + 𝟏𝟔 𝒂𝒃 + 𝒃𝒄 + 𝒄𝒂 𝟐
> 0
⇒ 𝒂𝟐 + 𝒃𝟐 + 𝒄𝟐 − 𝟒𝒂𝒃 − 𝟒𝒃𝒄 − 𝟒𝒄𝒂 𝟐
> 0
which is true. (Proved)

Solution 2 by Nguyen Minh Triet-Quang Ngai-Vietnam
𝒙 𝒚 𝒛
Let 𝒂 = ; 𝒃 = , 𝒄 = then abc=1
𝒚 𝟐 𝒙

The inequality becomes


𝟗 𝒂 𝟐 + 𝒃𝟐 + 𝒄𝟐 𝟐
≥ 𝟖 𝒂 + 𝒃 + 𝒄 𝒂𝟑 + 𝒃𝟑 + 𝒄𝟑 − 𝟑𝒂𝒃𝒄
⇔ 𝟗 𝒂 𝟐 + 𝒃𝟐 + 𝒄𝟐 ≥ 𝟖 𝒂 + 𝒃 + 𝒄 𝟐
𝒂𝟐 + 𝒃𝟐 + 𝒄𝟐 − 𝒂𝒃 − 𝒃𝒄 − 𝒄𝒂
⇔ 𝟗 𝒂 𝟐 + 𝒃𝟐 + 𝒄𝟐 𝟐

≥ 𝟖 𝒂𝟐 + 𝒃𝟐 + 𝒄𝟐 + 𝟐𝒂𝒃 + 𝟐𝒃𝒄 + 𝟐𝒄𝒂 𝒂𝟐 + 𝒃𝟐 + 𝒄𝟐 − 𝒂𝒃 − 𝒃𝒄 − 𝒄𝒂
⇔ 𝟗𝒕𝟐 ≥ 𝟖 𝒕 + 𝟐𝟗 𝒕 − 𝟗
𝟐 𝟐 𝟐
(where 𝒕 = 𝒂 + 𝒃 + 𝒄 ≥ 𝒂𝒃 + 𝒃𝒄 + 𝒄𝒂 )
𝟗 = 𝒂𝒃 + 𝒃𝒄 = 𝒄𝒂
⇔ 𝟗𝒕𝟐 ≥ 𝟖𝒕𝟐 + 𝟖𝟗𝒕 − 𝟏𝟔𝟗𝟐
𝟐
⇔ 𝒕 − 𝟒𝟗 ≥ 𝟎 True ⇒ q.e.d.
𝒕 = 𝟒𝟗
The equality holds at

𝒂𝒃𝒄 = 𝟏
𝟒 𝟐
𝒂= 𝒃= 𝒄
𝟐 𝟐 𝟐 𝟐𝟓 + 𝟏𝟎 𝟐𝟏 𝟓 + 𝟐𝟏
⇔ 𝒂 + 𝒃 + 𝒄 = 𝟒𝒂𝒃 + 𝒃𝒄 + 𝒄𝒂 ⇔
𝒂𝒃𝒄 = 𝟏 𝟒 𝟐
𝒂= 𝒃= 𝒄
𝟐𝟓 − 𝟏𝟎 𝟐𝟏 𝟓 − 𝟐𝟏

64
SOCIETATEA DE ȘTIINȚE MATEMATICE DIN ROMÂNIA
ROMANIAN MATHEMATICAL SOCIETY

Filiala Mehedinți - Mehedinți Branch


www.ssmrmh.ro

𝒙 𝟒 𝒚 𝟐 𝒛
⇔ = ⋅ = ⋅
𝒚 𝟐𝟓 ± 𝟏𝟎 𝟐𝟏 𝟐 𝟓 ± 𝟐𝟏 𝒙
𝟓 ± 𝟐𝟏 𝟓 ± 𝟐𝟏
⇔𝒚= 𝒙= 𝒛
𝟐 𝟐
And permutations.


47. If 𝒂, 𝒃, 𝒄 > 0, 𝒂𝟐 + 𝒃𝟐 + 𝒄𝟐 = 𝟑 then:
𝟏 𝟏 𝟏 𝟗 𝟏 𝟏 𝟏
𝟑
+ 𝟑
+ 𝟑
≥ −𝟒 𝟒
+ 𝟒
+ 𝟒

𝒂+𝟏 𝒃+𝟏 𝒄+𝟏 𝟖 𝒂+𝟏 𝒃+𝟏 𝒄+𝟏
Proposed by Daniel Sitaru – Romania
Solution 1 by Leonard Giugiuc – Romania
Solution 2 by Soumava Chakraborty – Kolkata – India
Solution 3 by Saptak Bhattacharya – Kolkata-India
Solution 4 by Soumitra Mandal – Chandar Nagore – India
Solution 1 by Leonard Giugiuc – Romania
𝟏
By AM-QM, 𝒂 + 𝒃 + 𝒄 ≤ 𝟑. The functions
𝒙.𝟏 𝟑
𝟏
and are convex on 𝟎, ∞ .
𝒙.𝟏 𝟒

By Jensen,
𝟏 𝟑 𝟑 𝟑
𝟑
≥ 𝟑
≥ 𝟑
=
𝒂+𝟏 𝒂+𝒃+𝒄 𝟏+𝟏 𝟖
𝒄𝒚𝒄 +𝟏
𝟑
and
𝟏 𝟏𝟐 𝟏𝟐 𝟑
𝟒 𝟒
≥ 𝟒
≥ 𝟑
=
𝒂+𝟏 𝒂+𝒃+𝒄 𝟏+𝟏 𝟒
𝒄𝒚𝒄 +𝟏
𝟑

65
SOCIETATEA DE ȘTIINȚE MATEMATICE DIN ROMÂNIA
ROMANIAN MATHEMATICAL SOCIETY

Filiala Mehedinți - Mehedinți Branch


www.ssmrmh.ro

Solution 2 by Soumava Chakraborty – Kolkata – India


𝟏 𝟗 𝟏
𝒂, 𝒃, 𝒄 > 0, 𝒂𝟐 + 𝒃𝟐 + 𝒄𝟐 = 𝟑 ⇒ 𝟑
≥ −𝟒
𝒂+𝟏 𝟖 𝒂+𝟏 𝟒
𝟏 𝟒 𝟏.𝟐 𝟐
𝟑
+ ≥ (Bergstrom’s inequality)
𝒂.𝟏 𝒂.𝟏 𝟒 𝒂.𝟏 𝟑 . 𝒂.𝟏 𝟒

𝟗
=
𝒂+𝟏 𝟑+ 𝒂+𝟏 𝟒
𝟏 𝟒 𝟗
∴ 𝟑
+ 𝟒

𝒂+𝟏 𝒂+𝟏 𝒂+𝟏 𝟑+ 𝒂+𝟏 𝟒
𝟏 𝟏
∴ it suffices to prove: 𝟑. 𝟒
≥ (1)
𝒂.𝟏 𝒂.𝟏 𝟖
𝟏
Now, let 𝒇 𝒙 = 𝟑.
𝒙>0
𝒙.𝟏 𝒙.𝟏 𝟒

𝟐 𝟏𝟎𝒙𝟐 + 𝟑𝟓𝒙 + 𝟑𝟏
𝒇•• 𝒙 = > 0, ∵ 𝒙 > 0
𝒙+𝟏 𝟓 𝒙+𝟐 𝟑
∴ applying Jensen’s inequality
𝟏 𝒂+𝒃+𝒄
𝒇 𝒂 ≥𝒇
𝟑 𝟑
𝟏 𝟑
⇒ ≥ 𝟑 𝟒

𝒂+𝟏 𝟑+ 𝒂+𝟏 𝟒
𝒂+𝒃+𝒄 𝒂+𝒃+𝒄
+𝟏 + +𝟏
𝟑 𝟑
𝟑𝟓
=
𝟑 𝒂+𝒃+𝒄+𝟑 𝟑+ 𝒂+𝒃+𝒄+𝟑 𝟒
𝟐
Now, 𝒂 ≤𝟑 𝒂𝟐 = 𝟗 ⇒ 𝒂 ≤ 𝟑
𝟏 𝟑𝟓 𝟑𝟑 𝟏
∴ 𝟑. 𝟒
≥ 𝟑. 𝟒
= = ⇒ (1) is true
𝒂.𝟏 𝒂.𝟏 𝟑 𝟑.𝟑 𝟑.𝟑 𝟔𝟑 𝟖

(Proved)
Solution 3 by Saptak Bhattacharya – Kolkata-India

66
SOCIETATEA DE ȘTIINȚE MATEMATICE DIN ROMÂNIA
ROMANIAN MATHEMATICAL SOCIETY

Filiala Mehedinți - Mehedinți Branch


www.ssmrmh.ro

𝒂 𝒂𝟐
≤ ⇒ 𝒂≤𝟑⇒ 𝒂 + 𝟏 ≤ 𝟔;
𝟑 𝟑
𝟑 𝟖𝟏 𝟖𝟏 𝟑
Let 𝒂 + 𝟏 = 𝒙, 𝒙≤𝟔⇒ 𝒙 ≤𝟖⋅ ⇒ 𝟑

𝟑 𝒙 𝟖

𝟏 𝟐 𝟏 𝟐 𝟏 𝟐
𝟑 𝟐 𝟗 𝟒 𝟏 𝟗
𝒙 𝒙 𝒙
⇒ ≥ (AM ≥ HM)⇒ + ≥ ⇒ + ≥
𝒙 𝟖 𝒙 𝒙 𝟖 𝒙𝟒 𝒙𝟑 𝟖

[∵ 𝒙𝟐 = 𝟐 𝒙; then Titu’s lemma]


𝟏 𝟗 𝟏 𝟏 𝟗 𝟏
⇒ ≥ −𝟒 ⇒ ≥ −𝟒
𝒙𝟑 𝟖 𝒙𝟒 𝒂+𝟏 𝟑 𝟖 𝒂+𝟏 𝟒
(Proved)
Solution 4 by Soumitra Mandal – Chandar Nagore – India

𝟑 𝒂𝟐 + 𝒃𝟐 + 𝒄𝟐 ≥ 𝒂 + 𝒃 + 𝒄 ⇒ 𝟑 ≥ 𝒂 + 𝒃 + 𝒄
we know,
7𝟑
𝟏 7𝟑
𝒂+𝟏+𝒃+𝟏+𝒄+𝟏 𝟐𝟕 𝟐𝟕
𝒂+𝟏 ≥ = 𝟑

𝟑 𝟑 𝒂+𝒃+𝒄+𝟑 𝟐𝟏𝟔
𝒄𝒚𝒄

𝟏 𝟑
∴ 𝟑

𝒂+𝟏 𝟖
𝒄𝒚𝒄

Similarly,
7𝟒
𝟏 7𝟒
𝒂+𝒃+𝒄+𝟑 𝟖𝟏 𝟖𝟏
𝒂+𝟏 ≥ = 𝟒

𝟑 𝟑 𝒂+𝒃+𝒄+𝟑 𝟏𝟐𝟗𝟔
𝒄𝒚𝒄

𝟏 𝟑
∴𝟒 𝟒

𝒂+𝟏 𝟒
𝒄𝒚𝒄

𝟏 𝟏 𝟗 𝟏 𝟗 𝟏
∴ 𝟑
+𝟒 𝟒
≥ ⇒ 𝟑
≥ −𝟒
𝒂+𝟏 𝒂+𝟏 𝟖 𝒂+𝟏 𝟖 𝒂+𝟏 𝟒
𝒄𝒚𝒄 𝒄𝒚𝒄 𝒄𝒚𝒄 𝒄𝒚𝒄

67
SOCIETATEA DE ȘTIINȚE MATEMATICE DIN ROMÂNIA
ROMANIAN MATHEMATICAL SOCIETY

Filiala Mehedinți - Mehedinți Branch


www.ssmrmh.ro

(Proved)
48. If 𝒙, 𝒚, 𝒛 > 0 then:
𝒙𝒚
𝟒 𝒙𝟐 + 𝒚𝟐 𝒛 + 𝟒𝒙𝒚𝒛 𝟐
≥ 𝟐𝟕𝒙𝒚𝒛
𝒙+𝒚
Proposed by Daniel Sitaru – Romania
Solution 1 by Mihalcea Andrei Stefan-Romania
Solution 2 by Soumitra Mandal-Chandar Nagore-India
Solution 3 by Redwane El Mellass-Morroco

Solution 1 by Mihalcea Andrei Stefan-Romania
𝒙𝒚
𝟒 𝒙𝟐 + 𝒚𝟐 𝒛 + 𝟒𝒙𝒚𝒛 𝟐
≥ 𝟐𝟕𝒙𝒚𝒛|: 𝒙𝒚𝒛 > 0
𝒙+𝒚
𝒙 𝒚 𝟏
𝟒 + +𝟒 𝒙 𝒚 ≥ 𝟐𝟕
𝒚 𝒙 + +𝟐
𝒚 𝒙
𝒙 𝒚 𝟒
We’ll prove: 𝟒 + +𝒙 𝒚 ≥ 𝟗
𝒚 𝒙 . .𝟐
𝒚 𝒙

𝒙 𝒚 𝒏𝒐𝒕
+ = 𝜶 ≥ 𝟐
𝒚 𝒙
𝟒𝜶𝟐 − 𝜶 − 𝟏𝟒 ≥ 𝟎 ⇔ 𝜶 − 𝟐 𝟒𝜶 + 𝟕 ≥ 𝟎, true (𝜶 ≥ 𝟐)
Solution 2 by Soumitra Mandal-Chandar Nagore-India
𝒙𝒚
𝟒 𝒛 𝒙𝟐 + 𝒚𝟐 + 𝟒𝒙𝒚𝒛 𝟐
≥ 𝟐𝟕𝒙𝒚𝒛
𝒙+𝒚
𝒄𝒚𝒄 𝒄𝒚𝒄

𝟒𝒙𝒚
⇔𝟒 𝒛 𝒙𝟐 + 𝒚𝟐 − 𝟔𝒙𝒚𝒛 ≥ 𝒙𝒚𝒛 𝟑 − 𝟐

𝒙+𝒚
𝒄𝒚𝒄 𝒄𝒚𝒄

68
SOCIETATEA DE ȘTIINȚE MATEMATICE DIN ROMÂNIA
ROMANIAN MATHEMATICAL SOCIETY

Filiala Mehedinți - Mehedinți Branch


www.ssmrmh.ro

𝟐 𝟐
𝟐
𝒙−𝒚 𝒙−𝒚 𝒙−𝒚 𝟐
⇔𝟒 𝒙 𝒚−𝒛 ≥ 𝒙𝒚𝒛 𝟐
⇔𝟒 ≥
𝒙+𝒚 𝒙𝒚 𝒙+𝒚
𝒄𝒚𝒄 𝒄𝒚𝒄 𝒄𝒚𝒄 𝒄𝒚𝒄
𝟐
𝟐 𝟒 𝒙.𝒚 7𝒙𝒚
⇔ 𝒄𝒚𝒄 𝒙−𝒚 ≥ 𝟎, which is true
𝒙𝒚 𝒙.𝒚 𝟐
𝟐
∵𝟒 𝒙+𝒚 − 𝒙𝒚 ≥ 𝟏𝟓𝒙𝒚 > 0
Hence,
𝒙𝒚
𝟒 𝒛 𝒙𝟐 + 𝒚𝟐 + 𝟒𝒙𝒚𝒛 𝟐
≥ 𝟐𝟕𝒙𝒚𝒛
𝒙+𝒚
𝒄𝒚𝒄 𝒄𝒚𝒄

(proved)
Solution 3 by Redwane El Mellass-Morroco
𝒙 𝒚 𝒙𝒚
The inequality ⇔ 𝟒 + + 𝒙u𝒚 𝟐
≥ 𝟐𝟕
𝒚 𝒙
𝟐

𝟏 𝟐
Let 𝒇 𝒙 = 𝟒 𝒙 + + 𝟏 for 𝒙 > 0.
𝒙 𝒙.
𝒙

𝟏 𝟏 𝟏
Since 𝒇• 𝒙 = 𝟐 𝟏 − 𝟐− 𝟏 𝟐
with 𝒙 + ≥ 𝟏
𝒙𝟐 𝒙. 𝒙
𝒙

we get ∀𝒙 > 0 : 𝒇 𝒙 ≥ 𝒇 𝟏 = 𝟗.
𝒙
∴ 𝑳𝑯𝑺 ≥ 𝒇 ≥ 𝟐𝟕 with equality if
𝒚

𝒙 𝒚 𝒛
= = = 𝟏 ⇒ 𝒙 = 𝒚 = 𝒛 > 0
𝒚 𝒙 𝒙
49. If 𝒂, 𝒃, 𝒄 ∈ 𝟏, ∞ then:
𝟏
𝐥𝐨𝐠 𝒂𝒃 𝒆 + 𝐥𝐨𝐠 𝒃𝒄 𝒆 + 𝐥𝐨𝐠 𝒄𝒂 𝒆 ≤ 𝐥𝐨𝐠 𝒂 𝒆 + 𝐥𝐨𝐠 𝒃 𝒆 + 𝐥𝐨𝐠 𝒄 𝒆
𝟐
Proposed by Daniel Sitaru – Romania
Solution 1 by Myagmarsuren Yadamsuren – Darkhan – Mongolia

69
SOCIETATEA DE ȘTIINȚE MATEMATICE DIN ROMÂNIA
ROMANIAN MATHEMATICAL SOCIETY

Filiala Mehedinți - Mehedinți Branch


www.ssmrmh.ro

Solution 2 by Ravi Prakash - New Delhi – India


Solution 1 by Myagmarsuren Yadamsuren – Darkhan – Mongolia
𝟏
⋅ 𝐥𝐨𝐠 𝒂 𝒆 + 𝐥𝐨𝐠 𝒃 𝒆 + 𝐥𝐨𝐠 𝒄 𝒆
𝐥𝐨𝐠 𝒂𝒃 𝒆 + 𝐥𝐨𝐠 𝒃𝒄 𝒆 + 𝐥𝐨𝐠 𝒄𝒂 𝒆 ≤
𝟐
𝟏 𝟏 𝟏 𝟏 𝟏 𝟏 𝟏
+ + ≤ + +
𝐥𝐧 𝒂𝒃 𝐥𝐧 𝒃𝒄 𝐥𝐧 𝒄𝒂 𝟐 𝐥𝐧 𝒂 𝐥𝐧 𝒃 𝐥𝐧 𝒄
𝟏 𝟏 𝟏 𝟏 𝟏
+ + ≤ ⋅
𝐥𝐧 𝒂 + 𝐥𝐧 𝒃 𝐥𝐧 𝒃 + 𝐥𝐧 𝒄 𝐥𝐧 𝒄 + 𝐥𝐧 𝒂 𝟐 𝐥𝐧 𝒂
𝟏+𝟏 𝟐 𝟏+𝟏 𝟐 𝟏+𝟏 𝟐 𝑪𝒂𝒖𝒄𝒉𝒚
+ + ≤
𝟒 ⋅ 𝐥𝐧 𝒂 + 𝐥𝐧 𝒃 𝟒 ⋅ 𝐥𝐧 𝒃 + 𝐥𝐧 𝒄 𝟒 ⋅ 𝐥𝐧 𝒄 + 𝐥𝐧 𝒂
𝟏 𝟐 𝟐 𝟐 𝟏 𝟏 𝟏 𝟏
≤ ⋅ + + = + +
𝟒 𝐥𝐧 𝒂 𝐥𝐧 𝒃 𝐥𝐧 𝒄 𝟐 𝐥𝐧 𝒂 𝐥𝐧 𝒃 𝐥𝐧 𝒄
Solution 2 by Ravi Prakash - New Delhi – India
Let 𝒙 = 𝐥𝐨𝐠 𝒆 𝒂 , 𝒚 = 𝐥𝐨𝐠 𝒆 𝒃 , 𝒛 = 𝐥𝐨𝐠 𝒆 𝒄. As 𝒂, 𝒃, 𝒄 > 1, 𝑥, 𝑦, 𝑧 > 0
𝐥𝐨𝐠 𝒆 𝟏
Now, 𝐥𝐨𝐠 𝒂𝒃 𝒆 = = , etc. [change of base]
𝐥𝐨𝐠 𝒂.𝐥𝐨𝐠 𝒃 𝒙.𝒚

We have:
𝟐𝒙𝒚 𝟏 𝟏 𝟏
≤ 𝒙+𝒚 ⇒ ≤ 𝒙+𝒚
𝒙+𝒚 𝟐 𝒙+𝒚 𝟒
𝟏 𝟏 𝟏
Thus, ≤ 𝒙+𝒚 = 𝒙
𝒙.𝒚 𝟒 𝟐

𝟏
𝐥𝐨𝐠 𝒂𝒃 𝒆 ≤ 𝐥𝐨𝐠 𝒂 𝒆
𝟐
50. If 𝒂, 𝒃, 𝒄 > 0 then:
𝟐 𝟐𝟒
𝟏 𝒂+𝒃
≤𝟑
𝒂 − 𝒂𝒃 + 𝒃𝟐
𝟐 𝟔 𝒂𝟐 + 𝒃𝟐
Proposed by Daniel Sitaru – Romania
Solution 1 by Leonard Giugiuc – Romania

70
SOCIETATEA DE ȘTIINȚE MATEMATICE DIN ROMÂNIA
ROMANIAN MATHEMATICAL SOCIETY

Filiala Mehedinți - Mehedinți Branch


www.ssmrmh.ro

Solution 2 by Soumava Chakraborty-Kolkata-India


Solution 1 by Leonard Giugiuc – Romania
𝟏 𝒂.𝒃 𝟐
By Cauchy, 𝒂𝟐 + 𝒃𝟐 𝟐
≤ 𝒂 + 𝒃 𝒂𝟑 + 𝒃𝟑 ⇒ ≤ ⇒
𝒂𝟐 7𝒂𝒃.𝒃𝟐 𝒂𝟐 .𝒃𝟐
𝟐
𝟏𝟐
𝟏 𝒂+𝒃 𝟏
≤ ⇒ ≤
𝒂𝟐 − 𝒂𝒃 + 𝒃𝟐 𝟔 𝒂𝟐 + 𝒃𝟐 𝒂𝟐 − 𝒂𝒃 + 𝒃𝟐 𝟔
𝒄𝒚𝒄

𝟐
𝟏𝟐
𝒂+𝒃
≤ .
𝒂𝟐 + 𝒃𝟐
𝒄𝒚𝒄

By AM-QM,
𝟐
𝟐𝟒
𝒂+𝒃 𝒂+𝒃
≤𝟑 .
𝒂𝟐 + 𝒃𝟐 𝒂𝟐 + 𝒃𝟐
𝒄𝒚𝒄 𝒄𝒚𝒄

Solution 2 by Soumava Chakraborty-Kolkata-India


𝟐
𝟏 𝟏
≤𝟑
𝒂𝟐 − 𝒂𝒃 + 𝒃𝟐 𝟔 𝒂𝟐 − 𝒂𝒃 + 𝒃𝟐 𝟏𝟐

𝟐
∵ 𝒙 ≤𝟑 𝒙𝟐

𝟏 𝒂.𝒃 𝟐𝟒
Let’s prove: ≤ (1)
𝒂𝟐 7𝒂𝒃.𝒃𝟐 𝟏𝟐 𝒂𝟐 .𝒃𝟐

𝟏 𝒂+𝒃 𝟐
⇔ 𝟐 ≤ 𝟐
𝒂 − 𝒂𝒃 + 𝒃𝟐 𝒂 + 𝒃𝟐 𝟐
⇔ 𝒂𝟒 + 𝒃𝟒 + 𝟐𝒂𝟐 𝒃𝟐 ≤ 𝒂 + 𝒃 𝒂𝟑 + 𝒃𝟑 = 𝒂𝟒 + 𝒃𝟒 + 𝒂𝟑 𝒃 + 𝒂𝒃𝟑
⇔ 𝒂𝟐 + 𝒃𝟐 ≥ 𝟐𝒂𝒃 ⇔ 𝒂 − 𝒃 𝟐
≥ 𝟎 → true
𝟏 𝒃.𝒄 𝟐𝟒
Similarly, ≤ (2) and
𝒃𝟐 7𝒃𝒄.𝒃𝟐 𝟏𝟐 𝒃𝟐 .𝒄𝟐

71
SOCIETATEA DE ȘTIINȚE MATEMATICE DIN ROMÂNIA
ROMANIAN MATHEMATICAL SOCIETY

Filiala Mehedinți - Mehedinți Branch


www.ssmrmh.ro

𝟏 𝒄.𝒂 𝟐𝟒
≤ (3)
𝒄𝟐 7𝒄𝒂.𝒂𝟐 𝟏𝟐 𝒄𝟐 .𝒂𝟐

𝟏 𝒂.𝒃 𝟐𝟒
(1) + (2) + (3) ⇒ ≤ (4)
𝒂𝟐 7𝒂𝒃.𝒃𝟐 𝟏𝟐 𝒂𝟐 .𝒃𝟐

𝟐 𝟐𝟒
𝟏 𝟏 𝒂+𝒃
∴ ≤𝟑 ≤𝟑
𝒂𝟐 − 𝒂𝒃 + 𝒃𝟐 𝟔 𝒂𝟐 − 𝒂𝒃 + 𝒃𝟐 𝟏𝟐 𝒂𝟐 + 𝒃𝟐
(using (4))
(proved)
𝟏
51. For the positive real numbers 𝒂, 𝒃, 𝒄 ∈ , 𝒏 , where 𝒏 ∈ ℕ, prove that
𝒏
𝟐 𝒏𝟐 7𝟏 𝟑 𝒏𝟐
𝟑𝒏 ⋅ 𝒏 ⋅ 𝒂𝒃𝒄 ≥ 𝒂 + 𝒃 + 𝒄
Proposed by Abhay Chandra-India
Solution by Leonard Giugiuc-Romania:
It’s equivalent to
𝟑𝒏𝟐 𝐥𝐧 𝟑 + 𝟑 𝒏𝟐 − 𝟏 𝐥𝐧 𝒏 + 𝐥𝐧 𝒂 + 𝐥𝐧 𝒃 + 𝐥𝐧 𝒄 − 𝟑𝒏𝟐 𝐥𝐧 𝒂 + 𝒃 + 𝒄 = 𝟎
𝟏
Let the function 𝒇: , 𝒏 → 𝑹,
𝒏

𝒇 𝒂 = 𝟑𝒏𝟐 𝐥𝐧 𝟑 + 𝟑 𝒏𝟐 − 𝟏 𝐥𝐧 𝒏 + 𝐥𝐧 𝒂 + 𝐥𝐧 𝒃 + 𝐥𝐧 𝒄 − 𝟑𝒏𝟐 𝐥𝐧 𝒂 + 𝒃 + 𝒄 .
Then
𝟏 𝟑𝒏𝟐 𝟏7𝟑𝒏𝟐 𝒂.𝒃.𝒄
𝒇ʹ 𝒂 = − = .
𝒂 𝒂.𝒃.𝒄 𝒂 𝒂.𝒃.𝒄

𝟑𝒏𝟐 7𝟏
But 𝟑𝒏𝟐 − 𝟏 𝒂 ≥ > 𝟐𝒏 ≥ 𝒃 + 𝒄,
𝒏
𝟏
∀𝒂 ∈ , 𝒏 , hence 𝒇 is strictly decreasing
𝒏

⇒ 𝐦𝐢𝐧 𝒇 = 𝒇 𝒏 =
= 𝟑𝒏𝟐 𝐥𝐧 𝟑 + 𝟑 𝒏𝟐 − 𝟏 𝐥𝐧 𝒏 + 𝐥𝐧 𝒏 + 𝐥𝐧 𝒃 + 𝐥𝐧 𝒄 − 𝟑𝒏𝟐 𝐥𝐧 𝒏 + 𝒃 + 𝒄
Apply twice more this procedure and get the desired result. Equality if

72
SOCIETATEA DE ȘTIINȚE MATEMATICE DIN ROMÂNIA
ROMANIAN MATHEMATICAL SOCIETY

Filiala Mehedinți - Mehedinți Branch


www.ssmrmh.ro

𝒂 = 𝒃 = 𝒄 = 𝒏.
52. If 𝒂, 𝒃, 𝒄, 𝒅 > 0 then:
𝟔
𝟑 𝒂+𝒃+𝒄+𝒅 >2 𝒂𝟑 𝒃 𝒃 + 𝒄 𝒃 + 𝒄 + 𝒅

Proposed by Daniel Sitaru – Romania


Solution 1 by Anas Adlany - El Jadida – Morroco
Solution 2 by Henry Ricardo - New York – USA
Solution 3 by Myagmarsuren Yadamsuren – Darkhan – Mongolia
Solution 1 by Anas Adlany - El Jadida – Morroco
𝟔 𝟑𝒂 + 𝒃 + 𝒃 + 𝒄 + 𝒃 + 𝒄 + 𝒅
𝟐 𝒂𝟑 𝒃 𝒃 + 𝒄 𝒃 + 𝒄 + 𝒅 ≤ 𝟐 ⋅ =
𝟔
𝟒𝒄 𝟐𝒅
=𝒂+𝒃+ +
𝟔 𝟔
𝟔 𝟒𝒄 𝟐𝒅
⇒ 𝟐 𝒂𝟑 𝒃 𝒃 + 𝒄 𝒃 + 𝒄 + 𝒅 ≤ 𝒂+𝒃+ + <3 𝒂+𝒃+𝒄+𝒅
𝟔 𝟔
(true)
Solution 2 by Henry Ricardo - New York – USA
The AM – GM inequality gives us
𝟔 𝒂+𝒂+𝒂+𝒃+ 𝒃+𝒄 + 𝒃+𝒄+𝒅
𝟐 𝒂𝟑 𝒃 𝒃 + 𝒄 𝒃 + 𝒄 + 𝒅 < 2 =
𝟔
𝒄𝒚𝒄𝒍𝒊𝒄 𝒄𝒚𝒄𝒍𝒊𝒄

𝟑𝒂 + 𝟑𝒃 + 𝟐𝒄 + 𝒅 𝟏
=𝟐 = 𝟑 𝒂+𝒃 +𝟐 𝒄+ 𝒅 =
𝟔 𝟑
𝒄𝒚𝒄𝒍𝒊𝒄 𝒄𝒚𝒄𝒍𝒊𝒄 𝒄𝒚𝒄𝒍𝒊𝒄 𝒄𝒚𝒄𝒍𝒊𝒄

𝟏
= ⋅ 𝟗 𝒂 + 𝒃 + 𝒄 + 𝒅 = 𝟑 𝒂 + 𝒃 + 𝒄 + 𝒅 .
𝟑
Solution 3 by Myagmarsuren Yadamsuren – Darkhan – Mongolia
𝒂; 𝒃; 𝒄; 𝒅 > 0

73
SOCIETATEA DE ȘTIINȚE MATEMATICE DIN ROMÂNIA
ROMANIAN MATHEMATICAL SOCIETY

Filiala Mehedinți - Mehedinți Branch


www.ssmrmh.ro

𝟔
𝟑⋅ 𝒂>2⋅ 𝒂𝟑 𝒃 𝒃 + 𝒄 ⋅ 𝒃 + 𝒄 + 𝒅

𝟗⋅ 𝒂+𝒃+𝒄+𝒅
𝟑⋅ 𝒂+𝒃+𝒄+𝒅 =𝟐⋅ =
𝟔
𝟑𝒂 + 𝟑𝒃 + 𝟐𝒄 + 𝒅 𝟑𝒂 + 𝒃 + 𝒃 + 𝒄 + 𝒃 + 𝒄 + 𝒅
=𝟐 =𝟐⋅ ≥
𝟔 𝟔
𝟔
≥ 𝟐 𝒂𝟑 ⋅ 𝒃 ⋅ 𝒃 + 𝒄 ⋅ 𝒃 + 𝒄 + 𝒅
𝑪𝒂𝒖𝒄𝒉𝒚


𝟏 𝟏 𝟏 𝟏
53. If 𝒂, 𝒃, 𝒄, 𝒅 ∈ 𝟎, ∞ , + + + = 𝟏
𝟏.𝒂𝟐 𝟏.𝒃𝟐 𝟏.𝒄𝟐 𝟏.𝒅𝟐

then: 𝒂 + 𝒃 + 𝒄 + 𝒅 ≥ 𝟔
Proposed by Daniel Sitaru – Romania
Solution by Myagmarsuren Yadamsuren – Mongolia
1)
𝒏
𝟏
=𝟏
𝟏 + 𝒙𝟐𝒊
𝒊±𝟏
𝒏 𝒏 𝑨𝒔𝒔𝒖𝒎𝒆
𝟏 𝒙𝟐𝒊 +𝟏 𝟏
𝒙𝒊 ≥ 𝒏 − 𝟏 ⋅ ⇔ ≥𝒏⋅
𝒙𝒊 𝒙𝒊 𝒙𝒊
𝒊±𝟏 𝒊±𝟏
𝒙 𝟏 ≥ 𝒙𝟐 ≥ ⋯ ≥ 𝒙𝒏
𝟏 𝟏 𝟏
≤ ≤⋯≤ Chebyshev ⇒
𝒙𝟏 𝒙𝟐 𝒙𝒏
𝒙𝟏 𝒙𝟐 𝒙𝒏
≤ ≤⋯≤
𝟏.𝒙𝟐𝟏 𝟏.𝒙𝟐𝟐 𝟏.𝒙𝟐𝒏
𝒏 𝒏 𝒏 𝒏
𝒙𝒊 𝟏 𝒙𝒊 𝟏 𝟏
⋅ ≤𝒏⋅ ⋅ =𝒏⋅ = 𝒏 ∗
𝒙𝟐𝒊 + 𝟏 𝒙𝒊 𝒙𝟐𝒊 + 𝟏 𝒙𝒊 𝒙𝟐𝒊 + 𝟏
𝒊±𝒏 𝒊±𝒏 𝒊±𝒏 𝒊±𝒏
𝒏 𝒏
𝒙𝟐𝒊 + 𝟏 𝒙𝒊 𝑪𝒂𝒖𝒄𝒉𝒚 𝒏 𝒙𝟐𝒊 + 𝟏 𝒙𝒊
⋅ 𝟐
≥ 𝒏⋅ ⋅𝒏⋅ = 𝒏𝟐 ∗∗
𝒙𝒊 𝒙𝒊 + 𝟏 𝒙𝒊 𝒙𝟐𝒊 +𝟏
𝒊±𝒏 𝒊±𝒏

74
SOCIETATEA DE ȘTIINȚE MATEMATICE DIN ROMÂNIA
ROMANIAN MATHEMATICAL SOCIETY

Filiala Mehedinți - Mehedinți Branch


www.ssmrmh.ro
𝒏 𝒏 𝒏
𝒙𝟐𝒊 + 𝟏 𝒙𝒊 𝒙𝒊 𝟏
∗ ; ∗∗ ⇒ ⋅ ≥𝒏⋅ ⋅ ⇒
𝒙𝒊 𝒙𝟐𝒊 + 𝟏 𝒙𝟐𝒊 + 𝟏 𝒙𝒊
𝒊±𝒏 𝒊±𝒏 𝒊±𝒏
𝒏 𝒏
𝒙𝟐𝒊 + 𝟏 𝟏
⇒ ≥𝒏⋅ ∗∗∗
𝒙𝒊 𝒙𝒊
𝒊±𝒏 𝒊±𝒏

2)
𝒏
𝟏 𝑪𝒂𝒖𝒄𝒉𝒚 𝟏 𝟏 𝟏
𝟏= ≤ ⋅ ⇒ ≥ 𝟐 ∗∗∗∗
𝟏 + 𝒙𝟐𝒊 𝟐 𝒙𝒊 𝒙𝒊
𝒊±𝒏
𝒏 𝒏 𝒏 𝒏
𝒙𝟐𝒊 + 𝟏 𝟏 𝟏
≥𝒏⋅ ⇔ 𝒙𝒊 ≥ 𝒏 − 𝟏 ⋅ ≥
𝒙𝒊 𝒙𝒊 𝒙𝒊
𝒊±𝒏 𝒊±𝒏 𝒊±𝒏 𝒊±𝒏
𝒏
∗∗∗∗
≥ 𝒏−𝟏 ⋅𝟐⇒ 𝒙𝒊 ≥ 𝒏 − 𝟏 ⋅ 𝟐
𝒊±𝒏

𝒏 = 𝟒 ⇒ 𝒙𝟏 + 𝒙𝟐 + 𝒙𝟑 + 𝒙𝟒 ≥ 𝟑 ⋅ 𝟐 = 𝟔 ⇒ 𝒂 + 𝒃 + 𝒄 + 𝒅 ≥ 𝟔
54. If 𝒂, 𝒃, 𝒄, 𝒅 > 0, 𝑎 + 𝑏 + 𝑐 + 𝑑 = 1 then:
𝒂𝟑 𝒃𝟑 𝒄𝟑 𝒅𝟑 𝟏
+ + + ≥
𝒃+𝒄 𝒄+𝒅 𝒅+𝒂 𝒂+𝒃 𝟖
Proposed by George Apostolopoulos - Messolonghi - Greece
Solution by Kevin Soto – Palacios – Huarmey – Peru
Si: 𝒂, 𝒃, 𝒄, 𝒅 > 0. 𝑎 + 𝑏 + 𝑐 + 𝑑 = 1. Probar que:
𝒂𝟑 𝒃𝟑 𝒄𝟑 𝒅𝟑 𝟏
+ + + ≥
𝒃+𝒄 𝒄+𝒅 𝒅+𝒂 𝒂+𝒃 𝟖
Por la desigualdad de Holder:
𝒂𝟑 𝒃𝟑 𝒄𝟑 𝒅𝟑
+ + + 𝒃+𝒄 + 𝒄+𝒅 + 𝒅+𝒂 + 𝒂+𝒃 𝟏 + 𝟏 + 𝟏 + ≥
𝒃+𝒄 𝒄+𝒅 𝒅+𝒂 𝒂+𝒃
≥ 𝒂 + 𝒃 + 𝒄 + 𝒅 𝟑

75
SOCIETATEA DE ȘTIINȚE MATEMATICE DIN ROMÂNIA
ROMANIAN MATHEMATICAL SOCIETY

Filiala Mehedinți - Mehedinți Branch


www.ssmrmh.ro

𝒂𝟑 𝒃𝟑 𝒄𝟑 𝒅𝟑 𝒂+𝒃+𝒄+𝒅 𝟑 𝟏
⇒ + + + ≥ =
𝒃+𝒄 𝒄+𝒅 𝒅+𝒂 𝒂+𝒃 𝟖 𝒂+𝒃+𝒄+𝒅 𝟖
(LQQD)
55. If 𝒙, 𝒚, 𝒛 > 0 then:
𝟐 𝒙 𝒙𝟐 𝒛𝟐 + 𝒚𝟒 + 𝒚 𝒚𝟐 𝒙𝟐 + 𝒛𝟒 + 𝒛 𝒛𝟐 𝒚𝟐 + 𝒙𝟒 > 𝑥𝑦 𝒚𝒛 + 𝒚𝒙 𝒛𝒙 + 𝒛𝒚 𝒙𝒚

Proposed by Daniel Sitaru – Romania


Solution 1 by Myagmarsuren Yadamsuren – Mongolia
Solution 2 by Nguyen Minh Triet - Quang Ngai – Vietnam
Solution 1 by Myagmarsuren Yadamsuren – Mongolia

𝟐 𝒙 ⋅ 𝒙𝟐 𝒛𝟐 + 𝒚𝟒 = 𝟏𝟐 + 𝟏𝟐 ⋅ 𝒙𝟒 𝒛𝟐 + 𝒚𝟒 ⋅ 𝒙𝟐 ≥

≥ 𝒙𝟐 𝒛 + 𝒚𝟐 𝒙 = 𝒙𝟐 𝒛 + 𝒚𝟐 𝒙 + 𝒚𝟐 𝒙 + 𝒛𝟐 𝒚 + 𝒛𝟐 𝒚 + 𝒙𝟐 𝒛 =

= 𝒙𝟐 𝒛 + 𝒚𝟐 𝒙 + 𝒚𝟐 𝒙 + 𝒛𝟐 𝒚 + 𝒛𝟐 𝒚 + 𝒙𝟐 𝒛 ≥ 𝟐 ⋅ 𝒙𝒚 ⋅ 𝒙𝒛 +
+𝟐 𝒙𝒚 ⋅ 𝒚𝒛 + 𝟐𝒙𝒛 ⋅ 𝒛𝒚 > 𝑥𝑦 𝒙𝒛 + 𝒚𝒙 𝒛𝒙 + 𝒛𝒚 𝒙𝒚
Solution 2 by Nguyen Minh Triet - Quang Ngai – Vietnam
By AM – GM inequality, we have:

𝒙𝟐 𝒛𝟐 + 𝒚𝟒 ≥ 𝟐𝒙𝒛𝒚𝟐 ⇒ 𝒙 𝟐 ⋅ 𝒙𝟐 𝒛𝟐 + 𝒚𝟒 ≥ 𝟐𝒙𝒚 𝒙𝒛 (1)

Similarly, we get: 𝒚 𝟐 ⋅ 𝒚𝟐 𝒙𝟐 + 𝒛𝟒 ≥ 𝒚𝒛 𝒙𝒚 (2)

𝒛 𝟐 ⋅ 𝒙𝟐 𝒚𝟐 + 𝒙𝟒 ≥ 𝟐𝒙𝒛 𝒙𝒚 (3)
(1) + (2) + (3) ⇒ 𝑳𝑯𝑺 ≥ 𝟐 𝑹𝑯𝑺 > 𝑅𝐻𝑆
q.e.d.
56. If 𝒂, 𝒃, 𝒄 > 0 then the following relationship holds:

76
SOCIETATEA DE ȘTIINȚE MATEMATICE DIN ROMÂNIA
ROMANIAN MATHEMATICAL SOCIETY

Filiala Mehedinți - Mehedinți Branch


www.ssmrmh.ro

𝒂 + 𝒃 + 𝟐𝒄 𝒂 + 𝒄 𝒃 + 𝒄
𝒂+𝒃+𝒄≥
𝟐 𝒂+𝒄 𝟐+𝟐 𝒃+𝒄 𝟐
Proposed by Daniel Sitaru – Romania
Solution 1 by Kevin Soto Palacios – Huarmey – Peru , Solution 2 by Anas Adlany-El
Jadida-Morroco , Solution 3 by Sk Rejuan-West Bengal-India , Solution 4 by
Nirapada Pal-India , Solution 5 by Myagmarsuren Yadamsuren-Darkhan-Mongolia
Solution 1 by Kevin Soto Palacios – Huarmey – Peru
Si: 𝒂, 𝒃, 𝒄 > 0. Probar la siguiente desigualdad:
𝒂 + 𝒃 + 𝟐𝒄 𝒂 + 𝒄 𝒃 + 𝒄
𝒂+𝒃+𝒄≥
𝟐 𝒂+𝒄 𝟐+𝟐 𝒃+𝒄 𝟐
Desde que: 𝒂, 𝒃, 𝒄 > 0. Por: MA ≥ MG:
𝟐 𝟐
𝟐 𝒂+𝒄 +𝟐 𝒃+𝒄 ≥ 𝟒 𝒂 + 𝒄 𝒃 + 𝒄 →
𝒂.𝒃.𝟐𝒄 𝒂.𝒄 𝒃.𝒄 𝟐𝒄.𝒂.𝒃
→ 𝟐 .𝟐 𝟐
≤ = 𝒂 + 𝒃 + 𝒄 … (LQQD)
𝟐 𝒂.𝒄 𝒃.𝒄 𝟒

Solution 2 by Anas Adlany-El Jadida-Morroco


We have 𝟐 𝑨𝟐 + 𝑩𝟐 ≥ 𝟒𝑨𝑩, so if we take
𝑨 = 𝒂 + 𝒄, 𝑩 = 𝒃 + 𝒄, we get
𝒂 + 𝒃 + 𝟐𝒄 𝒂 + 𝒄 𝒃 + 𝒄
=
𝟐 𝒂+𝒄 𝟐+ 𝒃+𝒄 𝟐
𝑨 + 𝑩 𝑨𝑩 𝑨+𝑩
= ≤ = 𝒂
𝟐 𝑨𝟐 + 𝑩𝟐 𝟒

Solution 3 by Sk Rejuan-West Bengal-India
We have to prove

77
SOCIETATEA DE ȘTIINȚE MATEMATICE DIN ROMÂNIA
ROMANIAN MATHEMATICAL SOCIETY

Filiala Mehedinți - Mehedinți Branch


www.ssmrmh.ro

𝒂 + 𝒃 + 𝟐𝒄 𝒂 + 𝒄 𝒃 + 𝒄
≤ 𝒂 + 𝒃 + 𝒄
𝟐 𝒂+𝒄 𝟐+𝟐 𝒃+𝒄 𝟐
𝒂.𝒃.𝒄 𝒂.𝒄 𝒃.𝒄
𝑳𝑯𝑺 = (1)
𝟐 𝒂.𝒄 𝟐 .𝟐 𝒃.𝒄 𝟐

𝟐 𝟐 𝟏
Now, 𝒂 + 𝒄 + 𝒃+𝒄 ≥ 𝒂 + 𝒃 + 𝟐𝒄 𝟐
𝟐

𝟏 𝟐
⇒ ≤
𝒂+𝒄 𝟐+ 𝒃+𝒄 𝟐 𝒂+𝒃+𝒄 𝟐
𝒂 + 𝒃 + 𝟐𝒄 𝒂 + 𝒄 𝒃 + 𝒄 𝟐 𝒂 + 𝒃 + 𝟐𝒄 𝒂 + 𝒄 𝒃 + 𝒄
⇒ ≤ =
𝟐 𝒂+𝒄 𝟐+𝟐 𝒃+𝒄 𝟐 𝟐 𝒂 + 𝒃 + 𝟐𝒄 𝟐
𝒂+𝒄 𝒃+𝒄 𝟏
= ≤ 𝒂 + 𝒃 + 𝟐𝒄
𝒂 + 𝒃 + 𝟐𝒄 𝟒
[by G.M ≤ A.M]
Solution 4 by Nirapada Pal-India
𝒂.𝒃.𝟐𝒄 𝒂.𝒄 𝒃.𝒄 𝒂.𝒃.𝟐𝒄 𝒂.𝒄 𝒃.𝒄
𝟐 .𝟐 𝟐
≤ =
𝟐 𝒂.𝒄 𝒃.𝒄 𝒂.𝒃.𝟐𝒄 𝟐

as 𝟐 𝒂𝟐 + 𝒃𝟐 ≥ 𝒂 + 𝒃 𝟐
𝒂.𝒄 𝒃.𝒄 𝟏 𝟐 𝟏 𝒂.𝒄 . 𝒃.𝒄
= = 𝟏 𝟏 ≤ = as 𝑯𝑴 ≤ 𝑨𝑴
𝒂.𝒃.𝟐𝒄 𝟐 . 𝟐 𝟐
𝒂u𝒄 𝒃u𝒄

𝟏
= 𝒂 + 𝒃 + 𝟐𝒄 = 𝒂 + 𝒃 + 𝒄
𝟒
Solution 5 by Myagmarsuren Yadamsuren-Darkhan-Mongolia
𝒂+𝒄=𝒙 𝒙+𝒚+𝒛
𝒄 + 𝒂 = 𝒚 𝒂 + 𝒃 + 𝒄 =
𝟐
𝒂+𝒃=𝒛
𝒙+𝒚+𝒛 𝒙 + 𝒚 ⋅ 𝒙𝒚

𝟐 𝟐 𝒙𝟐 + 𝒚𝟐
𝒙.𝒚 ⋅𝒙𝒚
𝒙+𝒚+𝒛≥ (ASSURE)
𝒙𝟐 .𝒚𝟐

78
SOCIETATEA DE ȘTIINȚE MATEMATICE DIN ROMÂNIA
ROMANIAN MATHEMATICAL SOCIETY

Filiala Mehedinți - Mehedinți Branch


www.ssmrmh.ro

𝒙+𝒚 𝒚+𝒛 𝒛+𝒙 𝒙 + 𝒚 ⋅ 𝒙𝒚


+ + ≥
𝟐 𝟐 𝟐 𝒙𝟐 + 𝒚𝟐
𝒙.𝒚 𝒙.𝒚 ⋅𝒙𝒚
≥ (ASSURE)
𝟐 𝒙𝟐 .𝒚𝟐
𝟏 𝒙𝒚
≥ ⇒ 𝒙𝟐 + 𝒚𝟐 ≥ 𝟐𝒙𝒚 (TRUE)
𝟐 𝒙𝟐 .𝒚𝟐

Similarly
𝒚+𝒛 𝒚 + 𝒛 𝒛𝒚
≥ 𝟐
𝟐 𝒚 + 𝒛𝟐
𝒛+𝒙 𝒛 + 𝒙 ⋅ 𝒛𝒙

𝟐 𝒛𝟐 + 𝒙𝟐
𝒙 + 𝒚 ⋅ 𝒙𝒚
𝒙+𝒚+𝒛≥
𝒚𝟐 + 𝒙𝟐
57. If 𝒂, 𝒃, 𝒄 ∈ 𝟐, ∞ then:
𝒂𝒃 − 𝟒 𝟏𝟐 𝟏 𝟏 𝟏
+ ≥ + +
𝒄(𝒂 + 𝒃) 𝒂𝒃𝒄 𝒂 𝒃 𝒄
Proposed by Daniel Sitaru – Romania
Solution by Redwane El Mellass-Morroco
𝟏 𝟏 𝟏 𝟏
∴+ ≤ + = 𝟏 ⇒ 𝒂 + 𝒃 ≤ 𝒂𝒃
𝒂 𝒃 𝟐 𝟐
𝟏 𝟏 𝒂𝒃 − 𝟒 𝟏 𝟏
⇒ ≥ ⇒ − ≥ 𝟎
𝒂 + 𝒃 𝒂𝒃 𝒄 𝒂 + 𝒃 𝒂𝒃
𝒂𝒃 − 𝟒 𝟒 𝟏 𝒂𝒃 − 𝟒 𝟏𝟐 𝟏
⇒ + − ≥𝟎⇒ + ≥ .
𝒄 𝒂+𝒃 𝒂𝒃𝒄 𝒄 𝒄 𝒂+𝒃 𝒂𝒃𝒄 𝒂
𝟏 𝟏 𝟏 𝟏 𝟏 𝟏
with equality if and only if + = + = + = 𝟏 ⇒ 𝒂 = 𝒃 = 𝒄 = 𝟐.
𝒂 𝒃 𝒃 𝒄 𝒄 𝒂

58. If 𝒂, 𝒃, 𝒄 ∈ (𝟎, 𝟐) then:

79
SOCIETATEA DE ȘTIINȚE MATEMATICE DIN ROMÂNIA
ROMANIAN MATHEMATICAL SOCIETY

Filiala Mehedinți - Mehedinți Branch


www.ssmrmh.ro

𝟏 𝟏 𝟏 𝟏 𝒂 𝒃 𝒄
𝒃𝒆𝒂7𝟐 + 𝒄𝒆𝒃7𝟐 + 𝒂𝒆𝒄7𝟐 ≤ + +
𝒆 𝒄 𝒂 𝒃
Proposed by Daniel Sitaru – Romania
Solution by Redwane El Mellass-Morrocco
𝟏
Let 𝒇 𝒙 = 𝒙𝒆𝒙F𝟐 , 𝟎 < 𝑥 < 2.
Since:
𝟏
𝒙 − 𝟏 𝒙 − 𝟑 𝒆𝒙7𝟐
𝒇• 𝒙 = ,
𝒙−𝟐 𝟐
𝟏
we get 𝒇 𝒙 ≤ 𝒇 𝟏 = .
𝒆

So:
𝒃 𝟏 𝒃
𝒇 𝒂 ≤ .
𝒂 𝒆 𝒂
𝒄𝒚𝒄 𝒄𝒚𝒄

59. If 𝒂, 𝒃, 𝒄 > 0 then:


𝒂𝟐 + 𝒃𝟐 𝒂𝒃
+ 𝟏𝟏 >6 𝒂𝒃 + 𝒃𝒄 + 𝒄𝒂
𝒂+𝒃 𝒂+𝒃
Proposed by Daniel Sitaru – Romania
Solution 1 by Mihalcea Andrei Ștefan – Romania , Solution 2 by Ravi Prakash - New
Delhi – India , Solution 3 by Seyran Ibrahimov – Maasilli – Azerbaidjian
Solution 4 by Soumitra Mandal - Chandar Nagore – India
Solution 5 by Abdallah El Farissi – Bechar – Algerie
Solution 1 by Mihalcea Andrei Ștefan – Romania
𝒂𝟐 .𝒃𝟐 𝒂𝒃
Let’s prove: + 𝟏𝟏 ≥ 𝟔 𝒂𝒃
𝒂.𝒃 𝒂.𝒃
𝒏𝒐𝒕
𝒂 + 𝒃 = 𝒔

80
SOCIETATEA DE ȘTIINȚE MATEMATICE DIN ROMÂNIA
ROMANIAN MATHEMATICAL SOCIETY

Filiala Mehedinți - Mehedinți Branch


www.ssmrmh.ro

𝒏𝒐𝒕
𝒂𝒃 = 𝒑
𝟐
⇔ 𝒔𝟐 − 𝟔𝒔 𝒑 + 𝟗𝒑 > 9 ⇔ 𝒔 − 𝟑 𝒑 ≥ 𝟎
𝒃 𝟑± 𝟓
𝒂 + 𝒃 = 𝟑 𝒂𝒃 ⇒ 𝒂 =
𝟐
𝒄 𝟑± 𝟓
if 𝒃 + 𝒄 = 𝟑 𝒃𝒄 ⇒ 𝒃 =
𝟐
𝒂 𝟑± 𝟓
𝒄 + 𝒂 = 𝟑 𝒂𝒄 ⇒ 𝒄 =
𝟐

Anyway we will have the signs in 𝟑 ± 𝟓, it will result that a rational number is
equal to an irrational number. Contradiction ⇒ 𝑳𝑯𝑺 > 𝑅𝐻𝑆
Solution 2 by Ravi Prakash - New Delhi – India
Consider
𝒂𝟐 + 𝒃𝟐 𝟏𝟏𝒂𝒃
+ − 𝟔 𝒂𝒃 =
𝒂+𝒃 𝒂+𝒃
𝟏
= 𝒂𝟐 + 𝒃𝟐 + 𝟐𝒂𝒃 − 𝟔 𝒂𝒃 𝒂 + 𝒃 + 𝟗𝒂𝒃 =
𝒂+𝒃
𝟏 𝟐
= 𝒂 + 𝒃 𝟐 − 𝟔 𝒂𝒃 𝒂 + 𝒃 + 𝟑 𝒂𝒃 =
𝒂+𝒃
𝟏 𝟐
= 𝒂 + 𝒃 − 𝟑 𝒂𝒃 ≥ 𝟎
𝒂+𝒃
𝒂𝟐 + 𝒃𝟐 𝟏𝟏𝒂𝒃
∴ + ≥ 𝟔 𝒂𝒃
𝒂+𝒃 𝒂+𝒃
Similarly for other two terms.
Solution 3 by Seyran Ibrahimov – Maasilli – Azerbaidjian
𝒂𝟐 + 𝒃𝟐 𝟏𝟏𝒂𝒃 𝒂+𝒃 𝟐 𝟗𝒂𝒃 𝑨𝑴7𝑮𝑴
+ = + ≥ 𝟔 𝒂𝒃
𝒂+𝒃 𝒂+𝒃 𝒂+𝒃 𝒂+𝒃

81
SOCIETATEA DE ȘTIINȚE MATEMATICE DIN ROMÂNIA
ROMANIAN MATHEMATICAL SOCIETY

Filiala Mehedinți - Mehedinți Branch


www.ssmrmh.ro

Solution 4 by Soumitra Mandal - Chandar Nagore – India


𝒂𝟐 + 𝒃𝟐 𝒂𝒃
+ 𝟏𝟏 >6 𝒂𝒃
𝒂+𝒃 𝒂+𝒃
𝒄𝒚𝒄 𝒄𝒚𝒄 𝒄𝒚𝒄

𝟐
𝒂+𝒃 + 𝟗𝒂𝒃 − 𝟔 𝒂 + 𝒃 𝒂𝒃
⇔ > 0
𝒂+𝒃
𝒄𝒚𝒄
𝟐
𝒂 + 𝒃 − 𝟑 𝒂𝒃
⇔ > 0
𝒂+𝒃
𝒄𝒚𝒄

which is true
𝒂𝟐 + 𝒃𝟐 𝒂𝒃
∴ + 𝟏𝟏 >6 𝒂𝒃
𝒂+𝒃 𝒂+𝒃
𝒄𝒚𝒄 𝒄𝒚𝒄 𝒄𝒚𝒄

(Proved)
Solution 5 by Abdallah El Farissi – Bechar – Algerie
𝒂 𝟐 + 𝒃𝟐 𝒂𝒃 𝒂𝒃
+ 𝟏𝟏 = 𝒂+𝒃 +𝟗 ≥ 𝟔 𝒂𝒃
𝒂+𝒃 𝒂+𝒃 𝒂+𝒃
𝟑 𝟓.𝟕
and equality if and only if 𝒂𝟐 − 𝟕𝒂𝒃 + 𝒃𝟐 = 𝟎, 𝒂 = 𝒃 then
𝟐

𝒂𝟐 + 𝒃𝟐 𝒂𝒃
+ 𝟏𝟏 ≥𝟔 𝒂𝒃 + 𝒃𝒄 + 𝒂𝒄
𝒂+𝒃 𝒂+𝒃
𝒂𝟐 − 𝟕𝒂𝒃 + 𝒃𝟐 = 𝟎
and equality if and only if 𝒃𝟐 − 𝟕𝒃𝒄 + 𝒄𝟐 = 𝟎 it follow that
𝒄𝟐 − 𝟕𝒄𝒂 + 𝒂𝟐 = 𝟎
𝟑 𝟓.𝟕
𝒂= 𝒃
𝟐
𝟑 𝟓.𝟕
𝒃= 𝒄 this is contradiction then
𝟐
𝟑 𝟓.𝟕
𝒄= 𝒂
𝟐

82
SOCIETATEA DE ȘTIINȚE MATEMATICE DIN ROMÂNIA
ROMANIAN MATHEMATICAL SOCIETY

Filiala Mehedinți - Mehedinți Branch


www.ssmrmh.ro

𝒂𝟐 + 𝒃𝟐 𝒂𝒃
+ 𝟏𝟏 >6 𝒂𝒃 + 𝒃𝒄 + 𝒂𝒄
𝒂+𝒃 𝒂+𝒃
60. If 𝒂, 𝒃, 𝒄 ≥ 𝟏 then:
𝟐
𝐥𝐨𝐠 𝒂 ⋅ 𝐥𝐨𝐠 𝒃 ≥ 𝐥𝐨𝐠 𝟐 𝒂 𝐥𝐨𝐠 𝟐 𝒂𝒃𝒄 − 𝟐 𝐥𝐨𝐠 𝟐 𝒂

Proposed by Daniel Sitaru – Romania



Solution 1 by Mihalcea Andrei Ștefan – Romania
Solution 2 by Soumava Chakraborty – Kolkata – India
Solution 3 by Abdallah El Farissi – Bechar – Algerie
Solution 1 by Mihalcea Andrei Ștefan – Romania
𝒙 = 𝐥𝐧 𝒂 ≥ 𝟎
Take 𝒚 = 𝐥𝐧 𝒃 ≥ 𝟎
𝒛 = 𝐥𝐧 𝒄 ≥ 𝟎
𝟐
The inequality ⇔ 𝒙𝒚 ≥ 𝒙𝟐 𝟐 𝒙𝒚 − 𝒙𝟐
⇔ 𝒈𝟐 ≥ 𝒑𝟐 − 𝟐𝒈 𝟒𝒒 − 𝒑𝟐
⇔ 𝒈𝟐 ≥ 𝟒𝒈𝒑𝟐 − 𝒑𝟒 − 𝟖𝒈𝟐 + 𝟐𝒈𝒑𝟐 ≥ 𝟎
⇔ 𝟐𝒈 − 𝒑𝟐 𝟐
≥ 𝟎 true

Solution 2 by Soumava Chakraborty – Kolkata – India
Let 𝐥𝐨𝐠 𝒂 = 𝒖, 𝐥𝐨𝐠 𝒃 = 𝒗, 𝐥𝐨𝐠 𝒄 = 𝒘 𝒖, 𝒗, 𝒘 ≥ 𝟎
𝟐
Given inequality ⇔ 𝒖𝒗 ≥ 𝒖𝟐 𝒖 𝟐
−𝟐 𝒖𝟐
𝟐
⇔ 𝒖𝒗 ≥ 𝒖𝟐 𝟐 𝒖𝒗 − 𝒖𝟐

Let 𝒖𝒗 = 𝒙 and 𝒖𝟐 = 𝒚

83
SOCIETATEA DE ȘTIINȚE MATEMATICE DIN ROMÂNIA
ROMANIAN MATHEMATICAL SOCIETY

Filiala Mehedinți - Mehedinți Branch


www.ssmrmh.ro

∴ given inequality ⇔ 𝒙𝟐 ≥ 𝒚 𝟐𝒙 − 𝒚 ⇔ 𝒙 − 𝒚 𝟐
≥ 𝟎
which is true (Proved)
N.B.: Inequality is true ∀ 𝒂, 𝒃, 𝒄 > 0
Solution 3 by Abdallah El Farissi – Bechar – Algerie
𝟐
𝟐
𝐥𝐨𝐠 𝒂 𝐥𝐨𝐠 𝒃 − 𝐥𝐨𝐠 𝒂 ≥ 𝟎

then
𝟐
𝐥𝐨𝐠 𝒂 𝐥𝐨𝐠 𝒃 ≥ 𝐥𝐨𝐠 𝟐 𝒂 𝟐 𝐥𝐨𝐠 𝒂 𝐥𝐨𝐠 𝒃 − 𝐥𝐨𝐠 𝟐 𝒂

= 𝐥𝐨𝐠 𝟐 𝒂 𝐥𝐨𝐠 𝟐 𝒂𝒃𝒄 − 𝟐 𝐥𝐨𝐠 𝟐 𝒂

61. If 𝒂, 𝒃, 𝒄 > 0 then:

𝒂𝟐 + 𝒃𝟐 𝒂 𝒃 𝟐 + 𝒄𝟐 𝒃 𝒄𝟐 + 𝒂𝟐 𝒄 𝒂 𝒃 𝒄
+ + 𝟑 ≤ + +
𝒂𝟑 + 𝒃𝟑 𝒃 𝟑 + 𝒄𝟑 𝒄 + 𝒂𝟑 𝒃 𝒄 𝒂

Proposed by Daniel Sitaru – Romania


Solution by Redwane El Mellass – Morroco
𝒕𝟐 .𝒕𝟔
Let 𝒇 𝒕 = − 𝒕, 𝒕 > 0.
𝟏.𝒕𝟔

7𝒕 𝒕7𝟏 𝟐 𝒕𝟒 .𝒕𝟑 .𝒕𝟐 .𝒕.𝟏


Since 𝒇 𝒕 = ≤ 𝟎
𝟏.𝒕𝟔

𝒂 𝒂𝟐 + 𝒃𝟐 𝒃 𝒂
∴ 𝒇 ≤𝟎⇒ ≤
𝒃 𝒂𝟑 + 𝒃𝟑 𝒃
𝒃 𝒄 𝒄
with equality if and only if = = = 𝟏 ⇒ 𝒂 = 𝒃 = 𝒄 > 0.
𝒂 𝒃 𝒂

62. If 𝒙, 𝒚, 𝒛 > 0 then:

84
SOCIETATEA DE ȘTIINȚE MATEMATICE DIN ROMÂNIA
ROMANIAN MATHEMATICAL SOCIETY

Filiala Mehedinți - Mehedinți Branch


www.ssmrmh.ro

𝒙𝟐 − 𝒙𝒚 + 𝒚𝟐 𝒙𝒚

𝒚𝟐 𝒛𝟐 𝒚𝟒 − 𝒚𝟑 𝒛 + 𝒚𝟐 𝒛𝟐 − 𝒚𝒛𝟑 + 𝒛𝟒
Proposed by Daniel Sitaru – Romania
Solution 1 by Mihalcea Andrei Ștefan – Romania
Solution 2 by Myagmarsuren Yadamsuren-Darkhan-Mongolia
Solution 3 by Soumitra Mandal - Chandar Nagore – India
Solution 1 by Mihalcea Andrei Ștefan – Romania
𝒙𝒚
𝑳𝑯𝑺 ≥
𝒚𝟐 𝒛𝟐
𝒙𝒚 𝒙𝒚
We’ll prove: ≥
𝒚𝟐 𝒛𝟐 𝒚𝟒 .𝒛𝟒 7𝒚𝟑 𝒛7𝒚𝒛𝟑 .𝒚𝟐 𝒛𝟐

⇔ 𝒚𝟒 − 𝒚𝟑 𝒛 + 𝒛𝟒 − 𝒚𝒛𝟑 ≥ 𝟎 ⇔ 𝒚 − 𝒛 𝟐
𝒚𝟐 + 𝒚𝒛 + 𝒛𝟐 ≥ 𝟎 true
Equality for 𝒂 = 𝒃 = 𝒄
Solution 2 by Myagmarsuren Yadamsuren-Darkhan-Mongolia
1. 𝒙𝟐 − 𝒙𝒚 + 𝒚𝟐 = 𝒙𝟐 + 𝒚𝟐 − 𝒙𝒚 ≥ 𝟐𝒙𝒚 − 𝒙𝒚 = 𝒙𝒚
𝑪𝒂𝒖𝒄𝒉𝒚

2. 𝒚𝟐 − 𝒛𝟐 𝟐
≥ 𝟎
𝟏 𝟒 𝟒 𝟐 𝟐 𝟒 𝟒
𝒚𝟒 + 𝒛𝟒
𝟎≤ 𝒚 +𝒛 −𝒚 𝒛 =𝒚 +𝒛 − − 𝒚𝟐 𝒛𝟐 =
𝟐 𝟐
𝒚𝟐 + 𝒛𝟐 𝟐
𝒚𝟐 + 𝒛𝟐 𝑪𝒂𝒖𝒄𝒉𝒚
= 𝒚 𝟒 + 𝒛𝟒 − = 𝒚𝟒 + 𝒛𝟒 − 𝒚𝟐 + 𝒛𝟐 ≤
𝟐 𝟐
≤ 𝒚𝟒 + 𝒛𝟒 − 𝒛𝒚 ⋅ 𝒚𝟐 + 𝒛𝟐
𝒚𝟒 + 𝒛𝟒 − 𝒚𝟑 𝒛 − 𝒚𝒛𝟑 ≥ 𝟎
𝒚𝟒 + 𝒛𝟒 − 𝒚𝟑 𝒛 + 𝒚𝟐 𝒛𝟐 − 𝒚𝒛𝟑 ≥ 𝒚𝟐 𝒛𝟐
𝟏 ; 𝟐
𝒙𝟐 7𝒙𝒚.𝒚𝟐 𝒙𝒚
3. ≥
𝒚𝟐 𝒛𝟐 𝒚𝟒 7𝒚𝟑 𝒛.𝒚𝟐 𝒛𝟐 7𝒚𝒛𝟑 .𝒛𝟒

85
SOCIETATEA DE ȘTIINȚE MATEMATICE DIN ROMÂNIA
ROMANIAN MATHEMATICAL SOCIETY

Filiala Mehedinți - Mehedinți Branch


www.ssmrmh.ro


Solution 3 by Soumitra Mandal - Chandar Nagore – India
𝒙𝟐 − 𝒙𝒚 + 𝒚𝟐 𝒙𝒚

𝒚𝟐 𝒛𝟐 𝒚𝟒 − 𝒚𝟑 𝒛 + 𝒚𝟐 𝒛𝟐 − 𝒚𝒛𝟑 + 𝒛𝟒
𝒄𝒚𝒄 𝒄𝒚𝒄

𝒙𝟑 + 𝒚𝟑 𝒙𝒚 𝒚 + 𝒛
⇔ ≥ ⇔
𝒚𝟐 𝒛𝟐 𝒙 + 𝒚 𝒚𝟓 + 𝒛𝟓
𝒄𝒚𝒄 𝒄𝒚𝒄

𝒙𝟑 .𝒚𝟑 𝒚𝟓 .𝒛𝟓 7𝒙𝒚𝟑 𝒛𝟐 𝒚.𝒛 𝒙.𝒚


⇔ 𝒄𝒚𝒄 ≥ 𝟎 … (1)
𝒚𝟐 𝒛𝟐 𝒙.𝒚 𝒚𝟓 .𝒛𝟓

Now, 𝒙𝟑 + 𝒚𝟑 𝒚𝟓 + 𝒛𝟓 − 𝒙𝒚𝟑 𝒛𝟐 𝒙 + 𝒚 𝒚 + 𝒛
𝟏 𝟑
𝟏 𝟓
≥ 𝒙+𝒚 ⋅ 𝒚+𝒛 − 𝒙𝒚𝟑 𝒛𝟐 𝒙 + 𝒚 𝒚 + 𝒛 ≥ 𝟎
𝟒 𝟏𝟔
Hence, (1) is established.
𝒙𝟐 − 𝒙𝒚 + 𝒚𝟐 𝒙𝒚
∴ ≥
𝒚𝟐 𝒛𝟐 𝒚𝟒 − 𝒚𝟑 𝒛 + 𝒚𝟐 𝒛𝟐 − 𝒚𝒛𝟑 + 𝒛𝟒
𝒄𝒚𝒄 𝒄𝒚𝒄

(proved)

63. If 𝒂, 𝒃, 𝒄 ≥ 𝟏 then:
𝒃 𝟏 + 𝟐𝒂
𝐥𝐨𝐠 𝒂𝒃 ⋅ 𝒃𝒄 ⋅ 𝒄𝒂 + 𝟔 ≥𝟑 𝒂+𝒃+𝒄
𝟏 + 𝟒𝒂 + 𝒂𝟐
Proposed by Daniel Sitaru – Romania
Solution 1,2 by Leonard Giugiuc – Romania
Solution 3 by Soumitra Mandal-Chandar Nagore-India
Solution 1 by Leonard Giugiuc – Romania
𝟔𝒃 𝟏.𝟐𝒂
Lemma: 𝒃 𝐥𝐧 𝒂 + ≥ 𝟑𝒃, ∀𝒂, 𝒃 ≥ 𝟏
𝒂𝟐 .𝟒𝒂.𝟏

86
SOCIETATEA DE ȘTIINȚE MATEMATICE DIN ROMÂNIA
ROMANIAN MATHEMATICAL SOCIETY

Filiala Mehedinți - Mehedinți Branch


www.ssmrmh.ro

Lemma’s proof:
𝟔𝒃 𝟏.𝟐𝒂
For all 𝒂 ≥ 𝟏, let 𝒇 𝒃 = 𝒃 𝐥𝐧 𝒂 + − 𝟑𝒃 on 𝟏, ∞ ;
𝒂𝟐 .𝟒𝒂.𝟏

𝟔 𝟏 + 𝟐𝒂
𝒇• 𝒃 = 𝐥𝐧 𝒂 + − 𝟑
𝒂𝟐 + 𝟒𝒂 + 𝟏
Now let 𝒈 𝒂 = 𝟐 𝒂𝟐 + 𝟒𝒂 + 𝟏 𝒇• 𝒃 = 𝟐 𝒂𝟐 + 𝟒𝒂 + 𝟏 𝐥𝐧 𝒂 − 𝟑𝒂𝟐 + 𝟑
on 𝟏, ∞ . We have:
𝟏 𝟒 𝟏
𝒈• 𝒂 = 𝟐 𝒂 + 𝟐 𝐥𝐧 𝒂 − 𝟓𝒂 + 𝟒 + and 𝒈•• 𝒂 = 𝟐 𝐥𝐧 𝒂 − 𝟑 + − > 0
𝒂 𝒂 𝒂𝟐
¥ ¶7· ¸
𝑔••• 𝑎 = ≥ 0,∀𝒂 ≥ 𝟏 ⇒ 𝒈•• 𝒂 ≥ 𝟎, 𝒈• 𝒂 ≥ 𝟎, 𝒈(𝒂) ≥ 𝟎
¶¹

𝒇 is strictly increasing.
𝒈(𝒂)
Since 𝒇 𝟏 = ≥ 𝟎 the conclusion follows.
𝒂𝟐 .𝟒𝒂.𝟏

Similarly, the other two inequalities. Adding up, we are done.


Solution 2 by Leonard Giugiuc – Romania
𝟔 𝟏.𝟐𝒙
Lemma: The function 𝒇 𝒙 = 𝐥𝐧 𝒙 + is strictly increasing on 𝟏, ∞ .
𝒙𝟐 .𝟒𝒙.𝟏

𝟏 𝟏𝟐 𝒙𝟐 .𝒙.𝟏
Lemma’s proof: 𝒇• 𝒙 = − .
𝒙 𝒙𝟐 .𝟒𝒙.𝟏 𝟐

We have:
𝒇• 𝒙 ≥ 𝟎 ⇔ 𝒙𝟐 + 𝒙 + 𝟏 + 𝟑𝒙 𝟐
≥ 𝟏𝟐𝒙 𝒙𝟐 + 𝒙 + 𝟏 ,
which is true by AM – GM for 𝒖 = 𝒙𝟐 + 𝒙 + 𝟏 and 𝒗 = 𝟑𝒙.
Back to the problem.
By the lemma, 𝒇 𝒙 ≥ 𝒇 𝟏 = 𝟑 ∀𝒙 ≥ 𝟏. We get:
𝟔𝒃 𝟏.𝟐𝒂
𝒃𝒇 𝒂 + 𝒄𝒇 𝒃 + 𝒂𝒇 𝒄 ≥ 𝟑 𝒂 + 𝒃 + 𝒄 ⇒ 𝐥𝐧 𝒂𝒃 𝒃𝒄 𝒄𝒂 + 𝒄𝒚𝒄 𝒂𝟐 .𝟒𝒂.𝟏 ≥ 𝟑 𝒂 + 𝒃 + 𝒄 .


Solution 3 by Soumitra Mandal-Chandar Nagore-India

87
SOCIETATEA DE ȘTIINȚE MATEMATICE DIN ROMÂNIA
ROMANIAN MATHEMATICAL SOCIETY

Filiala Mehedinți - Mehedinți Branch


www.ssmrmh.ro

𝟔 𝟏.𝟐𝒙
Let 𝒇 𝒙 = 𝐥𝐧 𝒙 + − 𝟑 for all 𝒙 ∈ 𝟏, ∞
𝟏.𝟒𝒙.𝒙𝟐

𝟏 𝟏𝟐 𝟏𝟐 𝟏 + 𝟐𝒙 𝒙 + 𝟐
∴ 𝒇• 𝒙 = + 𝟐

𝒙 𝟏 + 𝟒𝒙 + 𝒙 𝟏 + 𝟒𝒙 + 𝒙𝟐 𝟐
𝟏 𝟏𝟐 𝟏.𝒙.𝒙𝟐 𝒙7𝟏 𝟒
= − = ≥ 𝟎 for all 𝒙 ∈ 𝟏, ∞
𝒙 𝟏.𝟒𝒙.𝒙𝟐 𝟐 𝒙 𝟏.𝟒𝒙.𝒙𝟐 𝟐

now 𝒇 is continuous on 𝟏, ∞ and 𝒇• 𝒙 ≥ 𝟎 for all 𝒙 ∈ 𝟏, ∞


∴ 𝒇 𝒙 ≥ 𝒇 𝟏 = 𝟎
∴ for all 𝒂, 𝒃, 𝒄 ∈ 𝟏, ∞ then 𝒂𝒇 𝒄 + 𝒃𝒇 𝒂 + 𝒄𝒇 𝒃 ≥ 𝟎
𝒃 𝟏 + 𝟐𝒂
∴ 𝐥𝐧 𝒂𝒃 𝒃𝒄 𝒄𝒂 + 𝟔 ≥𝟑 𝒂+𝒃+𝒄
𝟏 + 𝟒𝒂 + 𝒂𝟐
𝒄𝒚𝒄

(Proved)
64. If 𝒂, 𝒃, 𝒄 ∈ 𝟎, ∞ then:
𝒂𝟑 + 𝒃𝟑 𝟏 + 𝒂𝒃 𝟏 𝟏 𝟏
≤ + +
𝒂𝟐 + 𝒃𝟒 𝒂𝟒 + 𝒃𝟐 𝒂𝒃 𝒃𝒄 𝒄𝒂
Proposed by Daniel Sitaru – Romania
Solution by Seyran Ibrahimov – Maasilli – Azerbaidjian
𝒂𝟑 + 𝒃𝟑 𝟏 + 𝒂𝒃 𝟏

𝒂𝟐 + 𝒃𝟒 𝒂𝟒 + 𝒃𝟐 𝒂𝒃
𝟏 𝒂𝟑 + 𝒃𝟑 𝟏 + 𝒂𝒃
≥ 𝟐
𝒂𝒃 𝒂 + 𝒃𝟒 𝒂𝟒 + 𝒃𝟐
𝒂𝟐 + 𝒃𝟒 𝒂𝟒 + 𝒃𝟐 ≥ 𝒂𝒃 + 𝒂𝟐 𝒃𝟐 𝒂𝟑 + 𝒃𝟑
𝒂𝟔 + 𝒂𝟐 𝒃𝟐 + 𝒂𝟒 𝒃𝟒 + 𝒃𝟔 ≥ 𝒂𝟒 𝒃 + 𝒂𝒃𝟒 + 𝒂𝟓 𝒃𝟐 + 𝒂𝟐 𝒃𝟓
𝒂𝟔 + 𝒂𝟐 𝒃𝟐 ≥ 𝟐𝒂𝟒 𝒃
𝒂𝟔 + 𝒂𝟒 𝒃𝟒 ≥ 𝟐𝒂𝟓 𝒃𝟐
+ proved(AM-GM)
𝒃𝟔 + 𝒂𝟐 𝒃𝟐 ≥ 𝟐𝒃𝟒 𝒂
𝒃𝟔 + 𝒂𝟒 𝒃𝟒 ≥ 𝟐𝒃𝟓 𝒂𝟐

88
SOCIETATEA DE ȘTIINȚE MATEMATICE DIN ROMÂNIA
ROMANIAN MATHEMATICAL SOCIETY

Filiala Mehedinți - Mehedinți Branch


www.ssmrmh.ro

65. If 𝒂, 𝒃, 𝒄, 𝒅 > 0, 𝑎𝑏𝑐𝑑 = 1 then:

𝒂−𝟏 𝒂−𝟐 𝒂 − 𝟑 ≥ 𝟔

Proposed by Daniel Sitaru – Romania


Solution 1 by Mihalcea Andrei Ștefan – Romania
Solution 2 by Seyran Ibrahimov-Maasilli-Azerbaidjian
Solution 1 by Mihalcea Andrei Ștefan – Romania
𝟐
Inequality (2) 𝒂−𝟒 𝒂 −𝟐 𝒂 + 𝟑 ≥ 𝟎
𝟒
true because 𝒂 ≥ 𝟒 𝒂𝒃𝒄𝒅 = 𝟒
Solution 2 by Seyran Ibrahimov-Maasilli-Azerbaidjian
𝒙 = 𝒂 + 𝒃 + 𝒄 + 𝒅 ≥ 𝟒
𝒙 − 𝟏 𝒙 − 𝟐 𝒙 − 𝟑 ≥ 𝟔
𝒙 − 𝟏 𝒙𝟐 − 𝟓𝒙 + 𝟔 ≥ 𝟔
𝒙𝟑 − 𝟓𝒙𝟐 + 𝟔𝒙 − 𝒙𝟐 + 𝟓𝒙 − 𝟔 ≥ 𝟔
𝒙𝟑 − 𝟔𝒙𝟐 + 𝟏𝟏𝒙 − 𝟏𝟐 ≥ 𝟎
𝒙𝟐 𝒙 − 𝟒 − 𝟐𝒙 𝒙 − 𝟒 + 𝟑 𝒙 − 𝟒 ≥ 𝟎
𝒙 − 𝟒 𝒙𝟐 − 𝟐𝒙 + 𝟑 ≥ 𝟎 ⇒ (+ ⋅ +) ≥ 𝟎
therefore 𝒙 ≥ 𝟒 ⇒ Equality
𝒙𝟐 − 𝟐𝒙 + 𝟑𝒙 = 𝒙 − 𝟏 𝟐
+ 𝟐

66. If 𝒂, 𝒃, 𝒄 ∈ ℝ, 𝒂𝒃𝒄 = 𝟏 then:
𝟏 𝒄 𝟏 𝒂 𝟏 𝒃
𝒂− + 𝒃− + 𝒄− + < 4
𝒂 𝒃 𝒃 𝒄 𝒄 𝒂
Proposed by Daniel Sitaru – Romania

89
SOCIETATEA DE ȘTIINȚE MATEMATICE DIN ROMÂNIA
ROMANIAN MATHEMATICAL SOCIETY

Filiala Mehedinți - Mehedinți Branch


www.ssmrmh.ro

Solution by Redwane El Mellass-Morroco


𝟏 𝒄
Let 𝜟 𝒂, 𝒃, 𝒄 ∈ ℝ∗ = 𝒂 − + .
𝒂 𝒃

𝒂 𝟏 𝒂 𝟐 𝟏 𝟐 𝟏
𝜟 𝒂, 𝒃, 𝒄 = 𝒄 − 𝒃− 𝜟 𝒂, 𝒃, 𝒄 = − 𝒃− 𝒄𝟐 + − 𝟏
𝒄 𝒃 𝒄 𝒃 𝒂𝟐

(1)
𝒃 𝟏 𝒃 𝟐 𝟏 𝟐 𝟏
∴ 𝜟 𝒃, 𝒄, 𝒂 = 𝒂 − 𝒄− ⇒ 𝜟 𝒂, 𝒃, 𝒄 = − 𝒄− 𝒂𝟐 + − 𝟏 (2)
𝒂 𝒄 𝒂 𝒄 𝒃𝟐

𝒄 𝟏 𝒄 𝟐 𝟏 𝟐 𝟏
𝜟 𝒄, 𝒂, 𝒃 = 𝒃 − 𝒂− 𝜟 𝒂, 𝒃, 𝒄 = − 𝒂− 𝒃𝟐 + − 𝟏 (3)
𝒃 𝒂 𝒃 𝒂 𝒄𝟐

Now lets study some cases about 𝒂 , 𝒃 and 𝒄 :


𝟏
If 𝒂 ≤ 𝒃 ≤ 𝒄 : ∴ 𝒂 ≤ 𝟏 ≤ 𝒄 ⇒ 𝒄𝟐 + − 𝟏 > 0
𝒂𝟐

𝒂 𝟐 𝟏 𝒂 𝟐 𝟏 𝟏
then (1) ⇒ 𝜟 𝒂, 𝒃, 𝒄 ≤ 𝒄𝟐 + 𝟐
−𝟏 < 𝒄𝟐 + = 𝒂𝟐 + ≤ 𝟐.
𝒄 𝒂 𝒄 𝒂𝟐 𝒄𝟐

By the same idea, if 𝒃 ≤ 𝒄 ≤ 𝒂 we use (2) and if 𝒄 ≤ 𝒂 ≤ 𝒃 we use (3).


𝟏
𝒂𝟐 + −𝟏>0
𝒃𝟐
If 𝒄 ≤ 𝒃 ≤ 𝒂 : ∴ 𝒄 ≤ 𝟏 ≤ 𝒂 ⇒ 𝟏

𝒃𝟐 + 𝟐
−𝟏>0
𝒄

𝒃 𝟐 𝟏 𝒃 𝟐 𝟏 𝟏
If 𝒃 ≤ 𝟏: ∴ (2) ⇒ 𝜟 𝒂, 𝒃, 𝒄 ≤ 𝒂𝟐 + −𝟏 < 𝒂𝟐 + = 𝒃𝟐 + ≤ 𝟐
𝒂 𝒃𝟐 𝒂 𝒃𝟐 𝒂𝟐

𝒄 𝟐 𝟏 𝒄 𝟐 𝟏 𝟏
If 𝒃 ≥ 𝟏: ∴ (3) ⇒ 𝜟 𝒂, 𝒃, 𝒄 ≤ 𝒃𝟐 + −𝟏 < 𝒃𝟐 + = 𝒄𝟐 + ≤ 𝟐
𝒃 𝒄𝟐 𝒃 𝒄𝟐 𝒃𝟐

By the same idea, if 𝒂 ≤ 𝒄 ≤ 𝒃 we use (1) and (3) and if 𝒃 ≤ 𝒂 ≤ 𝒄 we


use (1) and (2).
Finally 𝜟 𝒂, 𝒃, 𝒄 ∈ ℝ∗ 𝒂𝒃𝒄 = 𝟏 < 2 < 4.
67. Let 𝒙, 𝒚, 𝒛 be positive real numbers such that: 𝒙𝒚𝒛 = 𝟏. Prove that:
𝒙 𝒚 𝒛 𝟐 𝒙.𝒚.𝒛 𝟓
+ + + ≥ (1)
𝟐 𝒙𝟒 .𝒚𝟒 .𝟒𝒙𝒚 𝟐(𝒚𝟒 .𝒛𝟒 ).𝟒𝒚𝒛 𝟐 𝒛𝟒 .𝒙𝟒 .𝟒𝒛𝒙 𝟑 𝟐

Proposed by Hoang Le Nhat Tung – Hanoi – Vietnam

90
SOCIETATEA DE ȘTIINȚE MATEMATICE DIN ROMÂNIA
ROMANIAN MATHEMATICAL SOCIETY

Filiala Mehedinți - Mehedinți Branch


www.ssmrmh.ro

Solution by Hoang Le Nhat Tung – Hanoi – Vietnam


* Since Inequality Cauchy – Schwarz. We have:
𝟐
𝟐 𝒙𝟒 + 𝒚𝟒 + 𝟐𝒙𝒚 ≤ 𝟏𝟐 + 𝟏𝟐 𝟐 𝒙𝟒 + 𝒚𝟒 + 𝟒𝒙𝟐 𝒚𝟐 =

= 𝟒 𝒙𝟒 + 𝟐𝒙𝟐 𝒚𝟐 + 𝒚𝟒 = 𝟒 𝒙𝟐 + 𝒚𝟐 𝟐
⇔ 𝟐 𝒙𝟒 + 𝒚𝟒 + 𝟐𝒙𝒚 ≤ 𝟐 𝒙𝟐 + 𝒚𝟐 ⇔ 𝟐 𝒙𝟒 + 𝒚𝟒 + 𝟒𝒙𝒚 ≤ 𝟐 𝒙𝟐 + 𝒙𝒚 + 𝒚𝟐
𝟏 𝟏
⇔ ≥ ⇔
𝟐 𝒙𝟒 + 𝒚𝟒 + 𝟒𝒙𝒚 𝟐 𝒙𝟐 + 𝒙𝒚 + 𝒚𝟐
𝒙 𝒙
⇔ ≥ (2)
𝟐 𝒙𝟒 .𝒚𝟒 .𝟒𝒙𝒚 𝟐 𝒙𝟐 .𝒙𝒚.𝒚𝟐

- Similar:
𝒚 𝒚 𝒛 𝒛
≥ ; ≥ (3)
𝟐 𝒙𝟒 .𝒚𝟒 .𝟒𝒚𝒛 𝟐 𝒚𝟐 .𝒚𝒛.𝒛𝟐 𝟐 𝒛𝟒 .𝒙𝟒 .𝟒𝒛𝒙 𝟐 𝒛𝟐 .𝒛𝒙.𝒙𝟐

- Since (2), (3):


𝒙 𝒚 𝒛
⇒ + + ≥
𝟐 𝒙𝟒 + 𝒚𝟒 + 𝟒𝒙𝒚 𝟐 𝒚𝟒 + 𝒛𝟒 + 𝟒𝒚𝒛 𝟐 𝒛𝟒 + 𝒙𝟒 + 𝟒𝒛𝒙
𝒙 𝒚 𝒛
≥ + + (4)
𝟐 𝒙𝟐 .𝒙𝒚.𝒚𝟐 𝟐 𝒚𝟐 .𝒚𝒛.𝒛𝟐 𝟐 𝒛𝟐 .𝒛𝒙.𝒙𝟐

- Other, Since inequality Cauchy – Schwarz:


𝒙 𝒚 𝒛
+ + =
𝒙𝟐 + 𝒙𝒚 + 𝒚𝟐 𝒚𝟐 + 𝒚𝒛 + 𝒛𝟐 𝒛𝟐 + 𝒛𝒙 + 𝒙𝟐
𝒙𝟐 𝒚𝟐 𝒛𝟐
= 𝟑 + + ≥
𝒙 + 𝒙𝟐 𝒚 + 𝒙𝒚𝟐 𝒚𝟑 + 𝒚𝟐 𝒛 + 𝒚𝒛𝟐 𝒛𝟑 + 𝒛𝟐 𝒙 + 𝒛𝒙𝟐
𝒙+𝒚+𝒛 𝟐
≥ 𝟑
𝒙 + 𝒙𝟐 𝒚 + 𝒙𝒚𝟐 + 𝒚𝟑 + 𝒚𝟐 𝒛 + 𝒚𝒛𝟐 + 𝒛𝟑 + 𝒛𝟐 𝒙 + 𝒛𝒙𝟐
𝒙 𝒚 𝒛
⇔ 𝟐 + + ≥
𝒙 + 𝒙𝒚 + 𝒚𝟐 𝒚𝟐 + 𝒚𝒛 + 𝒛𝟐 𝒛𝟐 + 𝒛𝒙 + 𝒙𝟐

91
SOCIETATEA DE ȘTIINȚE MATEMATICE DIN ROMÂNIA
ROMANIAN MATHEMATICAL SOCIETY

Filiala Mehedinți - Mehedinți Branch


www.ssmrmh.ro

𝒙+𝒚+𝒛 𝟐
≥ 𝟐
𝒙 𝒙 + 𝒚 + 𝒛 + 𝒚𝟐 𝒙 + 𝒚 + 𝒛 + 𝒛𝟐 𝒙 + 𝒚 + 𝒛
𝒙 𝒚 𝒛 𝒙.𝒚.𝒛 𝟐 𝒙.𝒚.𝒛
⇔ + + ≥ = (5)
𝒙𝟐 .𝒙𝒚.𝒚 𝟐 𝒚𝟐 .𝒚𝒛.𝒛 𝟐 𝒛𝟐 .𝒛𝒙.𝒙𝟐 𝒙.𝒚.𝒛 𝒙𝟐 .𝒚𝟐 .𝒛𝟐 𝒙𝟐 .𝒚𝟐 .𝒛𝟐

- Since (4), (5):


𝒙 𝒚 𝒛 𝒙.𝒚.𝒛
⇒ + + ≥
𝟐 𝒙𝟒 .𝒚𝟒 .𝟒𝒙𝒚 𝟐 𝒚𝟒 .𝒛𝟒 .𝟒𝒚𝒛 𝟐 𝒛𝟒 .𝒙𝟒 .𝟒𝒛𝒙 𝟐 𝒙𝟐 .𝒚𝟐 .𝒛𝟐

𝒙 𝒚 𝒛 𝟐 𝒙.𝒚.𝒛
⇔ + + + ≥
𝟐(𝒙𝟒 .𝒚𝟒 ).𝟒𝒙𝒚 𝟐 𝒚𝟒 .𝒛𝟒 .𝟒𝒚𝒛 𝟐 𝒛𝟒 .𝒙𝟒 .𝟒𝒛𝒙 𝟑

𝒙.𝒚.𝒛 𝟐 𝒙.𝒚.𝒛
≥ + (6)
𝟐 𝒙𝟐 .𝒚𝟐 .𝒛𝟐 𝟑

- Since Inequality AM-GM for 5 positive real numbers we have:


𝒙+𝒚+𝒛 𝟐 𝒙+𝒚+𝒛
+ =
𝟐 𝒙𝟐 + 𝒚𝟐 + 𝒛𝟐 𝟑
𝒙+𝒚+𝒛 𝒙+𝒚+𝒛 𝒙+𝒚+𝒛 𝒙+𝒚+𝒛 𝒙+𝒚+𝒛
= + + + + ≥
𝟐 𝒙𝟐 + 𝒚𝟐 + 𝒛𝟐 𝟔 𝟔 𝟔 𝟔
𝟓 𝒙.𝒚.𝒛 𝒙.𝒚.𝒛 𝒙.𝒚.𝒛 𝒙.𝒚.𝒛 𝒙.𝒚.𝒛
≥𝟓⋅ ⋅ ⋅ ⋅ ⋅
𝟐 𝒙𝟐 .𝒚𝟐 .𝒛𝟐 𝟔 𝟔 𝟔 𝟔

𝒙+𝒚+𝒛 𝟐 𝒙+𝒚+𝒛 𝟓 𝒙+𝒚+𝒛 𝟓


⇔ + ≥𝟓⋅
𝟐 𝒙𝟐 + 𝒚𝟐 + 𝒛𝟐 𝟑 𝟔 𝟒 ⋅ 𝟐 𝒙𝟐 + 𝒚𝟐 + 𝒛𝟐
𝟐
- Since inequality: 𝒙𝒚 + 𝒚𝒛 + 𝒛𝒙 ≥ 𝟑𝒙𝒚𝒛 𝒙 + 𝒚 + 𝒛 and supposed:
𝒙𝒚𝒛 = 𝟏. We have:
𝒙 + 𝒚 + 𝒛 𝒙𝟐 + 𝒚𝟐 + 𝒛𝟐 = 𝟏 ⋅ 𝒙 + 𝒚 + 𝒛 𝒙𝟐 + 𝒚𝟐 + 𝒛𝟐 =
𝟐
𝟐 𝟐 𝟐
𝒙𝒚 + 𝒚𝒛 + 𝒛𝒙
= 𝒙𝒚𝒛 𝒙 + 𝒚 + 𝒛 𝒙 + 𝒚 + 𝒛 ≤ ⋅ 𝒙𝟐 + 𝒚𝟐 + 𝒛𝟐 =
𝟑
𝒙𝟐 + 𝒚𝟐 + 𝒛𝟐 𝒙𝒚 + 𝒚𝒛 + 𝒛𝒙 𝒙𝒚 + 𝒚𝒛 + 𝒛𝒙
=
𝟑

92
SOCIETATEA DE ȘTIINȚE MATEMATICE DIN ROMÂNIA
ROMANIAN MATHEMATICAL SOCIETY

Filiala Mehedinți - Mehedinți Branch


www.ssmrmh.ro

𝒙𝟐 .𝒚𝟐 .𝒛𝟐 𝒙𝒚.𝒚𝒛.𝒛𝒙 𝒙𝒚.𝒚𝒛.𝒛𝒙


⇔ 𝒙 + 𝒚 + 𝒛 𝒙𝟐 + 𝒚𝟐 + 𝒛𝟐 ≤ (8)
𝟒

- Other, Inequality AM-GM for 3 positive real numbers:


𝟐
𝒙+𝒚+𝒛 = 𝒙𝟐 + 𝒚𝟐 + 𝒛𝟐 + 𝒙𝒚 + 𝒚𝒛 + 𝒛𝒙 + 𝒙𝒚 + 𝒚𝒛 + 𝒛𝒙 ≥
𝟑
≥𝟑⋅ 𝒙𝟐 + 𝒚𝟐 + 𝒛𝟐 ⋅ 𝒙𝒚 + 𝒚𝒛 + 𝒛𝒙 ⋅ 𝒙𝒚 + 𝒚𝒛 + 𝒛𝒙
𝟔
⇔ 𝒙+𝒚+𝒛 ≥ 𝟐𝟕 𝒙𝟐 + 𝒚𝟐 + 𝒛𝟐 𝒙𝒚 + 𝒚𝒛 + 𝒛𝒙 𝒙𝒚 + 𝒚𝒛 + 𝒛𝒙
𝒙.𝒚.𝒛 𝟔
⇔ 𝒙𝟐 + 𝒚𝟐 + 𝒛𝟐 𝒙𝒚 + 𝒚𝒛 + 𝒛𝒙 𝒙𝒚 + 𝒚𝒛 + 𝒛𝒙 ≤ (9)
𝟐𝟕
𝒙.𝒚.𝒛 𝟔 𝒙.𝒚.𝒛 𝟔
- Since (8), (9): ⇒ 𝒙 + 𝒚 + 𝒛 𝒙𝟐 + 𝒚𝟐 + 𝒛𝟐 ≤ =
𝟐𝟕⋅𝟑 𝟖𝟏

𝒙.𝒚.𝒛 𝟓 𝒙.𝒚.𝒛 𝟓
⇔ 𝒙𝟐 + 𝒚𝟐 + 𝒛𝟐 ≤ ⇔ ≥ 𝟖𝟏 (10)
𝟖𝟏 𝒙𝟐 .𝒚𝟐 .𝒛𝟐

𝒙.𝒚.𝒛 𝟐 𝒙.𝒚.𝒛 𝟓 𝟖𝟏 𝟓 𝟑𝟒 𝟓 𝟏 𝟓
- Since (7), (10): ⇒ + ≥𝟓⋅ =𝟓⋅ =𝟓⋅ = (11)
𝟐 𝒙𝟐 .𝒚𝟐 .𝒛𝟐 𝟑 𝟔𝟒 ⋅𝟐 𝟐⋅𝟐𝟒 ⋅𝟑𝟒 𝟐𝟓 𝟐

- Since (6), (11):


𝒙 𝒚 𝒛 𝟐 𝒙.𝒚.𝒛 𝟓
⇒ + + + ≥
𝟐(𝒙𝟒 .𝒚𝟒 ).𝟒𝒙𝒚 𝟐 𝒚𝟒 .𝒛𝟒 .𝟒𝒚𝒛 𝟐 𝒛𝟒 .𝒙𝟒 .𝟒𝒛𝒙 𝟑 𝟐

⇒ Inequality (1) True and we get the desired result.


+ Equality occurs if:
𝒙, 𝒚, 𝒛 > 0; 𝑥𝑦𝑧 = 1
𝟐(𝒙𝟒 + 𝒚𝟒 ) = 𝟐𝒙𝒚; 𝟐 𝒚𝟒 + 𝒛𝟒 = 𝟐𝒚𝒛; 𝟐 𝒛𝟒 + 𝒙𝟒 = 𝟐𝒛𝒙
𝟏 𝟏 𝟏
= =
⇔ 𝒙𝟐 + 𝒙𝒚 + 𝒚𝟐 𝒚𝟐 + 𝒚𝒛 + 𝒛𝟐 𝒛𝟐 + 𝒙𝒛 + 𝒙𝟐 ⇔ 𝒙 = 𝒚 = 𝒛 = 𝟏
𝒙+𝒚+𝒛 𝒙+𝒚+𝒛
=
𝟐 𝒙𝟐 + 𝒚𝟐 + 𝒛𝟐 𝟔
𝒙𝒚 = 𝒚𝒛 = 𝒛𝒙
𝒙𝟐 + 𝒚𝟐 + 𝒛𝟐 = 𝒙𝒚 + 𝒚𝒛 + 𝒛𝒙
68. If 𝒙, 𝒚, 𝒛 > 0 then:
𝒙 𝟑 𝟑 𝒙
≤ , ≤
𝟕𝒙 + 𝟓 𝒚 + 𝒛 𝟏𝟕 𝟏𝟗 𝟓𝒙 + 𝟕 𝒚 + 𝒛

93
SOCIETATEA DE ȘTIINȚE MATEMATICE DIN ROMÂNIA
ROMANIAN MATHEMATICAL SOCIETY

Filiala Mehedinți - Mehedinți Branch


www.ssmrmh.ro

Proposed by Daniel Sitaru – Romania


Solution 1 by Abdallah El Farissi-Bechar-Algerie
Solution 2 by Ravi Prakash-New Delhi-India
Solution 3 by Imad Zak-Saida-Lebanon
Solution 1 by Abdallah El Farissi-Bechar-Algerie
𝒂
Let 𝒇 𝒂 = , 𝒂 > 0 we have
𝟐𝒂.𝟓 𝒙.𝒚.𝒛

7𝟐𝟎 𝒙.𝒚.𝒛
𝒇•• 𝒂 = 𝟑 < 0 then 𝒇 is concave function,
𝟐𝒂.𝟓 𝒙.𝒚.𝒛

𝒙 𝒇 𝒙 +𝒇 𝒚 +𝒇 𝒛 𝒙+𝒚+𝒛 𝟑
=𝟑 ≤ 𝟑𝒇 =
𝟕𝒙 + 𝟓 𝒚 + 𝒛 𝟑 𝟑 𝟏𝟕
𝒂 𝟕
Let 𝒈 𝒂 = ,𝒂 < 𝒙 + 𝒚 + 𝒛 , we have
𝟕 𝒙.𝒚.𝒛 7𝟐𝒂 𝟐

𝟐𝟖 𝒙.𝒚.𝒛
𝒈•• 𝒂 = ≥ 𝟎, then 𝒈 is convex function,
𝟕 𝒙.𝒚.𝒛 7𝟐𝒂 𝟑

𝒙 𝒇 𝒙 +𝒇 𝒚 +𝒇 𝒛 𝒙+𝒚+𝒛 𝟑
=𝟑 ≥ 𝟑𝒇 =
𝟓𝒙 + 𝟕 𝒚 + 𝒛 𝟑 𝟑 𝟏𝟗

Solution 2 by Ravi Prakash-New Delhi-India
Consider
𝒙 𝟏 𝟏𝟕𝒙 − 𝟕𝒙 − 𝟓 𝒚 + 𝒛 𝟓 𝒙−𝒚 +𝟓 𝒙−𝒛
𝑬𝟏 = − = =
𝟓 𝒚 + 𝒛 + 𝟕𝒙 𝟏𝟕 𝟏𝟕 𝟕𝒙 + 𝟓 𝒚 + 𝒛 𝟏𝟕 𝟕𝒙 + 𝟓𝒚 + 𝟓𝒛
Similarly,
𝟓 𝒚7𝒙 .𝟓 𝒚7𝒛 𝟓 𝒛7𝒙 .𝟓 𝒛7𝒚
𝑬𝟐 = and 𝑬𝟐 =
𝟏𝟕 𝟕𝒚.𝟓𝒙.𝟓𝒛 𝟏𝟕 𝟏𝟕𝒛.𝟓𝒙.𝟓𝒚
𝟏𝟕
Now, 𝑬𝟏 + 𝑬𝟐 + 𝑬𝟑
𝟓

94
SOCIETATEA DE ȘTIINȚE MATEMATICE DIN ROMÂNIA
ROMANIAN MATHEMATICAL SOCIETY

Filiala Mehedinți - Mehedinți Branch


www.ssmrmh.ro

𝟏 𝟏 𝟏 𝟏
= 𝒙−𝒚 − + 𝒚−𝒛 − +
𝟕𝒙 + 𝟓𝒚 + 𝟓𝒛 𝟕𝒚 + 𝟓𝒙 + 𝟓𝒛 𝟕𝒚 + 𝟓𝒙 + 𝟓𝒛 𝟕𝒛 + 𝟓𝒙 + 𝟓𝒚
𝟏 𝟏 −𝟐 𝒙 − 𝒚 𝟐
+ 𝒛−𝒙 − = +
𝟕𝒛 + 𝟓𝒙 + 𝟓𝒚 𝟕𝒙 + 𝟓𝒚 + 𝟓𝒛 𝟕𝒙 + 𝟓𝒚 + 𝟓𝒛 𝟓𝒙 + 𝟕𝒚 + 𝟓𝒛
−𝟐 𝒚 − 𝒛 𝟐 −𝟐 𝒛 − 𝒙 𝟐
+ + ≤ 𝟎
𝟓𝒙 + 𝟕𝒚 + 𝟓𝒛 𝟓𝒙 + 𝟓𝒚 + 𝟕𝒛 𝟓𝒙 + 𝟓𝒚 + 𝟕𝒛 𝟕𝒙 + 𝟓𝒚 + 𝟓𝒛
𝒙 𝟑
∴ ≤
𝟕𝒙 + 𝟓 𝒚 + 𝒛 𝟏𝟕
Next, consider
𝟏 𝒙 𝟕 𝒚 + 𝒛 + 𝟓𝒙 − 𝟏𝟗𝒙 𝟕 𝒚 − 𝒙 + 𝟕 𝒛 − 𝒙
𝑭𝟏 = − = =
𝟏𝟗 𝟓𝒙 + 𝟕 𝒚 + 𝒛 𝟏𝟗 𝟓𝒙 + 𝟕𝒚 + 𝟕𝒛 𝟏𝟗 𝟓𝒙 + 𝟕𝒚 + 𝟕𝒛
Similarly,
𝟕 𝒛−𝒚 +𝟕 𝒙−𝒚 𝟕 𝒙−𝒛 +𝟕 𝒚−𝒛
𝑭𝟐 = , 𝑭𝟑 =
𝟏𝟗 𝟓𝒚 + 𝟕𝒙 + 𝟕𝒛 𝟏𝟗 𝟓𝒛 + 𝟕𝒙 + 𝟕𝒚
𝟏𝟗
∴ 𝑭𝟏 + 𝑭𝟐 + 𝑭𝟑
𝟕
𝟏 𝟏 𝟏 𝟏
= 𝒚−𝒙 − + 𝒙−𝒛 − +
𝟓𝒙 + 𝟕𝒚 + 𝟕𝒛 𝟓𝒚 + 𝟕𝒙 + 𝟕𝒛 𝟓𝒛 + 𝟕𝒙 + 𝟕𝒚 𝟓𝒙 + 𝟕𝒚 + 𝟕𝒛
𝟏 𝟏 −𝟐 𝒚 − 𝒙 𝟐
+ 𝒛−𝒚 − = +
𝟓𝒚 + 𝟕𝒙 + 𝟕𝒛 𝟓𝒛 + 𝟕𝒙 + 𝟕𝒚 𝟓𝒙 + 𝟕𝒚 + 𝟕𝒛 𝟓𝒚 + 𝟕𝒙 + 𝟕𝒛
−𝟐 𝒙 − 𝒛 𝟐 −𝟐 𝒛 − 𝒚 𝟐
+ + ≤ 𝟎
𝟓𝒛 + 𝟕𝒙 + 𝟕𝒚 𝟓𝒙 + 𝟕𝒚 + 𝟕𝒛 𝟓𝒚 + 𝟕𝒙 + 𝟕𝒛 𝟓𝒛 + 𝟕𝒙 + 𝟕𝒚
𝒙 𝟑
⇒ ≥
𝟓𝒙 + 𝟕 𝒚 + 𝒛 𝟏𝟗
Solution 3 by Imad Zak-Saida-Lebanon
𝒙
Let 𝒇 𝒙 = for 𝒙 ∈ 𝟎; 𝟑 we have:
𝟐𝒙.𝟏𝟓

𝟏𝟓𝒙 𝟐 7𝟑𝟎 𝒙7𝟏 𝟐 𝟏𝟓𝒙 𝟐


𝒇 𝒙 − + = ≤𝟎⇒𝒇 𝒙 ≤ + … (1)
𝟐𝟖𝟗 𝟐𝟖𝟗 𝟐𝟖𝟗 𝟐𝒙.𝟓 𝟐𝟖𝟗 𝟐𝟖𝟗
𝒙
Let 𝒈 𝒙 = for 𝒙 ∈ 𝟎; 𝟑 we have:
𝟐𝟏7𝟐𝒙

95
SOCIETATEA DE ȘTIINȚE MATEMATICE DIN ROMÂNIA
ROMANIAN MATHEMATICAL SOCIETY

Filiala Mehedinți - Mehedinți Branch


www.ssmrmh.ro

𝟐𝟏𝒙 𝟐 𝟒𝟐 𝒙7𝟏 𝟐 𝟐𝟏𝒙 𝟐


𝒈 𝒙 − − = ≥𝟎⇒𝒈 𝒙 ≥ − … (2)
𝟑𝟔𝟏 𝟑𝟔𝟏 𝟑𝟔𝟏 𝟐𝟏7𝟐𝒙 𝟑𝟔𝟏 𝟑𝟔𝟏

Both inqualities are homogeneous, so let 𝒙 + 𝒚 + 𝒛 = 𝟑


? 𝟑
Ineq (1) ⇔ 𝒇 𝒙 ≤ acc. to (1) we may write!
𝟏𝟕
𝟏𝟓𝒙 𝟐 𝟏𝟓 𝒙.𝒚.𝟑 𝟔 𝟒𝟓.𝟔 𝟑
𝒇 𝒙 ≤ + = + = = Q.E.D.
𝟐𝟖𝟗 𝟐𝟖𝟗 𝟐𝟖𝟗 𝟐𝟖𝟗 𝟐𝟖𝟗 𝟏𝟕
?! 𝟑
Ineq (2) ⇔ 𝒈 𝒙 ≥ acc. to (2) we may affirm
𝟏𝟗
𝟐𝟏𝒙 𝟐 𝟐𝟏 𝟑 7𝟔 𝟓𝟕 𝟑
𝒈 𝒙 ≥ − = = = Q.E.D.
𝟑𝟔𝟏 𝟑𝟔𝟏 𝟑𝟔𝟏 𝟑𝟔𝟏 𝟏𝟗

≪=≫ when 𝒙 = 𝒚 = 𝒛 = 𝟏 or 𝒙 = 𝒚 = 𝒛 in general


𝟏𝟓𝒙 𝟐
N.B.: 𝒚 = + is the tangent, at 𝒙 = 𝟏 to 𝑮𝒇
𝟐𝟖𝟗 𝟐𝟖
𝟐𝟏𝒙 𝟐
𝒚= − // // // // // // to 𝑮𝒈
𝟑𝟔𝟏 𝟑𝟔𝟏


69. Let 𝒂, 𝒃, 𝒄 be positive real numbers such that: 𝒂 + 𝒃 + 𝒄 = 𝟑.
Prove that:
𝒂𝟑 𝒃𝟑 𝒄𝟑 𝟏
+ + ≥
𝒃 𝟐𝒃𝟐 − 𝒃𝒄 + 𝟐𝒄𝟐 𝟐 𝒄 𝟐𝒄𝟐 − 𝒄𝒂 + 𝟐𝒂𝟐 𝟐 𝒂 𝟐𝒂𝟐 − 𝒂𝒃 + 𝟐𝒃𝟐 𝟐 𝟑
Proposed by Hoang Le Nhat Tung – Hanoi – Vietnam
Solution 1 by proposer
Solution 2 by Anh Tai Tran – Hanoi – Vietnam
Solution 3 by Leonard Giugiuc – Romania
Solution 1 by proposer
* We have inequality:
𝒂𝟒 + 𝒃𝟒 + 𝒄𝟒 + 𝒂𝒃𝒄 𝒂 + 𝒃 + 𝒄 ≥ 𝒂𝒃 𝒂𝟐 + 𝒃𝟐 + 𝒃𝒄 𝒃𝟐 + 𝒄𝟐 + 𝒄𝒂 𝒄𝟐 + 𝒂𝟐 (2)

96
SOCIETATEA DE ȘTIINȚE MATEMATICE DIN ROMÂNIA
ROMANIAN MATHEMATICAL SOCIETY

Filiala Mehedinți - Mehedinți Branch


www.ssmrmh.ro

- (2) 𝒂𝟒 + 𝒃𝟒 + 𝒄𝟒 + 𝒂𝒃𝒄 𝒂 + 𝒃 + 𝒄 ≥ 𝒂𝒃 𝒂𝟐 + 𝒃𝟐 + 𝒃𝒄 𝒃𝟐 + 𝒄𝟐 + 𝒄𝒂 𝒄𝟐 + 𝒂𝟐
⇔ 𝒂𝟒 + 𝒃𝟒 + 𝒄𝟒 + 𝒂𝒃𝒄 𝒂 + 𝒃 + 𝒄 − 𝒂𝒃 𝒂𝟐 + 𝒃𝟐 − 𝒃𝒄 𝒃𝟐 + 𝒄𝟐 − 𝒄𝒂 𝒄𝟐 + 𝒂𝟐 ≥ 𝟎
⇔ 𝒂𝟐 𝒂𝟐 − 𝒂𝒃 − 𝒂𝒄 + 𝒃𝒄 + 𝒃𝟐 𝒃𝟐 − 𝒃𝒄 − 𝒃𝒂 + 𝒄𝒂 + 𝒄𝟐 𝒄𝟐 − 𝒄𝒂 − 𝒄𝒃 + 𝒂𝒃 ≥ 𝟎
⇔ 𝒂𝟐 𝒂 − 𝒃 𝒂 − 𝒄 + 𝒃𝟐 𝒃 − 𝒂 𝒃 − 𝒄 + 𝒄𝟐 𝒄 − 𝒂 𝒄 − 𝒃 ≥ 𝟎 (3)
* 𝒂 ≥ 𝒃 ≥ 𝒄 > 0.
𝒄≤𝒂 𝒄−𝒂≤𝟎
+ We have: ⇔ ⇒ 𝒄 − 𝒂 𝒄 − 𝒃 ≥ 𝟎 ⇔ 𝒄𝟐 𝒄 − 𝒂 𝒄 − 𝒃 ≥ 𝟎 (4)
𝒄≤𝒃 𝒄−𝒃≤𝟎
+ We have: 𝒂𝟐 𝒂 − 𝒃 𝒂 − 𝒄 + 𝒃𝟐 𝒃 − 𝒂 𝒃 − 𝒄 = 𝒂 − 𝒃 𝒂𝟐 𝒂 − 𝒄 − 𝒃𝟐 𝒃 − 𝒄
⇔ 𝒂𝟐 𝒂 − 𝒃 𝒂 − 𝒄 + 𝒃 𝟐 𝒃 − 𝒂 𝒃 − 𝒄 = 𝒂 − 𝒃 𝒂𝟑 − 𝒃 𝟑 − 𝒄 𝒂𝟐 − 𝒃 𝟐
= 𝒂−𝒃 𝒂 − 𝒃 𝒂𝟐 + 𝒂𝒃 + 𝒃𝟐 − 𝒄 𝒂 − 𝒃 𝒂 + 𝒃
= 𝒂 − 𝒃 ⋅ 𝒂 − 𝒃 𝒂𝟐 + 𝒂𝒃 + 𝒃𝟐 − 𝒂𝒄 − 𝒃𝒄
𝟐
= 𝒂−𝒃 𝒂𝟐 + 𝒂𝒃 + 𝒃𝟐 − 𝒂𝒄 − 𝒃𝒄 (5)
𝒂 ≥ 𝒃 ≥ 𝒄 > 0 ⇒ 𝒂 − 𝒄 ≥ 𝟎; 𝒃 − 𝒄 ≥ 𝟎
+ Therefore: 𝒂𝟐 + 𝒂𝒃 + 𝒃𝟐 − 𝒂𝒄 − 𝒃𝒄 = 𝒂 𝒂 − 𝒄 + 𝒃 𝒃 − 𝒄 + 𝒂𝒃 ≥ 𝒂𝒃 > 0.
𝟐
𝒂 − 𝒃 ≥ 𝟎; ∀𝒂, 𝒃 ∈ 𝑹
𝟐
⇒ 𝒂−𝒃 𝒂𝟐 + 𝒂𝒃 + 𝒃𝟐 − 𝒂𝒄 − 𝒃𝒄 ≥ 𝟎 (5):
⇒ 𝒂𝟐 𝒂 − 𝒃 𝒂 − 𝒄 + 𝒃𝟐 𝒃 − 𝒂 𝒃 − 𝒄 ≥ 𝟎 (6)
- Since (4), (6): ⇒ 𝒂𝟐 𝒂 − 𝒃 𝒂 − 𝒄 + 𝒃𝟐 𝒃 − 𝒂 𝒃 − 𝒄 + 𝒄𝟐 𝒄 − 𝒂 𝒄 − 𝒃 ≥ 𝟎
⇒ Inequality (3) True ⇒ 𝟐 True.
- Since inequality AM-GM for 2 positive real numbers:
𝒂𝒃 𝒂𝟐 + 𝒃𝟐 + 𝒃𝒄 𝒃𝟐 + 𝒄𝟐 + 𝒄𝒂 𝒄𝟐 + 𝒂𝟐 ≥ 𝒂𝒃 ⋅ 𝟐𝒂𝒃 + 𝒃𝒄 ⋅ 𝟐𝒃𝒄 + 𝒄𝒂 ⋅ 𝟐𝒄𝒂 =
= 𝟐 𝒂𝟐 𝒃𝟐 + 𝒃𝟐 𝒄𝟐 + 𝒄𝟐 𝒂𝟐 (7)
* Since (7):
⇒ 𝒂𝟒 + 𝒃𝟒 + 𝒄𝟒 + 𝒂𝒃𝒄 𝒂 + 𝒃 + 𝒄 ≥ 𝟐 𝒂𝟐 𝒃𝟐 + 𝒃𝟐 𝒄𝟐 + 𝒄𝟐 𝒂𝟐
⇔ 𝒂𝟒 + 𝒃𝟒 + 𝒄𝟒 + 𝟐 𝒂𝟐 𝒃𝟐 + 𝒃𝟐 𝒄𝟐 + 𝒄𝟐 𝒂𝟐 ≥ 𝟒 𝒂𝟐 𝒃𝟐 + 𝒃𝟐 𝒄𝟐 + 𝒄𝟐 𝒂𝟐 − 𝒂𝒃𝒄 𝒂 + 𝒃 + 𝒄
⇔ 𝒂𝟐 + 𝒃 𝟐 + 𝒄 𝟐 𝟐
≥ 𝟒 𝒂𝟐 𝒃𝟐 + 𝒃𝟐 𝒄𝟐 + 𝒄𝟐 𝒂𝟐 − 𝒂𝒃𝒄 𝒂 + 𝒃 + 𝒄
𝟐
𝒂𝟐 .𝒃𝟐 .𝒄𝟐
⇔ ≥ 𝟏 (8)
𝟒 𝒂𝟐 𝒃𝟐 .𝒃𝟐 𝒄𝟐 .𝒄𝟐 𝒂𝟐 7𝒂𝒃𝒄 𝒂.𝒃.𝒄

97
SOCIETATEA DE ȘTIINȚE MATEMATICE DIN ROMÂNIA
ROMANIAN MATHEMATICAL SOCIETY

Filiala Mehedinți - Mehedinți Branch


www.ssmrmh.ro

- Since inequality Cauchy – Schwarz. We have:


𝒂𝟑 𝒃𝟑 𝒄𝟐
+ +
𝒃 𝟐𝒃𝟐 − 𝒃𝒄 + 𝟐𝒄𝟐 𝟐 𝒄 𝟐𝒄𝟐 − 𝒄𝒂 + 𝟐𝒂𝟐 𝟐 𝒂 𝟐𝒂𝟐 − 𝒂𝒃 + 𝟐𝒃𝟐 𝟐
𝟐 𝟐 𝟐
𝒂𝟐 𝒃𝟐 𝒄𝟐
𝟐𝒃 − 𝒃𝒄 + 𝟐𝒄𝟐
𝟐 𝟐𝒄 − 𝒄𝒂 + 𝟐𝒂𝟐
𝟐 𝟐𝒂 − 𝒂𝒃 + 𝟐𝒃𝟐
𝟐
= + + ≥
𝒂𝒃 𝒃𝒄 𝒄𝒂
𝟐
𝒂𝟐 𝒃𝟐 𝒄𝟐
. .
𝟐𝒃𝟐 F𝒃𝒄u𝟐𝒄𝟐 𝟐𝒄𝟐 F𝒄𝒂u𝟐𝒂𝟐 𝟐𝒂𝟐 F𝒂𝒃u𝟐𝒃𝟐
≥ (9)
𝒂𝒃.𝒃𝒄.𝒄𝒂

- Other, since inquality Cauchy – Schwarz:


𝒂𝟐 𝒃𝟐 𝒄𝟐
+ +
𝟐𝒃𝟐 − 𝒃𝒄 + 𝟐𝒄𝟐 𝟐𝒄𝟐 − 𝒄𝒂 + 𝟐𝒂𝟐 𝟐𝒂𝟐 − 𝒂𝒃 + 𝟐𝒃𝟐
𝒂𝟒 𝒃𝟒 𝒄𝟒
= 𝟐 𝟐 + + ≥
𝟐𝒂 𝒃 − 𝒂𝟐 𝒃𝒄 + 𝟐𝒄𝟐 𝒂𝟐 𝟐𝒃𝟐 𝒄𝟐 − 𝒂𝒃𝟐 𝒄 + 𝟐𝒂𝟐 𝒃𝟐 𝟐𝒄𝟐 𝒂𝟐 − 𝒂𝒃𝒄𝟐 + 𝟐𝒃𝟐 𝒄𝟐
𝒂𝟐 + 𝒃 𝟐 + 𝒄 𝟐 𝟐

𝟐𝒂𝟐 𝒃𝟐 − 𝒂𝟐 𝒃𝒄 + 𝟐𝒄𝟐 𝒂𝟐 + 𝟐𝒃𝟐 𝒄𝟐 − 𝒂𝒃𝟐 𝒄 + 𝟐𝒂𝟐 𝒃𝟐 + 𝟐𝒄𝟐 𝒂𝟐 − 𝒂𝒃𝒄𝟐 + 𝟐𝒃𝟐 𝒄𝟐
𝒂𝟐 𝒃𝟐 𝒄𝟐
⇔ + + ≥
𝟐𝒃𝟐 − 𝒃𝒄 + 𝟐𝒄𝟐 𝟐𝒄𝟐 − 𝒄𝒂 + 𝟐𝒂𝟐 𝟐𝒂𝟐 − 𝒂𝒃 + 𝟐𝒃𝟐
𝟐
𝒂𝟐 .𝒃𝟐 .𝒄𝟐
≥ (10)
𝟒 𝒂𝟐 𝒃𝟐 .𝒃𝟐 𝒄𝟐 .𝒄𝟐 𝒂𝟐 7𝒂𝒃𝒄 𝒂.𝒃.𝒄

𝒂𝟐 𝒃𝟐 𝒄𝟐
- Since (8), (9): ⇒ + + ≥ 𝟏 (11)
𝟐𝒃𝟐 7𝒃𝒄.𝟐𝒄 𝟐 𝟐𝒄𝟐 7𝒄𝒂.𝟐𝒂 𝟐 𝟐𝒂𝟐 7𝒂𝒃.𝟐𝒃𝟐

𝒂𝟑 𝒃𝟑 𝒄𝟑 𝟏𝟐
- Since (9), (11): ⇒ + + ≥
𝒃 𝟐𝒃𝟐 7𝒃𝒄.𝟐𝒄𝟐 𝟐 𝒄 𝟐𝒄𝟐 7𝒄𝒂.𝟐𝒂𝟐 𝟐 𝒂 𝟐𝒂𝟐 7𝒂𝒃.𝟐𝒃𝟐 𝟐 𝒂𝒃.𝒃𝒄.𝒄𝒂

𝒂𝟑 𝒃𝟑 𝒄𝟑 𝟏
⇔ 𝟐 + 𝟐 + 𝟐 ≥ (12)
𝟐 𝒄 𝟐𝒄𝟐 −𝒄𝒂+𝟐𝒂𝟐 𝟐 𝒂𝒃.𝒃𝒄.𝒄𝒂
𝒃 𝟐𝒃 −𝒃𝒄+𝟐𝒄𝟐 𝒂 𝟐𝒂𝟐 −𝒂𝒃+𝟐𝒃

- Since inequality AM-GM we have:


𝒂𝟐 .𝒃𝟐 𝒃𝟐 .𝒄𝟐 𝒄𝟐 .𝒂𝟐 𝟐𝒂𝒃 𝟐𝒃𝒄 𝟐𝒄𝒂
𝒂 𝟐 + 𝒃𝟐 + 𝒄𝟐 = + + ≥ + + = 𝒂𝒃 + 𝒃𝒄 + 𝒄𝒂
𝟐 𝟐 𝟐 𝟐 𝟐 𝟐

⇔ 𝒂𝟐 + 𝒃𝟐 + 𝒄𝟐 ≥ 𝒂𝒃 + 𝒃𝒄 + 𝒄𝒂 ⇔ 𝒂𝟐 + 𝒃𝟐 + 𝒄𝟐 + 𝟐 𝒂𝒃 + 𝒃𝒄 + 𝒄𝒂 ≥ 𝟑 𝒂𝒃 + 𝒃𝒄 + 𝒄𝒂
𝟐
𝟐
𝒂+𝒃+𝒄 𝟑𝟐
⇔ 𝒂+𝒃+𝒄 ≥ 𝟑 𝒂𝒃 + 𝒃𝒄 + 𝒄𝒂 ⇔ 𝒂𝒃 + 𝒃𝒄 + 𝒄𝒂 ≤ = = 𝟑
𝟑 𝟑
(because 𝐚 + 𝐛 + 𝐜 = 𝟑) (13)

98
SOCIETATEA DE ȘTIINȚE MATEMATICE DIN ROMÂNIA
ROMANIAN MATHEMATICAL SOCIETY

Filiala Mehedinți - Mehedinți Branch


www.ssmrmh.ro

𝒂𝟑 𝒃𝟑 𝒄𝟑 𝟏
- Since (12), (13): ⇒ 𝟐 + 𝟐 + 𝟐 ≥
𝒃 𝟐𝒃𝟐 7𝒃𝒄.𝟐𝒄𝟐 𝒄 𝟐𝒄𝟐 7𝒄𝒂.𝟐𝒂𝟐 𝒂 𝟐𝒂𝟐 7𝒂𝒃.𝟐𝒃𝟐 𝟑

⇒ Inequality (1) true and we get the desired result.


𝒂+𝒃+𝒄=𝟑
𝒂=𝒃=𝒄>0
𝒂𝟐 𝒃𝟐 𝒄𝟐
+ Equality occurs if: 𝟐𝒃 F𝒃𝒄u𝟐𝒄𝟐
𝟐 𝟐𝒄 F𝒄𝒂u𝟐𝒂𝟐
𝟐 𝟐𝒂 F𝒂𝒃u𝟐𝒃𝟐
𝟐 ⇔ 𝒂 = 𝒃 = 𝒄 = 𝟏.
= =
𝒂𝒃 𝒃𝒄 𝒄𝒂
𝟏 𝟏 𝟏
= =
𝟐𝒃𝟐 7𝒃𝒄.𝟐𝒄𝟐 𝟐𝒄𝟐 7𝒄𝒂.𝟐𝒂𝟐 𝟐𝒂𝟐 7𝒂𝒃.𝟐𝒃𝟐

Solution 2 by Anh Tai Tran – Hanoi – Vietnam


𝟐 𝟐
𝒂𝟐 𝒂𝟐
𝒂𝟑 𝟐𝒃𝟐 − 𝒃𝒄 + 𝟐𝒄𝟐 𝟐𝒃𝟐 − 𝒃𝒄 + 𝟐𝒄𝟐
= ≥
𝒃 𝟐𝒃𝟐 − 𝒃𝒄 + 𝟐𝒄𝟐 𝟐 𝒂𝒃 𝒂𝒃
𝟐
𝒂𝟐
𝟐𝒃𝟐 − 𝒃𝒄 + 𝟐𝒄𝟐 𝑻𝟐
≥ ≥
𝒂+𝒃+𝒄 𝟐 𝟑
𝟑
We will prove:
𝒂𝟐
𝑻= ≥ 𝟏
𝟐𝒃𝟐 − 𝒃𝒄 + 𝟐𝒄𝟐
𝒂𝟐 𝟐
𝑻≥ ≥ 𝟏
𝟒 𝒃𝒄 𝟐 − 𝒂𝒃𝒄 𝒂

↔ 𝒂𝟒 + 𝒂𝒃𝒄 𝒂≥ 𝒂𝒃 𝟐

It’s true by Shur and AM-GM

𝑳𝑯𝑺 ≥ 𝒂𝒃 𝒂𝟐 + 𝒃𝟐 ≥ 𝟐 𝒂𝒃 𝟐

Solution 3 by Leonard Giugiuc – Romania


By Cauchy,

99
SOCIETATEA DE ȘTIINȚE MATEMATICE DIN ROMÂNIA
ROMANIAN MATHEMATICAL SOCIETY

Filiala Mehedinți - Mehedinți Branch


www.ssmrmh.ro

𝟐
𝟑 𝟐
𝒂 𝒂
𝒂𝒃 + 𝒃𝒄 + 𝒄𝒂 ≥ .
𝒃 𝟐𝒃𝟐 − 𝒃𝒄 + 𝟐𝒄𝟐 𝟐 𝟐𝒃𝟐 − 𝒃𝒄 + 𝟐𝒄𝟐
𝒄𝒚𝒄 𝒄𝒚𝒄

Suffice it to show that

𝒂𝟐 𝒂𝒃 + 𝒃𝒄 + 𝒄𝒂

𝟐𝒃𝟐 − 𝒃𝒄 + 𝟐𝒄𝟐 𝟑
𝒄𝒚𝒄

Apply Jensen and get


𝒂𝟐 𝒂 𝟐 + 𝒃𝟐 + 𝒄𝟐 𝟐
≥ .
𝟐𝒃𝟐 − 𝒃𝒄 + 𝟐𝒄𝟐 𝟒 𝒂𝟐 𝒃𝟐 + 𝒃𝟐 𝒄𝟐 + 𝒄𝟐 𝒂𝟐 − 𝟑𝒂𝒃𝒄
𝒄𝒚𝒄
𝟐
𝒂𝟐 .𝒃𝟐 .𝒄𝟐
We’ll show that ≥ 𝟏 ⇔
𝟒 𝒂𝟐 𝒃𝟐 .𝒃𝟐 𝒄𝟐 .𝒄𝟐 𝒂𝟐 7𝟑𝒂𝒃𝒄

⇔ 𝟏 + 𝟐𝒕𝟐 𝟐
≥ 𝟒 𝟏 − 𝒕𝟐 𝟐
− 𝟑𝒑, where
𝒂𝒃 + 𝒃𝒄 + 𝒄𝒂 = 𝟑 𝟏 − 𝒕𝟐 , 𝟎 ≤ 𝒕 < 1 and 𝒂𝒃𝒄 = 𝒑.
𝟏
Case 1: ≤ 𝒑 ≤ . Then
𝟐

𝟒 𝟏 − 𝒕𝟐 𝟐
−𝟑 𝟏+𝒕 𝟐
𝟏 − 𝟐𝒕 ≥ 𝟒 𝟏 − 𝒕𝟐 𝟐
− 𝟑𝒑 and
𝟏 + 𝟐𝒕𝟐 𝟐
− 𝟒 𝟏 − 𝒕𝟐 𝟐
−𝟑 𝟏+𝒕 𝟐
𝟏 − 𝟐𝒕 = 𝟑𝒕𝟐 𝟏 − 𝟐𝒕 ≥ 𝟎
𝟏
Case 2: ≤ 𝒑 < 1. Then
𝟐

𝟏 + 𝟐𝒕𝟐 𝟐
≥ 𝟒 𝟏 − 𝒕𝟐 𝟐
≥ 𝟒 𝟏 − 𝒕𝟐 𝟐
− 𝟑𝒑.
𝒂𝒃.𝒃𝒄.𝒄𝒂
Because ≥ , done.
𝟑

70. If 𝒂, 𝒃, 𝒄 > 0 then:


𝟐 𝟐
𝟏 𝟏 𝟏 𝟏 𝟏 𝟏 𝒂+𝒃+𝒄 𝟑𝟔
+ + + + + ≥
𝒂 𝒃 𝒄 𝒂 𝟐 𝒃𝟐 𝒄𝟐 𝒂𝒃 + 𝒃𝒄 + 𝒄𝒂 𝟑
𝒂𝒃𝒄 𝒂𝒃𝒄
Proposed by Daniel Sitaru – Romania

100
SOCIETATEA DE ȘTIINȚE MATEMATICE DIN ROMÂNIA
ROMANIAN MATHEMATICAL SOCIETY

Filiala Mehedinți - Mehedinți Branch


www.ssmrmh.ro

Solution 1 by Soumava Chakraborty – Kolkata – India


Solution 2 by proposer
Solution 1 by Soumava Chakraborty – Kolkata – India
𝟐
𝒂𝒃 + 𝒃𝒄 + 𝒄𝒂 𝒂𝟐 𝒃𝟐 𝒂+𝒃+𝒄 𝟐
𝑳𝑯𝑺 = +
𝒂𝟐 𝒃𝟐 𝒄𝟐 𝒂𝟐 𝒃𝟐 𝒄𝟐 𝒂𝒃 + 𝒃𝒄 + 𝒄𝒂 𝟐

𝒂𝒃 𝟐 𝒂𝟐 𝒃𝟐 𝒂+𝒃+𝒄 𝟐
= +
𝒂 𝟒 𝒃𝟒 𝒄𝟒 𝒂 𝟐 𝒃𝟐 𝒄𝟐
𝟑 𝟐 𝟑 𝟑 𝟐
𝑨7𝑮 𝟑 𝒂 𝟐 𝒃𝟐 𝒄𝟐 ⋅ 𝟑 𝒂 𝟒 𝒃𝟒 𝒄𝟒 𝟑 𝒂𝒃𝒄
≥ +
𝒂 𝟒 𝒃𝟒 𝒄𝟒 𝒂 𝟐 𝒃𝟐 𝒄𝟐
𝟑
𝟐𝟕 𝒂𝟖 𝒃𝟖 𝒄𝟖 𝟗 𝟐𝟕 𝟗
= + = +
𝒂 𝟒 𝒃𝟒 𝒄𝟒 𝟑
𝒂𝟒 𝒃𝟒 𝒄𝟒
𝟑
𝒂𝟒 𝒃𝟒 𝒄𝟒
𝟑
𝒂𝟒 𝒃𝟒 𝒄𝟒
𝟑𝟔 𝟑𝟔
=𝟑 = 𝟑
𝒂𝟒 𝒃𝟒 𝒄𝟒 𝒂𝒃𝒄 𝒂𝒃𝒄
(Proved)
Solution 2 by proposer
𝟐
𝟏 𝟏 𝟏 𝒂+𝒃+𝒄
+ + + =
𝒂𝟐 𝒃𝟐 𝒄𝟐 𝒂𝒃 + 𝒃𝒄 + 𝒄𝒂
𝟐
𝟏 𝟏 𝟏
𝟏 𝟏 𝟏 𝟐
𝟏 𝟏 𝟏 + +
= + + −𝟐 + + + 𝒂𝒃 𝒃𝒄 𝒂𝒄 =
𝒂 𝒃 𝒄 𝒂𝒃 𝒃𝒄 𝒄𝒂 𝟏 𝟏 𝟏
+ +
𝒂 𝒃 𝒄
𝟒 𝟐 𝟐
𝟏 𝟏 𝟏 𝟏 𝟏 𝟏 𝟏 𝟏 𝟏 𝟏 𝟏 𝟏
+ + −𝟐 + + + + + + +
= 𝒂 𝒃 𝒄 𝒂𝒃 𝒃𝒄 𝒄𝒂 𝒂 𝒃 𝒄 𝒂𝒃 𝒃𝒄 𝒂𝒄 =
𝟏 𝟏 𝟏 𝟐
+ +
𝒂 𝒃 𝒄

101
SOCIETATEA DE ȘTIINȚE MATEMATICE DIN ROMÂNIA
ROMANIAN MATHEMATICAL SOCIETY

Filiala Mehedinți - Mehedinți Branch


www.ssmrmh.ro

𝟐
𝟏 𝟏 𝟏 𝟐
𝟏 𝟏 𝟏 𝟏 𝟏 𝟏 𝟏 𝟏 𝟏 𝟐
+ +
𝒂 𝒃 𝒄
− + +
𝒂𝒃 𝒃𝒄 𝒂𝒄 𝟐 + 𝒃𝟐 + 𝒄𝟐 + 𝒂𝒃 + 𝒃𝒄 + 𝒂𝒄
= = 𝒂
𝟏 𝟏 𝟏 𝟏 𝟏 𝟏 𝟐
+ + + +
𝒂 𝒃 𝒄 𝒂 𝒃 𝒄
𝟐 𝟐
𝟏 𝟏 𝟏 𝟏 𝟏 𝟏 𝒂+𝒃+𝒄
+ + + + + =
𝒂 𝒃 𝒄 𝒂 𝟐 𝒃𝟐 𝒄𝟐 𝒂𝒃 + 𝒃𝒄 + 𝒄𝒂
𝟐
𝟐
𝟏 𝟏 𝟏 𝟏 𝟏 𝟏 𝟔 𝟏 𝟏 𝟏 𝟏 𝟏 𝟏
= + + + + + ≥ 𝟔 ⋅ ⋅ ⋅ ⋅ ⋅ =
𝒂𝟐 𝒃𝟐 𝒄𝟐 𝒂𝒃 𝒃𝒄 𝒄𝒂 𝒂𝟐 𝒃𝟐 𝒄𝟐 𝒂𝒃 𝒃𝒄 𝒄𝒂

𝟑 𝟏 𝟑𝟔
= 𝟑𝟔 =
𝒂𝟒 𝒃𝟒 𝒄𝟒 𝒂𝒃𝒄 𝟑 𝒂𝒃𝒄


71. Let 𝒂, 𝒃, 𝒄 be positive real numbers such that 𝒂 + 𝒃 + 𝒄 = 𝟑.
Prove that:
𝒂𝟐 𝒃𝟐 𝒄𝟐 𝟑
+ + >
𝒃𝟒 + 𝟒 𝒄𝟒 + 𝟒 𝒂𝟒 + 𝟒 𝟓
Proposed by Anish Ray-Santaragachi-India
Solution 1 by Anish Ray-Santaragachi-India
Solution 2 by Soumitra Mandal-Chandar Nagore-India
Solution 3 by Henry Ricardo - New York – USA
Solution 1 by Anish Ray-Santaragachi-India
Given that 𝒂, 𝒃, 𝒄 > 0
We can write,
𝟏 𝟏 𝟏 𝟏
𝒃𝟐 + 𝟐 𝟐
> 𝒃𝟒 + 𝟒 ⇒ > ⇒ >
𝒃𝟒 + 𝟒 𝒃𝟐 + 𝟐 𝟐
𝒃𝟒 + 𝟒 𝒃𝟐 + 𝟐

102
SOCIETATEA DE ȘTIINȚE MATEMATICE DIN ROMÂNIA
ROMANIAN MATHEMATICAL SOCIETY

Filiala Mehedinți - Mehedinți Branch


www.ssmrmh.ro

So,
𝒂𝟐 𝒂𝟐
>
𝒃𝟒 + 𝟒 𝒃𝟐 + 𝟐
𝒄𝒚𝒄 𝒄𝒚𝒄

Now, By Bergstorm’s Lemma, we get that


𝒂𝟐 𝒂+𝒃+𝒄 𝟐
≥ 𝟐
𝒃𝟐 + 𝟐 𝒂 + 𝒃𝟐 + 𝒄𝟐 + 𝟔
𝒄𝒚𝒄

Now, for 𝒂, 𝒃, 𝒄 > 0


𝟐
𝒂+𝒃+𝒄 > 𝒂𝟐 + 𝒃𝟐 + 𝒄𝟐 ⇒ 𝒂 + 𝒃 + 𝒄 𝟐
+ 𝟔 > 𝒂𝟐 + 𝒃𝟐 + 𝒄𝟐 + 𝟔
so,
𝟏 𝟏 𝟏
> =
𝒂𝟐 + 𝒃𝟐 + 𝒄𝟐 + 𝟔 𝒂+𝒃+𝒄 𝟐 +𝟔 𝟏𝟓
which implies,
𝒂+𝒃+𝒄 𝟐 𝒂+𝒃+𝒄 𝟐 𝟗 𝟑
> = =
𝒂 𝟐 + 𝒃𝟐 + 𝒄𝟐 + 𝟔 𝒂 + 𝒃 + 𝒄 𝟐 + 𝟔 𝟏𝟓 𝟓
therefore,
𝒂𝟐 𝟑

𝒃𝟐 + 𝟐 𝟓
𝒄𝒚𝒄

Thus,
𝒂𝟐 𝟑
>
𝒄𝒚𝒄
𝒃𝟒 + 𝟒 𝟓

Solution 2 by Soumitra Mandal-Chandar Nagore-India


Let 𝒂, 𝒃, 𝒄 ≥ 𝟎 and 𝒂 + 𝒃 + 𝒄 = 𝟑 then
𝒂𝟐 𝟑
>
𝒄𝒚𝒄
𝒃𝟒 + 𝟒 𝟓

103
SOCIETATEA DE ȘTIINȚE MATEMATICE DIN ROMÂNIA
ROMANIAN MATHEMATICAL SOCIETY

Filiala Mehedinți - Mehedinți Branch


www.ssmrmh.ro

𝒂𝟐 𝒂𝟐
=
𝒄𝒚𝒄
𝒃𝟒 + 𝟒 𝒄𝒚𝒄 𝒃𝟐 − 𝟐𝒃 + 𝟐 𝒃𝟐 + 𝟐𝒃 + 𝟐

𝒂𝟐 𝒂𝟒 𝒂 𝟐 + 𝒃𝟐 + 𝒄𝟐 𝟐
≥ = ≥
𝒃𝟐 + 𝟐 𝒂𝟐 𝒃𝟐 + 𝟐𝒂𝟐 𝟐 𝟐
𝒄𝒚𝒄 𝒂 𝒃 + 𝟐 𝒄𝒚𝒄 𝒂
𝟐
𝒄𝒚𝒄 𝒄𝒚𝒄

∴ we need to prove,
𝟐

𝟓 𝒂𝟐 >3 𝒂𝟐 𝒃𝟐 + 𝟔 𝒂𝟐
𝒄𝒚𝒄 𝒄𝒚𝒄 𝒄𝒚𝒄

𝟐
𝟐
⇔𝟓 𝒂𝟐 >3 𝒂𝟐 𝒃𝟐 + 𝒂+𝒃+𝒄 𝟐
𝒂𝟐
𝟑
𝒄𝒚𝒄 𝒄𝒚𝒄 𝒄𝒚𝒄

𝟐 𝟐

⇔ 𝟏𝟓 𝒂 >9 𝒂𝟐 𝒃𝟐 + 𝟐 𝒂 𝒂𝟐
𝒄𝒚𝒄 𝒄𝒚𝒄 𝒄𝒚𝒄 𝒄𝒚𝒄

⇔ 𝟏𝟓 𝒂𝟒 + 𝟐𝟏 𝒂𝟐 𝒃𝟐 > 2 𝒂𝟐 +𝟒 𝒂𝒃 𝒂𝟐
𝒄𝒚𝒄 𝒄𝒚𝒄 𝒄𝒚𝒄 𝒄𝒚𝒄 𝒄𝒚𝒄

⇔ 𝟏𝟑 𝒂𝟒 + 𝟏𝟕 𝒂𝟐 𝒃𝟐 > 4 𝒂𝒃 𝒂𝟐 … 𝟏
𝒄𝒚𝒄 𝒄𝒚𝒄 𝒄𝒚𝒄 𝒄𝒚𝒄

we need to prove (1). Now we have


𝟐 𝟐

𝟏𝟑 𝒂𝟒 + 𝟏𝟕 𝒂𝟐 𝒃𝟐 = 𝟏𝟏 𝒂𝟒 + 𝟏𝟏 𝒂𝟐 𝒃𝟐 + 𝟐 𝒂𝟐 +𝟐 𝒂𝒃 − 𝟏𝟐𝒂𝒃𝒄
𝒄𝒚𝒄 𝒄𝒚𝒄 𝒄𝒚𝒄 𝒄𝒚𝒄 𝒄𝒚𝒄 𝒄𝒚𝒄

𝟐
≥ 𝟐𝟐 𝒂𝟐 𝒃 + 𝟒 𝒂𝒃 𝒂𝟐 − 𝟏𝟐𝒂𝒃𝒄
𝒄𝒚𝒄 𝒄𝒚𝒄 𝒄𝒚𝒄

104
SOCIETATEA DE ȘTIINȚE MATEMATICE DIN ROMÂNIA
ROMANIAN MATHEMATICAL SOCIETY

Filiala Mehedinți - Mehedinți Branch


www.ssmrmh.ro

since,

𝒂𝟒 ≥ 𝒂𝟐 𝒃𝟐
𝒄𝒚𝒄 𝒄𝒚𝒄

≥ 𝟐𝟐𝒂𝒃𝒄 𝒂 + 𝒃 + 𝒄 + 𝟒 𝒂𝒃 𝒂𝟐 − 𝟏𝟐𝒂𝒃𝒄 = 𝟒 𝒂𝒃 𝒂𝟐 + 𝟓𝟒𝒂𝒃𝒄


𝒄𝒚𝒄 𝒄𝒚𝒄 𝒄𝒚𝒄 𝒄𝒚𝒄

>4 𝒂𝒃 𝒂𝟐
𝒄𝒚𝒄 𝒄𝒚𝒄

hence statement (1) is true.


𝒂𝟐 𝟑
∴ >
𝒄𝒚𝒄
𝒃𝟐 + 𝟒 𝟓

(proved)
Solution 3 by Henry Ricardo - New York – USA
Without loss of generality we may assume that 𝒂 ≥ 𝒃 ≥ 𝒄. It follows that
𝟏 𝟏 𝟏
𝒂𝟐 ≥ 𝒃𝟐 ≥ 𝒄𝟐 and ≤ ≤ .
𝒂𝟐 .𝟒 𝒃𝟐 .𝟒 𝒄𝟐 .𝟒

Now the Rearrangement Inequality give us


𝒂𝟐 𝒃𝟐 𝒄𝟐 𝒂𝟐 𝒂𝟐 𝒃𝟐 𝒄𝟐
+ + ≥ + + +
𝒃𝟐 + 𝟒 𝒄𝟐 + 𝟒 𝒂𝟐 + 𝟒 𝒂𝟐 + 𝟒 𝒂𝟐 + 𝟒 𝒃𝟐 + 𝟒 𝒄𝟐 + 𝟒
It can be seen graphically (and proved with some tedious algebra/analysis) that
𝒙𝟐
the curve given by 𝒚 = lies on or above the tangent line to the curve at,
𝒙𝟐 .𝟒

𝟗 𝟓 𝟓
𝒙 = 𝟏, 𝒚 = 𝒙−𝟏 + , on the interval 𝟎, 𝟑 . Thus we have
𝟐𝟓 𝟓

𝒂𝟐 𝟗 𝟓 𝟓
≥ 𝒂−𝟏 +
𝒃𝟐 + 𝟒 𝟐𝟓 𝟓
𝒄𝒚𝒄𝒍𝒊𝒄 𝒄𝒚𝒄𝒍𝒊𝒄

105
SOCIETATEA DE ȘTIINȚE MATEMATICE DIN ROMÂNIA
ROMANIAN MATHEMATICAL SOCIETY

Filiala Mehedinți - Mehedinți Branch


www.ssmrmh.ro

𝟗 𝟓 𝟏𝟐 𝟓
= 𝒂−
𝟐𝟓 𝟐𝟓
𝒄𝒚𝒄𝒍𝒊𝒄

𝟐𝟕 𝟓 𝟏𝟐 𝟓 𝟑 𝟓 𝟑
= − = > .
𝟐𝟓 𝟐𝟓 𝟓 𝟓

𝒂𝟓 .𝒃𝟓 .𝒄𝟓 .𝒅𝟓
72. Prove that: 𝒂 + 𝒃 + 𝒄 + 𝒅 ≤ , 𝒂, 𝒃, 𝒄, 𝒅 > 0
𝒂𝒃𝒄𝒅

Proposed by Daniel Sitaru – Romania


Solution by Kunihiko Chikaya – Tokyo – Japan
𝒂 𝟓 + 𝒂 𝟓 + 𝒃𝟓 + 𝒄𝟓 + 𝒅𝟓 𝟓
≥ 𝒂𝟓 𝒂𝟓 𝒃𝟓 𝒄𝟓 𝒅𝟓 = 𝒂𝟐 𝒃𝒄𝒅
𝟓
𝒂 𝟓 + 𝒃𝟓 + 𝒃𝟓 + 𝒄𝟓 + 𝒅𝟓 𝟓
≥ 𝒂𝟓 𝒃𝟓 𝒃𝟓 𝒄𝟓 𝒅𝟓 = 𝒂𝒃𝟐 𝒄𝒅
𝟓
𝒂 𝟓 + 𝒃𝟓 + 𝒄𝟓 + 𝒄𝟓 + 𝒅𝟓 𝟓
≥ 𝒂𝟓 𝒃𝟓 𝒄𝟓 𝒄𝟓 𝒅𝟓 = 𝒂𝒃𝒄𝟐 𝒅
𝟓
𝒂 𝟓 + 𝒃𝟓 + 𝒄𝟓 + 𝒅𝟓 + 𝒅𝟓 𝟓
≥ 𝒂𝟓 𝒃𝟓 𝒄𝟓 𝒅𝟓 𝒅𝟓 = 𝒂𝒃𝒄𝒅𝟐
𝟓
𝟓(𝒂𝟓 .𝒃𝟓 .𝒄𝟓 .𝒅𝟓 )
⨁ ≥ 𝒂𝒃𝒄𝒅(𝒂 + 𝒃 + 𝒄 + 𝒅)
𝟓

73. If 𝒙, 𝒚, 𝒛 > 0 then:


𝒙𝟑 𝒚𝟑 𝒛𝟑 𝒙𝟐 𝒚𝟐 𝒛𝟐 𝒙 𝒚 𝒛
𝟐 𝟑 + 𝟑 + 𝟑 + 𝟏𝟖 > 3 𝟐 + 𝟐 + 𝟐 + 𝟑 + +
𝒚 𝒛 𝒙 𝒚 𝒛 𝒙 𝒚 𝒛 𝒙
Proposed by Daniel Sitaru – Romania
Solution 1 by Soumava Chakraborty – Kolkata – India
Solution 2 by Myagmarsuren Yadamsuren – Darkhan – Mongolia
Solution 3 by Aditya Narayan Sharma – Kanchrapara – India

106
SOCIETATEA DE ȘTIINȚE MATEMATICE DIN ROMÂNIA
ROMANIAN MATHEMATICAL SOCIETY

Filiala Mehedinți - Mehedinți Branch


www.ssmrmh.ro

Solution 4 by Seyran Ibrahimov – Maasilli – Azerbaidian


Solution 1 by Soumava Chakraborty – Kolkata – India
𝒙𝟑 𝒙𝟐 𝒙
𝒙, 𝒚, 𝒛 > 0 ⇒ 𝟐 + 𝟏𝟖 > 3 +𝟑
𝒚𝟑 𝒚𝟐 𝒚
Let 𝒇 𝒕 = 𝟐𝒕𝟑 − 𝟑𝒕𝟐 − 𝟑𝒕 + 𝟔 ∀ 𝒕 > 0

𝟑 𝟑 𝟐𝟒 𝑨7𝑮 𝟑 𝟐𝟒
Now, 𝒕 + 𝒕 + ≥ 𝟑𝒕𝟐 > 5𝒕𝟐
𝟓 𝟓

𝟐𝟒 𝟔 𝟔
∴ 𝟐𝒕𝟑 + − 𝟑𝒕𝟐 − 𝟑𝒕 + > 2𝒕𝟐 − 𝟑𝒕 +
𝟓 𝟓 𝟓
𝟑 𝟑 𝟑 𝟗
= 𝟐 𝒕𝟐 − 𝒕 + > 2 𝒕𝟐 − 𝒕 +
𝟐 𝟓 𝟐 𝟏𝟔
𝟐
𝟑
=𝟐 𝒕− > 0 ⇒ 𝟐𝒕𝟑 − 𝟑𝒕𝟐 − 𝟑 + 𝟔 > 0 ∀𝒕 > 0
𝟒
⇒ 𝟐𝒕𝟑 + 𝟔 > 3𝒕𝟐 + 𝟑𝒕 ∀ 𝒕 > 0
𝒙𝟑 𝒙𝟐 𝒙 𝒙
∴𝟐 𝟑
+𝟔>3 + 𝟑 (1) (putting 𝒕 = )
𝒚 𝒚𝟐 𝒚 𝒚

𝒚𝟑 𝒚𝟐 𝒚 𝒚
𝟐 𝟑
+𝟔>3 + 𝟑 (2) (putting 𝒕 = )
𝒛 𝒛𝟐 𝒛 𝒛
𝒛𝟑 𝒛𝟐 𝒛 𝒛
𝟐 𝟑
+𝟔>3 + 𝟑 (3) (putting 𝒕 = )
𝒙 𝒙𝟐 𝒙 𝒙

𝒙𝟑 𝒙𝟐 𝒙
(1) + (2) + (3) ⇒ 𝟐 𝟑
+ 𝟏𝟖 > 3 +𝟑
𝒚 𝒚𝟐 𝒚

Solution 2 by Myagmarsuren Yadamsuren – Darkhan – Mongolia


If 𝒙, 𝒚, 𝒛 > 0
𝒙𝟑 𝒙𝟐 𝒙
𝟐⋅ + 𝟏𝟖 > 3 ⋅ +𝟑⋅
𝒚𝟐 𝒚𝟐 𝒚
𝟐𝒕𝟑 + 𝟔 > 3𝒕𝟐 + 𝟑𝒕 (ASSURE)

107
SOCIETATEA DE ȘTIINȚE MATEMATICE DIN ROMÂNIA
ROMANIAN MATHEMATICAL SOCIETY

Filiala Mehedinți - Mehedinți Branch


www.ssmrmh.ro

𝟑
𝟏 𝟑 𝟏 𝟑 𝑪𝒂𝒖𝒄𝒉𝒚
𝟐𝒕 + 𝟔 = 𝒕 + 𝒕 + 𝟒 + 𝒕𝟑 + 𝟏 + 𝟏 ≥
𝟐 𝟐
≥ 𝟑𝒕𝟐 + 𝟑𝒕
𝒙 𝒚 𝒛
𝒕𝟏 = ; 𝒕𝟐 = ; 𝒕𝟑 =
𝒚 𝒛 𝒙
𝟑 𝟑 𝟑

𝟐⋅ 𝒕𝟑𝒊 + 𝟏𝟖 > 3 ⋅ 𝒕𝟐𝒊 + 𝟑 ⋅ 𝒕𝒊


𝒊±𝟏 𝒊±𝟏 𝒊±𝟏

Solution 3 by Aditya Narayan Sharma – Kanchrapara – India


𝒙𝟑 𝒚𝟑 𝒛𝟑 𝒙𝟐 𝒚𝟐 𝒛𝟐 𝒙 𝒚 𝒚
𝟐 𝟑 + 𝟑 + 𝟑 + 𝟏𝟖 > 3 𝟐 + 𝟐 + 𝟐 + + +
𝒚 𝒛 𝒙 𝒚 𝒛 𝒙 𝒚 𝒛 𝒙
𝒙 𝒚 𝒛
Let = 𝒂, = 𝒃, = 𝒄
𝒚 𝒛 𝒙

∴ 𝒂𝒃𝒄 = 𝟏 with 𝒂, 𝒃, 𝒄 > 0


To prove, 𝟐 𝒂𝟑 + 𝒃𝟑 + 𝒄𝟑 + 𝟗 − 𝟑 𝒂𝟐 + 𝒃𝟐 + 𝒄𝟐 + 𝒂 + 𝒃 + 𝒄 > 0
Now define,
𝒇 𝒂, 𝒃, 𝒄 = 𝟐 𝒂𝟑 + 𝒃𝟑 + 𝒄𝟑 + 𝒛 − 𝟑 𝒂𝟐 + 𝒃𝟐 + 𝒄𝟐 + 𝒂 + 𝒃 + 𝒄
Evaluating first partial derivatives,
𝝏𝒇
= 𝟔𝒂𝟐 − 𝟔𝒂 − 𝟑 = 𝟎 (1)
𝝏𝒂
𝝏𝒇
= 𝟔𝒃𝟐 − 𝟔𝒃 − 𝟑 = 𝟎 (2)
𝝏𝒃
𝝏𝒇
= 𝟔𝒄𝟐 − 𝟔𝒄 − 𝟑 = 𝟎 (3)
𝝏𝒄

All of the equations are identical, thus we have same solution for 𝒂, 𝒃, 𝒄.
∴ 𝒂 = 𝒃 = 𝒄
Now to justify that 𝒂 = 𝒃 = 𝒄 is a minimum we consider the hessian matrix,

108
SOCIETATEA DE ȘTIINȚE MATEMATICE DIN ROMÂNIA
ROMANIAN MATHEMATICAL SOCIETY

Filiala Mehedinți - Mehedinți Branch


www.ssmrmh.ro

𝟏𝟐𝒂 − 𝒃 𝟎 𝟎
𝑯 𝒇 𝒂, 𝒃, 𝒄 = 𝟎 𝟏𝟐𝒃 − 𝟔 𝟎
𝟎 𝟎 𝟏𝟐𝒄 − 𝒃
𝟐𝒂 − 𝟏 𝟎 𝟎
=𝟔 𝟎 𝟐𝒃 − 𝟏 𝟎
𝟎 𝟎 𝟐𝒄 − 𝟏
Now, from the solutions of (1), (2), (3) we know,
𝟐𝒂 − 𝟏 > 0, 2𝒃 − 𝟏 > 0, 2𝒄 − 𝟏 > 0
because we neglect the negative roots because 𝒂, 𝒃, 𝒄 > 0
Solution 4 by Seyran Ibrahimov – Maasilli – Azerbaidian
𝒙 𝒚 𝒛
Equivalent to: = 𝒂; = 𝒄; = 𝒃
𝒚 𝒛 𝒙

𝟐𝒂𝟑 + 𝟔 > 𝟑𝒂𝟐 + 𝟑𝒂

𝟐𝒂𝟑 + 𝟔 > 3𝒂𝟐 + 𝟑𝒂


𝟑𝒂𝟐 + 𝟑𝒂 − 𝟐𝒂𝟑 < 6
𝒇 𝒂 = 𝟑𝒂𝟐 + 𝟑𝒂 − 𝟐𝒂𝟑
𝒇• 𝒂 = −𝟔𝒂𝟐 + 𝟔𝒂 + 𝟑 = 𝟎
𝟑+𝟏 𝟑−𝟏
𝒂𝟏 = ; 𝒂𝟐 =
𝟐 𝟐



Ã7· Ã.·
−∞; ↓ and ; +∞ ↓
¥ ¥

109
SOCIETATEA DE ȘTIINȚE MATEMATICE DIN ROMÂNIA
ROMANIAN MATHEMATICAL SOCIETY

Filiala Mehedinți - Mehedinți Branch


www.ssmrmh.ro

3−1 3+1
𝑓 𝑎 ⇒ ; ↑
2 2
3−1 3−1 8−3 3
𝑓ÆÇÈ ⇒𝑓 =
2 2 2

3+1 3+1 4+3 3 ?


𝑓Æ¶É ⇒𝑓 = < 6
2 2 2
(Proved)
74. If 𝒂, 𝒃, 𝒄 ∈ (𝟎, ∞) and 𝒂 + 𝒃 + 𝒄 = 𝟏 then

𝟏 𝟏
≥ 𝝀+𝟑 , 𝝀 ∈ 𝟏, 𝟑
𝒂 𝝀+𝒂
𝒄𝒚𝒄 𝒄𝒚𝒄

Proposed by Marin Chirciu – Romania


Solution 1 by proposer
Solution 2 by Soumitra Mandal - Chandar Nagore – India
Solution 1 by proposer
𝒙𝟐 𝒚𝟐 𝒙.𝒚 𝟐
Using Bergström inequality + ≥ , where 𝒙, 𝒚, 𝒂, 𝒃 > 0 with equality if and only if
𝒂 𝒃 𝒂.𝒃


𝒙 𝒚 𝟏𝟐 𝟑𝟐 𝟏.𝟑 𝟐 𝟏 𝟏𝟔 𝟗
= we obtain + ≥ ⇔ ≥ − .
𝒂 𝒃 𝒂 𝝀 𝒂.𝝀 𝒂 𝝀.𝒂 𝝀
𝟏 𝟏 𝟏 𝟏 𝟏 𝟏 𝟐𝟕
It follows + + ≥ 𝟏𝟔 + + − .
𝒂 𝒃 𝒄 𝝀.𝒂 𝝀.𝒃 𝝀.𝒄 𝝀

It is enough to prove that


𝟏 𝟏 𝟏 𝟐𝟕 𝟏 𝟏 𝟏
𝟏𝟔 + + − ≥ 𝟑+𝝀 + +
𝝀+𝒂 𝝀+𝒃 𝝀+𝒄 𝝀 𝝀+𝒂 𝝀+𝒃 𝝀+𝒄
𝟏 𝟏 𝟏 𝟐𝟕
⇔ 𝟏𝟑 − 𝝀 + + ≥
𝝀+𝒂 𝝀+𝒃 𝝀+𝒄 𝝀

110
SOCIETATEA DE ȘTIINȚE MATEMATICE DIN ROMÂNIA
ROMANIAN MATHEMATICAL SOCIETY

Filiala Mehedinți - Mehedinți Branch


www.ssmrmh.ro

𝟏 𝟏 𝟏 𝟐𝟕
⇔ + + ≥ , which follows from means inequality 𝑨𝒎 ≥ 𝑯𝒎 , or from
𝝀.𝒂 𝝀.𝒃 𝝀.𝒄 𝝀 𝟏𝟑7𝝀

Bergström inequality for three reports.


𝟏 𝟏 𝟏 𝟗 𝟗 𝟐𝟕
Indeed + + ≥ = ≥ , where the latter inequality is equivalent
𝝀.𝒂 𝝀.𝒃 𝝀.𝒄 𝟑𝝀.𝒂.𝒃.𝒄 𝟑𝝀.𝟏 𝝀 𝟏𝟑7𝝀

𝟗 𝟐𝟕
to ≥ ⇔ 𝝀 𝟏𝟑 − 𝝀 ≥ 𝟑 𝟑𝝀 + 𝟏 ⇔ 𝝀𝟐 − 𝟒𝝀 + 𝟑 ≤ 𝟎 ⇔ 𝝀 ∈ 𝟏, 𝟑 .
𝟑𝝀.𝟏 𝝀 𝟏𝟑7𝝀
𝟏 𝟑 𝟏 𝟑 𝟏 𝟑
The equality holds if and only if = , = , = , 𝒂 + 𝒃 + 𝒄 = 𝟏, wherefrom
𝒂 𝝀 𝒃 𝝀 𝒄 𝝀
𝟏
we obtain 𝝀 = 𝟏 and 𝒂 = 𝒃 = 𝒄 = .
𝟑

Solution 2 by Soumitra Mandal - Chandar Nagore – India

𝟏 𝟏
𝝀+𝟑 = 𝝀+𝟑
𝒂+𝝀 𝝀 𝒂+𝒃+𝒄 +𝒂
𝒄𝒚𝒄 𝒄𝒚𝒄

𝟏
= 𝝀+𝟑
𝟏 + 𝝀 𝒂 + 𝝀𝒃 + 𝝀𝒄
𝒄𝒚𝒄

𝝀+𝟑 𝟏+𝝀 𝝀 𝝀 𝝀+𝟑 𝟏 𝟏 𝟏


≤ 𝟐
+ + = + +
𝟏 + 𝟑𝝀 𝒂 𝒃 𝒄 𝟏 + 𝟑𝝀 𝒂 𝒃 𝒄
𝒄𝒚𝒄

we need to prove,

𝝀+𝟑 𝟏 𝟏
≤ ⇔ 𝟏 ≤ 𝝀
𝟏 + 𝟑𝝀 𝒂 𝒂
𝒄𝒚𝒄 𝒄𝒚𝒄

which is true ∵ 𝝀 ∈ 𝟏, 𝟑

𝟏 𝟏
∴ ≥ 𝝀+𝟑
𝒂 𝝀+𝒂
𝒄𝒚𝒄 𝒄𝒚𝒄

(Proved)

111
SOCIETATEA DE ȘTIINȚE MATEMATICE DIN ROMÂNIA
ROMANIAN MATHEMATICAL SOCIETY

Filiala Mehedinți - Mehedinți Branch


www.ssmrmh.ro

75. Let 𝒂, 𝒃 and 𝒄 be positive real numbers such that 𝒂𝒃𝒄 = 𝟏. Prove that:
𝟏 𝟏 𝟔
+ ≥
𝒂𝟑 + 𝒃𝟑 + 𝒄𝟑 𝒂𝒃 + 𝒃𝒄 + 𝒄𝒂 𝒂 𝟐 + 𝒃𝟐 + 𝒄𝟐 𝟐
Proposed by Nguyen Phuc Tang - Dong Thap – Vietnam
Solution 1 by Khung Long Xanh - Da Nang – Vietnam
Solution 2 by Sladjan Stankovic – Macedonia
Solution 1 by Khung Long Xanh - Da Nang – Vietnam
By Cauchy :

𝟏 𝟏 𝟏
+ ≥𝟐
𝒂𝟑 + 𝒃𝟑 + 𝒄𝟑 𝒂𝒃 + 𝒃𝒄 + 𝒄𝒂 𝒂𝟑 + 𝒃𝟑 + 𝒄𝟑 𝒂𝒃 + 𝒃𝒄 + 𝒄𝒂

𝟏 𝟔
𝟐 ≥
𝒂𝟑 + 𝒃𝟑 + 𝒄𝟑 𝒂𝒃 + 𝒃𝒄 + 𝒄𝒂 𝒂 𝟐 + 𝒃𝟐 + 𝒄𝟐 𝟐

⇔ 𝟗 𝒂𝟑 + 𝒃𝟑 + 𝒄𝟑 𝒂𝒃 + 𝒃𝒄 + 𝒄𝒂 ≤ 𝒂𝟐 + 𝒃𝟐 + 𝒄𝟐 𝟒
⇔ 𝟖𝟏 𝒂𝟑 + 𝒃𝟑 + 𝒄𝟑 𝟐
𝒂𝒃 + 𝒃𝒄 + 𝒄𝒂 𝟐
≤ 𝒂𝟐 + 𝒃𝟐 + 𝒄𝟐 𝟖 (*)
By Cauchy – Schwarz :
𝒂 𝟑 + 𝒃𝟑 + 𝒄𝟑 𝟐
≤ 𝒂 𝟒 + 𝒃𝟒 + 𝒄𝟒 𝒂 𝟐 + 𝒃𝟐 + 𝒄𝟐
𝟐
𝒂𝒃 + 𝒃𝒄 + 𝒄𝒂 ≤ 𝟑 𝒂 𝟐 𝒃 𝟐 + 𝒃𝟐 𝒄𝟐 + 𝒄𝟐 𝒂 𝟐 ≤ 𝒂 𝟐 𝒃 𝟐 + 𝒃𝟐 𝒄𝟐 + 𝒄𝟐 𝒂 𝟐 𝟐
( 𝒂𝟐 𝒃𝟐 + 𝒃𝟐 𝒄𝟐 + 𝒄𝟐 𝒂𝟐 ≥ 𝟑)
⇒ 𝑳𝑯𝑺 ≤ 𝟖𝟏 𝒂𝟒 + 𝒃𝟒 + 𝒄𝟒 𝒂𝟐 𝒃𝟐 + 𝒃𝟐 𝒄𝟐 + 𝒄𝟐 𝒂𝟐 𝟐
⋅ 𝒂 𝟐 + 𝒃𝟐 + 𝒄𝟐
𝟐𝟕 𝒂𝟒 + 𝒃𝟒 + 𝒄𝟒 𝒂𝟐 𝒃𝟐 + 𝒃𝟐 𝒄𝟐 + 𝒄𝟐 𝒂𝟐 𝟐
≤ 𝒂𝟐 + 𝒃𝟐 + 𝒄𝟐 𝟔 (AM-GM)
⇒ ≤ 𝟖𝟏 𝒂𝟒 + 𝒃𝟒 + 𝒄𝟒 𝒂𝟐 𝒃𝟐 + 𝒃𝟐 𝒄𝟐 + 𝒄𝟐 𝒂𝟐 𝟐
⋅ 𝒂𝟐 + 𝒃𝟐 + 𝒄𝟐 ≤
≤ 𝟑 ⋅ 𝒂 𝟐 + 𝒃𝟐 + 𝒄𝟐 𝟕
≤ 𝒂 𝟐 + 𝒃𝟐 + 𝒄𝟐 𝟖
(*)

112
SOCIETATEA DE ȘTIINȚE MATEMATICE DIN ROMÂNIA
ROMANIAN MATHEMATICAL SOCIETY

Filiala Mehedinți - Mehedinți Branch


www.ssmrmh.ro

Solution 2 by Sladjan Stankovic – Macedonia


𝟏 𝟏 𝟔
𝒂, 𝒃, 𝒄 > 0, 𝑎𝑏𝑐 = 1 ⇒ + ≥
𝒂𝟑 + 𝒃𝟑 + 𝒄𝟑 𝒂𝒃 + 𝒃𝒄 + 𝒄𝒂 𝒂 𝟐 + 𝒃𝟐 + 𝒄𝟐 𝟐
𝒂+𝒃+𝒄=𝒑≥𝟑
Facts: 𝒓 = 𝒂𝒃𝒄 = 𝟏 ⇒ (1)
𝒂 + 𝒃𝟐 + 𝒄𝟐 = 𝒑𝟐 − 𝟐𝒒 ≥ 𝟑
𝟐

𝟏 𝒂.𝒃.𝒄 𝒂𝒃.𝒃𝒄.𝒄𝒂
≥ ; 𝒂, 𝒃, 𝒄 ≥ 𝟎 (2)
𝒂𝟑 .𝒃𝟑 .𝒄𝟑 𝒂𝟐 .𝒃𝟐 .𝒄𝟐 𝟑


𝟏 𝒂.𝒃.𝒄 𝒂𝒃.𝒃𝒄.𝒄𝒂
𝒂, 𝒃, 𝒄 ≥ 𝟎. Prove that: ≥
𝒂𝟑 .𝒃𝟑 .𝒄𝟑 𝒂𝟐 .𝒃𝟐 .𝒄𝟐 𝟑

𝒑=𝒂+𝒃+𝒄=𝟏
Inequlity is homogeneous ⇒ 𝟏
𝒒 = 𝒂𝒃 + 𝒃𝒄 + 𝒂𝒄 ≤
𝟑

𝟏 𝒑𝒒 𝟏 𝟐
𝑰: ≥ 𝟐 ⇔ ≥
𝒑𝟑 − 𝟑𝒑𝒒 + 𝟑𝒓 𝒑 − 𝟐𝒒 𝟑 𝟏 − 𝟑𝒒 + 𝟑𝒓 𝟏 − 𝟐𝒒 𝟑
𝒒𝟐 ≥ 𝟑𝒑𝒓 ⇒ 𝟑𝒓 ≤ 𝒒𝟐
𝟑 𝟑
⇔ 𝟏 − 𝟐𝒒 ≥ 𝒒 𝟏 − 𝟑𝒒 + 𝟑𝒓 ⇔ 𝟏 − 𝟐𝒒 ≥ 𝒒 𝟏 − 𝟑𝒒 + 𝒒𝟐 ⇔
⇔ 𝟗𝒒𝟑 − 𝟏𝟓𝒒𝟐 + 𝟕𝒒 − 𝟏 ≤ 𝟎 (*)⇔ 𝒒 − 𝟏 𝟑𝒒 − 𝟏 𝟐
≤ 𝟎 (*) True

𝟏 𝟏 𝟐 𝒑𝒒 𝒑𝟐 − 𝟐𝒒 𝟏 𝒑𝒒 𝒑𝟐 − 𝟐𝒒 𝟏 𝑨𝑴7𝑮𝑴
+ ≥ ⋅ + = + ≥
𝒂𝟑 + 𝒃𝟑 + 𝒄𝟑 𝒂𝒃 + 𝒃𝒄 + 𝒄𝒂 𝒑𝟐 − 𝟐𝒒 𝟑 𝒑𝟐 − 𝟐𝒒 𝒒 𝒑𝟐 − 𝟐𝒒 𝟒 𝒒

𝒑𝒒 𝒑𝟐 − 𝟐𝒒 𝟏 𝟐 𝟏 𝟔
≥𝟐⋅ 𝟐 𝟒
⋅ = 𝟐 𝟐
⋅ 𝒑 𝒑𝟐 − 𝟐𝒒 ≥
𝒑 − 𝟐𝒒 𝒒 𝒑 − 𝟐𝒒 𝒂𝟐 + 𝒃 𝟐 + 𝒄𝟐 𝟐

Equality case 𝟏, 𝟏, 𝟏 .

76. Let 𝒙, 𝒚, 𝒛 > 0 and 𝟐 𝒙𝒚𝒛 + 𝒙 + 𝒚 + 𝒛 = 𝟏 then
𝒙𝒚 𝒚𝒛 𝒛𝒙 𝟑
1. + + ≥
𝒛 𝒙 𝒚 𝟒

113
SOCIETATEA DE ȘTIINȚE MATEMATICE DIN ROMÂNIA
ROMANIAN MATHEMATICAL SOCIETY

Filiala Mehedinți - Mehedinți Branch


www.ssmrmh.ro
𝒙 𝒚 𝒛
2. + + ≥ 𝟏𝟐
𝒚𝒛 𝒛𝒙 𝒙𝒚

Proposed by Mihalcea Andrei Ștefan – Romania


Solution by Soumitra Mandal – Chandar Nagore – India
Let 𝒙 + 𝒚 + 𝒛 = 𝟑𝒑𝟐 where 𝒑 > 0 then

𝟐
𝒙+𝒚+𝒛
𝒙 + 𝒚 + 𝒛 + 𝟐 𝒙𝒚𝒛 = 𝟏 ⇒ 𝟏 ≤ 𝒙 + 𝒚 + 𝒛 + 𝟐 = 𝟑𝒑𝟐 + 𝟐𝒑𝟑
𝟑

⇒ 𝟐 𝒑𝟑 + 𝟏 + 𝟑 𝒑𝟐 − 𝟏 ≥ 𝟎 ⇒ 𝒑 + 𝟏 𝟐𝒑𝟐 + 𝒑 − 𝟏 ≥ 𝟎

𝟐
𝟏 𝟑
⇒ 𝒑+𝟏 𝟐𝒑 − 𝟏 ≥ 𝟎 ⇒ 𝒑 ≥ ⇒𝒙+𝒚+𝒛≥
𝟐 𝟒
𝒙𝒚 𝒛𝒙 𝒚𝒛 𝒛𝒙 𝒙𝒚 𝒚𝒛
Applying A.M ≥ G.M, + ≥ 𝟐𝒙, + ≥ 𝟐𝒛 and + ≥ 𝟐𝒚
𝒛 𝒚 𝒙 𝒚 𝒛 𝒙

𝒙𝒚 𝟑
∴ ≥𝒙+𝒚+𝒛≥
𝒛 𝟒
𝒄𝒚𝒄

(Proved)

Let 𝒙𝒚𝒛 = 𝒂𝟔 where 𝒂 > 0 now, 𝒙 + 𝒚 + 𝒛 + 𝟐 𝒙𝒚𝒛 = 𝟏 ⇒

⇒ 𝟏 ≥ 𝟑 𝟑 𝒙𝒚𝒛 + 𝟐 𝒙𝒚𝒛
𝟏
⇒ 𝟐𝒂𝟑 + 𝟑𝒂𝟐 ≤ 𝟏 ⇒ 𝟎 ≥ 𝟐𝒂𝟑 + 𝟑𝒂𝟐 − 𝟏 ⇒ 𝒂 + 𝟏 𝟐
𝟐𝒂 − 𝟏 ≤ 𝟎 ⇒ 𝒂 ≤
𝟐
𝟏
⇒ 𝒙𝒚𝒛 ≤ . Now applying A.M ≥ G.M,
𝟔𝟒

𝒙 𝒚 𝟐 𝒚 𝒛 𝟐
+ ≥ , + ≥
𝒚𝒛 𝒛𝒙 𝒛 𝒛𝒙 𝒙𝒚 𝒙
𝒛 𝒙 𝟐
and + ≥ so,
𝒙𝒚 𝒚𝒛 𝒚

𝒙 𝟏 𝟏 𝟏 𝟑
≥ + + ≥ ≥ 𝟏𝟐
𝒚𝒛 𝒙 𝒚 𝒛 𝟑
𝒙𝒚𝒛
𝒄𝒚𝒄

114
SOCIETATEA DE ȘTIINȚE MATEMATICE DIN ROMÂNIA
ROMANIAN MATHEMATICAL SOCIETY

Filiala Mehedinți - Mehedinți Branch


www.ssmrmh.ro

(Proved)
77. If 𝒂, 𝒃, 𝒄, 𝒅, 𝒆 ∈ 𝟎, ∞ then:
𝒂−𝒄 𝒃−𝒅 𝒄−𝒆 𝒅−𝒂 𝒆−𝒃
+ + + + ≥ 𝟎
𝒃+𝒄 𝒄+𝒅 𝒅+𝒆 𝒆+𝒂 𝒂+𝒃
Proposed by Daniel Sitaru – Romania
Solution by Kevin Soto Palacios – Huarmey-Peru
Si: 𝒂, 𝒃, 𝒄, 𝒅, 𝒆 ∈< 0, ∞ > . Probar que:
𝒂−𝒄 𝒃−𝒅 𝒄−𝒆 𝒅−𝒂 𝒆−𝒃
+ + + + ≥ 𝟎
𝒃+𝒄 𝒄+𝒅 𝒅+𝒆 𝒆+𝒂 𝒂+𝒃
𝒂−𝒄 𝒃−𝒅 𝒄−𝒆 𝒅−𝒂 𝒆−𝒃
⇒ +𝟏 + +𝟏 + +𝟏 + +𝟏 + + 𝟏 ≥ 𝟓
𝒃+𝒄 𝒄+𝒅 𝒅+𝟑 𝒆+𝒂 𝒂+𝒃
𝒂+𝒃 𝒃+𝒄 𝒄+𝒅 𝒅+𝒆 𝒆+𝒂
⇒ + + + + ≥ 𝟓
𝒃+𝒄 𝒄+𝒅 𝒅+𝒆 𝒆+𝒂 𝒂+𝒃
Desde que: 𝒂, 𝒃, 𝒄 > 0. Por las desigualdades entre las medias:
𝑴𝑨 ≥ 𝑴𝑮

𝒂+𝒃 𝒃+𝒄 𝒄+𝒅 𝒅+𝒆 𝒆+𝒂 𝟓 𝒂+𝒃 𝒃+𝒄 𝒄+𝒅 𝒅+𝒆 𝒆+𝒂
⇒ + + + + ≥𝟓
𝒃+𝒄 𝒄+𝒅 𝒅+𝒆 𝒆+𝒂 𝒂+𝒃 𝒃+𝒄 𝒄+𝒅 𝒅+𝒆 𝒆+𝒂 𝒂+𝒃

𝒂+𝒃 𝒃+𝒄 𝒄+𝒅 𝒅+𝒆 𝒆+𝒂


⇒ + + + + ≥ 𝟓
𝒃+𝒄 𝒄+𝒅 𝒅+𝒆 𝒆+𝒂 𝒂+𝒃
⇒ La igualdad se alcanza cuando: 𝒂 = 𝒃 = 𝒄 = 𝒅 = 𝒆
78. Prove that for all positive real numbers 𝒂, 𝒃, 𝒄, 𝒅 such that:
𝒂 𝟐 𝒃 𝟐 𝒄 𝟐 𝒅 𝟐
+ + + + ≥ 𝟏
𝒃+𝒄+𝒅 𝟑 𝒄+𝒅+𝒂 𝟑 𝒅+𝒂+𝒃 𝟑 𝒂+𝒃+𝒄 𝟑
Proposed by Nguyen Viet Hung – Hanoi – Vietnam
Solution by Kevin Soto Palacios – Huarmey – Peru
Siendo: 𝒂, 𝒃, 𝒄, 𝒅 ℝ. . Probar la siguiente desigualdad:

115
SOCIETATEA DE ȘTIINȚE MATEMATICE DIN ROMÂNIA
ROMANIAN MATHEMATICAL SOCIETY

Filiala Mehedinți - Mehedinți Branch


www.ssmrmh.ro

𝒂 𝟐 𝒃 𝟐 𝒄 𝟐 𝒅 𝟐
+ + + + ≥ 𝟏
𝒃+𝒄+𝒅 𝟑 𝒂+𝒄+𝒅 𝟑 𝒂+𝒃+𝒅 𝟑 𝒂+𝒃+𝒄 𝟑
Sean: 𝒃 + 𝒄 + 𝒅 = 𝟑𝒙 ≥ 𝟎, 𝒂 + 𝒄 + 𝒅 = 𝟑𝒚 ≥ 𝟎, 𝒂 + 𝒃 + 𝒅 = 𝟑𝒛 ≥ 𝟎,
𝒂 + 𝒃 + 𝒄 = 𝟑𝒘 ≥ 𝟎
Por lo tanto:
𝒂 = 𝒚 + 𝒛 + 𝒘 − 𝟐𝒙 ≥ 𝟎, 𝒃 = 𝒛 + 𝒙 + 𝒘 − 𝟐𝒚 ≥ 𝟎, 𝒄 = 𝒙 + 𝒚 + 𝒘 − 𝟐𝒛 ≥ 𝟎,
𝒅 = 𝒙 + 𝒚 + 𝒛 − 𝟐𝒘 ≥ 𝟎
La desigualdad es equivalente:
𝒚 + 𝒛 + 𝒘 − 𝟐𝒙 𝟐 𝒛 + 𝒙 + 𝒘 − 𝟐𝒚 𝟐 𝒙 + 𝒚 + 𝒘 − 𝟐𝒛 𝟐 𝒙 + 𝒚 + 𝒛 − 𝟐𝒘 𝟐
+ + + + ≥ 𝟏
𝟑𝒙 𝟑 𝟑𝒚 𝟑 𝟑𝒛 𝟑 𝟑𝒘 𝟑

𝒚+𝒛+𝒘 𝒛+𝒙+𝒘 𝒙+𝒚+𝒘 𝒙+𝒚+𝒛
⇒ ≥ 𝟏
𝟑𝒙 𝟑𝒚 𝟑𝒛 𝟑𝒘

⇒ 𝒚 + 𝒛 + 𝒘 𝒛 + 𝒙 + 𝒘 𝒙 + 𝒚 + 𝒘 𝒙 + 𝒚 + 𝒛 ≥ 𝟖𝟏𝒙𝒚𝒛𝒘 ⇔
⇔ (Válido por: 𝑴𝑨 ≥ 𝑴𝑮)

79. Prove the following inequality holds for all positive real numbers 𝒂, 𝒃, 𝒄
𝟏 𝟏 𝟏 𝟏 𝟏 𝟏 𝟏
+ + + ≥ +
𝟑 𝒂+𝒃 𝒃+𝒄 𝒄+𝒂 𝟒𝒂 + 𝒃 + 𝒄 𝒂 + 𝟐𝒃 + 𝟑𝒄 𝒂 + 𝟑𝒃 + 𝟐𝒄
𝒄𝒚𝒄 𝒄𝒚𝒄

Proposed by Nguyen Viet Hung – Hanoi – Vietnam


Solution by Leonard Giugiuc – Romania
We’ll prove first that

𝒙𝟑 𝒚𝟑 + 𝒙𝒚𝒛 𝒙𝟑 + 𝒚𝟑 + 𝒛𝟑 ≥ 𝒙𝒚𝒛 𝒙𝒚 𝒙 + 𝒚 , ∀𝒙, 𝒚, 𝒛 > 0


𝒄𝒚𝒄 𝒄𝒚𝒄

Indeed, by AM-GM, 𝒙𝒚𝒛 𝒙𝟑 + 𝒚𝟑 + 𝒛𝟑 ≥ 𝟑𝒙𝟐 𝒚𝟐 𝒛𝟐 .

116
SOCIETATEA DE ȘTIINȚE MATEMATICE DIN ROMÂNIA
ROMANIAN MATHEMATICAL SOCIETY

Filiala Mehedinți - Mehedinți Branch


www.ssmrmh.ro

We apply Schur’s inequality for the positive numbers 𝒙𝒚, 𝒚𝒛 and 𝒛𝒙 and get

𝒙𝟑 𝒚𝟑 + 𝟑𝒙𝟐 𝒚𝟐 𝒛𝟐 ≥ 𝒙𝒚𝒛 𝒙𝒚 𝒙 + 𝒚
𝒄𝒚𝒄 𝒄𝒚𝒄

Hence proved.
From above, for all 𝒕 ∈ 𝟎, 𝟏 we have, for 𝒙 = 𝒕𝒂 , 𝒚 = 𝒕𝒃 and 𝒛 = 𝒕𝒄 , that

𝒕𝟑 𝒂.𝒃
+ 𝒕𝟑 𝒃.𝒄
+ 𝒕𝟑 𝒄.𝒂
+ 𝒕𝟒𝒂.𝒃.𝒄 ≥ 𝒕𝒂.𝟐𝒃.𝟑𝒄 + 𝒕𝒂.𝟑𝒃.𝟐𝒄 ⇒
𝒄𝒚𝒄 𝒄𝒚𝒄
𝟏 𝟏
𝟏 𝟑 𝒂.𝒃 𝟏
𝒕 + 𝒕𝟑 𝒃.𝒄 + 𝒕𝟑 𝒄.𝒂 + 𝒕 𝟒𝒂.𝒃.𝒄
𝒅𝒕 ≥ 𝒕𝒂.𝟐𝒃.𝟑𝒄 + 𝒕𝒂.𝟑𝒃.𝟐𝒄 𝒅𝒕 ⇒
𝒕 𝒕
𝟎 𝒄𝒚𝒄 𝟎 𝒄𝒚𝒄

𝟏 𝟏 𝟏 𝟏 𝟏 𝟏 𝟏
+ + + ≥ + .
𝟑 𝒂+𝒃 𝒃+𝒄 𝒄+𝒂 𝟒𝒂 + 𝒃 + 𝒄 𝒂 + 𝟐𝒃 + 𝟑𝒄 𝒂 + 𝟑𝒃 + 𝟐𝒄
𝒄𝒚𝒄 𝒄𝒚𝒄

80. Let 𝒂, 𝒃, 𝒄, 𝒅 be non-negative real numbers. Prove that:

𝒂𝟐 + 𝒃𝟐 𝒄𝟐 + 𝒅𝟐 + 𝟐 𝒂𝒃𝒄𝒅 ≥ 𝒂 + 𝒃 𝒄𝒅 + 𝒄 + 𝒅 𝒂𝒃
Proposed by Nguyen Viet Hung – Hanoi – Vietnam

Solution by Kevin Soto Palacios – Huarmey – Peru
Sean: 𝒂, 𝒃, 𝒄, 𝒅 números ℝ no negativos. Probar la siguiente desigualdad:

𝒂𝟐 + 𝒃𝟐 𝒄𝟐 + 𝒅𝟐 + 𝟐𝒂𝒃𝒄𝒅 ≥ 𝒂 + 𝒃 𝒄𝒅 + 𝒄 + 𝒅 𝒂𝒃
Por: 𝑴𝑪 ≥ 𝑴𝑨
𝒂+𝒃 𝒄+𝒅
𝒂𝟐 + 𝒃𝟐 𝒄𝟐 + 𝒅𝟐 + 𝟐𝒂𝒃𝒄𝒅 ≥ + 𝟐𝒂𝒃𝒄𝒅 ≥ 𝒂 + 𝒃 𝒄𝒅 + 𝒄 + 𝒅 𝒂𝒃
𝟐
Sean: 𝒂 = 𝒙𝟐 ≥ 𝟎, 𝒃 = 𝒚𝟐 ≥ 𝟎, 𝒄 = 𝒛𝟐 ≥ 𝟎, 𝒅 = 𝒘𝟐 ≥ 𝟎
La desigualdad es equivalente:

𝒂 + 𝒃 𝒄 + 𝒅 + 𝟒𝒂𝒃𝒄𝒅 ≥ 𝟐 𝒂 + 𝒃 𝒄𝒅 + 𝟐 𝒄 + 𝒅 𝒂𝒃

117
SOCIETATEA DE ȘTIINȚE MATEMATICE DIN ROMÂNIA
ROMANIAN MATHEMATICAL SOCIETY

Filiala Mehedinți - Mehedinți Branch


www.ssmrmh.ro

𝒙𝟐 + 𝒚𝟐 𝒛𝟐 + 𝒘𝟐 + 𝟒𝒙𝒚𝒛𝒘 ≥ 𝟐𝒙𝟐 𝒛𝒘 + 𝟐𝒚𝟐 𝒛𝒘 + 𝟐𝒛𝟐 𝒙𝒚 + 𝟐𝒘𝟐 𝒙𝒚


𝟐 𝟐 𝟐 𝟐
𝒙𝒛 + 𝒙𝒘 + 𝒚𝒘 + 𝒚𝒛 + 𝟒𝒙𝒚𝒛𝒘 ≥
≥ 𝟐𝒙𝟐 𝒛𝒘 + 𝟐𝒚𝟐 𝒛𝒘 + 𝟐𝒛𝟐 𝒙𝒚 + 𝟐𝒘𝟐 𝒙𝒚
⇒ 𝒙𝟐 𝒛𝟐 + 𝒘𝟐 − 𝟐𝒘𝒛 + 𝒚𝟐 𝒛𝟐 + 𝒘𝟐 − 𝟐𝒘𝒛 − 𝟐𝒙𝒚 𝒛𝟐 + 𝒘𝟐 − 𝟐𝒘𝒛 ≥ 𝟎
𝟐
⇔ 𝒛−𝒘 𝒙𝟐 + 𝒚𝟐 − 𝟐𝒙𝒚 = 𝒛 − 𝒘 𝟐
𝒙−𝒚 𝟐
≥ 𝟎 → (Lo cual es cierto)

𝟏 𝟏 𝟏 𝟏
81. If 𝒂, 𝒃, 𝒄, 𝒅 ∈ 𝟎, ∞ , + + = , 𝒂 ≠ 𝒅, 𝒃 ≠ 𝒅, 𝒄 ≠ 𝒅 then:
𝒂 𝒃 𝒄 𝒅

𝒂+𝒃 𝒃+𝒄 𝒄+𝒂


≥ 𝟐𝟕
𝒂−𝒅 𝒃−𝒅 𝒄−𝒅
Proposed by Daniel Sitaru – Romania
Solution by Kevin Soto Palacios-Huarmey-Peru
𝟏 𝟏 𝟏 𝟏 𝒂.𝒃 𝒄7𝒅 𝒂.𝒃 𝒂𝒃
Del dato: + + = → = → = (A)
𝒂 𝒃 𝒄 𝒅 𝒂𝒃 𝒄𝒅 𝒄7𝒅 𝒄𝒅
𝒃.𝒄 𝒂7𝒅 𝒃.𝒄 𝒃𝒄 𝒂.𝒄 𝒃7𝒅 𝒂.𝒄 𝒂𝒄
De forma análoga: = → = 𝑩 ∧ = → = 𝑪
𝒃𝒄 𝒂𝒅 𝒂7𝒅 𝒂𝒅 𝒂𝒄 𝒃𝒅 𝒃7𝒅 𝒃𝒅

Multiplicando: 𝑨 𝑩 𝑪
𝒂+𝒃 𝒃+𝒄 𝒄+𝒂 𝒂𝒃 𝒃𝒄 𝒂𝒄 𝒂𝒃𝒄
= ⋅ ⋅ = 𝟑
𝒂−𝒅 𝒃−𝒅 𝒄−𝒅 𝒄𝒅 𝒂𝒅 𝒃𝒅 𝒅
Desde que: 𝒂, 𝒃, 𝒄, 𝒅 ∈< 0, ∞ >.
Aplicamos: 𝑴𝑨 ≥ 𝑴𝑮

𝟏 𝟏 𝟏 𝟑 𝟏 𝟏 𝟑 𝟏 𝟏 𝟐𝟕 𝒂𝒃𝒄
+ + ≥𝟑 → ≥𝟑 → 𝟑≥ → 𝟑 ≥ 𝟐𝟕
𝒂 𝒃 𝒄 𝒂𝒃𝒄 𝒅 𝒂𝒃𝒄 𝒅 𝒂𝒃𝒄 𝒅

82. If 𝒂, 𝒃, 𝒄, 𝒅 ∈ 𝟎, ∞ , 𝒂𝒃𝒄𝒅 = 𝟏 then:


𝟓 𝟓 𝟓 𝟓
𝒂+𝒃+𝒄 𝒃+𝒄+𝒅 𝒄+𝒅+𝒂 𝒅+𝒂+𝒃
𝟒
+ 𝟒
+ 𝟒
+ 𝟒
≥ 𝟏𝟐
𝒃+𝒄+𝒅 𝒄+𝒅+𝒂 𝒅+𝒂+𝒃 𝒂+𝒃+𝒄

118
SOCIETATEA DE ȘTIINȚE MATEMATICE DIN ROMÂNIA
ROMANIAN MATHEMATICAL SOCIETY

Filiala Mehedinți - Mehedinți Branch


www.ssmrmh.ro

Proposed by Daniel Sitaru – Romania


Solution by Kevin Soto Palacios-Huarmey-Peru
Si: 𝒂, 𝒃, 𝒄, 𝒅, 𝒆 ∈ < 0, ∞ >, 𝑎𝑏𝑐𝑑 = 1. Probar que:
𝟓 𝟓 𝟓 𝟓
𝒂+𝒃+𝒄 𝒃+𝒄+𝒅 𝒄+𝒅+𝒂 𝒅+𝒂+𝒃
𝟒
+ 𝟒
+ 𝟒
+ 𝟒
≥ 𝟏𝟐
𝒃+𝒄+𝒅 𝒄+𝒅+𝒂 𝒅+𝒂+𝒃 𝒂+𝒃+𝒄
Desde que: 𝒂, 𝒃, 𝒄, 𝒅, 𝒆 ∈ < 0, ∞ >
Por: 𝑴𝑨 ≥ 𝑴𝑮
𝟓 𝟓 𝟓 𝟓
𝒂+𝒃+𝒄 𝒃+𝒄+𝒅 𝒄+𝒅+𝒂 𝒅+𝒂+𝒃
𝟒
+ 𝟒
+ 𝟒
+ 𝟒

𝒃+𝒄+𝒅 𝒄+𝒅+𝒂 𝒅+𝒂+𝒃 𝒂+𝒃+𝒄
𝟒
≥𝟒 𝒂+𝒃+𝒄 𝒃+𝒄+𝒅 𝒄+𝒅+𝒂 𝒅+𝒂+𝒃
𝟑
Pero por: 𝑴𝑨 ≥ 𝑴𝑮: 𝒂 + 𝒃 + 𝒄 ≥ 𝟑 𝒂𝒃𝒄 𝑨
𝟑
𝒃 + 𝒄 + 𝒅 ≥ 𝟑 𝒃𝒄𝒅 𝑩
𝟑
𝒄 + 𝒅 + 𝒂 ≥ 𝟑 𝒄𝒅𝒂 𝑪
𝟑
𝒅 + 𝒂 + 𝒃 ≥ 𝟑 𝒅𝒂𝒃 𝑫
→ Multiplicando: 𝑨 𝑩 𝑪 𝑫
𝒂 + 𝒃 + 𝒄 𝒃 + 𝒄 + 𝒅 𝒄 + 𝒅 + 𝒂 𝒅 + 𝒃 + 𝒂 ≥ 𝟖𝟏𝒂𝒃𝒄𝒅 = 𝟖𝟏 ⇔
𝟒
⇔ 𝒂 + 𝒃 + 𝒄 𝒃 + 𝒄 + 𝒅 𝒄 + 𝒅 + 𝒂 𝒅 + 𝒂 + 𝒃 ≥ 𝟑
Por transitividad:
𝟓 𝟓 𝟓 𝟓
𝒂+𝒃+𝒄 𝒃+𝒄+𝒅 𝒄+𝒅+𝒂 𝒅+𝒂+𝒃
𝟒
+ 𝟒
+ 𝟒
+ 𝟒

𝒃+𝒄+𝒅 𝒄+𝒅+𝒂 𝒅+𝒂+𝒃 𝒂+𝒃+𝒄
𝟒
≥𝟒 𝒂 + 𝒃 + 𝒄 𝒃 + 𝒄 + 𝒅 𝒄 + 𝒅 + 𝒂 𝒅 + 𝒂 + 𝒃 = 𝟏𝟐
83. If 𝒂, 𝒃, 𝒄, 𝒅 ∈ 𝟎, ∞ then:
𝟐 𝟓 𝟓 𝟓 𝟓
𝒂𝒃 + 𝒄𝒅 ≤ 𝒃 𝒂𝒃𝟒 + 𝒅 𝒄𝒅𝟒 𝒂 𝒂𝟒 𝒃 + 𝒄 𝒄𝟒 𝒅

119
SOCIETATEA DE ȘTIINȚE MATEMATICE DIN ROMÂNIA
ROMANIAN MATHEMATICAL SOCIETY

Filiala Mehedinți - Mehedinți Branch


www.ssmrmh.ro

Proposed by Daniel Sitaru – Romania


Proposed by Kevin Soto Palacios
Si: 𝒂, 𝒃, 𝒄, 𝒅 ∈ < 0, ∞ >. Probar que:
𝟐 𝟓 𝟓 𝟓 𝟓
𝒂𝒃 + 𝒄𝒅 ≤ 𝒃 𝒂𝒃𝟒 + 𝒅 𝒄𝒅𝟒 𝒂 𝒂𝟒 𝒃 + 𝒄 𝒄𝟒 𝒅

Por la desigualdad entre las medias:


Usando: Cauchy – Schwarz
Sean: 𝒙𝟏 , 𝒙𝟐 , … , 𝒙𝒏 , 𝒚𝟏 , 𝒚𝟐 , … , 𝒚𝒏 números reales, se cumple:
𝒙𝟏 𝒚𝟏 + 𝒙𝟐 𝒚𝟐 + ⋯ + 𝒙𝒏 𝒚𝒏 𝟐
≤ 𝒙𝟐𝟏 + 𝒙𝟐𝟐 + ⋯ + 𝒙𝟐𝒏 𝒚𝟐𝟏 + 𝒚𝟐𝟐 + ⋯ + 𝒚𝟐𝒏 𝑨
𝒙𝟏 𝒙𝟐
La igualdad se alacanza cuando: =
𝒚𝟏 𝒚𝟐
𝟏𝟎 𝟏𝟎 𝟏𝟎 𝟏𝟎
Sea: 𝒙𝟏 = 𝒃 𝒂𝒃𝟒 , 𝒚𝟏 = 𝒂 𝒂𝟒 𝒃, 𝒙𝟐 = 𝒅 𝒄𝒅𝟒 , 𝒚𝟐 = 𝒄 𝒄𝟒 𝒅
Por la tanto, reemplazando en 𝑨 :
𝟐 𝟓 𝟓 𝟓 𝟓
𝒂𝒃 + 𝒄𝒅 ≤ 𝒃 𝒂𝒃𝟒 + 𝒅 𝒄𝒅𝟒 𝒂 𝒂𝟒 𝒃 + 𝒄 𝒄𝟒 𝒅
𝟏𝟎 𝟏𝟎
𝒃 𝒂𝒃𝟒 𝒅 𝒄𝒅𝟒
La igualdad se alcanza cuando: 𝟏𝟎 = 𝟏𝟎 ⇔ 𝒃𝒄 = 𝒂𝒅
𝒂 𝒃𝒂𝟒 𝒄 𝒄𝟒 𝒅

84. Let 𝒂, 𝒃, 𝒄, 𝒅 be positive real numbers such that:


𝟏 𝟏 𝟏 𝟏
+ + + ≥ 𝟑.
𝒂+𝟏 𝒃+𝟏 𝒄+𝟏 𝒅+𝟏
Prove that:

𝒂 + 𝒃 + 𝒄 + 𝒅 + 𝟒 ≥ 𝒂 + 𝒃 + 𝒄 + 𝒅
Proposed by Nguyen Viet Hung – Hanoi – Vietnam
Solution by Kevin Soto Palacios – Huarmey – Peru
Sean 𝒂, 𝒃, 𝒄, 𝒅 números: ℝ. , de tal manera se cumple que:

120
SOCIETATEA DE ȘTIINȚE MATEMATICE DIN ROMÂNIA
ROMANIAN MATHEMATICAL SOCIETY

Filiala Mehedinți - Mehedinți Branch


www.ssmrmh.ro

𝟏 𝟏 𝟏 𝟏
+ + + ≥ 𝟑
𝒂+𝟏 𝒃+𝟏 𝒄+𝟏 𝒅+𝟏
Probar que:

𝒂 + 𝒃 + 𝒄 + 𝒅 + 𝟒 ≥ 𝒂 + 𝒃 + 𝒄 + 𝒅
Siendo: 𝒂, 𝒃, 𝒄, 𝒅 > 0. Por la desigualdad de Cauchy Schwarz:
𝒂 𝒃 𝒄 𝒅 𝟐
𝒂+𝟏 + 𝒃+𝟏 + 𝒄+𝟏 + 𝒅+𝟏 + + + ≥ 𝒂 + 𝒃 + 𝒄 + 𝒅 ... (A)
𝒂.𝟏 𝒃.𝟏 𝒄.𝟏 𝒅.𝟏

Ahora bien:
𝒂 𝒃 𝒄 𝒅 𝟏 𝟏 𝟏 𝟏
+ + + = 𝟏− + 𝟏− + 𝟏− + 𝟏− ≤ 𝟒 − 𝟑 = 𝟏 ... (B)
𝒂.𝟏 𝒃.𝟏 𝒄.𝟏 𝒅.𝟏 𝟏.𝒂 𝟏.𝒃 𝟏.𝒄 𝟏.𝒅

De (B) ∧ (A) ...


𝟐
𝒂+ 𝒃+ 𝒄+ 𝒅
⇒ 𝒂+𝒃+𝒄+𝒅+𝟒 ≥ ≥
𝒂 𝒃 𝒄 𝒅
+ + +
𝒂+𝟏 𝒃+𝟏 𝒄+𝟏 𝒅+𝟏
𝟐
𝒂+ 𝒃+ 𝒄+ 𝒅 𝟐
≥ = 𝒂+ 𝒃+ 𝒄+ 𝒅
𝟏
Por transitividad: ⇒
𝟐
⇒ 𝒂+𝒃+𝒄+𝒅+𝟒 ≥ 𝒂+ 𝒃+ 𝒄+ 𝒅 ⇔

⇔ 𝒂 + 𝒃 + 𝒄 + 𝟒 ≥ 𝒂 + 𝒃 + 𝒄 + 𝒅 … (LQQD)
85. If 𝒂, 𝒃, 𝒄, 𝒙, 𝒚, 𝒛 ∈ 𝟎, ∞ then:
𝟏 𝟏
𝒂 𝒃 𝒂.𝒃 𝒂 𝒃 𝒄 𝒂.𝒃.𝒄
𝒂+𝒃 𝒙 𝒚 + 𝒄𝒛 ≥ 𝒂 + 𝒃 + 𝒄 𝒙 𝒚 𝒛
Proposed by Daniel Sitaru – Romania
Solution by Hamza Mahmood-Lahore-Pakistan
𝟏
Let 𝒘𝟏 = 𝒂 + 𝒃, 𝒘𝟐 = 𝒄, 𝑿𝟏 = 𝒙𝒂 𝒚𝒃 𝒂u𝒃
, 𝑿𝟐 = 𝒛,
Now 𝒘𝟏 , 𝒘𝟐 , 𝑿𝟏 , 𝑿𝟐 ∈ 𝟎, ∞

121
SOCIETATEA DE ȘTIINȚE MATEMATICE DIN ROMÂNIA
ROMANIAN MATHEMATICAL SOCIETY

Filiala Mehedinți - Mehedinți Branch


www.ssmrmh.ro

By Weighted AM-GM inequality:


𝒘𝟏 𝑿𝟏 + 𝒘𝟐 𝑿𝟐 𝟏
𝒘 𝒘 𝒘𝟏 .𝒘𝟐
≥ 𝑿𝟏 𝟏 ⋅ 𝑿𝟐 𝟐
𝒘𝟏 + 𝒘𝟐
𝟏 𝟏
𝒂 𝒃 𝒂.𝒃 𝟏
𝒂+𝒃 𝒙 𝒚 + 𝒄𝒛 𝒂 𝒃 𝒂.𝒃⋅ 𝒂.𝒃
𝒂.𝒃.𝒄
⇒ ≥ 𝒙 𝒚 ⋅ 𝒛𝒄
𝒂+𝒃+𝒄
𝟏 𝟏
𝒂 𝒃 𝒂.𝒃 𝒂 𝒃 𝒄 𝒂.𝒃.𝒄
⇒ 𝒂+𝒃 𝒙 𝒚 + 𝒄𝒛 ≥ 𝒂 + 𝒃 + 𝒄 𝒙 𝒚 𝒛
𝟏
Equality holds when 𝑿𝟏 = 𝑿𝟐 ⇒ 𝒙𝒂 𝒚𝒃 𝒂u𝒃
= 𝒛 ⇒ 𝒙𝒂 𝒚𝒃 = 𝒛𝒂.𝒃

86. Let 𝒂, 𝒃, 𝒄 be positive real numbers. Prove that
𝟏 𝟏 𝟏 𝟏
+ + ≤
𝒂𝟑 + 𝟖𝒂𝒃𝒄 𝒃𝟑 + 𝟖𝒂𝒃𝒄 𝒄𝟑 + 𝟖𝒂𝒃𝒄 𝟑𝒂𝒃𝒄
Proposed by Nguyen Viet Hung, Hanoi University of Science, Vietnam
Solution 1 by Kevin Soto Palacios – Huarmey – Vietnam
Siendo: 𝒂, 𝒃, 𝒄 números ℝ. . Probar que:
𝟏 𝟏 𝟏 𝟏
+ + ≤
𝒂𝟑 + 𝟖𝒂𝒃𝒄 𝒃𝟑 + 𝟖𝒂𝒃𝒄 𝒄𝟑 + 𝟖𝒂𝒃𝒄 𝟑𝒂𝒃𝒄
𝒃𝒄 𝒂𝒄 𝒂𝒃 𝟏
⇒ 𝟐 + 𝟐 + 𝟐 ≤
𝒂 + 𝟖𝒃𝒄 𝒃 + 𝟖𝒂𝒄 𝒄 + 𝟖𝒂𝒃 𝟑
𝒃𝒄 𝟏 𝒂𝟐
⇒ 𝟐 = −
𝒂 + 𝟖𝒃𝒄 𝟖 𝟖 𝒂𝟐 + 𝟖𝒃𝒄
𝟏 𝒂𝟐 𝟏 𝒃𝟐 𝟏 𝒄𝟐 𝟏
− + − + − ≤
𝟖 𝟖 𝒂𝟐 + 𝟖𝒃𝒄 𝟖 𝟖 𝒃𝟐 + 𝟖𝒂𝒄 𝟖 𝟖 𝒄𝟐 + 𝟖𝒂𝒃 𝟑
𝒂𝟐 𝒃𝟐 𝒄𝟐 𝟏
Demostraremos que: + + ≥
𝒂𝟐 .𝟖𝒃𝒄 𝒃𝟐 .𝟖𝒂𝒄 𝒄𝟐 .𝟖𝒂𝒃 𝟑

Por desigualdad de Cauchy:

122
SOCIETATEA DE ȘTIINȚE MATEMATICE DIN ROMÂNIA
ROMANIAN MATHEMATICAL SOCIETY

Filiala Mehedinți - Mehedinți Branch


www.ssmrmh.ro

𝒂𝟐 𝒃𝟐 𝒄𝟐 𝒂+𝒃+𝒄 𝟐 𝟏
+ 𝟐 + 𝟐 ≥ ≥ ⇒
𝒂𝟐 + 𝟖𝒃𝒄 𝒃 + 𝟖𝒂𝒄 𝒄 + 𝟖𝒂𝒃 𝒂𝟐 + 𝟖 𝒂𝒃 𝟑
𝟐
⇒𝟑 𝒂+𝒃+𝒄 ≥ 𝒂𝟐 + 𝟖 𝒂𝒃

⇔ 𝒂𝟐 ≥ 𝒂𝒃 (LQQD). Por lo tanto:


𝟏 𝒂𝟐 𝟏 𝒃𝟐 𝟏 𝒄𝟐 𝟑 𝟏 𝟏
− + − + − ≤ − ≤
𝟖 𝟖 𝒂𝟐 + 𝟖𝒃𝒄 𝟖 𝟖 𝒃𝟐 + 𝟖𝒂𝒄 𝟖 𝟖 𝒄𝟐 + 𝟖𝒂𝒃 𝟖 𝟐𝟒 𝟑
(LQQD)
Solution 2 by Soumitra Moukherjee - Chandar Nagore – India
𝟏 𝟏 𝟏 𝟏
≥ ⇔ − 𝟑 ≥ 𝟎
𝟑𝒂𝒃𝒄 𝒂𝟑 + 𝟖𝒂𝒃𝒄 𝟗𝒂𝒃𝒄 𝒂 + 𝟖𝒂𝒃𝒄
𝒄𝒚𝒄 𝒄𝒚𝒄

𝒂𝟐 − 𝒃𝒄 𝒂−𝒃 𝒂+𝒄 + 𝒂−𝒃 𝒂+𝒄


⇔ ≥𝟎⇔ ≥ 𝟎
𝟗𝒂𝒃𝒄 𝒂𝟐 + 𝟖𝒃𝒄 𝟏𝟖𝒂𝒃𝒄 𝒂𝟐 + 𝟖𝒃𝒄
𝒄𝒚𝒄 𝒄𝒚𝒄

𝟏 𝒂−𝒃 𝒂+𝒄 𝒂+𝒃 𝒂−𝒄


⇔ 𝟐
+ ≥ 𝟎
𝟏𝟖𝒂𝒃𝒄 𝒂 + 𝟖𝒃𝒄 𝒂𝟐 + 𝟖𝒃𝒄
𝒄𝒚𝒄

𝟏 𝒂−𝒃 𝒂+𝒄 𝒃−𝒂 𝒃+𝒄


⇔ 𝟐
+ ≥ 𝟎
𝟏𝟖𝒂𝒃𝒄 𝒂 + 𝟖𝒃𝒄 𝒃𝟐 + 𝟖𝒂𝒄
𝒄𝒚𝒄

𝒂 − 𝒃 𝟕𝒄 𝒂𝟐 − 𝒃𝟐 + 𝟖𝒄𝟐 𝒂 − 𝒃 − 𝒂𝒃 𝒂 − 𝒃
⇔ ≥ 𝟎
𝟏𝟖𝒂𝒃𝒄 𝒂𝟐 + 𝟖𝒃𝒄 𝒂𝟐 + 𝟖𝒂𝒄
𝒄𝒚𝒄

𝒂7𝒃 𝟐
⇔ 𝒄𝒚𝒄 𝟏𝟖𝒂𝒃𝒄 𝟕𝒂𝒄 + 𝟕𝒃𝒄 + 𝟖𝒄𝟐 − 𝒂𝒃 ≥ 𝟎, which is true
𝟏 𝟏
so, ≥ (proved)
𝟑𝒂𝒃𝒄 𝒂𝟑 .𝟖𝒂𝒃𝒄

87. If 𝒂, 𝒃, 𝒄 ∈ 𝟎, ∞ , 𝒂𝟐 + 𝒃𝟐 + 𝒄𝟐 = 𝟒 − 𝒂𝒃𝒄 then:


𝟐+𝒂 𝟐+𝒃 𝟐+𝒄
≥ 𝟑 𝟑
𝟐−𝒂 𝟐−𝒃 𝟐−𝒄

123
SOCIETATEA DE ȘTIINȚE MATEMATICE DIN ROMÂNIA
ROMANIAN MATHEMATICAL SOCIETY

Filiala Mehedinți - Mehedinți Branch


www.ssmrmh.ro

Proposed by Daniel Sitaru, Romania


Solution by Kevin Soto Palacios – Huarmey – Peru
Si: 𝐚, 𝐛, 𝐜 ∈< 𝟎, ∞ >, de tal manera que: 𝐚𝟐 + 𝐛𝟐 + 𝐜 𝟐 + 𝐚𝐛𝐜 = 𝟒. Probar que:
𝟐.𝐚 𝟐.𝐛 𝟐7𝐜
≥ 𝟑 𝟑 ... (A)
𝟐7𝐚 𝟐7𝐛 𝟐.𝐜

Desde que: 𝐚, 𝐛, 𝐜 > 𝟎


Si: 𝐀 + 𝐁 + 𝐂 = 𝛑
𝟒 𝐜𝐨𝐬 𝟐 𝐀 + 𝟒 𝐜𝐨𝐬 𝟐 𝐁 + 𝟒 𝐜𝐨𝐬 𝟐 𝐂 + 𝟖 𝐜𝐨𝐬 𝐀 𝐜𝐨𝐬 𝐁 𝐜𝐨𝐬 𝐂 = 𝟒
Sean: 𝐚 = 𝟐 𝐜𝐨𝐬 𝐀 > 𝟎, 𝐛 = 𝟐 𝐜𝐨𝐬 𝐁 > 𝟎, 𝐜 = 𝟐 𝐜𝐨𝐬 𝐂 > 𝟎
Reemplazando en ... (A)
𝐀 𝐁 𝐂
𝟐 𝟏 + 𝐜𝐨𝐬 𝐀 𝟐 𝟏 + 𝐜𝐨𝐬 𝐁 𝟐 𝟏 + 𝐜𝐨𝐬 𝐂 𝟒 𝐜𝐨𝐬 𝟐 𝟒 𝐜𝐨𝐬 𝟐 𝟒 𝐬𝐞𝐧𝟐
= 𝟐 𝟐 𝟐 =
𝟐 𝟏 − 𝐜𝐨𝐬 𝐀 𝟐 𝟏 − 𝐜𝐨𝐬 𝐁 𝟐 𝟏 − 𝐜𝐨𝐬 𝐂 𝐀 𝐁 𝐂
𝟒 𝐬𝐞𝐧𝟐 𝟒 𝐬𝐞𝐧𝟐 𝟒 𝐜𝐨𝐬 𝟐
𝟐 𝟐 𝟐
𝐀 𝐁 𝐂 𝟐
= 𝐜𝐨𝐭 𝐜𝐨𝐭 𝐜𝐨𝐭 ≥ 𝟐𝟕 > 𝟑 𝟑 ... (LQQD)
𝟐 𝟐 𝟐


88. Prove that for all positive real numbers 𝒂, 𝒃, 𝒄:
𝟐 𝒂𝒃𝒄 𝒂𝒃 + 𝒃𝒄 + 𝒄𝒂
+ 𝟑 ≥
𝟑 𝒂 + 𝒃𝟑 + 𝒄𝟑 𝒂𝟐 + 𝒃𝟐 + 𝒄𝟐
Proposed by Adil Abdullayev – Baku – Azerbaidian
Solution by Nguyen Viet Hung – Hanoi – Vietnam
The required inequality may be written as:
𝒂𝒃 + 𝒃𝒄 + 𝒄𝒂 𝟏 𝒂𝒃𝒄
𝟏− ≥ −
𝒂 𝟐 + 𝒃𝟐 + 𝒄𝟐 𝟑 𝒂 𝟑 + 𝒃𝟑 + 𝒄𝟑
which is equivalent to:

124
SOCIETATEA DE ȘTIINȚE MATEMATICE DIN ROMÂNIA
ROMANIAN MATHEMATICAL SOCIETY

Filiala Mehedinți - Mehedinți Branch


www.ssmrmh.ro

𝒂𝟐 + 𝒃𝟐 + 𝒄𝟐 − 𝒂𝒃 − 𝒃𝒄 − 𝒄𝒂 𝒂 + 𝒃 + 𝒄 𝒂𝟐 + 𝒃𝟐 + 𝒄𝟐 − 𝒂𝒃 − 𝒃𝒄 − 𝒄𝒂

𝒂𝟐 + 𝒃𝟐 + 𝒄𝟐 𝟑 𝒂 𝟑 + 𝒃𝟑 + 𝒄𝟑
It suffices to show that:
𝟏 𝒂+𝒃+𝒄

𝒂 𝟐 + 𝒃𝟐 + 𝒄𝟐 𝟑 𝒂 𝟑 + 𝒃𝟑 + 𝒄𝟑
Or
𝟑 𝒂 𝟑 + 𝒃𝟑 + 𝒄𝟑 ≥ 𝒂 𝟐 + 𝒃𝟐 + 𝒄𝟐 𝒂 + 𝒃 + 𝒄
But this is true by Tchebyshev’s inequality and we are done.

89. If 𝒂, 𝒃, 𝒄 ∈ 𝟎, ∞ then:
𝟑 𝟔𝒂𝒃𝒄
𝟐𝒂 + 𝟓 𝟐𝒃 + 𝟓 𝟐𝒄 + 𝟓 ≥ + 𝟓
𝒂𝒃 + 𝒃𝒄 + 𝒄𝒂
Proposed by Daniel Sitaru – Romania
Solution 1 by Kevin Soto Palacios – Huarmey – Peru
Solution 2 by Pham Quy –Quang Ngai- Vietnam

Solution 1 by Kevin Soto Palacios – Huarmey – Peru
Si: 𝐚, 𝐛, 𝐜 ∈< 𝟎, ∞ >. Probar que:
𝟑 𝟔𝐚𝐛𝐜
𝟐𝐚 + 𝟓 𝟐𝐛 + 𝟓 𝟐𝐜 + 𝟓 ≥ + 𝟓
𝐚𝐛 + 𝐛𝐜 + 𝐚𝐜
Por la desigualdad de Holder:
𝟑 𝟑 𝟑
𝟐𝐚 + 𝟓 𝟐𝐛 + 𝟓 𝟐𝐜 + 𝟓 ≥ 𝟖𝐚𝐛𝐜 + 𝟏𝟐𝟓
𝟑 𝟑 𝟔𝐚𝐛𝐜
⇒ 𝟐𝐚 + 𝟓 𝟐𝐛 + 𝟓 𝟐𝐜 + 𝟓 ≥ 𝟐 𝐚𝐛𝐜 + 𝟓 ≥ + 𝟓
𝐚𝐛 + 𝐛𝐜 + 𝐚𝐜
Por lo cual solo quenda demostrar:

125
SOCIETATEA DE ȘTIINȚE MATEMATICE DIN ROMÂNIA
ROMANIAN MATHEMATICAL SOCIETY

Filiala Mehedinți - Mehedinți Branch


www.ssmrmh.ro

𝟑 𝟔𝐚𝐛𝐜 𝟑
𝟐 𝐚𝐛𝐜 ≥ ⇔ 𝟐 𝐚𝐛 + 𝐛𝐜 + 𝐚𝐜 ≥
𝐚𝐛 + 𝐛𝐜 + 𝐚𝐜
𝟑 𝟑 𝟐
≥ 𝟐 𝐚𝐛𝐜 𝟑 𝐚𝐛𝐜 = 𝟔𝐚𝐛𝐜 ... (LQQD)


Solution 2 by Pham Quy –Quang Ngai- Vietnam
If 𝒂, 𝒃, 𝒄 ∈ 𝟎, ∞ then
𝟑 𝟔𝒂𝒃𝒄
𝟐𝒂 + 𝟓 𝟐𝒃 + 𝟓 𝟐𝒄 + 𝟓 ≥ + 𝟓
𝒂𝒃 + 𝒃𝒄 + 𝒄𝒂
We have:
𝟑 𝟑 𝟔𝒂𝒃𝒄
𝒂𝒃 + 𝒃𝒄 + 𝒄𝒂 ≥ 𝟑 𝒂𝒃𝒄 𝟐 (AM-GM)⇒ 𝟐 𝒂𝒃𝒄 ≥
𝒂𝒃.𝒃𝒄.𝒄𝒂

𝟑 𝟑
𝟐𝒂 + 𝟓 𝟐𝒃 + 𝟓 𝟐𝒄 + 𝟓 ≥ 𝟐 𝒂𝒃𝒄 + 𝟓 (Holder inequality)
𝟑 𝟑 𝟔𝒂𝒃𝒄
⇒ 𝟐𝒂 + 𝟓 𝟐𝒃 + 𝟓 𝟐𝒄 + 𝟓 ≥ 𝟐 𝒂𝒃𝒄 + 𝟓 ≥ + 𝟓 (q.e.d)
𝒂𝒃.𝒃𝒄.𝒄𝒂

The equality holds at 𝒂 = 𝒃 = 𝒄



90. If 𝒂, 𝒃, 𝒄 ∈ 𝟎, ∞ then:
𝟒𝒂 𝟑𝒃 𝒂 𝒄 𝒃
𝒄 + ≥ 𝟏𝟐 + 𝟑 + +
𝒃𝟐 𝒂𝟐 𝒄 𝒃 𝒂
Proposed by Daniel Sitaru – Romania
Solution by Soumava Chakraborty – Kolkata – India
𝟒𝒂 𝟑𝒃 𝟒𝒂 𝟑𝒃 𝟒𝒃 𝟑𝒄 𝟒𝒄 𝟑𝒂
𝒄 + = 𝒄 + + 𝒂 + + 𝒃 +
𝒃𝟐 𝒂𝟐 𝒃𝟐 𝒂𝟐 𝒄𝟐 𝒃𝟐 𝒂 𝟐 𝒄𝟐
𝟒𝒂𝒄 𝟒𝒂𝒃𝒄 𝟒𝒂𝒃 𝟑𝒂𝒃 𝟑𝒃𝒄 𝟑𝒄𝒂 𝒂𝒃
= + + + + + = 𝟕
𝒃𝟐 𝒂𝟐 𝒄𝟐 𝒄𝟐 𝒂𝟐 𝒃𝟐 𝒄𝟐
𝟒𝒂𝒄 𝟒𝒃𝒄 𝟒𝒂𝒃 𝟑
𝑨𝑴 ≥ 𝑮𝑴 ⇒ + + ≥ 𝟑 𝟒𝟑 = 𝟏𝟐 (1)
𝒃𝟐 𝒂𝟐 𝒄𝟐

126
SOCIETATEA DE ȘTIINȚE MATEMATICE DIN ROMÂNIA
ROMANIAN MATHEMATICAL SOCIETY

Filiala Mehedinți - Mehedinți Branch


www.ssmrmh.ro

𝒂𝒃 𝒂𝒃 𝒄𝒂 𝟑 𝒂𝟑 𝒂
Again, 𝑨𝑴 ≥ 𝑮𝑴 ⇒ + + ≥𝟑 =𝟑 (2)
𝒄𝟐 𝒄𝟐 𝒃𝟐 𝒄𝟑 𝒄

𝒃𝒄 𝒃𝒄 𝒂𝒃 𝟑 𝒃𝟑 𝒃
𝑨𝑴 ≥ 𝑮𝑴 ⇒ 𝟐
+ 𝟐
+ ≥𝟑 =𝟑 (3)
𝒂 𝒂 𝒄𝟐 𝒂𝟑 𝒂

𝒄𝒂 𝒄𝒂 𝒃𝒄 𝒄𝟑 𝒄
𝑨𝑴 ≥ 𝑮𝑴 ⇒ + + ≥𝟑 =𝟑 (4)
𝒃𝟐 𝒃𝟐 𝒂𝟐 𝒃𝟑 𝒃

𝒂𝒃 𝒂 𝒄 𝒃
𝟏 + 𝟐 + 𝟑 + 𝟒 ⇒𝟕 ≥ 𝟏𝟐 + 𝟑 + +
𝒄𝟐 𝒄 𝒃 𝒂
91. Let 𝒂, 𝒃, 𝒄 > 0 and sum of any two is not zero, then:
𝟑 𝟐
𝟏 𝟏 𝟏 𝟏 𝟑 𝒂 + 𝒃 + 𝒄 + 𝒂𝒃𝒄
+ + + 𝟑 >
𝒂 + 𝒃 𝒃 + 𝒄 𝒄 + 𝒂 𝟑 𝒂𝒃𝒄 𝒂+𝒃+𝒄 𝟑
Proposed by Soumitra Mandal - Chandar Nagore – India
Solution by Hung Nguyen Viet – Hanoi – Vietnam
By Chauchy – Schwarz inequality we have
𝟑 𝟐
𝟏 𝟏 𝟏 𝟏 𝒄𝟐 𝒂𝟐 𝒃𝟐 𝒂𝒃𝒄
+ + + 𝟑 = 𝟐 + 𝟐 + 𝟐 +
𝒂 + 𝒃 𝒃 + 𝒄 𝒄 + 𝒂 𝟑 𝒂𝒃𝒄 𝒄 𝒂 + 𝒃 𝒂 𝒃+𝒄 𝒃 𝒄+𝒂 𝟑𝒂𝒃𝒄
𝟑 𝟐
𝒂+𝒃+𝒄+ 𝒂𝒃𝒄

𝒂𝟐 𝒃 + 𝒄 + 𝒃𝟐 𝒄 + 𝒂 + 𝒄𝟐 𝒂 + 𝒃 + 𝟑𝒂𝒃𝒄
It’s suffices to show that
𝟏 𝟑

𝒂𝟐 𝒃 + 𝒄 + 𝒃𝟐 𝒄 + 𝒂 + 𝒄𝟐 𝒂 + 𝒃 + 𝟑𝒂𝒃𝒄 𝒂+𝒃+𝒄 𝟑
which is equivalent to
𝒂𝟑 + 𝒃𝟑 + 𝒄𝟑 + 𝟑 𝒂 + 𝒃 𝒃 + 𝒄 𝒄 + 𝒂 ≥ 𝟑 𝒂𝒃 𝒂 + 𝒃 + 𝒃𝒄 𝒃 + 𝒄 + 𝒄𝒂 𝒄 + 𝒂 + 𝟑𝒂𝒃𝒄
or
𝒂𝟑 + 𝒃𝟑 + 𝒄𝟑 ≥ 𝟑𝒂𝒃𝒄
But this result is clearly true by AM-GM inequality.

127
SOCIETATEA DE ȘTIINȚE MATEMATICE DIN ROMÂNIA
ROMANIAN MATHEMATICAL SOCIETY

Filiala Mehedinți - Mehedinți Branch


www.ssmrmh.ro

Therefore the proof is complete. Note that the equality can’t occur.
92. If 𝒂, 𝒃, 𝒄 ∈ ℕ − 𝟎, 𝟏, 𝟐 then:
𝒂+𝒃+𝟐 𝒃+𝒄+𝟐 𝒄+𝒂+𝟐
𝟖 𝐥𝐨𝐠 𝒂 𝒂 + 𝟏 𝐥𝐨𝐠 𝒃 𝒃 + 𝟏 𝐥𝐨𝐠 𝒄 𝒄 + 𝟏 <
𝒂𝒃𝒄
Proposed by Daniel Sitaru – Romania
Solution by Safal Das Biswas – Chinsurah – India
𝟏
Take a function 𝒇 𝒙 = 𝒙𝒙F𝟏 . We will Check the The maximum value of this
function 𝒇 𝒙 within the domain 𝒙 ∈ 𝟏, 𝟐 .
𝟏 𝟏
𝟏 𝐥𝐧 𝒙
Now 𝒇 𝒙 = 𝒙𝒙F𝟏 . So 𝒇• 𝒙 = 𝒙𝒙F𝟏 −
𝒙 𝒙7𝟏 𝒙7𝟏 𝟐
𝟏
𝟏 𝐥𝐧 𝒙
Now at maximum 𝒇• 𝒙 = 𝟎 then 𝒙𝒙F𝟏 − = 𝟎.
𝒙 𝒙7𝟏 𝒙7𝟏 𝟐
𝒙F𝟏 𝟏 𝟏
𝟏 𝐥𝐧 𝒙 𝒙7𝟏 𝒆
As 𝒙 > 1 then, = implies = 𝐥𝐧 𝒙, or 𝒆 𝒙 = 𝒙, or 𝒆𝟏7𝒙 = 𝒙 or, = 𝒆𝒙 ,
𝒙 𝒙7𝟏 𝒙 𝒙
𝟏
or 𝒆 = 𝒙𝒆𝒙 , this is only possible for 𝒙 = 𝟏 but since 𝒙 > 1 we can claim that the
𝒇• 𝒙 = 𝟎 when 𝐥𝐢𝐦 𝒙 → 𝟏.
Thus we need to find
𝐥𝐢𝐦 𝒇 𝒙 for the max value of 𝒇 within the domain 𝒙 ∈ 𝟏, 𝟐 ,
𝒙→𝟏
𝟏 𝐥𝐧 𝒙 𝟏
𝐥𝐢𝐦 𝐥𝐢𝐦
𝐥𝐢𝐦 𝒇 𝒙 = 𝐥𝐢𝐦 𝒙 𝒙7𝟏 =𝒆 𝒙→𝟏𝒙7𝟏 =𝒆 𝒙→𝟏𝒙 = 𝒆
𝒙→𝟏 𝒙→𝟏

Thus 𝒇 𝒙 < 3 ∀ 𝟐 ≥ 𝒙 > 1.


Now assume 𝒑, 𝒒, 𝒓 ∈ ℝ satisfying 𝒂𝒑 = 𝒂 + 𝟏,
𝒃𝒒 = 𝒃 + 𝟏 and 𝒄𝒓 = 𝒄 + 𝟏.
As 𝒂𝒑 > 𝑎 ∀ 𝒂 ∈ ℕ then 𝒑 > 1 likely 𝒒 > 1 and also 𝒓 > 1.
As, 𝒂, 𝒃, 𝒄 ∈ ℕ − 𝟏, 𝟐 then 𝒂𝟐 > 𝑎 + 1 = 𝒂𝒑 so 𝒑 < 2, likely for 𝒒 and 𝒓.

128
SOCIETATEA DE ȘTIINȚE MATEMATICE DIN ROMÂNIA
ROMANIAN MATHEMATICAL SOCIETY

Filiala Mehedinți - Mehedinți Branch


www.ssmrmh.ro

Thus we have, 𝟏 > 𝒑, 𝒒, 𝒓 < 2.


Since we know that 𝒂, 𝒃, 𝒄 ∈ ℕ − 𝟏, 𝟐 then 𝒂, 𝒃, 𝒄 ≥ 𝟑.
This implies 𝒂, 𝒃, 𝒄 > 𝑓 𝒙 ∀ 𝒙 ∈ 𝟏, 𝟐 as 𝒑, 𝒒, 𝒓 ∈ 𝟏, 𝟐 then 𝒂 > 𝑓 𝒑 or,
𝟏
𝒂>𝒑 𝒑F𝟏 , or 𝒂𝒑7𝟏 > 𝑝 then 𝒂𝒑 > 𝑎𝑝, likely 𝒃𝒒 > 𝑏𝑞 and 𝒄𝒓 > 𝑐𝑟

𝒂+𝒃+𝟐 = 𝒂𝒑 + 𝒃𝒒 ≥ 𝟖𝒂𝒑 𝒃𝒒 𝒄𝒓 > 8𝒂𝒃𝒄𝒑𝒒𝒓


𝒂,𝒃,𝒄 𝒄𝒚𝒄𝒍𝒊𝒄 𝒂𝒑 ,𝒃𝒒 ,𝒄𝒓 𝒄𝒚𝒄𝒍𝒊𝒄

So we have the following


𝒂+𝒃+𝟐
> 8𝒑𝒒𝒓 = 𝟖 𝐥𝐨𝐠 𝒂 𝒂 + 𝟏
𝒄
𝒂,𝒃,𝒄 𝒄𝒚𝒄𝒍𝒊𝒄 𝒂,𝒃,𝒄 𝒄𝒚𝒄𝒍𝒊𝒄


93. If 𝒙, 𝒚, 𝒛 ∈ ℕ − 𝟎, 𝟏 then:
𝒙u𝒚 𝒚u𝒛 𝒛u𝒙
𝟐𝒙 + 𝟐𝒚 + 𝟐𝒛 + 𝟐𝒙.𝒚.𝒛 > 𝟏𝟔𝒙𝒚 + 𝟏𝟔𝒚𝒛 + 𝟏𝟔𝒛𝒙 + 𝟏
Proposed by Daniel Sitaru – Romania
Solution by Ravi Prakash – New Delhi – India
𝟐𝒙 + 𝟐𝒚 + 𝟐𝒛 + 𝟐𝒙.𝒚.𝒛 − 𝟐𝒙.𝒚 − 𝟐𝒚.𝒛 − 𝟐𝒛.𝒙 − 𝟏
= 𝟐𝒙 − 𝟏 𝟐𝒚 − 𝟏 𝟐𝒛 − 𝟏 > 0
⇒ 𝟐𝒙 + 𝟐𝒚 + 𝟐𝒛 + 𝟐𝒙.𝒚.𝒛 > 𝟐𝒙.𝒚 + 𝟐𝒚.𝒛 + 𝟐𝒛.𝒙 + 𝟏
𝒙u𝒚 𝟐𝒙𝒚
𝒙u𝒚
But: 𝟐𝒙.𝒚 = 𝟒 𝟐 ≥ 𝟒𝒙u𝒚 = 𝟏𝟔𝒙𝒚 . Hence:
𝟏 𝟏 𝟏
𝟐𝒙 + 𝟐𝒚 + 𝟐𝒛 + 𝟐𝒙.𝒚.𝒛 > 𝟏𝟔𝒙𝒚 𝒙.𝒚 + 𝟏𝟔𝒚𝒛 𝒚.𝒛 + 𝟏𝟔𝒛𝒙 𝒛.𝒙 + 𝟏
𝟏.𝒂 𝟏.𝒃 𝟏.𝒄 𝒂𝒃𝒄. 𝒂𝒃𝒄
94. Given 𝒂, 𝒃, 𝒄 ∈ 𝟏, +∞ prove that: ≥ 𝟐.
𝒂. 𝒂 𝒃. 𝒃 𝒄. 𝒄

Proposed by Daniel Sitaru-Romania


Solutions by Ngô Minh Ngọc Bảo-Vietnam

129
SOCIETATEA DE ȘTIINȚE MATEMATICE DIN ROMÂNIA
ROMANIAN MATHEMATICAL SOCIETY

Filiala Mehedinți - Mehedinți Branch


www.ssmrmh.ro

Solution 1
We have:

𝟏 + 𝒂 𝟏 + 𝒃 𝟏 + 𝒄 𝒂𝒃𝒄 + 𝒂𝒃𝒄
≥ 𝟐 ⇔
𝒂+ 𝒂 𝒃+ 𝒃 𝒄+ 𝒄
𝟏
𝐥𝐧 𝒂 + 𝟏 + 𝐥𝐧 𝒂 − 𝒂+ 𝒂 + 𝐥𝐧 𝒂𝒃𝒄 + 𝟏 ≥ 𝐥𝐧 𝟐
𝟐
𝟏
Considering function: 𝒇 𝒕 = 𝐥𝐧 𝒕 + 𝟏 + 𝐥𝐧 𝒕 − 𝐥𝐧 𝒕 + 𝒕 , ∀ 𝒕 ∈ 𝟏, +∞
𝟐

𝟏 𝟏 𝟐 𝒕+𝟏
⇒ 𝒇ʹ 𝒕 = + − =
𝟐𝒕 𝟏 + 𝒕 𝟐𝒕 𝒕 + 𝟏

𝒕+𝟏 𝒕 + 𝟏 + 𝟐𝒕 𝒕+𝟏 − 𝟐 𝒕+𝟏 𝒕+𝟏


= =
𝟐𝒕 𝒕 + 𝟏 𝒕+𝟏
𝒕+𝟐 𝒕−𝟏
= > 𝟎
𝟐 𝒕 𝒕+𝟏 𝒕+𝟏
𝟏
⇒ 𝐥𝐧 𝒂 + 𝟏 + 𝐥𝐧 𝒂 − 𝐥𝐧 𝒂 + 𝒂 + 𝐥𝐧 𝒂𝒃𝒄 + 𝟏 ≥
𝟐
≥ 𝟑 𝐥𝐧 𝟐 − 𝟑 𝐥𝐧 𝟐 + 𝐥𝐧 𝟐 = 𝐥𝐧 𝟐 𝒂𝒃𝒄 + 𝟏 ≥ 𝟐
Solution 2
𝟏.𝒂 𝟏.𝒃 𝟏.𝒄 𝒂𝒃𝒄. 𝒂𝒃𝒄
We have: 𝒇 𝒂, 𝒃, 𝒄 = , we need to prove 𝒇 𝒂, 𝒃, 𝒄 ≥ 𝟐.
𝒂. 𝒂 𝒃. 𝒃 𝒄. 𝒄

Use: 𝟏 + 𝒙𝟑 𝟏 + 𝒚𝟑 𝟏 + 𝒛𝟑 ≥ 𝟏 + 𝒙𝒚𝒛 𝟑 , 𝒙, 𝒚, 𝒛 > 𝟎 we have:


𝟑 𝟑
𝟏 + 𝒂 𝟏 + 𝒃 𝟏 + 𝒄 ≥ 𝟏 + 𝒂𝒃𝒄

And 𝒂 + 𝒂 𝒃 + 𝒃 𝒄 + 𝒄 ≤ 𝒂 + 𝒂 𝒃 + 𝒃 𝒄 + 𝒄 = 𝟖𝒂𝒃𝒄
𝟑 𝟑
𝟏 + 𝒂𝒃𝒄 𝒂𝒃𝒄 + 𝒂𝒃𝒄 𝟏 𝟑 𝟑 𝟏
⇒ 𝒇 𝒂, 𝒃, 𝒄 ≥ = 𝟏 + 𝒂𝒃𝒄 𝟏+ =
𝟖𝒂𝒃𝒄 𝟖 𝒂𝒃𝒄

130
SOCIETATEA DE ȘTIINȚE MATEMATICE DIN ROMÂNIA
ROMANIAN MATHEMATICAL SOCIETY

Filiala Mehedinți - Mehedinți Branch


www.ssmrmh.ro

𝟑 𝟑 𝟑
𝟏 𝟑 𝟑 𝟏 + 𝒂𝒃𝒄 𝟏 𝟑 𝟑 𝟏 𝟔
= 𝟏 + 𝒂𝒃𝒄 + = 𝟏 + 𝒂𝒃𝒄 + + 𝒂𝒃𝒄 ≥
𝟖 𝟔
𝒂𝒃𝒄 𝟖 𝟔
𝒂𝒃𝒄
𝟏 𝟑 𝟑
≥ 𝟏+𝟏 + 𝟐 = 𝟐
𝟖
95. Prove that for all positive real numbers 𝒙, 𝒚, 𝒛
𝒙𝟑 + 𝒚 𝟑 + 𝒛 𝟑 𝟑

𝒙 𝒚+𝒛 𝟐 𝟒
Proposed by Nguyen Viet Hung – Hanoi – Vietnam
Solution 1 by Kevin Soto Palacios – Huarmey – Peru
Solution 2 by Marian Dincă – Romania
Solution 1 by Kevin Soto Palacios – Huarmey – Peru
Probar para todos los números 𝐑. 𝐱. 𝐲. 𝐳:
𝐱𝟑 + 𝐲𝟑 + 𝐳𝟑 𝟑

𝐱 𝐲+𝐳 𝟐 𝟒
Supongamos sin pérdida de generalidad que:
𝐱 + 𝐲 + 𝐳 = 𝟑 ⇔ 𝟎 < 𝐱, 𝐲, 𝐳 < 𝟑
Por: MP ≥ MA
𝟗 𝐱𝟑 + 𝐲𝟑 + 𝐳𝟑 ≥ 𝐱 + 𝐲 + 𝐳 𝟑

⇔ 𝟗 𝐱 𝟑 + 𝐲 𝟑 + 𝐳 𝟑 ≥ 𝟐𝟕 ⇔ 𝐱 𝟑 + 𝐲 𝟑 + 𝐳 𝟑 ≥ 𝟑
Lo cual solo nos falta probar que:
𝟐 𝟐
𝟒≥𝐱 𝐲+𝐳 ⇔𝟒≥𝐱 𝟑−𝐱 ⇔ 𝟒 ≥ 𝟗𝐱 − 𝟔𝐱 𝟐 + 𝐱 𝟑
⇒ −𝐱 𝟑 + 𝟔𝐱 𝟐 − 𝟗𝐱 + 𝟒 ≥ 𝟎 ⇔ −𝐱 𝟑 + 𝟔𝐱 𝟐 − 𝟗𝐱 + 𝟒 = 𝐱 − 𝟏 𝟐
𝟒 − 𝐱 ≥ 𝟎 ...
(LQQD)
La igualdad se alcanza cuando: 𝐱 = 𝐲 = 𝐳

131
SOCIETATEA DE ȘTIINȚE MATEMATICE DIN ROMÂNIA
ROMANIAN MATHEMATICAL SOCIETY

Filiala Mehedinți - Mehedinți Branch


www.ssmrmh.ro

Solution 2 by Marian Dincă – Romania


𝐱𝟑 + 𝐲𝟑 + 𝐳𝟑 𝟑

𝐱 𝐲+𝐳 𝟐 𝟒
𝐲.𝐳 𝟑
𝐲𝟑 + 𝐳𝟑 ≥ 𝟐 − proof elementary of Jensen inequality and use AM – GM
𝟐
𝟑 𝟑
𝟑 𝟑
𝐲+𝐳 𝟑 𝟑
𝐲+𝐳
𝟑
𝐲+𝐳 𝟑
𝐱 +𝐲 +𝐳 ≥𝐱 +𝟐 =𝐱 + +
𝟐 𝟐 𝟐
𝟑 𝟑 𝟐
𝟑 𝐲+𝐳 𝐲+𝐳 𝐲+𝐳 𝟑
𝟑 𝐱𝟑 ⋅ ⋅ = 𝟑𝐱 = 𝐱 𝐲 + 𝐳 𝟐
𝟐 𝟐 𝟐 𝟒
96. Let 𝒙, 𝒚, 𝒛 be positive real numbers such that 𝒙𝒚𝒛 = 𝟏. Prove that:
𝟏 𝟏 𝟏 𝟏
+ + ≥
𝟏 + 𝟖𝒙 𝟏 + 𝟖𝒚 𝟏 + 𝟖𝒛 𝟑
Proposed by Nguyen Viet Hung – Hanoi – Vietnam
Solution 1,2 by Kevin Soto Palacios – Huarmey – Peru
Solution 1 by Kevin Soto Palacios – Huarmey – Peru
Sean: 𝐱, 𝐲, 𝐳 números ℝ. , de tal manera que 𝐱𝐲𝐳 = 𝟏. Probar que:
𝟏 𝟏 𝟏 𝟏
+ + ≥
𝟏 + 𝟖𝐱 𝟏 + 𝟖𝐲 𝟏 + 𝟖𝐳 𝟑
𝐚 𝐛 𝐜
Sea: 𝐱 = > 𝟎, 𝐲 = > 𝟎, 𝐳 = > 𝟎 ⇔ 𝐚, 𝐛, 𝐜 > 𝟎
𝐛 𝐜 𝐚

La desigualdad es equivalente:
𝟏 𝟏 𝟏 𝟏
⇒ + + ≥
𝟖𝐚 𝟖𝐛 𝟖𝐜 𝟑
𝟏+ 𝟏+
𝐛 𝟏+ 𝐜 𝐚
𝐛 𝐜 𝐚 𝟏
⇒ + + ≥
𝐛 + 𝟖𝐚 𝐜 + 𝟖𝐛 𝐚 + 𝟖𝐜 𝟑

132
SOCIETATEA DE ȘTIINȚE MATEMATICE DIN ROMÂNIA
ROMANIAN MATHEMATICAL SOCIETY

Filiala Mehedinți - Mehedinți Branch


www.ssmrmh.ro

𝐛𝟐 𝐜𝟐 𝐚 𝟏
⇒ 𝟐 + 𝟐 + 𝟐 ≥
𝐛 + 𝟖𝐚𝐛 𝐜 + 𝟖𝐜𝐛 𝐚 + 𝟖𝐚𝐜 𝟑
Por desigualdad de Bergstrom’s (Cauchy):
𝐚+𝐛+𝐜 𝟐 𝟏
⇒ 𝟐 ≥ ⇔
𝐚 + 𝐛 𝟐 + 𝐜 𝟐 + 𝟖𝐚𝐛 + 𝟖𝐛𝐜 + 𝟖𝐚𝐜 𝟑
𝟐
⇔𝟑 𝐚+𝐛+𝐜 ≥ 𝐚𝟐 + 𝐛𝟐 + 𝐜 𝟐 + 𝟖𝐚𝐛 + 𝟖𝐛𝐜 + 𝟖𝐚𝐜
⇒ 𝟑𝐚𝟐 + 𝟑𝐛𝟐 + 𝟑𝐜 𝟐 + 𝟔𝐚𝐛 + 𝟔𝐛𝐜 + 𝟔𝐚𝐜 ≥ 𝐚𝟐 + 𝐛𝟐 + 𝐜 𝟐 + 𝟖𝐚𝐛 + 𝟖𝐜 + 𝟖𝐚𝐜
⇒ 𝐚𝟐 + 𝐛𝟐 + 𝐜 𝟐 ≥ 𝐚𝐛 + 𝐛𝐜 + 𝐚𝐜 … (LQQD)
Solution 2 by Kevin Soto Palacios – Huarmey – Peru
Sean 𝒙, 𝒚, 𝒛 números ℝ. , de tal manera que: 𝒙𝒚𝒛 = 𝟏. Probar que:
𝟏 𝟏 𝟏 𝟏
+ + ≥
𝟏 + 𝟖𝒙 𝟏 + 𝟖𝒚 𝟏 + 𝟖𝒛 𝟑
Multiplicando 𝟏 + 𝟖𝒙 𝟏 + 𝟖𝒚 𝟏 + 𝟖𝒛 :
𝟑 𝟏 + 𝟖𝒚 𝟏 + 𝟖𝒛 + 𝟏 + 𝟖𝒙 𝟏 + 𝟖𝒛 + 𝟏 + 𝟖𝒙 𝟏 + 𝟖𝒚 ≥
≥ 𝟏 + 𝟖𝒙 𝟏 + 𝟖𝒚 𝟏 + 𝟖𝒛
⇒ 𝟑 𝟑 + 𝟏𝟔 𝒙 + 𝒚 + 𝒛 + 𝟔𝟒𝒙𝒚 + 𝟔𝟒𝒚𝒛 + 𝟔𝟒𝒙𝒛 ≥
≥ 𝟏𝟑 + 𝟏𝟐 𝒙𝟖 𝒙 + 𝒚 + 𝒛 + 𝟏𝒙𝟔𝟒 𝒙𝒚 + 𝒚𝒛 + 𝒙𝒛 + 𝟓𝟏𝟐𝒙𝒚𝒛
⇒ 𝟗 + 𝟒𝟖 𝒙 + 𝒚 + 𝒛 + 𝟏𝟗𝟐 𝒙𝒚 + 𝒚𝒛 + 𝒛𝒙 ≥
≥ 𝟏 + 𝟖 𝒙 + 𝒚 + 𝒛 + 𝟔𝟒 𝒙𝒚 + 𝒚𝒛 + 𝒙𝒛 + 𝟓𝟏𝟐
⇒ 𝟒𝟎 𝒙 + 𝒚 + 𝒛 + 𝟏𝟐𝟖 𝒙𝒚 + 𝒚𝒛 + 𝒙𝒛 ≥ 𝟓𝟎𝟒
Desde que: 𝒙, 𝒚, 𝒛 > 0
Por: MA ≥ 𝑴𝑮
𝟑
⇒ 𝟒𝟎 𝒙 + 𝒚 + 𝒛 + 𝟏𝟐𝟖 𝒙𝒚 + 𝒚𝒛 + 𝒙𝒛 ≥ 𝟒𝟎𝒙𝟑 𝟑 𝒙𝒚𝒛 + 𝟏𝟐𝟖𝒙𝟑 𝒙𝒚𝒛 𝟐 =
= 𝟏𝟐𝟎 + 𝟑𝟖𝟒 = 𝟓𝟎𝟒 … (LQQD)

133
SOCIETATEA DE ȘTIINȚE MATEMATICE DIN ROMÂNIA
ROMANIAN MATHEMATICAL SOCIETY

Filiala Mehedinți - Mehedinți Branch


www.ssmrmh.ro

97. If 𝒂, 𝒃, 𝒄 ∈ 𝟎, ∞ then:
𝒄 𝒂−𝒃 𝟐
𝒄 𝟖 𝒂𝟐 + 𝒃𝟐 ≤ 𝟒 𝒂𝒃 + 𝒃𝒄 + 𝒄𝒂 +
𝒂+𝒃
Proposed by Daniel Sitaru – Romania
Solution 1 by Ravi Prakash - New Delhi – India
Solution 2 by Richdad Phuc – Hanoi – Vietnam
Solution 3 by Soumitra Mandal - Chandar Nagore – India
Solution 1 by Ravi Prakash - New Delhi – India
Consider
𝟐 𝟐 𝟐
𝟐 𝐚+𝐛 + 𝐚−𝐛 − 𝟖 𝐚𝟐 + 𝐛𝟐 𝐚 + 𝐛 𝟐
=
= 𝟑𝐚𝟐 + 𝟑𝐛𝟐 + 𝟐𝐚𝐛 𝟐
− 𝟖 𝐚𝟐 + 𝐛𝟐 𝐚𝟐 + 𝐛𝟐 + 𝟐𝐚𝐛 =
= 𝟗𝐚𝟒 + 𝐚𝐛𝟒 + 𝟒𝐚𝟐 𝐛𝟐 + 𝟏𝟐𝐚𝟑 𝐛 + 𝟏𝟐𝐚𝐛𝟑 + 𝟏𝟖𝐚𝟐 𝐛𝟐 −
−𝟖 𝐚𝟒 + 𝐛𝟒 + 𝟐𝐚𝟐 𝐛𝟐 + 𝟐𝐚𝟑 𝐛 + 𝟐𝐚𝐛𝟑 =
= 𝐚𝟒 + 𝐛𝟒 + 𝟔𝐚𝟐 𝐛𝟐 − 𝟒𝐚𝟑 𝐛 − 𝟒𝐚𝐛𝟑 =
= 𝐚𝟐 + 𝐛𝟐 − 𝟐𝐚𝐛 𝟐
= 𝐚−𝐛 𝟒
≥ 𝟎
𝟐 𝟐
⇒𝟐 𝐚+𝐛 + 𝐚−𝐛 ≥ 𝟖 𝐚𝟐 + 𝐛𝟐 𝐚 + 𝐛 𝟐
𝐚−𝐛 𝟐
⇒ 𝟖 𝐚𝟐 + 𝐛𝟐 ≤ +𝟐 𝐚+𝐛
𝐚+𝐛
𝐜 𝐚−𝐛 𝟐
⇒𝐜 𝟖 𝐚𝟐 + 𝐛𝟐 ≤ 𝟐 𝐚𝐜 + 𝐛𝐜 +
𝐚+𝐛
𝟐
𝐜 𝐚−𝐛
⇒ 𝐜 𝟖 𝐚𝟐 + 𝐛𝟐 ≤ + 𝟒 𝐚𝐛 + 𝐛𝐜 + 𝐜𝐚
𝐚+𝐛
Solution 2 by Richdad Phuc – Hanoi – Vietnam
We have:

134
SOCIETATEA DE ȘTIINȚE MATEMATICE DIN ROMÂNIA
ROMANIAN MATHEMATICAL SOCIETY

Filiala Mehedinți - Mehedinți Branch


www.ssmrmh.ro

𝐜 𝟖 𝐚𝟐 + 𝐛 𝟐 − 𝟒 𝐚𝐛 + 𝐛𝐜 + 𝐜𝐚 = 𝟐𝐜 𝟐 𝐚𝟐 + 𝐛 𝟐 − 𝐚 + 𝐛 =
𝟐
𝟐𝐜 𝐚 − 𝐛
=
𝟐 𝐚𝟐 + 𝐛𝟐 + 𝐚+𝐛
By Cauchy – Schwarz

𝟐 𝐚𝟐 + 𝐛 𝟐 ≥ 𝐚 + 𝐛
𝐜 𝐚−𝐛 𝟐
⇒ 𝐜 𝟖 𝐚𝟐 + 𝐛𝟐 − 𝟒 𝐚𝐛 + 𝐛𝐜 + 𝐜𝐚 ≤
𝐚+𝐛
Done
Equality for hold 𝐚 = 𝐛 = 𝐜
Solution 3 by Soumitra Mandal - Chandar Nagore – India
𝟐 𝟐
𝒄 𝒂−𝒃 𝒄 𝒂−𝒃
+ 𝟒 𝒂𝒃 + 𝒃𝒄 + 𝒄𝒂 = +𝒄 𝒂+𝒃 + 𝟐 𝒂𝒃 + 𝒃𝒄 + 𝒄𝒂 =
𝒂+𝒃 𝒂+𝒃
𝒄𝒚𝒄 𝒄𝒚𝒄

𝟐 𝟐
𝟐𝒄 𝒂𝟐 + 𝒃 𝟐𝒄 𝒂𝟐 + 𝒃
= + 𝟐 𝒂𝒃 + 𝒃𝒄 + 𝒄𝒂 = +𝒄 𝒂+𝒃 ≥
𝒂+𝒃 𝒂+𝒃
𝒄𝒚𝒄 𝒄𝒚𝒄

𝟐
𝟐𝒄 𝒂𝟐 + 𝒃 𝟐
≥𝟐 𝒄 𝒂+𝒃 = 𝒄 𝟖 𝒂𝟐 + 𝒃
𝒂+𝒃
𝒄𝒚𝒄 𝒄𝒚𝒄

(proved)
98. If 𝒂, 𝒃, 𝒄 ∈ 𝟎, ∞ then:
𝟏 𝟏 𝟏 𝒂 𝒄 𝒃
𝒂 𝟒 + 𝒃𝟒 + 𝒄𝟒 𝟒
+ 𝟒+ 𝟒 ≥𝟐 + +
𝒂 𝒃 𝒄 𝒄 𝒃 𝒂
Proposed by Daniel Sitaru – Romania
Solution by Ravi Prakash - New Delhi – India

𝟒 𝟒 𝟒
𝟏 𝟏 𝟏 𝐚𝟒 𝐛𝟒 𝐚𝟒 𝐜 𝟒 𝐛𝟒 𝐜 𝟒
𝐚 +𝐛 +𝐜 + + =𝟑+ 𝟒+ 𝟒 + 𝟒+ 𝟒 + 𝟒 + 𝟒
𝐚𝟒 𝐛 𝟒 𝐜 𝟒 𝐛 𝐚 𝐜 𝐚 𝐜 𝐛

135
SOCIETATEA DE ȘTIINȚE MATEMATICE DIN ROMÂNIA
ROMANIAN MATHEMATICAL SOCIETY

Filiala Mehedinți - Mehedinți Branch


www.ssmrmh.ro

𝐚 𝐚𝟒 𝐛𝟒 𝐚
If < 𝟏, 𝟒
+ >𝟐>𝟐
𝐛 𝐛 𝐚𝟒 𝐛

Similarly for any expression < 𝟏 on RHS.


𝐚 𝐚𝟒 𝐛𝟒 𝐚𝟒
If ≥ 𝟏, 𝟒
+ 𝟒
+𝟏> + 𝟏
𝐛 𝐛 𝐚 𝐛𝟒

𝐚𝟒 𝐚𝟐 𝐚
and 𝟒
+𝟏≥𝟐 ≥ 𝟐 .
𝐛 𝐛𝟐 𝐛

Similarly for other expression ≥ 𝟏 on RHS.


In any case we get
𝟏 𝟏 𝟏 𝐚 𝐛 𝐜
𝐚𝟒 + 𝐛𝟒 + 𝐜 𝟒 + + > 𝟐 + +
𝐚𝟒 𝐛 𝟒 𝐜 𝟒 𝐛 𝐜 𝐚

99. Prove that for all positive real numbers 𝒂, 𝒃, 𝒄
𝟑
𝒂+𝒃+𝒄 𝟕 𝒃+𝒄 𝒄+𝒂 𝒂+𝒃
≥ + + + 𝟔
𝒂𝒃𝒄 𝟐 𝒂 𝒃 𝒄
Proposed by Nguyen Viet Hung – Hanoi – Vietnam
Solution by Kevin Soto Palacios – Huarmey – Peru
Probar para todos los números 𝐑. 𝐚, 𝐛, 𝐜:
𝟑
𝐚+𝐛+𝐜 𝟕 𝐛+𝐜 𝐜+𝐚 𝐚+𝐛
≥ + + + 𝟔
𝐚𝐛𝐜 𝟐 𝐚 𝐛 𝐜
Recordar la siguiente identidad:
𝟑
𝐚+𝐛+𝐜 = 𝐚𝟑 + 𝐛𝟑 + 𝐜 𝟑 + 𝟑𝐚𝐛 𝐚 + 𝐛 + 𝟑𝐛𝐜 𝐛 + 𝐜 + 𝟑𝐚𝐜 𝐚 + 𝐜 + 𝟔𝐚𝐛𝐜
Reemplazando en la desigualdad:
𝐚𝟑 + 𝐛𝟑 + 𝐜 𝟑 𝐛+𝐜 𝐜+𝐚 𝐚+𝐛 𝟕 𝐛+𝐜 𝐜+𝐚 𝐚+𝐛
+𝟑 + + +𝟔≥ + + + 𝟔
𝐚𝐛𝐜 𝐚 𝐛 𝐜 𝟐 𝐚 𝐛 𝐜
𝐚𝟑 + 𝐛𝟑 + 𝐜 𝟑 𝟏 𝐛 + 𝐜 𝐜 + 𝐚 𝐚 + 𝐛
≥ + + ⇔
𝐚𝐛𝐜 𝟐 𝐚 𝐛 𝐜

136
SOCIETATEA DE ȘTIINȚE MATEMATICE DIN ROMÂNIA
ROMANIAN MATHEMATICAL SOCIETY

Filiala Mehedinți - Mehedinți Branch


www.ssmrmh.ro

⇔ 𝟐𝐚𝟑 + 𝟐𝐛𝟑 + 𝟐𝐜 𝟑 ≥ 𝐚𝐛 𝐚 + 𝐛 + 𝐛𝐜 𝐛 + 𝐜 + 𝐚𝐜 𝐚 + 𝐜
Desde que: 𝐚, 𝐛, 𝐜 > 𝟎. Por: MA ≥ MG
𝐚𝟑 + 𝐚𝟑 + 𝐛𝟑 ≥ 𝟑𝐚𝟐 𝐛 ... (I)
𝐛𝟑 + 𝐛𝟑 + 𝐚𝟑 ≥ 𝟑𝐛𝟐 𝐚 ... (II)
Sumando (I) + (II): 𝐚𝟑 + 𝐛𝟑 ≥ 𝐚𝐛 𝐚 + 𝐛 ... (III)
Por lo tanto: 𝐛𝟑 + 𝐜 𝟑 ≥ 𝐛𝐜 𝐛 + 𝐜 ... (IV) ∧ 𝐚𝟑 + 𝐜 𝟑 ≥ 𝐚𝐜 𝐚 + 𝐜 ... (V)
Sumando: (III) + (IV) + (V):
𝟐𝐚𝟑 + 𝟐𝐛𝟑 + 𝟐𝐜 𝟑 ≥ 𝐚𝐛 𝐚 + 𝐛 + 𝐛𝐜 𝐛 + 𝐜 + 𝐚𝐜 𝐚 + 𝐜 ... (LQQD)

100. Prove that for all real numbers 𝒂, 𝒃, 𝒄
𝟐 𝟐
𝒂+𝒃+𝒄 𝒂𝒃 + 𝒃𝒄 + 𝒄𝒂 + 𝟏𝟖 𝒂 + 𝒃 + 𝒄 𝒂𝒃 + 𝒃𝒄 + 𝒄𝒂 𝒂𝒃𝒄
≥ 𝟒 𝒂 + 𝒃 + 𝒄 𝟑 𝒂𝒃𝒄 + 𝟒 𝒂𝒃 + 𝒃𝒄 + 𝒄𝒂 𝟑
+ 𝟐𝟕 𝒂𝒃𝒄 𝟐
Proposed by Nguyen Viet Hung – Hanoi – Vietnam
Solution by Kevin Soto Palacios – Huarmey – Peru

→ 𝑹𝑯𝑺 = 𝟒 𝒂𝟑 + 𝟑 𝒂 𝒂𝒃 − 𝟑𝒂𝒃𝒄 +

𝟑 𝟐
+𝟒 𝒂𝒃 +𝟑 𝒂𝒃 𝒂 𝒂𝒃𝒄 − 𝟑 𝒂𝒃𝒄 + 𝟐𝟕 𝒂𝒃𝒄 𝟐

→ 𝑹𝑯𝑺 = 𝟒 𝒂𝟑 𝒂𝒃𝒄 + 𝟒 𝒂𝒃 𝟑
+ 𝟐𝟒 𝒂 𝒂𝒃 𝒂𝒃𝒄 + 𝟑 𝒂𝒃𝒄 𝟐
𝟐
⇒ 𝑳𝑯𝑺 = 𝒂𝒃 𝒂 + 𝒃 + 𝒃𝒄 𝒃 + 𝒄 + 𝒂𝒄 𝒂 + 𝒄 + 𝟑𝒂𝒃𝒄 +
+𝟏𝟖 𝒂 + 𝒃 + 𝒄 𝒂𝒃 + 𝒃𝒄 + 𝒂𝒄 𝒂𝒃𝒄
𝟐 𝟐 𝟐
→ 𝑳𝑯𝑺 = 𝒂𝒃 𝒂+𝒃 + 𝟗 𝒂𝒃𝒄 + 𝟐𝒂𝒃𝒄 𝒂 𝒂 + 𝒃 𝒂 + 𝒄 +

+𝟔𝒂𝒃𝒄 𝒂𝒃 𝒂 + 𝒃 + 𝟏𝟖 𝒂 𝒂𝒃 𝒂𝒃𝒄

137
SOCIETATEA DE ȘTIINȚE MATEMATICE DIN ROMÂNIA
ROMANIAN MATHEMATICAL SOCIETY

Filiala Mehedinți - Mehedinți Branch


www.ssmrmh.ro

→ 𝑳𝑯𝑺 − 𝑹𝑯𝑺:
𝟐 𝟐
→ 𝒂𝒃 𝒂+𝒃 − 𝟒𝒂𝒃 − 𝟔 𝒂 𝒂𝒃 𝒂𝒃𝒄 + 𝟔𝒂𝒃𝒄 𝒂𝒃 𝒂 + 𝒃 +

+𝟐𝒂𝒃𝒄 𝒂 𝒂+𝒃 𝒂+𝒄 −𝟒 𝒂𝟑 𝒂𝒃𝒄 + 𝟔 𝒂𝒃𝒄 𝟐

𝟐 𝟐
→ 𝒂𝒃 𝒂−𝒃 − 𝟔𝒂𝒃𝒄 𝒂𝒃 𝒂 + 𝒃 + 𝟑𝒂𝒃𝒄 𝒂𝒃𝒄 +

+𝟔𝒂𝒃𝒄 𝒂𝒃 𝒂 + 𝒃 + 𝟐𝒂𝒃𝒄 𝒂 𝒂+𝒃 𝒂+𝒄 −𝟒 𝒂𝟑 𝒂𝒃𝒄 + 𝟔 𝒂𝒃𝒄 𝟐


𝟐 𝟐
→ 𝒂𝒃 𝒂−𝒃 + 𝟐𝒂𝒃𝒄 𝒂 𝒂+𝒃 𝒂+𝒄 −𝟒 𝒂𝟑 𝒂𝒃𝒄 − 𝟏𝟐 𝒂𝒃𝒄 𝟐
𝟐 𝟐
→ 𝒂𝒃 𝒂−𝒃 + 𝟐𝒂𝒃𝒄 𝒂𝟑 + 𝒂𝟐 𝒃 + 𝒂𝟐 𝒄 + 𝒂𝒃𝒄 −

−𝟒 𝒂𝟑 𝒂𝒃𝒄 − 𝟏𝟐 𝒂𝒃𝒄 𝟐

𝟐 𝟐
⇒ 𝒂𝒃 𝒂−𝒃 −𝟐 𝒂𝟑 𝒂𝒃𝒄 + 𝟐𝒂𝒃𝒄 𝒂𝟐 𝒃 + 𝒂𝟐 𝒄 − 𝟔 𝒂𝒃𝒄 𝟐

𝟐 𝟐
⇒ 𝒂𝒃 𝒂−𝒃 +𝟐 𝒂𝒃𝒄 −𝒂𝟑 + 𝒂𝟐 𝒃 + 𝒂𝟐 𝒄 − 𝒂𝒃𝒄

𝟐 𝟐
⇒ 𝒂𝒃 𝒂−𝒃 +𝟐 𝒂𝒃𝒄 −𝒂𝟐 𝒂 − 𝒃 + 𝒂𝒄 𝒂 − 𝒃 =

𝟐 𝟐
= 𝒂𝒃 𝒂−𝒃 +𝟐 𝒂𝒃 𝒂𝒄 𝒂 − 𝒃 𝒄 − 𝒂
𝟐 𝟐 𝟐 𝟐 𝟐 𝟐
𝒂𝒃 𝒂−𝒃 + 𝒃𝒄 𝒃−𝒄 + 𝒂𝒄 𝒄−𝒂 +
+𝟐𝒂𝒃𝒄 𝒂 𝒂 − 𝒃 𝒄 − 𝒂 + 𝒃 𝒃 − 𝒄 𝒂 − 𝒃 + 𝒄 𝒄 − 𝒂 𝒃 − 𝒄
𝟐
⇒ 𝒂𝒃 𝒂 − 𝒃 + 𝒃𝒄 𝒃 − 𝒄 + 𝒄𝒂 𝒄 − 𝒂 =
𝟐 𝟐 𝟐 𝟐
= 𝒂−𝒃 𝒂−𝒄 𝒃−𝒄 = 𝒂−𝒃 𝒂−𝒄 𝒃−𝒄 ≥ 𝟎
(LQQD)
La igualdad se alcanza cuando: 𝒂 = 𝒃 ∨ 𝒃 = 𝒄, 𝒄 = 𝒂

138
SOCIETATEA DE ȘTIINȚE MATEMATICE DIN ROMÂNIA
ROMANIAN MATHEMATICAL SOCIETY

Filiala Mehedinți - Mehedinți Branch


www.ssmrmh.ro

Its nice to be important but more important its to be


nice.
At this paper works a TEAM.
This is RMM TEAM.
To be continued!
Daniel Sitaru

139

S-ar putea să vă placă și